Está en la página 1de 431

1

2
UNIVERSIDAD ESTATAL A DISTANCIA
COLEGIO NACIONAL DE EDUCACIÓN A DISTANCIA
COORDINACIÓN ACADÉMICA

Antología de Matemáticas

Nivel Noveno

Año: 2018

3
Información administrativa
El CONED agradece al Msc. Jorge Alonso Díaz Porras oriundo de Heredia y graduado
de la Universidad Nacional y a la Lic. Annia María Marín Alvarado oriundo de Moravia
y graduada de la Universidad de Costa Rica por la elección y presentación de los
temas del presente material, así como el aporte a la educación secundaria a
distancia.
Las denominaciones empleadas en esta publicación la forma en que aparecen
presentados los datos, no implican de parte del CONED y la UNED juicio alguno
sobre la condición jurídica de personas o países, territorios, ciudades o de
autoridades

MATERIAL SIN FINES COMERCIALES PARA USO EXCLUSIVO DE


ESTUDIANTES DEL COLEGIO NACIONAL DE EDUCACIÓN A DISTANCIA

CONED
Dirección General: Clara Vila Santo Domingo
Coordinación Académica: Paola Mesén
Coordinador de área: Jorge Díaz Porras
Teléfonos 22-58-22-09 / 22-55-30-42 / 22-21-29-95
Página Web: http//www.coned.ac.cr
© 2018, CONED.

4
Tabla de contenido
Capítulo 1 Números (nivel 9)

Objetivos generales 7
Lista de conceptos claves 7
Introducción 8
Tema 1 Números Irracionales 5
Ejercicios 22,32,34
Tema 2 Números Reales 37
Ejercicios 43,51,59,62,65,67
Tema 3 Cantidades Grandes y Pequeñas 71
Ejercicios 76
Practica Unidad 1 79
Capítulo 2 Geometría (nivel 9)
Objetivos generales 87
Lista de conceptos claves 87
Introducción 88
Tema 1 Teorema de Pitágoras 88
Ejercicios 94,
Tema 2 Distancia Entre dos Puntos 105
Ejercicios 111
Tema 3 Trigonometría 115
Ejercicios 119,128,145,156
Tema 4 Visualización Espacial 163
Ejercicios 172,175
Práctica de la Unidad 2 187
Ejercicios 167
Capítulo 3 Relaciones y Algebra (nivel 9) 203
Objetivos Generales 203
Lista de Conceptos Claves 203
Introducción 204
Tema 1 Función Cuadrática 204
Ejercicios 211
Tema 2 Expresiones Algebraicas 217
Ejercicios 222,227,232,236,239,243,248,252,257,266
Tema 3 Ecuaciones Cuadráticas 301
Ejercicios 205,211, 216, 226,332
Tema 4 Funciones Cuadrática 335
Ejercicios 342,349
Práctica de la Unidad 2 349
Capítulo 4 Estadística Y Probabilidad (nivel 9) 363
Objetivos Generales 363
Lista de Conceptos Claves 363
Introducción 363
Tema 1 Variables 364
Ejercicios 383, 385
Tema 2 Probabilidad 401
Ejercicios 410
Practica Unidad 4 420
Bibliografía 430

5
6
Nuestro primer
desafío matemático,

Capítulo I Números un paso más para


aprender

Objetivos

Al finalizar el capítulo el estudiante deberá estar en capacidad de:

1. Comparar números reales en sus diferentes representaciones

2. Seleccionar y aplicar métodos y herramientas para calcular y operar con


números reales
3. Utilizar la estimación, el cálculo mental, el papel y lápiz o la calculadora, según sea el
caso, para el cálculo de operaciones con números enteros, racionales y reales.
4. Plantear y resolver problemas en diferentes contextos donde se requiera el uso
de las operaciones y representaciones numéricas.

5. Utilizar diferentes representaciones para identificar y representar números


racionales e irracionales

Conceptos clave

1. Irracionales 4.Cantidades 7.Reales


pequeñas
2. Racionales 8.Operaciones
5.Cantidades Grandes
3. Recta numérica 9.Periodica y no
6.Expancion decimal Periódica

7
 Introducción
Al ingresar al Tercer ciclo cada estudiante trae la habilidad de comparar y operar
tanto números naturales como números con expansión decimal hasta la
diezmilésima. La potenciación se trabaja en 6° Año pero
con ejemplos muy básicos, principalmente de cuadrados y
cubos perfectos. Con respecto a las fracciones, domina sus
diferentes representaciones y su operatoria. Conoce algunos
conceptos de la teoría de números, como por ejemplo
número primo, compuesto, divisores, múltiplos, entre otros.
La conceptualización de los números enteros, racionales,
irracionales y reales junto con su operatoria, son temas
fundamentales en este ciclo y en toda la enseñanza Secundaria.
En este ciclo se aborda el cálculo de sumas, restas, multiplicaciones, divisiones,
potenciación y radicación para los números enteros
Deseamos que este curso pueda resultarles de gran provecho y sobre todo de
motivación para avanzar en los cambios que en la enseñanza y aprendizaje de las
matemáticas requieren nuestros niños y jóvenes.

Tema 1 Números Irracionales

Situación Problema
Observe los siguientes números

1,217391304347826086695652173913043478260866956521739130434782608
66956521739130434782608669565217391304347826086695652173913043478
26086695652173913043478260866956521739130434782608669565217391304
34782608669565217391304347826086695652173913043478260866956521739
13043478260866956521739130434782608669565217391304347826086695652
17391304347826086695652173913043478260866956521739130434782608669
56521739130434782608669565217391304347826086695652173913043478260
86695652173913043478260866956521739130434782608669565217391304347
82608669565217391304347826086695652173913043478260866956521739130
43478260866956521739130434782608669565217391304347826086695652173
91304347826086695652173913043478260866956521739130434782608669565…

8
Caracterice estos números, determinando semejanzas y diferencias entre ellos.

Análisis de la actividad

Esta actividad tiene por objeto comenzar a establecer la existencia de números


irracionales mediante la exploración de su notación decimal. Es claro que después
de una minuciosa revisión de las cifras de los tres números anteriores se puede
destacar una característica común a ellas y es que su expansión decimal es infinita.
Con respecto a las diferencias se puede observar que los dos últimos números
tienen la particularidad de que su expansión decimal

Uno como tutor no tendrán dificultad en notar que el segundo y el tercer número
son aproximaciones de e y π respectivamente. Es probable que si esta actividad es
desarrollada por el estudiante, quizás reconozca el número π.

9
En el contexto estudiantil es importante que la actividad promueva la identificación
de números que son racionales y los que no lo son, apoyados en la experiencia que
éstos han adquirido en 8° Año en el tratamiento de los primeros.

Es importante que al identificar el primer número de esta actividad como racional,


hay que tener presente que se puede expresar como el cociente de dos números
enteros. Se puede cuestionar si los otros dos números pueden ser expresados de
esa forma.

Esta actividad puede servir como una oportunidad para introducir los números
irracionales por medio del componente histórico.
Todo lo anterior permite sentar una base para que se pueda formalizar con los
estudiantes el concepto de número irracional y sus diversas formas de
representación.
Nota: Previamente es importante discutir cómo el nacimiento de nuevos tipos de
números obedece a la necesidad del hombre por representar y modelar situaciones
que no se ajustan a la realidad o las formas de representación numérica existentes.

Repaso de conjuntos numéricos:

1) Números Naturales ( IN ): 0, 1, 2, 3, 4, 5, 6, 7, 8, 9, 10,


11, 12, …..

Ejercicio: Complete los espacios utilizando uno de los símbolos  


según corresponda:

7
a) -10 ____IN b) 884 _____IN c) _____IN d) 24,00_____IN
1
1 5 7 18
e) _____IN f) _____IN g)  ____IN h) _____IN
5 8 4 6
i) 9,25 _____IN

2) Números Enteros ( ):

……. –7 , –6 , –5 , –4 , –3 , –2 , –1 , 0 , 1 , 2 , 3 , 4 , 5 , 6 , 7 , 8 , ……

10
Ejercicio: Complete los espacios utilizando uno de los símbolos  , 
según corresponda:

12 2 4
a) _____ c) 8,72 _____ e)  _____ g) _____
1 11 5

b) 14, 6 _____ d) 8,727272… _____ f) –24,0 ______

*Nota: Los números naturales son un subconjunto de los números enteros.


Simbólicamente se escribe así: IN 

3) Números Racionales ( Q ):

El conjunto de los números racionales está formado por todos los números
naturales, los números enteros, las fracciones, los números con expansión
decimal finita y los números con expansión decimal infinita periódica.

 Ejemplos de números racionales:

1 8 1 2
 0 1 –7 2,125 –9,6 12,314 5,03 7 28
2 3 5 3

11
Ejercicio: Escriba  ó  según corresponda.

2
a) –9 _____ g) ______ IN
9
b) 12 _____ – h) 3,75 ______ Q.
8
c) _____ Q + i ) –2,9 ______
3
d) –3 _____ IN j ) 4 7,28 _______ IN
1 2
e) ______ k) 8 ______ Q
5 5
f) 16 ______ IN l ) –15 ______ Q –

*Nota: El conjunto de los números naturales es un subconjunto del conjunto de


los números enteros, y los números enteros son un subconjunto del conjunto de
los números racionales. Simbólicamente se escribe así:

IN   Q

También se puede representar por medio de un esquema como el siguiente:

Ejercicio: Escriba  ó  según corresponda.

a) IN  h) Q – ____ Q
b) IN _____ Q i) ____ IN
c)  Q j ) Q ____

Q+  Q – k) – ____

12
La clave

Un número irracional es un número que no se puede escribir como el


cociente indicado de dos números enteros (una fracción). Ejemplo: π es
un número irracional. Su valor aproximado es
3,1415926535897932384626433832795… La expansión decimal no sigue ningún
patrón, por lo que se dice que este tipo de números tiene expansión decimal infinita
no periódica.

Números irracionales famosos

Pi es un número irracional famoso. Se han


calculado más de un millón de cifras decimales y
sigue sin repetirse. Los primeros son estos:

3.1415926535897932384626433832795 (y
sigue...)

El número e (el número de Euler) es otro número


irracional famoso. Se han calculado muchas cifras
decimales de e sin encontrar ningún patrón. Los
primeros decimales son:

2.7182818284590452353602874713527 (y
sigue...)

La razón de oro es un número irracional. Sus


primeros dígitos son:

1.61803398874989484820... (y más...)

Varias escuelas celebran el día de Pi el 14 de marzo. Usa la caricatura para


sugerir por qué se usa ese día y a qué hora comienza la celebración. (En inglés el
nombre de la letra Pi suena pai.)

13
Muchas raíces cuadradas, cúbicas, etc. también son irracionales. Ejemplos

3 =1,7320508075688772935274463415059…

99 = 9,9498743710661995473447982100121…

Pero 4 = 2, y 9 = 3, así que no todas las raíces son irracionales


Los números irracionales son aquellos que tienen expansión decimal infinito y
no periódico.

Ejemplos de números irracionales:

a) –6,712935148862….. c) e = 2,7182818284….
e) 3 = 1,73205080…. b)  = 3,1415926535897…..
d) – 2 = –1,41421356…

CRITERIOS PARA IDENTIFICAR Y CONSTRUIR NÚMEROS IRRACIONALES

Son números irracionales:

Cualquier La raíz de cualquier Algunos números Todo


raíz de un número natural, que no especiales número
número es la enésima potencia cuya
primo de otro natural parte
5 , 6 7 , 8 19   3,141592... e  2,71828... decimal
sea
18 , 6 11, 10
2, 3 5 infinita y
no
periódica.
Ejercicios

1) Escriba  ó  según corresponda:

a) 4 ____ Q c) 3
9 ______ II e) 4
81 _______ II

b) – 100 _____ Q d) 5
2 ______ Q f) – 20 _______ II

14
2) Clasifique los siguientes números como racionales o irracionales:

a) 5 ______ e) 5 6 4 _______ i) 4, 21 8 ________

3 27
b) ______ f) 0 _______ j) 3 _______
7 8

1 25
c) –7, 5 ______ g) 3 _______ k) 4 _______
2 16

d) – 3 12 ______ h) – 529 _______ l) 0, 12434343… _______

Un número irracional es el que tiene infinitas cifras decimales no periódicas.


Además, no se puede expresar como una fracción.
El conjunto de los números irracionales se representa por II.

Ejemplos

¿Cuánto mide la diagonal de un cuadrado cuyo lado mide 1 cm?


Aplicamos el Teorema de Pitágoras y calculamos la diagonal del cuadrado:

Los números irracionales son números decimales con un número ilimitado de


cifras decimales no periódicas que no se pueden expresar en forma de fracción y,
por lo tanto, no son números racionales.

Puedes comprobar con tu calculadora que 2 tiene infinitas cifras decimales no


periódicas y no se puede expresar como una fracción. Por eso decimos 2 que es
un número irracional.

Al calcular números irracionales en la calculadora, debemos tener en cuenta que lo


que vemos en la pantalla es una aproximación. Observa el resultado de 2

15
Situación Problema
Demostrar que √2 es un número irracional.

Análisis de la actividad

Este tipo de actividad debe servir como un elemento que permite fomentar
procesos de argumentación matemática. Por otra parte, el incluir elementos de
teoría de números permite establecer conexiones con otros temas matemáticos.
Esta demostración se realizará por contradicción, la cual se encuentra en
Elementos de Euclides, el texto con un mayor número de ediciones publicadas
después de la biblia. Cabe destacar que existen otras demostraciones de índole
geométrica y analítica para esta misma situación.
Supóngase que √2=𝑝 donde el máximo común divisor de 𝑝 y de 𝑞 es 1.
Elevando al cuadrado a ambos miembros, se tiene

  p2
2
2  2
q
p2
2
q2
2q 2  p 2

De lo anterior, 𝑝2 es múltiplo de 2, por lo que 𝑝 es múltiplo de 2 y puede ser


expresado de la forma =2𝑘, k entero. Luego, 2𝑞2= (2𝑘)2=4𝑘2
𝑞2=2𝑘2
Con lo cual, 𝑞2 es múltiplo de 2 y 𝑞 es múltiplo de 2. Así, 𝑝 y 𝑞 son múltiplos de 2,
lo que contradice nuestra suposición inicial. Por lo tanto, √2 tiene que ser
irracional.

Nota: Esta demostración clásica permite trabajar algunos elementos de la lógica,


tales como el significado de la demostración por contradicción, donde se parte
de un supuesto verdadero hasta llegar a una proposición que contradice dicho
supuesto, así como los métodos de inducción y deducción. Además se utilizan
conceptos básicos de teoría de números.

Ejemplos Identifica y clasifica los siguientes números en racionales o


irracionales.
a) 4,3235325… Irracional b) 3,010010001… Irracional
c) 2,3334545… Racional d) 0,098 Racional
e) 9,17173797… Irracional f) 5,9865798657… Racional
¿La expresión (π+ 2) es un número racional o irracional?

16
Ejemplo

De acuerdo con lo estudiado hasta ahora es claro cuáles, de los números que se
muestran a continuación, son racionales o irracionales.

2
a)  0, 4 es un número racional
5

b) 8,15436 es un número racional

c) -2, 10100100010000… es un número irracional

d) 13
12  1, 21063... es un número irracional

e) 5, 1616161616… es un número racional

f) - 2 es un número irracional

g) e3 es un número irracional

h) sen 2 es un número irracional

Otros números irracionales

En sétimo y octavo año usted calculó la raíz cuadrada de números racionales como:
16
4, 49, , 0,25, 0,04, entre otros. Estos números tienen la característica de que se
9
obtienen al elevar un número racional al cuadrado.

Surge entonces la pregunta, ¿será posible calcular la raíz cuadrada de números


racionales que no son cuadrados perfectos? Considere la siguiente situación:

En la figura 3 se muestra un cuadrado formado por la unión de cuatro triángulos


isósceles rectángulos, cuyos catetos tienen longitud 1:

Recuerde:

La raíz cuadrada de un número


racional positivo a es el único
número positivo b que elevado al
cuadrado es igual a a .

a  a  b2  a

17
11
Cada uno de estos triángulos tiene un área de:  0,5
2

Como el área del cuadrado viene dada por la suma de las áreas de los cuatro
triángulos se tiene que el área del cuadrado es igual a 2:

A  0,5  0,5  0,5  0,5  2

Si x representa la medida del lado del cuadrado que se muestra en la figura 3 (que
corresponde a la hipotenusa de cada uno de los triángulos rectángulos) debe
cumplirse entonces que x 2  2 . Luego se concluye que debe existir un número cuyo
cuadrado sea igual a 2.

El único número positivo cuyo cuadrado es igual a 2 se representa por 2 .

El concepto de raíz cuadrada era conocido por los sumerios alrededor del año 3 000
a.C. Esta Civilización, por métodos que aún se desconocen, llegó a resolver
problemas prácticos de mucha complejidad para su época. Por la necesidad de
calcular el área de la superficie cuadrada de un terreno, cuando conocían su lado,
construyeron tablas de cuadrados. Crearon tablas de raíces, al darse cuenta de que
si conocían el área de una superficie cuadrada, entonces sabían de inmediato la
longitud de su lado.

Alrededor del siglo V a.C., los griegos habían demostrado que 2 no es racional,
es decir, no es posible expresarlo como el cociente de dos números enteros: 2 es
un número irracional.

Un número irracional es aquel que no se puede expresar como el cociente de dos


números enteros. Su expansión decimal es infinita y no periódica.

3 16 1
Son números racionales por ejemplo: , 7, ,4 y también
7 125 2

1,327272727..., 3,01201201 Son números irracionales por ejemplo: 2,  ,   e, y
3
también 1,320332033320..., 51,010320425617...

¿Cómo determinar la expansión decimal de 2 ? O sea, ¿cómo resolver la ecuación


x 2  2 ? o al menos ¿cómo encontrar un número racional que se aproxime lo más
posible a la solución de esta ecuación?

Para lograr una aproximación de 2 por números racionales, lo que es lo mismo,


buscar los primeros dígitos de su expansión decimal, se escogen dos números tales

18
que el cuadrado de uno de ellos sea menor que 2 y el cuadrado del otro sea mayor
que 2, como se muestra a continuación:

Si toma x  1 se tiene que 12  1 y si toma x  2 entonces 22  4 . Como 1 < 2 < 4 el


número que se busca debe ser mayor que 1 y menor que 2. Considere ahora
x  1,1 y x  1,9 y realice los cálculos entonces (1,1)2  1,21 y (1,9)2  3,61 ; luego se puede
concluir que el número está entre 1,1 y 1,9.

De esta manera es posible acercarse más y más al número 2 tomando números


mayores que 1,1 y menores que 1,9 y calculando sus cuadrados. En efecto:

Si x  1, 4 entonces x 2  1,96

Si x  1,5 entonces x 2  2, 25

Si x  1, 41 entonces x 2  1,9881

Si x  1, 45 entonces x 2  2,1025

Si x  1, 414 entonces x 2  1,9993

Si x  1, 43 entonces x 2  2,0449

Si x  1, 4142 entonces x 2  1,9996

Si x  1, 4143 entonces x 2  2,00024

Si se continúa averiguando números cuyos cuadrados se acerquen a 2, jamás se


logrará determinar aquel cuyo cuadrado sea exactamente igual a él, ya que su
expansión decimal es infinita y no periódica. Sin embargo, mediante el método
anterior, se pueden determinar excelentes aproximaciones. Por ejemplo
1,4142135, se llama una aproximación por defecto, pues 1,4142135  1,99999998
2

que es menor que 2. Si 10 m 2 2  1,4142136 se llama una aproximación por


exceso, dado que 1,4142136   2,00000001 y es mayor que 2.
2

Así x=1,4142135 satisface aproximadamente la ecuación x 2  2 . Lo mismo que x =


- 1,4142135, ya que  1,4142135  1,99999998 .
2

19
Para expresar el número cuyo cuadrado es igual a 2, se escribe x  2 de lo que se
concluye, que:

x 2  2 si y solo si x  2 ó x   2

Se dice también que la solución x es igual a más (menos) la raíz cuadrada de 2, lo


que se acostumbra escribir como:

x 2  2 si y solo si x   2

Ahora se sabe que 2 es un número irracional, y por ende su expansión decimal


es infinita y no periódica, a veces usted se podría ver en la necesidad de acercarse
a él mediante un número racional. En la práctica, si al resolver un problema del
entorno aparece un número irracional, es necesario aproximarlo por exceso o por
defecto a un racional. Puesto que no se acostumbra comprar en la ferretería 2
metros de cable o 4 10 m 2 de cerámica.

Así, ya se está en capacidad de decir que el lado del cuadrado de la figura 3 mide
aproximadamente 1,4142135 o que es igual a 2 .

Ejemplo 1

Determine la solución de la ecuación x 2  5 .

Solución

Si x 2  5 , entonces x   5 o bien x≈2,236068 o x≈-2,236068.

Determine la solución de la ecuación x 2  17 .

Solución

Si x 2  17 , entonces x   17 o bien x≈4,1231056 o

X≈-4,1231056.

Además de 2 , existen muchas otras raíces cuadradas de números racionales que


son irracionales. En general la raíz cuadrada de un número que no se pueda
expresar como el cuadrado de otro racional es un irracional.

20
Ejemplo:

 3 es irracional, porque 3 no es cuadrado perfecto.


5
 es irracional, porque no existe ningún número racional que elevado al
3
5
cuadrado sea igual a .
3
 0,1 es irracional porque no existe ningún número racional que elevado al
cuadrado sea igual a 0,
También son números irracionales:

 3
6 porque no existe ningún número racional que elevado al cubo sea igual a
6.
 5 16 porque 16 no se puede expresar como una La raíz n-ésima, n  , n
potencia de un número racional de exponente 5. par, de un número
2 racional positivo a es el
 3 porque no existe ningún número racional que
3 único número positivo
2 que elevado a la n es
elevado a la 3 sea igual a .
3 igual a a .
 7 10 porque no existe ningún número racional que
elevado a la 10 sea igual a 7.
n
a  b  bn  a
La raíz cúbica de un número que no es el cubo de un
La raíz n-ésima, n  , n
número racional es irracional; la raíz cuarta de un
impar, de un número
número que no es una potencia de cuatro de un número
racional a es el único
racional es un número irracional, etc. En general la raíz
número que elevado a la
n-ésima, a , de un número a , n
n
que no es una n es igual a a
potencia de n de un número racional, es un número n
a  b  bn  a
irracional y son llamados números algebraicos.

Ejemplo 2

a) Determine la solución de la ecuación x 3  5 .

Solución

 5
3
b) Si x 3  5 , entonces x  3 5 , puesto que
3
5 .

21
b) Determine el conjunto solución de la ecuación x 4  7 .

Solución

 7 
4
Si x 4  7 , entonces x   4 7 , puesto que 4
7.

c)Por lo tanto el conjunto solución de la ecuación es:

 4

7,  4 7 .

Determine la solución de la ecuación x15  5 .

Solución

Si x15  5 , entonces x  5 .
15

Ejemplo 3

Determine la longitud de los lados de un cuadrado cuya área es igual a 15 cm 2 .

Solución

Recuerde que si la longitud de los lados de un cuadrado es x cm , entonces su área A


es A  x 2 .Por lo tanto, si el área es igual a 15 cm 2 , se tiene que x 2  15 , entonces la
longitud de cada uno de los lados es x  15 .

Observe que x   15 también es una solución a la ecuación planteada, pero en


este caso no tiene sentido considerarla, ya que x representa la medida de un lado
del cuadrado y por ende una magnitud positiva.

Ejemplo 4

De acuerdo con lo estudiado hasta ahora es claro cuáles, de los números que se
muestran a continuación, son racionales o irracionales.

2
a)  0, 4 es un número racional
5

b) 8,15436 es un número racional

c) -2, 10100100010000… es un número irracional

22
d) 13
12  1, 21063... es un número irracional

e) 5, 1616161616… es un número racional

f) - 2 es un número irracional

g) e3 es un número irracional

h) sen 2 es un número irracional

Ejercicios

1) Diga cuáles de los siguientes números son irracionales y explique su


respuesta.

a) 3,14 b) 225 c) 2,12122122212222… d) 4


8


e) -4,123123123123… f)   e g) h) 22
3

i) -1,4142 j) log 5 k) 5
32 l) 2e  1

2) Use la calculadora para encontrar una expansión decimal que aproxime los
siguientes números irracionales:
3
5
a)   e b) -2  c) e 2 d) 3 5  7 e) f) sen 45°
3

3) Determine la solución de las siguientes ecuaciones:


a) x 3  8 b) x 6  2 c) x 7  1 d) x 5  25 e) x 5  32

El conjunto de los números irracionales, está formado por todos los números
estudiados en esta sección y aquellos que comparten la característica de ser
expresados como una expansión decimal infinito y no periódico.

23
Ejercicios

1) Diga cuáles de los siguientes números son irracionales y explique su


respuesta.
a) 3,14 b) 225 c) 2,12122122212222… d) 4 8


e) -4,123123123123… f)   e g) h) 22
3

i) -1,4142 j) log 5 k) 5
32 l) 2e  1

2) Use la calculadora para encontrar una expansión decimal que aproxime los
siguientes números irracionales:
3
5
a)   e b) -2  c) e 2 d) 3 5  7 e) f) sen 45°
3

3) Determine la solución de las siguientes ecuaciones:


a) x 3  8 b) x 6  2 c) x 7  1 d) x 5  25 e) x 5  32

4) Determine cuáles de los siguientes números son irracionales


1) 9 6) 6
7 11)1,010010001…

2) 10 7) 3
125 12) e
3) 16 8) 4
100 13) sen  45 

4) 22 9) log 5125 1
14)
7
5) 25 10) log 6 4 15) 0

6) Escribe 5 ejemplos para cada uno de los criterios que sirven en la


identificación de números irracionales.

24
7) Responde:

a. ¿Qué es un número irracional?

b. ¿Cuál es la diferencia entre número racional y número irracional?

c. ¿Por qué afirmamos que el número  es irracional?

8) Indica cuáles de las expresiones que siguen representan números racionales y


cuáles números irracionales.

a. 0,37 f. 2,2360679...

b. 0,13666... g. 

c. 5/13 h. 2  /3

d. 22/7 i. 2 + 3

e. 2 j. 9

9). Escribe en tu cuaderno falso (F) o verdadero (V) según corresponda. Justifica
tu respuesta.

a. 5 es un número racional. b. 2,5 es un número irracional.

c. 2 es un número racional d. 10 es un número irracional

e. Los números irracionales son racionales. f. Ningún número entero es racional

g. Algún número entero es racional. h. Ningún número irracional es entero

25
i. Todo número natural es entero j. Al menos un número irracional es racional

k. Algún número racional no es irracional l. Ningún número irracional es entero.

10) Copia este diagrama de Venn en tu cuaderno y ubica en la región


correspondiente cada uno de los siguientes números

Z N

a. 2,5 d. 18 g. 8/3

b. 6 e. 1,41 h. ¾

c. -3 f. 0 i. 25

11). Indica cuáles de las siguientes cantidades son una representación de número
irracional y cuáles no.

a. 5 e. 3
9

1
b. 4 f.
4

c. 9 g. 0,04

d. 3
4 h. 3
8

26
Historia
Uno de los pueblos de Mesopotamia, los sumerios, determinó la relación:

longitud de la circunferencia
3
longitud del diámetro

Así, si la circunferencia medía 60 codos, el diámetro debía medir 20 codos.


Descubrieron que si usaban otras unidades de medida, por ejemplo, la palma de la
mano, la relación era la misma: si la longitud de la
circunferencia era 150 palmas, entonces el diámetro debía
medir 50 palmas: De esta manera, los sumerios, habían
encontrado un número que miles de años después sería
llamado pi (π) por el gran matemático Suizo Leonard Euler
(1707-1783).

Posteriormente, alrededor del año 3 000 a.C., al aparecer


las fracciones, fue posible efectuar mediciones con mayor
precisión, lo que permitió a los matemáticos babilonios
trabajar este cociente como: El codo fue una unidad de
longitud usada en varias
longitud de la circunferencia 1
 3  o lo que es igual a 3,12.. culturas y era la distancia
longitud del diámetro 8 entre el codo y el final de la
mano abierta. Variaba de
Paralelamente a la sumeria se desarrolló la civilización
un lugar a otro. La figura
egipcia y alrededor del año 3 000 a.C., en un intento por
muestra el codo egipcio
determinar el área del círculo sus matemáticos
(0,45m) del Siglo XIV a.C.
establecieron que pi (π) era igual a:
que se encuentra en el
1 19 Museo de Louvre. París,
3   3,16 .
6 6 Francia.

Según la Biblia, durante el imperio de David y Salomón, los hebreos manejaron el


número pi (π) como 3.

Durante miles de años se creyó erróneamente que el número pi (π) se podía escribir
con una cantidad finita de decimales, es decir, que era un número racional. Esto,
debido a que los métodos de cálculo eran ineficientes y al hecho de que los
matemáticos recientemente estaban descubriendo las diferentes clases de
números.

27
Debido a problemas económicos, políticos y de migración, estas civilizaciones
desaparecieron y otras crecieron en su lugar. Una de ellas fue la griega y su cultura
se extendió aproximadamente, del año 700 a.C. al año 300 d.C. Pero aún ellos,
continuaron creyendo que pi (π) se podía expresar con una cantidad finita de
decimales. Por ejemplo, Arquímedes de Siracusa (287-212 a.C.) lo calculó como)

1 22
3   3,14 .
7 6

Al desaparecer el imperio griego resurge el romano (II a.C.- V d.C.) cuyo aporte al
desarrollo de la matemática fue muy pobre. Sus aportes se dieron en el campo de
las leyes y la administración.

Al surgir el imperio Árabe se enriquece la matemática y en particular el estudio del


número pi (π). Cuando desaparece a fines del Siglo XV florece el Renacimiento en
Europa y durante todo este tiempo los matemáticos continuaban creyendo que pi
(π) era un número racional. Sin embargo, conforme la humanidad iba mejorando
sus métodos de cálculo, los decimales de π iban en aumento y al no hallarse un
periodo en su expansión decimal, se empezó a dudar de su racionalidad. Observe
algunos datos indicando aproximaciones de π

Los Hindúes Siglo I   3, 004 Babilonios, (4 000 a.C.),   3

Los Chinos Siglo III   3,14159 Babilonios, (3 000 a.C.),   3,12

Los Hindúes Siglo IV   3,141692 Egipcios, (3 000 a.C.),   3,16

Los Chinos Siglo V   3,1415926 Hebreos, (4 000 a.C.),   3

Los Árabes Siglo XV   3,1415926535897932 Griegos, (300 a.C.),   3,14

Los Siglo XVI   3,14159265358979323


Italianos

Los Belgas Siglo XVII   3,14159265.... 35


decimales

Los ingleses Siglo   3,14159265.... 127


XVIII decimales

El matemático alemán Ferdinand Von Lindeman demostró, en 1882, que: el número


π no era racional, por lo tanto dicho número no podía expresarse como una fracción
y en consecuencia su expansión decimal tenía que ser infinita y no periódica. Es
decir, se estaba en presencia de un número irracional.

28
Situación Problema

Determine entre qué par de números naturales se encuentra el


número 33

Análisis de la actividad

Este problema tiene por objeto hacer uso de la estimación para realizar
aproximaciones de raíces cuadradas y se espera que el mismo se pueda generalizar
para los casos de raíces cúbicas, cuartas, etc. Aquí sería muy fácil hacer uso de la
calculadora, pero no se debe perder de vista que la estimación es una habilidad
muy importante que se debe ejercitar constantemente para facilitar la resolución
de ciertos ejercicios, por lo que esta actividad constituye una oportunidad para ello.
Se pueden retomar elementos trabajados por el estudiante en años anteriores. Por
ejemplo, la relación que existe entre las potencias y las raíces.

73  343  3 343  7 De ese modo, se puede recurrir a la noción de potencia para

ir realizando aproximaciones
22  4 32  9 4 2  16 52  25 62  36
25  33  36 con lo que
25  33  36

O sea , 33 se ubica enrtre 5 y 6 Como detalle complementario es interesante


exponer el método utilizado por los babilonios para aproximar una raíz cuadrada.

La clave

Utilice su calculadora para obtener una aproximación racional para los


números √999999999991 y √999999999999

b. ¿Considera usted que los números anteriores son iguales

En algunas ocasiones puede utilizarse la calculadora para obtener aproximaciones


decimales a los números irracionales con el objeto de determinar su posición en la
recta numérica. No obstante, debido a la limitada cantidad de dígitos que pueden
mostrarse en las pantallas de las calculadoras, no siempre es posible utilizarlas
para establecer el orden entre dos o más números.

Por ejemplo, en muchas calculadoras los números√999999999991 y aparecen


aproximados por el mismo número racional, a pesar de que ambas raíces son

29
distintas. Si deseamos saber cuál de las raíces anteriores es mayor, la calculadora
no será́ de mucha ayuda, por lo que tendremos que emplear un procedimiento
distinto.

En este caso, como los índices de las raíces son iguales, bastará con que
comparemos sus subradicales.

Así́ √999999999991 y √999999999999 pues 999 999 999 991 < 999 999 999
999.

Algunas veces es conveniente saber qué tan buenas son las aproximaciones que
utilizamos. Si tenemos dos aproximaciones racionales de un mismo número,
diremos que la mejor de estas aproximaciones es la que está más cerca de su
verdadero valor.

Cuando el valor que se utiliza para aproximar una cantidad es menor que su valor
real, se dice que se tiene una aproximación por defecto. Si el valor aproximado es
mayor que su valor real, se dice que se tiene una aproximación por exceso.

Ejemplo
1. Utilice su calculadora para determinar la segunda potencia de cada uno de
los números racionales que se anotaron en la primera fila de la siguiente
tabla. Anote cada resultado en la casilla que se ubica bajo el número
correspondiente
x 1 1,1 1,2 1,3 1,4 1,5 1,6 1,7 1,8 1,9 2
X2 1 4
2. ¿ entre cuáles de los números racionales del arreglo anterior se ubica 2
3. Explique por qué se puede asegurar 1,4 aproxima por defecto a 2
4. Explique por qué se puede asegurar 1,4 aproxima por exceso a 2
5. Utilice su calculadora para determinar la segunda potencia de cada uno de
los números racionales que se anotaron en la fila en la siguiente tabla. anote
cada resultado en la casilla que se ubica bajo el número correspondiente
x 1,4 1,41 1,42 1,43 1,44 1,45 1,46 1,47 1,48 1,49 1,5
X 2 1,96 2,25

¿Entre cuáles de los números racionales del arreglo anterior se ubica el


número√2? Observe

Si se tiene una aproximación por defecto y una por exceso, se puede utilizar su
1, 41  1, 42
semisuma para acelerar el procedimiento anterior  1, 415
2

30
Dado que 1, 415  2,002224  2 podemos asegurar que 1,41  2  1,415 . Tenemos
2

entonces que 1,415 se aproxima por exceso a 2 con dos cifras decimales con
exactitud

Ahora podemos calcular la semisuma de 1,41 y 1,415,

1, 41  1, 415
Esta vez  1, 4125 se aproxima por defecto al valor de 2 ( por
2
qué?)

Observe que con el conocimiento anterior obtuvimos una mejor


aproximación del valor real de 2 , si embargo , dado que 1,4125 y
1,415 coinciden hasta la segunda cifra decimal , solo tenemos con certeza que las
primeras dos cifras decimales de nuestra aproximaciones coinciden con el valor real
que buscamos

Es claro que este procedimiento puede repetirse tantas veces como sea necesario
a fin de obtener aproximaciones cada vez más cercanas al valor real de 2

2
Un numero Real puede tener diferentes representaciones , por ejemplo y
2
1
también . También podemos calcular expresiones como 52  3 donde obtenemos
2
22 . Podemos verificar también que 12  2 3

Analicemos la situación
¿ Entre que par de numeros consecutivos esta 50 ?Podemos ver que 50 es un
numero irracional que esta entre 7 y 8 más cercano a 7
. Determine tres números irracionales que están entre 9 y 10
Es sencillo como 81 es 9 y 100 es 10, todas las raíces cuadradas a partir de
81 y antes de 100 van a estar entre 9 y 10
Ejemplos

1. Determine dos números irracionales entre 12 y 13


Primero 122  144 y 132  169 entonces todas las raíces cuadradas después de
144 y antes de 169 están entre 12 y 13
145  12, 0415946......
Son números que están entre 12 y 13
168  12,9614813968....

31
Ejercicios
1) Escriba entre cuáles números enteros consecutivos se encuentra
cada una de las siguientes raíces:

a) ______ 21 ______ d) ______ – 5 1000 ______

b) ______ 4
925 ______ e) ______ – 209 ______

c) ______ 3
19 ______ f) ______ 3
254 ______

2) Clasifique los siguientes números como racionales o irracionales:

a) 5 ______ e) 5 6 4 _______ i) 4,21 8 ________

3 27
b) ______ f) 0 _______ j) 3 _______
7 8

1 25
c) –7, 5 ______ g) 3 _______ k) 4 _______
2 16

d) – 3 12 ______ h) – 529 _______ l) 0, 12434343… _______

3) Determine 3 numeros irracionales estre los siguinetes numeros


a) 5 y 6
b) 21y 22
c) -3y-4
d) -26y-27
e)
4) Pruebe que 2  5  6  8  3
5) Encuentre dos números consecutivos a y b que cumplan a  45  b
6) Determine entre cuales nuemeros enteros se encuntre los suiguientes
numeros irracionales

a. 3
7 b. 15 c. 3
27
d. 5 e. 3
17 f. -e

g.  3
4
h.  e  
2
i.  3  

32
7) ¿Cuál es el mayor de los números anteriores?

8) Ordene de forma descendente (de mayor a menor) los números de la parte


6.

9) Ubique los números de la parte 1 en la siguiente recta numérica

periódica. Es decir, se estaba en presencia de un número irracional.

10) Escriba dos números irracionales que estén entre los números dados.

1) 1 ___ , ___ 2 2) 2 ___ , ___ 3 3) 5 ___ , ___ 6 4)  9 ___ , ___ -8

12
5)  12 ___ , ___- 11 6) 4
16 ___, ___ 7) 4 ___, ___ 3
125
3

8) 23 ___, ___ 6 9)  10 ___, ____  9 10) e ____, ___ 

6 17
11)  5e ____, ___ 2 e 12) -e ____, ___ 13) ___, ____ - 2 e
5 2

Orden de los números irracionales

Los números irracionales al igual que los números enteros y racionales


mantienen una relación de orden, por lo tanto se puede comparar con
otros números y determinar cuáles son mayores o menores según sea
el caso.

10
Vamos a ordenar los siguientes números de menor a mayor: 29 9 2 3
2

29  5.385164807.....
10
5
Tenemos que 2
9 3
2 3  3.464101615...

10
Así podemos ordenar de menor a mayor 9 2 3 29
2

33
Recordemos que las cantidades están ordenadas de menor a mayor .Por
lo tanto siempre más a las izquierda van los menores y más a ala derecha
van las mayores

Ejemplos

1) 2 3  2 3 Por eso la misma expresión positiva es mayor que toda la


expresión negativa

6 6
 4 Recordemos que en su forma decimal es 1,2 por lo tanto es menor que 4
5 5

2 ) Complete con los signos <,>,= según corresponda

2 5  5 2 ya que 2 5  4,4721... 5 2  7,071....

Ejercicios

1. Complete utilizando los símbolos    , para cada uno de los


siguientes pares de números racionales e irracionales

34
35
36
Tema 2 Números Reales

Situación Problema

Observa los números y responde.

3
-27 16 1,353535… 7 
5

a) ¿Qué números son naturales? 16

b) ¿Qué números son enteros? -27 16

3
c) ¿Qué números son racionales? -27 16 1,353535…
5

d) ¿Qué números son irracionales? 7 

3
e) ¿Qué números son reales? -27 16 1,353535… 7 
5

La Clave

El Conjunto de los números reales

En forma paralela a la aparición de las fracciones, surge el concepto de número


irracional, de manera que cientos de años a.C., con excepción de los números
negativos, ya se tenía idea, aunque no completa de lo que actualmente se llaman
números reales. Recuerde que los números negativos aparecen tardíamente en
la historia, pues se cree que los primeros en referirse a ellos fueron los hindúes
alrededor del siglo V d. C. pero no fue sino a finales del siglo XV, que fueron
aceptados como verdaderos números.

A finales del siglo XIX, se tenía una idea bastante precisa de cómo eran todos los
números; así que cuando Georg Cantor creó la teoría de conjuntos se empezó a
hablar del Conjunto de los Números reales.

¿Cómo está conformado entonces el conjunto de los números reales?

Este conjunto está formado por todos los números que se han estudiado desde
sétimo año hasta este momento.

37
El conjunto de los números reales contiene a todos los números estudiados hasta
ahora, es decir:

ℕ⊂𝑍⊂ℚ⊂ℝ y I ⊂ R.

La expansión decimal caracteriza a los números racionales y a los irracionales.

Así; un número es irracional si su expansión decimal es infinita y no periódica, por


el contrario la expansión decimal de un racional es infinita periódica o finita.

De manera que, un número no puede ser al mismo tiempo racional e irracional.


Dicho de otra forma, un número puede ser solamente racional o solamente
irracional.

Esto significa que la intersección de ambos conjuntos es vacía, es decir:

ℚ ∩ 𝕀 = ∅.

Se define la unión de ambos conjuntos como el conjunto de los números reales

ℚ ∪ 𝕀 = ℝ.

La unión de los números racionales (Q) con los números irracionales (II) forma un
nuevo conjunto que se denomina el conjunto de los números reales (lR).

38
Ejemplos

El Conjunto de los Números Reales Positivos se denota por ℝ+ y se


define como:

ℝ+ = { 𝑥 ∈ ℝ ∶ 𝑥 > 0}.

ℝ+ consiste en aquellos números reales que tienen signo positivo. Son ejemplos
4
de números reales positivos 𝑒, 2, , √5 .
5
El Conjunto de los Números Reales Negativos se denota por ℝ− y se define
como:

ℝ− = { 𝑥 ∈ ℝ ∶ 𝑥 < 0}.

ℝ− consiste en aquellos números reales que tienen signo negativo. Son ejemplos
4
de números reales negativos −𝑒, −2, − , − √5.
5

Asimismo, se cumple que ℝ = ℝ− ∪ {0} ∪ ℝ+ .

Ejemplo

1. Simplifique al máximo cada expresión matemática. Indique a su vez si cada


expresión es un número racional o irracional.

a) √49 b) √8

Solución Solución

√49 = √72 = 7 ∈ ℚ . √8 = √22 ⋅ 2 = 2√2 ∈ 𝕀 .

c) 𝟑√25
1
d) 8 3

Solución Solución
𝟑 𝟑 𝟑 1
√25 = √22 = √25 ∈ 𝕀 . 3 3
83 = √8 = √23 = 2 ∈ ℚ .

e) 𝟐√−25
1
f) 64 2

Solución Solución

39
2 1 3
√−25 ∉ ℝ 642 = √64 = √26 = 4 ∈ ℚ .

g) 𝟒√32 Solución
2
h) 8− 3 Solución

√32 = √25 = √25 ⋅ 2 = 2 √2 ∈ 𝕀 . 8− 23 = 1 = 1 1 1


4 4 4 4

3 = =
√82 3√(23 )2 √26 23
3

1
= ∈ℚ.
8
2 3
i) √ √64 Solución j)
2

144
Solución
49
2 3
√ √64 = 6√64 = 6√26 = 2 ∈ ℚ.
2 144 √144 12
√ = = ∈ℚ.
49 √49 7

k) √−37 ⋅ 23 ⋅ 510 Solución


5

5 5
√−37 ⋅ 23 ⋅ 510 = √−35 ⋅ 32 ⋅ 23 ⋅ 510

5 5
= −3 ⋅ 52 √32 ⋅ 23 = −75 √72 ∈ 𝕀 .

3 −26 ⋅59
2. Simplifique al máximo √ e iindique a su vez si la expresión es un
78
número racional o irracional.

Solución

3 −26 ⋅ 59 3 −26 ⋅ 59 22 ⋅ 53 3 1
√ = √ 6 2 =− √ 2 ∈ 𝕀, ℚ
78 7 ⋅7 72 7

22 ⋅ 53 3 1 7 22 ⋅ 53 𝟑
− 2
√ 2⋅ =− √7 ∈ 𝕀.
7 7 7 73

500 𝟑
= √7 ∈ 𝕀
243

40
3. Discuta la validez o no validez de la siguiente proposición

2 2 2
√32 + 42 = √32 + √42 = 3 + 4 = 7.
Solución La proposición es falsa porque
2 2 2
√3 2 + 4 2 ≠ √3 2 + √4 2 .

4. Clasifica los siguientes números decimales en racionales o irracionales y explica


la razón:

a) 0,55555555...
b) 0,125689312...
c) 1,3525252...
d) 0,75

Solución:
a) 0,55555555... RACIONAL porque es un número decimal periódico y se puede
expresar en forma fraccionaria
b) 0,125689312... IRRACIONAL porque es un número decimal no periódico.
c) 1,3525252... RACIONAL porque es un número decimal periódico y se puede
expresar en forma fraccionaria
d) 0,75 RACIONAL porque es un número decimal exacto

5. Clasifica los siguientes números decimales en racionales o irracionales y explica


la razón:

a) 1,3030030003...
b) 2,1245124512...
c) 4,18325183251...
d) 6,1452453454...

Solución:
a) 1,3030030003... IRRACIONAL porque es un número decimal no periódico.
b) 2,1245124512... RACIONAL porque es un número decimal periódico y se
puede expresar en forma fraccionaria. Su periodo es 1245
c) 4,18325183251... RACIONAL porque es un número decimal periódico y se
puede expresar en forma fraccionaria. Su periodo es 18325
d) 6,1452453454... IRRACIONAL porque es un número decimal no periódico.

41
7. Clasifica los siguientes números decimales en racionales o irracionales y explica
la razón:
 3 1
a) b) 23 c) d)
2 3 100001

Solución:

a) IRRACIONAL porque el numerador de la fracción es un número decimal no
2
periódico.

b) 23 IRRACIONAL, ya que la solución de la raíz tiene ilimitadas cifras decimales


no periódicos.
3
c) IRRACIONAL, ya que el numerador de la fracción tiene ilimitadas cifras
3
decimales no periódicos.
1
d) RACIONAL porque el cociente de la fracción es un número decimal
100001
periódico.

8. Utilice los símbolos ∈, ∉, ⊂,  según corresponda

1 −2 −2
a) ( 2) _____ℚ . √64
√ b) (√ 8
) _____𝕀 .

Solución
Solución
−2
1 2
−2
( ) = (√2) = 2 ∈ ℚ . −2
√2 √64 8
(√ ) = (√ ) = (1) 2 = 1 ∈ ℚ .
8 8

c)
1
(1 + 𝜋) (1 + 𝑒) _____𝕀 . Solución d) ℤ+ _____ ℚ . Solución

⊂.

e) 𝕀 _____ ℝ . Solución ⊂. f) 𝕀 _____ ℚ . Solución  .

9. Escriba F si la opción es falsa y V si la opción es verdadera dentro del paréntesis.


Justifique su respuesta

a) ( ) Si 𝑥√2 ∈ ℚ entonces 𝑥 es un número irracional.

Solución

Verdadero. 𝑥 = √2 ∈ 𝕀 .

42
3 1
b) ( ) La expresión √− representa un número racional entero.
27

Solución

3 1 1
Falso. √− = − Representa un número racional no entero.
27 3

3
√−24
c) ( ) La expresión 3 representa un número entero.
√8

3
√−24 3 24 3
Verdadero. 3 = − √ 3 = − √8 = −2 ∈ ℤ .
√3

d) ( ) La expresión 5,345678489 …., en la cual no se encuentra un periodo, representa


un número racional.

Falso. El número es irracional.

10. Escriba la suma de dos números irracionales cuyo resultado sea un número
racional.

Solución

Una posibilidad es sumar un número con su opuesto. Por ejemplo: 3  3 = 0

5 5 0

11. Encuentre tres números reales que sean mayores que 2 y menores que 3.
Solución

Se pueden dar infinita cantidad de números. Por ejemplo:


7
5, e, , 4 17, 2, 425, 2, 734521786432...
3

43
12. Justifique, si la siguiente afirmación es falsa o verdadera.

  7
3, 4, ,   .
 2 3

Solución

La afirmación es falsa pues es un número irracional, en este caso solo aparece
2
escrito en forma de cociente pero no corresponde a una fracción pues no es el
cociente de dos números enteros.

Ejercicios Resuelve y marca con (✓) si el resultado pertenece al


conjunto indicado.

2. Evalúa las siguientes afirmaciones y escribe (V) verdadera o (F) falsa.

 69 Hay números enteros que no son racionales. ( )


 70 Existen números irracionales que no son números
reales. ( )
 71 Todo número real es racional o irracional. ( )
 72 Cualquier número decimal es un número real. ( )

44
3. Use los símbolos 𝕀, , , , , y complete las siguientes
expresiones:

 II 

II  II 

 II 

1
4. Encuentre tres números irracionales entre y 1.
2

5. Encuentre tres números irracionales entre -2 y-1.

6. Escriba F si es falso o V si es verdadero

 9  19 
( ) 3, 3 3  5, 16,   ( ) 3, 3 16, 25, 
 3  3

 19   19 
( ) 3, 3 16, 25, 𝕝 ( ) 3, 3 16, 17,  ,  
 3  5

( )  ( ) 

7. Determine en el siguiente cuadro a que conjuntos pertenece los siguientes


números

II II

 3 2 2

0 log 5 125

3
125 1
3

1,010010001… 2

45
2 13
3
8
3 5

4 25

3
8 e  3e
2

sen  60  tan(45)

sen  45   tan  30 
In e
cos  30 

Nota la raíz con índice par y el sub radical negativo no es un número real

Ejemplos

1 
4
256 

8. Escriba “F” o “V” según sea falso o verdadero:

2
a) Z _____ h) – 3 24  IR _____ m) Z  II ____
3

b) 64  IN _____ i) 25  IR _____ n) IR  Q _____

c) – 15  II _____ j ) 4
4096  IR _____ o) IR  II _____

1
d) –  Q _____ k ) IN  IR _____ p) II  Q _____
4

e) 5
243  IR _____ l) Z  II _____ q) Z  IR _____

f) –0,8 12  II _____ r) Q  IR _____

g) Si un número es racional, entonces también puede ser irracional. _____

46
9. Determine la notación decimal de cada uno de los siguientes números reales,
e indique si es racional( Q ) o irracional( II ).

3
a) – = _______________ f) 4 3 10 = _________________
8

3
b) 2 = _______________ g) –2 = _________________
4

c) – 27 = _________________ h) – + 7 = ________________

2
d) – 4 625 = _________________ i ) = __________________
7

13 15
e) = ___________________ j ) – = _______________________
3 13

Situación Problema

Dadas las coordenadas a, b, c, d, e, y f como se muestra en la


figura:

1. ¿Cuál es el punto más próximo de a ⋅ b?


1
2. ¿y de ?
d
f
3. ¿y de ?
2
4. ¿Quién es mayor e o |a|?

Análisis de la Actividad
Aunque esta actividad se puede resolver con relativa facilidad sustituyendo por
valores aproximados los números representados, se debe pensar en la justificación
de las conclusiones por medio de argumentos que hagan referencia al uso de las
propiedades de los números.
Por ejemplo, en el caso de la pregunta 1, se pueden considerar propiedades
como el signo de a ⋅ b, y sila magnitud de a ⋅ b es menor o mayor que la de bpara
concluirque el valor más cercano a dicha expresión es d.

47
En el caso de 2, se puede aprovechar que al ser d un valor ubicado entre 0 y 1,
1
representa una cantidad mayor que uno (inclusive mayor que 2), por lo que f es
d
el valor más cercano.
En En 3,basta determinar en la representación gráficael punto medio entre 0 y
f, con lo que e es el valor más próximo. En iv se aplica la noción de valor absoluto
para concluir que a está a una mayor distancia que e con respecto al cero. Así que
|a| >e

La clave

Recta numérica La recta numérica siempre va de valores menores a


valores mayores en sentido de izquierda a derecha. Podemos ubicar infinidad de
cantidades de ella

Dada una recta cualesquiera se puede establecer una correspondencia biunívoca


entre sus puntos y los números reales, es decir, que a cada punto de la recta se le
asocia un número real y a cada número real le corresponde un punto de la recta.
Históricamente este hecho se le atribuye a Richard Dedekind

48
Ejemplo
Trace una recta numérica y ubique en ella, de manera aproximada, los siguientes
números reales: 2,1, 3,0, 1, e, 2,  2 .

-e -2  2 -1 0 1 3 2
2) Para localizar estos puntos de la recta se busca una aproximación del número
que se desea ubicar.
Trace una recta numérica y ubique en ella, de forma aproximada, los siguientes

números reales: 3
11 ,  log17 , ,
2


 log17 0 1 2 3 11
2

3) Ubique en forma ordenada las letras correspondientes a cada número en la


siguiente recta.

Observación: en estos casos en importante el uso de la calculadora para tener


un valor aproximado en notación decimal de los números que no son enteros.

49
4) Si a= 2 Como 2 = 1,4142135. Se ubicara entre 1 y 2 antes de 1,5

𝑎
Para representar un número racional en la recta numérica se efectúa la operación
𝑏
𝑎 ÷ 𝑏 y se obtiene la expansión decimal aproximada por medio de la calculadora.

Para representar un número racional o irracional en la recta numérica se obtiene la


expansión decimal aproximada por medio de la calculadora.

Ejemplos

1. Represente en la recta numérica los siguientes números reales

a) 5, ̅̅
26̅̅

Solución

b)
π 3 34
− + √19 −
4 4

Solución
π 3 34
− + √19 − ≈ −5,04
4 4

50
Ejercicios

1. Use aproximaciones para localizar en la recta numérica , de manera


5 2e
aproximada, los siguientes números:  ,   e, 5 31,
2 3

2. Dibuje una recta numérica y represente en ella los siguientes


números reales:

1 5
–8 3 – – 2 –3 + 12 3
20 e–7
2 3
Sugerencia Se debe hallar la notación decimal que le corresponde a cada
número real, y luego se ubican estas cantidades en la recta numérica.

3. Ubique los números en la recta numérica

5
a) –8 c) – = –1, 6 e) –3 + 12 = 2,57522203…
3

g) e – 7 = –4,281718…

1 7
b) 3 = = 3,5 d) – 2 = –1,4142135….. f) 3
20 = 2,714417….
2 2

Ahora se ubican las cantidades obtenidas tomando en cuenta solo el primer


decimal:

51
4. Ubique en la recta numérica los siguientes números reales

5. Represente en la recta numérica los siguientes números reales

a. 5, ̅26
̅̅̅

π 3 34
b. − + √19 −
4 4

c. π + 𝑒 2

3
d. √54 − 5 − π

52
6. Encuentre cuales números enteros se encuentran las siguientes cantidades
2 a)  5 h) 2 3 4
a.
2
b. 60 b) 2 i) log 3 125
c) 5
50 g) -2e j)  sen  70 

7.Señales las cantidades anteriores en la recta numérica

7. Represente cada pareja de números irracionales en la recta numérica.


a) 2 y
2

2  3
b) y
4 2

 5 
c) y
2 3

53
Uso de los símbolos >, <, ≥, ≤ en ℝ

Es importante que se dominen las siguientes notaciones en ℝ

Concepto Notación Significado


𝑥 es mayor 𝑥>0 𝑥 ∈ ℝ+
que 0 o
que 𝑥 es
positivo
𝑥 es menor 𝑥<0 𝑥 ∈ ℝ−
que 0 o
que 𝑥 es
negativo
𝑥 es mayor 𝑥>𝑦 𝑥 − 𝑦 ∈ ℝ+
que 𝑦
𝑥 es menor 𝑥<𝑦 𝑦 − 𝑥 ∈ ℝ+
que 𝑦
𝑥 es mayor 𝑥≥𝑦 𝑥>𝑦o𝑥=𝑦
o igual que
𝑦
𝑥 es menor 𝑥≤𝑦 𝑥<𝑦o𝑥=𝑦
o igual que
𝑦
𝑦 está entre 𝑥 < 𝑦 < 𝑧 𝑥 < 𝑦, o 𝑦 <
𝑥 y 𝑧. 𝑧

La clave
Ley de Tricotomía en ℝ

La ley de la Tricotomía en ℝ permite comparar dos números reales y afirma que


para todo par de números reales 𝑎 y 𝑏: 𝑎 < 𝑏 ó 𝑎 = 𝑏 o 𝑎 > 𝑏.

Los números positivos están a la derecha del 0 y los números negativos están a
la izquierda del0.

54
Para comparar dos números racionales, se puede aplicar la siguiente propiedad:

Propiedad
Sean 𝑎, 𝑏, 𝑐, 𝑑 ∈ ℚ, 𝑏, 𝑑 > 0 𝑥 < 𝑦.
𝑎 𝑐
< 𝑑 si y solo sí 𝑎 ⋅ 𝑏 < 𝑏 ⋅ 𝑐.
𝑏

Para comparar dos números racionales e irracionales se puede obtener la expansión


decimal aproximada por medio de la calculadora y luego se aplica la siguiente
propiedad:

Propiedad
Sean 𝑥, 𝑦 ∈ ℝ. 𝑥 < 𝑦 si y solo si 𝑥
está a la izquierda del punto 𝑦 en
la recta numérica.

Ejemplos

1. Coloque los símbolos > ó < ó = según sea el caso.

5 6 1 −2
a) _____ b) _____
3 5 −8 9
Solución Solución
5 6 −1 −2
25 > 18 ⇒ 3 > 5 . −9 > −18 ⇒ > .
8 9

c) −4√18 _____ −
10
d) 16, ̅̅
26̅̅_____ 16,26
𝑒
Solución
10 Solución
−4√18 ≈ −16,97 < − ≈ −3,67 16, ̅26
̅̅̅ > 16,26.
𝑒

e) 2π + √6_____ π + √9
2 2

Solución
2 2
2π + √6 ≈ 8,73 > π + √9 ≈ 6,14.

55
2. Escriba F si la opción es falsa y V si la opción es verdadera dentro del paréntesis.
Justifique su respuesta

1. ( ) 5 ≥ 3.

Solución
Verdadera.

2. ( ) 5 < 𝑥 < 7 ⇒ 𝑥 > 5 o 𝑥 < 7.

Solución
Verdadera.

3. ( ) 𝑥 > 5 ⇒ 5 < 𝑥.

Solución
Verdadera.

4. ( ) 𝑥 ≤ 6 ⇒ 6 ≤ 𝑥.

Solución
Falsa. 𝑥 ≤ 6 ⇒ 6 ≥ 𝑥.
.

56
Ejercicios

1. Complete utilizando los símbolos  ,  para cada uno de los


siguientes pares de números Reales

2. Ordena de menor a mayor y representa en la recta real los siguientes números:

9 23 0,0256
a) –0,75 b) c) 3 d) e) 
4 20 2

3.) Compara y escribe >, < o = según corresponda.

57
4.) Compara y escribe >, < o = según corresponda.

5) Ordena de forma ascendente los siguientes números

Valor absoluto de un número real


Si 𝑥 ∈ ℝ se define el valor absoluto de 𝑥 por

|𝑥| = { 𝑥 si 𝑥 ≥ 0 .
−𝑥 si 𝑥 < 0

Propiedades
Si 𝑥 ∈ ℝ entonces
a) |𝑥| ≥ 0
b) |𝑥| = |−𝑥|.
c) |𝑥| = 0 si y solo si 𝑥 = 0
d) |𝑥 ⋅ 𝑦| = |𝑥| ⋅ |𝑦|.
𝑥 |𝑥|
e) |𝑦| = |𝑦| si 𝑦 ≠ 0.
f) |𝑥| = √𝑥 𝑛 si 𝑛 es un número
𝑛

par.
g) |𝑥 + 𝑦| ≤ |𝑥| + |𝑦|.

Ejemplos

1. Determine a) |−6| b) |±0| Solución


Solución |−6| = 6. |±0| = 0.
1
c) − |− |3 4|| Solución d) |1 − √3| Solución
1 − √3 < 0. Luego |1 − √3| = √3 − 1.
1 −13
− |− |3 || =
4 4
e) |−2 − √17| Solución
|−2 − √17| = 2 + √17.

58
2. Determine el valor exacto de
6 3
𝑟 = − |− | − |𝑒 −1 | + | √−8| .
7
Luego, localice el número en la recta numérica e indique si el número es racional
o irracional.
Solución
6 3 −6 1
− |− | − |𝑒 −1 | + | √−8| = + + 2.
7 7 𝑒
8 1 8𝑒 + 7
= + = ≈ 1,51 ∈ 𝕀 .
7 𝑒 7𝑒

3 .Calcule

2e - 3
57 , 4- 3
100 , 34 - 27 , e- .

Solución. Encuentre la expansión decimal de la cantidad que está dentro del


valor absoluto y luego use la definición
2e - 3
57  2e - 3
57 puesto que 2e - 3
57  1, 588  0

4- 3
100  4 + 3
100 ya que 4 3
100  0, 6415  0

34 - 27  34 - 27 pues 34 - 27  0

e -  e   se comprueba que e    0

Ejercicios
1. Calcule

a) 15 - 13 b) 12 - 17 c) 3- 13
d) 10 -  e) 15 + 13 f)  15  13

59
2. Determine el valor absoluto de

1.  -5+ 3  = 6.  -6 + 2  =

2.  -6 - 2  +  -5 - 3 -  - 13 = 7. - 4 + 16  +  -3 - 11 + 6 =

3. 3 2 - 1  -  5 = 8. 4 5 + 7 6 =


4. 5 - 6 +  7 - 8 =
  
3 5 4

9.
4 2 5

      
1 2 1 2 4 3
5. 3 5 4 10. 1 3

4 3 6 3 5 4

Situación Problema

De acuerdo a la recta numérica adjunta, ¿Cuál es un posible valor de a ?

( ) √10 ( ) √101 ( ) √95 ( ) √80

De acuerdo a la recta numérica adjunta, ¿Cuál es un posible valor de a ?

3
( ) 3
10 ( ) 3
6 ( ) 3 ( ) 3
29
2

60
Análisis de la actividad
De acuerdo a la recta numérica adjunta, ¿Cuál es un posible valor de a ?

( ) √10 ( ) √101 ( x ) √95 ( ) √80

De acuerdo a la recta numérica adjunta, ¿Cuál es un posible valor de a ?

3
( x ) 3
10 ( ) 3
6 ( ) 3 ( ) 3
29
2

La Clave

La raíz n-ésima, n  , n par, de un número racional positivo a es el


único número positivo que elevado a la n es igual a a .

n
a  b  bn  a

La raíz n-ésima, n  , n impar, de un número racional a es el único número que


elevado a la n es igual a a .

n
a  b  bn  a

Ejemplos

1. Determine la solución de la ecuación x 2  5 .

Solución

Si x 2  5 , entonces x   5 o bien x≈2,236068 o x≈-2,236068.

61
2. Determine la solución de la ecuación x 2  17 .

Solución

Si, x 2  17 entonces x   17 o bien x≈4,1231056 o


Todo número, positivo o
x≈-4,1231056. negativo, elevado a una
potencia par es positivo.
3. Determine la solución de la ecuación x 3  5 .
Todo número negativo
Solución
elevado a una potencia
impar es negativo.
 5
3
4. Si x 3  5 , entonces x  3 5 , puesto que 3
5 .
Todo número positivo
elevado a una potencia
Determine el conjunto solución de la ecuación x 4  7 .
impar es positivo.
Solución

 7 
4
5. Si x 4  7 , entonces x   4 7 , puesto que 4
7.

Por lo tanto el conjunto solución de la ecuación es:

 4

7,  4 7 .

6. Determine la solución de la ecuación x15  5 .

Solución

Si x15  5 , entonces x  5 .
15

Ejercicios

1) Determine el conjunto solución de las siguientes ecuaciones:


a) x 2  0, 0036 b) x 3  0, 001 c) x3  0, 001
d) x 3  7 e) x 4  64 f) x 4  17

2) Busque dos objetos cilíndricos (tarros) de diferentes bases. A continuación,


mida longitud de la circunferencia de la base y el diámetro de cada uno de ellos.
Tome las medidas lo más precisas posibles y calcule una aproximación al valor de
 en la misma manera que lo hicieron los sumerios. ¿Cuántos decimales exactos
halló para  ?

3) Se construye un jardín circular cuyo radio diámetro mide 16 metros y se desea


hacer una cerca con una malla que vale 5000 colones el metro.

62
a) Exprese el largo de la malla en términos de  .
b) ¿Cuál es el largo de la malla si se considera   3,14 ?
c) ¿Cuántos colones costará la malla si aproxima  por exceso a un número con
tres decimales?
d) ¿Qué diferencia en colones tendrá que pagar usted si el dueño del almacén usa
una aproximación de  con seis decimales?

4. Determine un valor para x en las siguientes ecuaciones. (Use la calculadora)

5. Ubique un número irracional con representación radical entre cada par de


números consecutivos en la siguiente recta numérica.

Operaciones con radicales

Operaciones

Para introducir una expresión radical en la calculadora el elemento más importante


es el índice, pues éste nos determina cuál botón se utiliza. Dependiendo de la
calculadora, los botones se pueden utilizar con raíces en su primera o segunda
función. Algunas tienen hasta una tercera función. El color de la función determina
cuál es el procedimiento para utilizarla:

63
 Si aparece en color blanco sobre el botón, se trata de la función principal,
por lo que simplemente éste se presiona y se utiliza con normalidad.
 Si la función se ve en color amarillo, es la segunda función, por lo que es
necesario oprimir primero SHIFT (ubicado arriba a la izquierda) y luego el
botón ubicado debajo de dicha función.
 Si la función se presenta en color rojo, se trata de la tercera función, por lo
cual es necesario oprimir primero ALPHA (ubicado a la derecha de SHIFT) y
luego el botón ubicado debajo de dicha función.

En la imagen adjunta, se aprecia la calculadora


más reciente. En ésta, la raíz cuadrada (de
índice2) es una función principal. En ese
mismo botón, la raíz cúbica (de índice3) es la
segunda función.
A su derecha, se ubica el botón para elevar al
cuadrado o al cubo (exponente 2 y 3
respectivamente). A la derecha de este botón
se ubica el que permite elevar a cualquier
potencia como función principal y radicar con
cualquier otro índice como segunda función.
De esta manera, se puede comprobar en la
calculadora el resultado de operaciones como
las siguientes:

1. √27 ∙ √8 = 6√2
3

3
2. √250 + √16 = 7√2
3 3

3. √243 − √48 = √3
4 4 4

3 √6
4. √ =
2 2
5. √27 ∙ √81 = 3 √3
4 3 12

2 2
6. (5√2 − 4) − 2 ∙ (3 − √2) = 44 −
28√2

O también se pueden resolver problemas,


como por ejemplo, determinar el valor de 𝑥 tal que 𝑥 5 = 30. Como la radicación
corresponde a la operación inversa de la potenciación, se tendría que 𝑥 = √30 =
5

1.0551130635362276 …

64
Ejemplos

1. Determine el perímetro de la siguiente figura

El resultado de la operación es de 17,4717 aproximadamente


Por lo tanto, el área del polígono es de aproximadamente 17,4717

2. Determine el área del siguiente rectángulo

El perímetro es 2( 7  5)  2  7 93  19,11

Nota: Hay infinidad de raíces que tienen solución en IR. Las


únicas raíces que no tienen solución son las que tienen de índice un número par y
subradical negativo.

Ejercicio: Halle la solución de los siguientes radicales e indique cuáles


no tienen solución.

a) 3
64 = __________

b) 25 = __________

c) 4
1296 = __________

d) 7
2187 = __________

e) 5
7776 = __________

f) 8
256 = ___________

65
Simplificación de Radicales:

Ejemplos: Simplifique al máximo los siguientes radicales.

a) 360 =
Solución:

1) Se factoriza el subradical:
360 2
180 2
90 2
45 5
9 3
3 3

En este ejemplo la raíz es cuadrada, es decir, que el índice es 2. Por consiguiente


con los factores se forman potencias elevadas a la 2 para que puedan “salir” de la
raíz. Al salir de la raíz el exponente se cancela porque se divide entre el índice.

360 = 22  2  5  32 =

23 25 =

6 10

b) 5
1944 =

Solución:

1944 2
972 2 5
1944 = 5
2  2  2  35 = (En este caso las
486 2 potencias deben
243 3 3 5
222 =
ser de
81 3
exponente 5 o
27 3 3 5
8
9 3 múltiplo de 5)
3 3
1

66
Ejercicios Utilice la calculadora para calcular las siguiente
operaciones

1) – 49 = 36
9) = 10) 36 =
81

11) 81 =
2) 121 =

3) 81 =

320 3
12) 3 = 13) 320 =
4) 64 = 343

5) – 64 = 14) 3
343 =

3
6) 103 =

15) – 200 =
169
7) 4
π4 =

8) – 252 =

67
2. Simplifique al máximo los siguientes radicales.

a) 6480 = c) 2 63 =
3 100

4
b) 4 3
5000 = d) 80 0,0001 =

3. Resuelva las siguientes operaciones con radicales:

a) 2 5 +7 5 = o) 4 3
54 – 3
250 + 5 3
128 =

b) 5 3 –9 3 = 1 3 1
p) 3
686 – 3
2 + 3
250 =
2 4 8

c) 4 10 + 2 10 – 5 10 = 3 – 2 27 =
q) 108 –
4 3 16

f) 4 3 –8 2 –7 2 =
r) – 2 + 1 18 – 2 =
9 3 25 49
g) 2 3 4 – 5 3
3 +3 3
4 =
j) 3 8 + 2 =
k) 6 27 – 5 48 =

68
4. Hallen el perímetro de cada una de las siguientes figuras

5.Utilice la calculadora para calcular las siguientes expresiones:

15 5
216 3 3 2
e)  f)  g)  h) 
5 3 5
108 1 3 3 2

2 3 1 5 1 1
i)  j)  k)  l) 
2 3 1 2 10  2 3 2

12.( 3  5) 20 2. 15 1 38. 2
m)  n)  p)  q)  r) 
3 5 20  20 5 3 3
3 19
35

32
s) 
12
235

6. Resuelvan cada uno de los siguientes cálculos combinados


1
1 2 3 4 3 1 1 5 
a) . 2    b)    c)   

3 1 2 2 1 5 7 5 7  4 
2
 2  2 2 8 8
d)   
 e)  f) 3 
 2  3  1 2 8 1

1

  1 
1
3 3
g) 6. 12 : 18 
4 2
h) 5. 5 :  .5 25 
3

5 

69
7. Halle el valor exacto del área de las siguientes figuras

8. Todas las figuras tiene áreas 1 Hallen las incógnitas indicadas con x .
Expresen todos los resultados sin radicales en el denominador

70
9. Efectúa las siguientes operaciones:

 2  32 - 50  18   3  108 - 27  48 
  5 - 
3 6 5 6 3  
 6 6 -6 3  6  3 

 4 6 - 4 3  6  3    180  162   20  18 
 25 9  1
 8     32  3 
 2 8  
 2 

10. Efectúa las siguientes operaciones:

 2 3 16  3 3 54 - 3
128  3
250  2 4 4375  4
9072 - 3 4 567  4
112

 5 4 176  3 4 891  6 4 6875  2 4 14256

 2 3 1029 - 5 3 192  3 3 648

Tema 3 Cantidades Grandes o


pequeña

Situación Problema

El sueño de un costarricense por


buscar una forma de viajar a Marte más
rápidamente está más cerca que nunca. El 26
de noviembre del 2011 a las 10:25 am, la
NASA envió a dicho planeta el Rover Curiosity
de la misión Mars Science Laboratory de la
NASA y después de aproximadamente 563
millones de kilómetros de recorrido liberó un
vehículo explorador que tocó suelo marciano el
6 de agosto del año 2012. Su propósito:
investigar si alguna vez este planeta ha tenido
condiciones favorables para el desarrollo de la
vida2 . El científico costarricense Franklin
Chang Díaz viene desarrollando con su equipo
de trabajo desde hace varios años un motor de
plasma que reduciría el tiempo de viaje a 39

71
días aproximadamente. Esto puede contribuir a que se puedan explorar planetas
distantes en un menor tiempo. Con base en la información anterior,
¿aproximadamente en cuántos metros por segundo permite el motor desarrollado
por el doctor Chang aumentar la velocidad para futuros viajes espaciales tomando
como referencia la desarrollada por el motor de la nave espacial Mars Science
Laboratory?

Análisis de la actividad

El propósito de este problema es el de introducir el uso de unidades de mayor


rango, que permitan expresar cantidades extremadamente “grandes”, como en
este caso la distancia entre dos puntos diferentes en el universo. De paso, se
podrían implementar actividades con cantidades sumamente pequeñas que
permitan también justificar el empleo de unidades inferiores al milímetro,
relacionadas al tamaño de las células, el radio de un nanotubo de carbono, etc. En
esta oportunidad, dado que los estudiantes trabajarán con cantidades muy grandes
y que la elaboración de una estrategia que permita dar solución al problema debe
ser lo primordial, el docente puede permitir el uso de una calculadora para facilitar
las labores de cálculo y así hacer un uso inteligente de la tecnología. Se pueden
conformar parejas de estudiantes para trabajar dicho problema, pues la idea es
brindar oportunidad al estudiante de intercambiar ideas con su compañero y
argumentar con propiedad su posición respecto a ellas. Esto permite activar los
procesos Comunicar y Razonar y argumentar. Para establecer dicha diferencia, los
estudiantes deben obtener las velocidades que se
desarrollarían con cada uno de los motores. Para el caso
del motor que llevó al Curiosity a Marte, primero hay que
notar que las unidades en las que se pide brindar la
respuesta no corresponden a las mencionadas en los datos
formulados en el problema. Es así como la acción
estudiantil debe ir orientada a realizar las conversiones
necesarias para expresar las unidades en la forma
requerida. Como la nave comenzó su viaje el 26 de
noviembre del 2011 y finalizó el 6 de agosto del 2012, los
estudiantes necesitan considerar primero la cantidad de
días que duró la nave en llegar a Marte. Para una mejor
representación de los procedimientos, se puede usar una
representación tabular como la siguiente: M

72
Luego, se procede a realizar la conversión del tiempo empleado a segundos,
pasando primero por las conversiones a horas y minutos. Dado que un día tiene 24
horas: 254 × 24 = 6 096 horas. Dado que una hora tiene 60 minutos: 6 096 × 60
= 365 760 minutos. Dado que un minuto tiene 60 segundos: 365 760 × 60 = 21
945 600 segundos. De ese modo, con el motor convencional se duraron
aproximadamente 21 945 600 s. Como se recorrieron 563 000 000 km, se realiza
la conversión a metros. Cada kilómetro tiene 1000 metros, con lo que se obtiene
583 000 000 × 1000 = 563 000 000 000 m. Finalmente, la velocidad aproximada
a la que viajó dicha nave se consigue al dividir la distancia por el tiempo, con lo
que 563 000 000 000 ÷ 21 945 600 ≈ 25 654,35 m / s. Ahora, para el motor que
desarrolla el doctor Franklin Chang, se efectúa igualmente la conversión a las
unidades correspondientes. En el caso del tiempo, se realiza la conversión de días
a segundos: 39 × 24 × 60 × 60 = 3 369 600 s. Considerando la conversión
realizada anteriormente para el caso de la distancia entre los planetas, se obtiene
finalmente que la velocidad que desarrollaría el motor del doctor Chang sería
aproximadamente de 563 000 000 000 ÷ 3 369 600 ≈ 167 082,15 m / s. Así, el
motor que implementará el doctor Chang permitiría aumentar la velocidad en 167
082,15 – 25 654,35 = 146 427,8 m / s aproximadamente. Esta actividad puede
complementarse con un problema que trabaje con cantidades muy pequeñas y así
durante las etapas de discusión y comunicación de resultados se pueden establecer
comparaciones y valorar la necesidad de incluir en nuestro sistema de medidas
otras que permitan facilitar la forma de expresar las cantidades que son muy
grandes o muy pequeñas, reflejando otros contextos que necesitan ser conocidos
por el estudiante.

La Clave
Cantidades muy grandes o muy pequeñas
En este apartado se trata de resolver problemas con unidades de medida y sus
múltiplos y submúltiplos. En la escuela se estudiaron tres múltiplos (kilo, hecto y
deca) y tres submúltiplos (deci, centi y mili). Sin embargo, la lista es mucho más
extensa. Para los múltiplos se tienen las siguientes equivalencias:

73
Prefijo Símbolo Equivalencia con la unidad Potencia
24
yotta 𝑌 1 000 000 000 000 000 000 000 000 10
zetta 𝑍 1 000 000 000 000 000 000 000 1021
exa 𝑅 1 000 000 000 000 000 000 1018
peta 𝑃 1 000 000 000 000 000 1015
tera 𝑇 1 000 000 000 000 1012
giga 𝐺 1 000 000 000 109
mega 𝑀 1 000 000 106
kilo 𝑘 1 000 103
hecto ℎ 100 102
deca 𝑑𝑎 10 101
Para los submúltiplos se tienen las siguientes equivalencias:

Prefijo Símbolo Equivalencia con la unidad Potencia


−1
deci 𝑑 0,1 10
centi 𝑐 0,01 10−2
mili 𝑚 0, 001 10−3
micro 𝜇 0, 000 001 10−6
nano 𝑛 0,000 000 001 10−9
pico 𝑝 0,000 000 000 001 10−12
femto 𝑓 0,000 000 000 000 001 10−15
atto 𝑎 0,000 000 000 000 000 001 10−18
zepto 𝑧 0,000 000 000 000 000 000 001 10−21
yocto 𝑦 0,000 000 000 000 000 000 000 001 10−24

Se debe recordar que al subir en cada tabla se debe dividir la cantidad dada por la
equivalencia y al bajar se multiplica la cantidad dada por la equivalencia.

Ejemplo
1 En un laboratorio de biología se está analizando un tipo de célula sanguínea
que presenta un diámetro de 1 000 000 000 00, a cuantos Gm equivale dicha
cantidad.

74
Primero: Se determina la posición de los prefijos, para determinar el valor que se
utiliza, se cuentan los espacios que hay entre los prefijos a utilizar:

2. Si se convierte de unidades pequeñas a unidades grandes se divide el


número entre el valor obtenido, como sigue a continuación:

3. Se puede resolver el siguiente problema:

Los ácaros son causantes de algunas enfermedades como alergias,


sarna y demás. Un ácaro tiene una longitud de 28 𝜇𝑚. Si se hace una
fila con ácaros, ¿cuántos se requieren para abarcar 10 𝑐𝑚?

Para determinar la respuesta, es necesario tener las dos medidas en la misma


unidad. La forma más fácil de hacerlo sería convertir ambas medidas a metros, que
es la unidad de medida con la que se está trabajando. Así, se tiene que
28 𝜇𝑚 = 28 ∙ 0,000 001 𝑚 = 0,000 028 𝑚
10 𝑐𝑚 = 10 ∙ 0,01 𝑚 = 0,1 𝑚

75
Luego, como la medida de la longitud de un ácaro es menor que la de la fila, se
divide la medida de la fila por la longitud de cada ácaro:
0,1
= 3571,42857
0,000 028

Por lo tanto, para cubrir los 10 𝑐𝑚 de la fila con ácaros, se requiere tener 3572 de
ellos
4.Mi servisio de correo electronico gratuito me ofrece una capacidad de
almacenamiento de 10 Gb , sin un disco compacto CD tiene una capacidad de
700MB ¿ cuantos discos compactos equivalesn a al capacidad de almacemamiento
de mi correo?

Un Gigabyte equivale a 230  1073741842 bytes


Un Megabyte a 220  1048576 bytes
10 Gigabyte equivale a 10  230 bytes
700Megabytes seria 10  2 20 bytes
Por lo que dividimos la capacidad del correo por la capacidad de cada CD
(10  230 )   700  220   14 (10  230 )   700  220   14 CD´S Aproximadamente

5.Un Cabello tiene 80000 nanometros de diametro ¿ Cual es la medida en metros?


Un Nanometro tiene 10-9 metros . Entonces el diametro de un cabello es
80000 109  0,00008 mts

Ejercicios

A. Realice las siguientes conversiones de unidades

1) Exprese 25 000 000 km a m

2) Convierta230000000000000000000 pm a m

3) Convierta 0,00235 Das a s

4) Convierta 900000000 g a Mg

5) Convierta 4 m a nm

76
6) A cuantos km equivalen 82000000 m

7) Exprese 40000000 s a s

8) Convierta 5000 dm a m ?

9) Convierta 200 Hm a m

10) Convierta 300 Mg a mg

11) Convierta 20 Kg a g

12) Cuantos nm equivalen a 0,000000058 km

2. En la siguiente tabla se muestran las distancias medias al Sol de cada uno de los
planetas del sistema solar

a. Cuál es la distancia en metros entre la Tierra y el Sol?

b. .cuántos ceros consecutivos se ubican al final del número que representa la


distancia en metros entre la Tierra y el Sol?

c. "Justifique la validez de la igualdad 149 600 000 000 m = 1496. 108


m."

d. ¿cuál es el factor de conversión correspondiente a los mega metros (Mm)?

77
3 Realice las siguientes conversiones

4. Resuelva los siguientes problemas


a. Las bacterias son microorganismo que tienen un tamaño tan pequeño que se
mide en micrómetros ( por lo general 0,5 y 5  m de longitud ) Exprese esa
longitud en picometros

b. En un laboratorio en Suiza se coloca un nanotubo de carbono de 50


nanómetros de ancho para derretir el cobre ¿ Cuál es esa medida es micrómetros

c. Mi laptop tiene un disco duro de 500 Gb , si mis archivos aproximadamente


ocupa 1000Mb y mis programas ocupan 150 Gb ¿ cuánto espacio libre hay en mi
disco

d. Acabo de comprar una laptop con un disco duro de un Terabyte . ¿ Cuantos


gigas tiene de capacidad de almacenamiento

e. Si mi disco duro tiene una capacidad de 1,5T ¿ cuantas llaves de 32G caben
en ese disco duro

78
Práctica de la unida 1
Selección única

1) Considere los siguientes números:


I. 23,534

II. 100, 01111...


III. 7,10100100001...

¿Cuáles de ellos corresponden a números con expansión decimal infinito no


periódico?
A) Solo el I C) Solo el III
B) El I y el II D) El II y el III

2) El número 3 11 en notación decimal es aproximadamente:


A) 3,32 C) 9, 95
B) 12, 00 D) 33, 00

3) Considere las siguientes proposiciones:


I. 3
0 es un número racional.

II. 2 es un número racional.

¿Cuáles de ellas son verdaderas?


A) Ambas C) Ninguna
B) Solo la I D)Solo la II

4) Considere las siguientes proposiciones:


I. 10  1 es un número racional.

II. 2,337445 es un número irracional.


¿Cuáles de ellas son verdaderas?
A) Ambas C) Ninguna
B) Solo la I D)Solo la II

79
5) Considere las siguientes relaciones:
I. 3
3  3 es un número racional.

II. 3,121121112...  3 2 es un número irracional.


¿Cuáles de ellas son verdaderas?
A) Ambas C)Ninguna
B) Solo la I D) Solo la II

6) Considere las siguientes proposiciones:


I. 3
5 es un número no real.
II. 3 es un número real.

¿Cuáles de ellas son verdaderas?


A) Ambas C)Ninguna
B) Solo la I D) Solo la II

7) Considere la siguiente representación gráfica:

De acuerdo con la representación anterior, un posible valor de “ A ” es:


A) 3
4 C) 3
27

B) 3
50 D) 3
64

 2  2
4 0

8) El resultado de corresponde a:
22

A) 1 C) 3
3 7
B) D)
4 4
3
6
9) El resultado de 3
9 es:
6
3
2
A) B) 3
9
2

33 2
B) C) 2 3 2
2

80
10) El resultado de 9 32  2 27  2 8 es:

A) 9 2  5 3
B) 32 2  5 3
C) 32 2  6 3
D) 28 2  18 3
11) De acuerdo con el Sistema Internacional de Medidas, 1 nanómetro equivale a:
A) 100 000 000 metros.
B) 1 000 000 000 metros.
C) 0, 000 000 001 metros.
D) 0, 000 000 000 1 metros.
12) De acuerdo con el Sistema Internacional de Medidas, el número 3 000 000 000
equivale a:
A) 3 gigas.
B) 3 teras.
C) 3 nanos.
D) 3 megas.
13) Considere las siguientes proposiciones:
I. " 0 " pertenece al conjunto de los números reales.
II. 4 pertenece al conjunto de los números irracionales.

De ellas, ¿cuál o cuáles son verdaderas?


A) Ambas
B) Ninguna
C) Solo la I
D) Solo la II

81
14) Un número irracional corresponde a:
1
A) 2 2
1
B) 4 2

 2
2
C)

 4
2
D)

15) Considere las siguientes proposiciones:


I. 2 representa un número real.
1
II. 4 3 representa un número irracional.

De ellas, ¿cuál o cuáles son verdaderas?


A) Ambas
B) Ninguna
C) Solo la I
D) Solo la II

16) Considere las siguientes proposiciones:


I.  8

II. 3e

De ellas, ¿cuál o cuáles son verdaderas?


A) Ambas
B) Ninguna
C) Solo la I
D) Solo la II
17) ¿Cuál de los siguientes números es no real?
A)  2
B) 3

C) 3
4

D)  3 5

82
18) ¿Cuál de los siguientes números tiene expansión decimal infinita no periódica?
1
A)
7

3
B)
13

C) 5

D) 25

19) Si " x " representa números irracionales con la condición 2  x  3 , entonces, un


valor de " x " es:
A) 4

B) 8

C) 9

D) 10

20) El número 3
6 se ubica entre:

A) 0 y 1
B) 1 y 2
C) 2 y 3
D) 3 y 4
21) La expresión 5  289 es equivalente a:

A) 294

B) 17 5
C) 1445

D) 5  17

22) La expresión 17  529 es equivalente a:


A) 512

B) 546

C) 17  23

D) 17  23

83
23) En el siguiente cuadro se presentan algunos dispositivos de memoria o de
almacenamiento de información (unidad de medida byte " b ") y su respectiva
capacidad de almacenamiento:

Nombre del
Capacidad de almacenamiento de información
dispositivo

Disco
700 Mb
compacto

Disco duro
1 Tb
externo

Llave maya 16 Gb

Micro SD 8000 Mb

Con base en el contexto dado, considere las siguientes proposiciones:


I. Tiene más capacidad de almacenamiento la llave maya que el disco duro
externo.
II. La capacidad de almacenamiento de la llave maya es mayor que la capacidad
de almacenamiento de la micro SD.
De ellas, ¿cuál o cuáles son verdaderas?
A) Ambas
B) Ninguna
C) Solo la I
D) Solo la II
24) Dos megámetros equivalen a:
A) 2x103 metros
B) 2x106 metros
C) 2x109 metros
D) 2 x1012 metros

84
85
86
Nuestro primer
desafío matemático,

Capítulo II Geometría un paso más para


aprender

Habilidades Específicas

Al finalizar el capítulo el estudiante deberá estar en capacidad de:

1. Utilizar nociones básicas de geometría analítica.

2. Determina las medidas de lados e Hipotenusa de un triángulo rectángulo


3. Aplicar las razones trigonométricas básicas (seno, coseno, tangente) y las
relaciones entre ellas en diferentes contextos.
4. Determina grados y radianes de un Angulo
5. Visualizar y aplicar características y propiedades de figuras geométricas
tridimensionales

Conceptos clave

1.Distancia entre dos 4.Senos 7.Ley de Senos


puntos
5.Cosenos 8.Piramides
2.Pitágoras
6.Tangentes 9.Prismas
3.Trigonometría

87
 Introducción
En este ciclo el abordaje de los contenidos conviene realizarse en principio mediante
la experimentación, para pasar posteriormente a la abstracción
y la argumentación matemática. A través de la experimentación
el estudiante podrá realizar conjeturas cuya validez deberá
argumentar apropiadamente y en algunos casos hacer pruebas
usando las propiedades matemáticas (especialmente en el
último año del ciclo).
Con el propósito de reforzar la actitud de confianza en la utilidad
de las matemáticas es importante la contextualización de los
contenidos geométricos y las conexiones con otras disciplinas.
Al finalizar este ciclo el estudiante será capaz de abstraer propiedades geométricas,
realizar conjeturas y probar algunas propiedades relacionadas especialmente con
triángulos y cuadrilátero

Tema 1 Teorema de Pitágoras

Situación Problema
En el Libro I de los Elementos de Euclides se consigna el Teorema de
Pitágoras de la siguiente manera:
“En todo triángulo rectángulo, la suma de los cuadrados construidos
sobre los catetos es igual al cuadrado construido sobre la hipotenusa.

88
Sin embargo, se sabe que el conocido Teorema de Pitágoras se desarrolló en otras
culturas a parte de la griega. Por ejemplo, en China se desarrolló con el nombre
del teorema de Kouku. Este aparece demostrado en un texto muy antiguo llamado
ChouPei.
Realice los siguientes pasos para realizar la demostración del teorema de KouKu
de la cultura China. A este procedimiento se le llama
“apilamiento de rectángulos”.
1. Construya un rectángulo ABCD de dimensiones a y b.
2. Trace la diagonal del rectángulo AC .A esta se le asignará la dimensión c.
3. Sobre la di agonal del rectángulo construya un cuadrado de dimensión c.
4. Construya tres rectángulos que tengan como diagonales los lados del
cuadrado de lado c, de modo que se forme un cuadrado mayor de lado a
+ b.
El cuadrado sobre la diagonal c consta de cuatro triángulos congruentes con el
triángulo ABC y un cuadrado menor.
Exprese la relación que existe entre el área del cuadrado de lado c y los cuatro
triángulos congruentes y el cuadrado menor que están dentro del cuadrado de lado
c.

Análisis de la actividad
La siguiente secuencia de imágenes muestra las construcciones propuestas.

La longitud del cuadrado mayor es igual a a+b. La última figura muestra que la
longitud del lado del cuadrado pequeño es a – b. Puede encontrarse el área del

89
cuadrado formado por las diagonales de los rectángulos sumando las áreas de los
cuatro triángulos congruentes y el área del cuadrado pequeño. El área de los
1
triángulos es 𝑎𝑏. El área del cuadrado es (𝑎 − )2 . Así se tiene la siguiente
2
relación:
Desarrollando la igualdad se tiene que:
c 2  2ab   a 2  2ab  b 2 
c 2  a 2  b2

La Clave
Teorema de Pitágoras
En un triángulo rectángulo se diferencia el nombre de sus lados de
acuerdo con su posición con respecto al ángulo recto. De esta manera, en el
triángulo adjunto se tiene:

Figura Nombres de los lados


Cateto (uno de los dos lados que
𝑎
forman el ángulo recto).
𝑐
𝑎 Cateto (uno de los dos lados que
𝑏
forman el ángulo recto).
Hipotenusa (lado que se ubica
𝑏 𝑐
frente al ángulo recto).

De acuerdo con el teorema de Pitágoras, la suma de los cuadrados de los catetos


de un triángulo rectángulo es igual al cuadrado de la medida de la hipotenusa. Con
la figura anterior, el teorema dice que

𝑎2 + 𝑏 2 = 𝑐 2
Para efecto de la resolución de problemas y ejercicios relacionados con el teorema
de Pitágoras es importante tener claro lo siguiente.

Características
a) Tiene un ángulo recto
b) Los lados que forman el ángulo de 90º se llaman catetos.
c) El lado más largo y opuesto al ángulo recto se llama hipotenusa.

90
Teorema de Pitágoras y las relaciones que se establecen entre sus lados
Representación geométrica Representación algebraica

Ejemplos

4
1. Determine la medida de la hipotenusa de un triángulo rectángulo si sabe
que sus catetos miden 5cm y 12cm.
Solución
Si x es la medida de la hipotenusa entonces 52  122  x 2
Luego x 2  25  144  169  x   169  x  13 .
Pero, recuerde que x representa la medida de un segmento por lo tanto es
positiva, así x  13 .
La hipotenusa mide 13cm.

2. Calcule la medida de uno de los catetos de un triángulo rectángulo si sabe


que el otro mide 30cm y la hipotenusa 34cm.
Solución
Si b es la medida del cateto usando el teorema de Pitágoras se puede afirmar
que 302  b 2  34 2 , es decir, 900  b 2  1156 Luego
b  1156  900  b  256  b   256 .
2 2

91
Recuerde que b representa la medida de un segmento por lo tanto es positiva,
así b  256  b  16 .

3. Si la hipotenusa de un triángulo rectángulo mide 16dm y uno de sus


catetos mide 8dm, calcule la medida del otro cateto.
Solución
Si a es la medida del cateto desconocido entonces
a  8  16  a  64  256  a  256  64  a  192
2 2 2 2 2 2

Luego a   192 por lo tanto a  192  26  3  23 3  8 3 .


Pues a debe ser positivo.
Así el otro cateto mide 8 3dm .

4. ¿Si una persona camina 7km hacia el norte, después 3km hacia el oeste y
luego 3km hacia el sur entonces a qué distancia está del punto de partida?
Solución
Es más sencillo resolver este problema si se hace un dibujo que ilustre la
situación
Observe que se formó un triángulo rectángulo de catetos 3km y 4km use
el teorema de Pitágoras d 2  32  42  d 2  9  16  25
luego d 2  25  d  25  d  5

Respuesta
La persona se encuentra a 5km de distancia del punto de
partida

5. En cada caso indique si el triángulo es rectángulo

a) Los lados del triángulo miden 5,12 y 13.

Solución
Se usa el recíproco del teorema de Pitágoras, la medida del lado mayor es 13
entonces se debe verificar si 132  12 2  52
Efectivamente: 132  122  52  169  144  25 por lo tanto el triángulo es
rectángulo

b) Sea el ABC cuyos lados miden 4, 5 y 6.


Solución
De nuevo se usa el recíproco del teorema de Pitágoras, la medida del lado
mayor es 6 entonces se debe verificar si 62  42  52 sin embargo se puede
observar que

92
62  42  52 puesto que 36  16  25 por lo tanto el triángulo no es rectángulo

6. Determine la medida de cada lado faltante en cada figura

a) 𝐵

𝑐 3

𝐴 𝐶
4

Solución

𝑐 = √32 + 42 = 5.

En general, si se conocen las medidas de los catetos 𝑎 y 𝑏 entonces la medida


de la hipotenusa es √𝑎2 + 𝑏 2.

b) 𝐵

8
𝑎

𝐴 𝐶
4

Solución

𝑎 = √82 − 42 = √48 = 4√3.

En general, si se conocen: la medida de un cateto 𝑎 y la medida de la


hipotenusa 𝑐 entonces la medida del otro cateto es √ℎ2 − 𝑎2 .

93
7. Una escalera de 3 m de longitud se coloca contra la pared para alcanzar
una ventana. Si el pie de la escalera está a 1 m de la base de la pared,
¿a qué altura aproximadamente se encuentra la ventana?
Ya que la escalera está contra la pared, se forma un
triángulo rectángulo que tiene como hipotenusa
la escalera, y como catetos la distancia del pie de la
escalera a la base de la pared y la altura de la pared
desde el piso hasta donde la escalera toca la pared.
Así, en el triángulo rectángulo la hipotenusa mide
3 m y uno de los catetos mide 1 m. Entonces, se
toma la expresión que indica la relación pitagórica y se
remplazan los valores correspondientes a la hipotenusa y
el cateto.

Ejercicios

Hallar la medida del lado que falta en cada uno de los siguientes triángulos
rectángulos:
a) c)

9cm 12cm 25cm x

x 20cm

40cm 9cm 15cm


b) d) 17cm

x x

94
e) f)
x

7cm 26cm 24cm


25cm

2)Utilice el teorema de Pitágoras para hallar la medida del lado que se le pide en
cada una de las siguientes figuras:

a) e)

x 14 2
4cm x

6cm

b) f)

15cm 15cm
x

8 2 cm

10cm
g) 7cm

c)
x 9cm
y
9cm x

12cm

95
h) 9cm

d) 12cm
x y 15cm
6cm

20cm
x

3. El triángulo de la figura adjunta es rectángulo isósceles.

a) Si a = 3 cm entonces c = _______ cm.


a c
b) Si c = 2 2 entonces a = _______ cm.
a
c) Si c = 6 cm entonces a = ________ cm.

d) Si a = 5 cm entonces el perímetro mide ______ cm.

e) Si c = 8 cm entonces el área es _________ cm2 .

4. Conteste las preguntas del cuadro


a) Determine la medida de la hipotenusa de un triángulo rectángulo si sabe
que sus catetos miden 8 cm y 12cm.
b) Calcule la medida de uno de los catetos de un triángulo rectángulo si sabe
que el otro mide 24cm y la hipotenusa 74cm.
c) Encuentre la medida de la hipotenusa de un triángulo rectángulo isósceles
cuyos catetos miden 15cm.
d) Encuentre la medida de los catetos de un triángulo rectángulo que están
en la relación 3 : 4 y cuya hipotenusa mide 15cm.
e) ¿A qué distancia se encuentra una persona de su punto de partida si
primero camina 5km al norte y luego 3km al este?
f) En cada caso indique si el triángulo es rectángulo
a) Los lados del triángulo miden 10, 14 y 15.
b) Sea el ABC cuyos lados miden 24, 25 y 7.
c) Las medidas de los lados del triángulo son: 3, 6 y 3 2 .

96
5. Determine el valor de los segmentos señalados con las variables "x" y " y"

97
6. Determina la medida del lado indicado en cada una de los siguientes triángulos
rectángulos.

7. Determina la altura (h) de cada una de las siguientes figuras.

8. Determina la medida de la diagonal (d) en cada una de las siguientes figuras.

98
9. Determina la medida de la pantalla de un televisor si su altura mide 14,2 cm y
su ancho miden 15,4 cm.

10. Halla la medida de los lados indicados en cada una de las siguientes
figuras.

11. Un automóvil recorre 15km hacia el norte, dobla hacia la derecha en ángulo
recto y continúa 5km más. Posteriormente dobla hacia el norte y recorre otros
10km, terminando con 14km hacia la izquierda en ángulo recto. ¿A qué distancia
se encuentra del punto original? ¿Cuánto camino recorrió?

12. Una persona camina 4km hacia el norte y 3km al oeste. Luego cambia hacia el
norte y camina 8km, por ´ultimo camina 6km más hacia el oeste. ¿A qué distancia
se encuentra del origen? ¿Cuánto camino recorrió esa persona?

13. El hueco de una ventana mide 41 pulgadas de ancho y 26 pulgadas de altura.


¿Puede introducirse por la ventana un mesa de ping-pong de 48 pulgadas de
ancho?

99
14. Una puerta mide 210cm de altura por 80cm de ancho. ¿Cuál es el ancho mayor
que puede tener un tablero para que quepa por esta puerta?

15. Una escalera de 4.5 metros se coloca contra una pared con la base de la
escalera a 2 metros de la pared. ¿A qué altura del suelo está la parte más alta de
la escalera?

16. Una escalera de 6 metros se apoya contra una pared, quedando la parte
superior de la misma a una altura de 5.4metros. ¿A qué distancia está el pie de la
escalera de la base de la pared?

17. Una escalera telescópica de 36 metros se apoya sobre un edificio en llamas.


La base de la escalera Está a 10 metros del edificio. ¿Qué altura alcanza la
escalera?

18. Las diagonales de un rombo miden 16cm y 10 cm respectivamente. ¿Cuánto


mide cada uno de los lados? Calcule el área del rombo

19.Saber utilizar el teorema de Pitágoras para calcular el cateto o la hipotenusa


de un triángulo rectángulo en el que conocemos dos de sus lados

100
20. Saber determinar triángulos rectángulos en distintas figuras del plano para
calcular, a través de Pitágoras, ciertas medidas desconocidas, asociadas a las
figuras.

21.Calcula el cuadrado de los tres lados de estos triángulos y comprueba en cuál


de ellos de cumple el teorema de Pitágoras.

22. Una escalera de 65 decímetros se apoya en una pared vertical de modo que
el pie de la escalera está a 25 decímetros de la pared. ¿Qué altura, en decímetros
alcanza la escalera

101
23. La cara frontal de una tienda de campaña es un triángulo isósceles cuya base mide
1,6 metros y cada uno de los lados iguales mide 170 centímetros. Calcula la altura en
centímetros de esa tienda de campaña.

24 .En un triángulo isósceles y rectángulo, los catetos miden 25 milímetros cada


uno, ¿Cuál es la medida de su hipotenusa?

25. La altura de una portería de fútbol reglamentaria es de 2,4 metros y la distancia


desde el punto de penalti hasta la raya de gol es de 10,8 metros. ¿Qué distancia
recorre un balón que se lanza desde el punto de penalti y se estrella en el punto
central del larguero?

102
103
104
Tema 2 Distancia Entre Dos Puntos

Situación Problema
Adrián y Fabián salen del colegio para su casa. Adrián camina
3km hacia el Este y 2km hacia el Norte y Fabián camina 1km al Oeste
y 5km al Norte. ¿A qué distancia se encuentra la casa de Adrián de la de Fabián?

Análisis de la Actividad

Si representamos gráficamente la situación en el plano cartesiano siendo el colegio


el punto de origen o referencia (0,0) entonces la casa de Fabián está ubicada en el
punto B (- 1,5) y la casa de Adrián está ubicada en el punto C(3,2).Luego, al estar
estos puntos ubicados en el sistema de coordenadas se puede encontrar un punto
tal que se forme un triángulo rectángulo en el cual la distancia entre las casas sea
la hipotenusa y el punto encontrado sea el vértice del ángulo recto. Ese punto
puede ser (3,5) (abscisa del punto C y ordenada del punto B) o también podría ser
(-1,2) (abscisa del punto B y ordenada del punto C). Para este caso utilizaremos el
punto (3,5) y le llamaremos A.

Al visualizar la imagen se puede notar que las medidas de los catetos que se forman
son 3 y 4 por lo que entonces la distancia entre las casas de Fabián y Adrián será
5. Como cada unidad equivale a un kilómetro esto quiere decir que la distancia
entre las casas es de 5km. Sin embargo, se quiere utilizar este problema para
institucionalizar la fórmula de distancia entre dos puntos coordenados, por lo que
entonces analizaremos este problema mediante la distancia entre coordenadas.
Como se dijo, el punto B es la casa de Fabián y C la casa de Adrián, entonces la

105
distancia entre las abscisas es d ( A, B)   4  1   3  2   10 1  3 y la distancia
2 2

entre las ordenadas es 5  2 ; ahora utilizando el teorema de Pitágoras se tiene que:

d ( AB )   x1  x2    y1  y2 
2 2

La clave

Fórmula de la distancia entre dos puntos coordenados

Sean A y B dos puntos en el plano cartesiano cuyas coordenadas son


respectivamente  x1 , y1  y  x2 , y2  entonces la distancia entre A y B es igual a:

d ( AB )   x1  x2    y1  y2 
2 2

Justificación:
Buscando un punto C, tal que el ángulo ACB sea el ángulo recto del triángulo ABC,
entonces el punto C puede ser:  x1, y1  o  x2 , y2 
Nota Suponga que “a”, “b” y “c” son lados de un triángulo, y que “c” es el lado
mayor. Entonces se cumple lo siguiente:

1) Si c2 = a2 + b2 , entonces el triángulo es un triángulo rectángulo.


(teorema de Pitágoras).

2) Si c2 > a2 + b2 , entonces el triángulo es obtusángulo.

3) Si c2 < a2 + b2 , entonces el triángulo es acutángulo.

Ejemplos

1. En cada caso indique si el triángulo es rectángulo


a) Los lados del triángulo miden 5,12 y 13.
Solución
Se usa el recíproco del teorema de Pitágoras, la medida del lado mayor es 13
entonces se debe verificar si 132  12 2  52
Efectivamente: 132  122  52  169  144  25 por lo tanto el triángulo es rectángulo

b) Sea el ABC cuyos lados miden 4, 5 y 6.


Solución
De nuevo se usa el recíproco del teorema de Pitágoras, la medida del lado mayor es
6 entonces se debe verificar si 62  42  52 sin embargo se puede observar que
62  42  52 puesto que 36  16  25 por lo tanto el triángulo no es rectángulo

106
2 Observe el siguiente cuadro

Sea C (x,y) entonces el segmento AC con BC forman un ángulo recto En la


siguiente figura se muestra un caso particular donde A(-1,5), B(3,2) y C(3,5 Determine
los pares ordenados del triangulo

Sea C (x,y) entonces el segmento AC con BC forman un


ángulo recto En la siguiente figura se muestra un caso particular
donde A(-1,5), B(3,2) y C(3,5)

3)Observe el siguiente cuadro

Posibles Posible
Comprobación Conclusión
medidas hipotenusa
242 + 72 = 252 El triángulo sí es
24, 7, 25 25
625 = 625 rectángulo

42 + 62 = 72 El triángulo no es
7, 4, 6 7
52 ≠ 49 rectángulo

402 + 92 = 412 El triángulo sí es


40, 41, 9 41
1681 = 1681 rectángulo

152 + 202 = 262 El triángulo no es


15, 20, 26 26
625 ≠ 9 rectángulo

107
4)Dadas las siguientes coordenadas de los vértices de un triángulo A(2,1), B(6,5) y
C(9,2), clasifique el triángulo de acuerdo a la medida de sus
ángulos y la medida de sus lados. Argumente su respuesta

Aunque para la resolución del problema no es necesaria


la representación gráfica, un primer acercamiento al
problema podría ser ubicar los puntos en el plano
cartesiano y unirlos con segmentos para formar un
triángulo

Aparentemente el triángulo de la imagen se visualiza como rectángulo y escaleno,


sin embargo hay que argumentar esta percepción con fundamentos matemáticos.
Para esto, se utilizará la fórmula de la distancia entre dos puntos conociendo sus
coordenadas y el teorema de Pitágoras
Con la fórmula de la distancia entre dos puntos se obtiene que

d ( AB)   2  6   1  5  4 2
2 2

d ( BC )  9  6   2  5 3 2
2 2

d ( AC )   2  9   1  2  5 2
2 2

     
2 2 2
Por lo tanto, como 5 2  4 2 3 2se concluye que efectivamente el
triángulo es rectángulo y como las medidas de sus lados son diferentes se clasifica
como escaleno

108
5. Demostrar que los puntos: A(3, 8); B(-11, 3) y C(-8, -2) son vértices de un
triángulo isósceles.

d AB  ( 3  11 ) 2  ( 8  3 ) 2  221

d BC  (11  8 ) 2  ( 3  2 ) 2  34

d AC  ( 3  8 ) 2 ( 8  2 ) 2  221

Como AB = AC  BC; el triángulo es isósceles

7. Hallar la distancia entre:

a) A (-2,3) y B (5,1)

d AB  ( 5 - (-2) ) 2  ( 3  1) 2  53

b). C (6, -1) y D (-4, -3)

d CD  (-4 - 6) 2  (3  (1)) 2  2 26

109
8. Demostrar que A(7,5), B(2,3) y C(6, -7) son vértices de un
triángulo rectángulo.

d AB  (2 7 ) 2  (3  5 ) 2  25  4  29
d BC  ( 6  2 ) 2  (7  3 ) 2  16  100  116
d AC  ( 6  7 ) 2  (7  5 ) 2  1  144  145

AB2  BC2  AC2 ; 29  116  145


El cuadrado de la hipotenusa (AC) es igual a la suma de
los cuadrados de los catetos (AB y BC).

8. Calcula la distancia entre los puntos


A (1, 2) y B (4, 3).

d ( A, B)   4  1   3  2   10
2 2

12.Calcula la distancia entre los puntos P 1 (7,


5) y P 2 (4, 1)

13.

110
Ejercicios

1) Hallar la distancia entre:


a) A ( 4,1 ) y B ( 3,-2 )
b) C ( -1,-5 ) y D ( 2,-3)

2) Determinar un punto que equidiste de: A ( 1,7 ); B ( 8,6 ) y C ( 7,-1 )

3) Hallar el perímetro del cuadrilátero cuyos vértices son: A( -3,-1 ); B( 0,3 );


C( 3,4 ) y D( 4,-1 )

4) Demostrar que :
a) A (0,1); B (3,5); C (7,2) y D (4,-2) son vértices de un cuadrado.

b) A ( 1,1 ); B ( 3,5 ); C ( 11,6 ) y D ( 9,2 ) son los vértices de


un paralelogramo.

5) Uno de los extremos de un segmento rectilíneo de longitud igual a 13 es


el punto A (-1, -5 ); si la abscisa del otro extremo es 2, hallar su ordenada (
dos soluciones ).

6) Dos de los vértices de un triángulo equilátero son los puntos A (3,1 ) y B ( -


1, 1 ); hallar las coordenadas del tercer vértice ( dos soluciones )

7) Hallar la longitud de las diagonales del paralelogramo que tiene como


vértices los puntos: A ( 0,0 ); B ( 3,0 ); C ( 4,2 ) y D ( 1,2 )

8) Demostrar que los puntos A (3,3 ); B ( -3,-3 ) y C ( -3 3 , 3 3 ) son


vértices de un triángulo equilátero.

9) Hallar el perímetro del triángulo cuyos vértices son: A(-2, 5 ), B( 4, 3 ) y


C( 7, -2 ).

10) Uno de los extremos de un segmento rectilíneo de longitud igual a


10 es el punto A (-3, 6 ); si la abscisa del otro extremo es ( 3 ), hallar su
ordenada ( dos soluciones ).

111
11. Halla la distancia entre A y B en cada caso:

a. A(-7, 4), B(6, 4) b. A(3, 4), B(3, 9) c. A(-5, 11), B(0, -1)

12. Calcula el valor de k para que la distancia de A(-1, 4) a B(k, 1) sea igual a 5.

13. Halla las coordenadas de dos puntos tales que la distancia entre ellos sea igual
a 4.

14. Calcula el perímetro de los siguientes triángulos y clasifícalos según la longitud


de sus lados:

a. A(-2, 2), B(1, 6), C(6, -6) b. A(-5, -2), B(0, 6), C(5, -2)

15. Usando las cuadrículas proporcionadas, determine la distancia entre los


puntos A y B en cada caso.

C) D)

112
16. Utilizando el teorema de Pitágoras resuelva los siguientes problemas, puede
ubicarse en el plano cartesiano para entender mejor algunos problemas.

1) Un automóvil recorre 15 km hacia el norte, dobla hacia la derecha en ángulo


recto y continúa 5km más. Posteriormente dobla hacia el norte y recorre otros
10km, terminando con 14 km hacia la izquierda en ángulo recto. ¿A qué distancia
se encuentra del punto original? ¿Cuánto camino recorrió?

2) Una persona camina 4km hacia el norte y 3km al oeste. Luego cambia hacia el
norte y camina 8km, por último camina 6km más hacia el oeste. ¿A qué distancia
se encuentra del origen? ¿Cuánto camino recorrió esa persona?

3) El hueco de una ventana mide 41 pulgadas de ancho y 26 pulgadas de altura.


¿Se Puede introducir por la ventana una mesa de ping-pong de 48 pulgadas de
ancho?

4) Una puerta mide 210 cm de altura por 80 cm de ancho. ¿Cuál es el ancho mayor
que puede tener una lámina para que pase por esta puerta?

5) Una persona viaja 8km al norte, 3km al oeste, 7 km al norte y 11km al este. ¿A
qué distancia está la persona del punto original? ¿Cuánto camino recorrió en su
totalidad?

113
114
Tema 3 Trigonometría

Situación Problema
Los estudiantes del nuevo gobierno estudiantil del Colegio El Progreso
se dieron cuenta que para trasladarse hacia el Pabellón 2 de su
Institución sólo existían gradas. Ellos tienen claro que de acuerdo a la ley 7600 se
deben construir rampas de acceso para personas con discapacidad. Es por esto que
le solicitan al director del colegio que ponga en regla esta situación para no afectar
los derechos de las personas.

De esta manera se asesoran y encuentran que de acuerdo a las regulaciones


existentes respecto a la construcción de rampas de acceso según la ley 7600 de
Costa Rica la rampa debe de tener una elevación máxima de 15°.

Si cada escalón tiene una altura de 22 cm y una profundidad de 25 cm, y deciden


hacer la rampa con exactamente 15° de elevación, ¿a qué distancia del primer
escalón deberá iniciar la rampa para cumplir con las especificaciones y qué longitud
tendrá

Análisis de la Actividad

Una estrategia de resolución es la de visualizar la situación como un problema


trigonométrico. Observe la siguiente imagen representativa:

115
Se conoce que el ángulo de elevación debe ser de 15° y la altura que deberá
alcanzar la rampa es de 66 cm debido a que la altura de cada grada es de 22 cm.
Sea AB la longitud de la rampa, el estudiante puede establecer la siguiente
relación:

66
sen 15  
AB
66
AB 
sen 15 
AB  255cm

Por lo tanto, la rampa tendrá una longitud de 255 cm o 2,55 m aproximadamente.


Para saber a qué distancia del primer escalón se debe iniciar la construcción de la
rampa se puede utilizar la siguiente relación:

66
tan 15  
AC
66
AC 
tan 15 
AC  246,32m

Ahora bien, como las gradas tienen una profundidad de 25 cm cada una, entonces
la rampa se deberá iniciar a 247 cm – 50 cm = 197 cm o 1,97 m del borde inferior
del primer escalón.

La Clave

Conversiones de medidas de ángulos

Hasta ahora, las medidas de los ángulos sólo se conocen en grados. Sin
embargo, las medidas de los ángulos se pueden expresar en otra unidad llamada
radianes (𝑟𝑎𝑑). La equivalencia se da al decir que 𝜋 𝑟𝑎𝑑 = 180°. Las conversiones se
pueden dar en dos direcciones diferentes: de grados a radianes y de radianes a
grados. Se estudia cada caso por separado.

 Si la medida 𝛼 está dada en grados, la medida en radianes se obtiene por


medio de la siguiente fórmula:
𝛼° ∙ 𝜋
𝑚𝑒𝑑𝑖𝑑𝑎 𝑑𝑒𝑙 á𝑛𝑔𝑢𝑙𝑜 = 𝑟𝑎
180

116
Ejemplo, se pueden hacer las siguientes conversiones:

Medida en grados Operación Resultado en radianes

4 4 1
∙𝜋 4 1
° 𝜋 𝑟𝑎𝑑
𝜋 = = 45
180 180 45

85 ∙ 𝜋 17𝜋 17𝜋
85° = 𝑟𝑎𝑑
180 36 36

125 ∙ 𝜋 25𝜋 25𝜋


125° = 𝑟𝑎𝑑
180 36 36

 Si la medida 𝛼 está dada en radianes, la medida en grados se obtiene por


medio de la siguiente fórmula:

𝛼 𝑟𝑎𝑑∙𝜋°
𝑚𝑒𝑑𝑖𝑑𝑎 𝑑𝑒𝑙 á𝑛𝑔𝑢𝑙𝑜 =
180

Ejemplo, se pueden hacer las siguientes conversiones:

Medida en radianes Operación Resultado en grados

5𝜋 5𝜋
∙ 180 900𝜋
𝑟𝑎𝑑 4 225°
4 = = 225
𝜋 4𝜋

3
3 ∙ 180 540 270 270°
𝑟𝑎𝑑 2 = =
2 𝜋 2𝜋 𝜋 𝜋

7𝜋 7𝜋
∙ 180 1260𝜋
𝑟𝑎𝑑 20 63°
20 =
𝜋 20𝜋

117
Ejemplo

1. Convierta las siguientes medidas a radianes.

a. 30°.
b. 45°.
c. 60°.
d. 250°.
Solución

𝜋 𝜋
a. 30° = 30° ⋅ = .
180° 6
𝜋 𝜋
b. 45° = 45° ⋅ = .
180° 4
𝜋 𝜋
c. 60° = 60° ⋅ = .
180° 3
2. Convierta las siguientes medidas a grados.

𝜋
a. .
4
𝜋
b. 2 .
5𝜋
c. .
6
d. 4 rad.

Solución
𝜋 𝜋 180°
a. = ⋅ = 45°.
4 4 𝜋
𝜋 𝜋 180°
b. = ⋅ = 90°.
2 2 𝜋
5𝜋 5𝜋 180°
c. = ⋅ = 150°.
6 6 𝜋
180°
d. 4 = 4 ⋅ = 229,29°.∎
𝜋

Ejemplo Para transformar de una unidad a otra, usamos la regla de tres:

180º  rad 180º  rad 40º  rad 4 rad 2 rad


  ejemplo: 40º a rad   y=  
xº y 40º y 180º 18 9

118
Ejercicios

1)Convierta los siguientes ángulos a radianes:

60º _________ 2) 220º _____________ 3) 315º _______________

2) Convierta los siguientes ángulos a grados:


5
1) 2 ___________ 2)  ________________
4
2
3)  _______________
3

3) Realice la conversión a Radianes de los siguientes ángulos

( a ) 45° ( d ) 140° ( g ) – 20° ( j ) 740° ( m ) – 5°

( b ) – 132° (e) – 570° ( h ) 311° (k)– 2280° ( n ) 8°

( c ) 90° ( f ) 210° ( i ) 360° ( l ) 120° ( ñ ) 190°

4) Realice la conversión a Grados de los siguientes ángulos

(a) 5 (d) π π ( j ) – 5π (m)– 11π


(g)–
2
π 5π π
(b) – (e)7 3π (k) (n)
4 (h) 3 3
4

8π 7π 9π
(c) 17 π (l) – (ñ) –
5 (i ) 6 2
( f ) 21 3

119
5) Convierta los siguientes ángulos de grados a radianes o de radianes a grados
según sea el caso.

a) 60º =

b) 
3
c) 150º =
5
d) 
6
e) 4 =
f) 225º =

g) 
2
e) - 600º

6) Transformar el ángulo de grados a rad:

1) 15º 2) 35º 3) 80º 4) 150º

5) 200º 6) 90º 7) 60º 8) 45º 9) 30º

7) Transformar el ángulo de rad a grados:

 
1) rad 2) rad 3) 3 rad
5 10

Razones trigonométricas

Para definir adecuadamente las razones trigonométricas de un ángulo, es


necesario determinar, en un triángulo rectángulo, qué nombre recibe cada
lado del triángulo con respecto a los ángulos agudos. Así, se tiene que

120
Figura Con respecto al ángulo dado
Medida del lado
∡𝛼 ∡𝛽

𝑎 Cateto adyacente Cateto opuesto

𝑏 Cateto opuesto Cateto adyacente

𝑐 Hipotenusa Hipotenusa

La diferenciación anterior se debe a que las razones trigonométricas utilizan las


medidas de los lados de un triángulo dependiendo de la posición que éstos ocupan
con respecto a uno de los ángulos agudos.Las razones trigonométricas siempre se
definen para un ángulo determinado. Si el ángulo se denota como ∡𝜃, se tiene:

Razón Abreviatura utilizada Definición


𝑚𝑒𝑑𝑖𝑑𝑎 𝑑𝑒𝑙 𝑐𝑎𝑡𝑒𝑡𝑜 𝑜𝑝𝑢𝑒𝑠𝑡𝑜
Seno sen(𝜃) = sen 𝜃
𝑚𝑒𝑑𝑖𝑑𝑎 𝑑𝑒 𝑙𝑎 ℎ𝑖𝑝𝑜𝑡𝑒𝑛𝑢𝑠𝑎
𝑚𝑒𝑑𝑖𝑑𝑎 𝑑𝑒𝑙 𝑐𝑎𝑡𝑒𝑡𝑜 𝑎𝑑𝑦𝑎𝑐𝑒𝑛𝑡𝑒
Coseno cos(𝜃) = cos 𝜃
𝑚𝑒𝑑𝑖𝑑𝑎 𝑑𝑒 𝑙𝑎 ℎ𝑖𝑝𝑜𝑡𝑒𝑛𝑢𝑠𝑎
𝑚𝑒𝑑𝑖𝑑𝑎 𝑑𝑒𝑙 𝑐𝑎𝑡𝑒𝑡𝑜 𝑜𝑝𝑢𝑒𝑠𝑡𝑜
Tangente tan(𝜃) = tan 𝜃
𝑚𝑒𝑑𝑖𝑑𝑎 𝑑𝑒𝑙 𝑐𝑎𝑡𝑒𝑡𝑜 𝑎𝑑𝑦𝑎𝑐𝑒𝑛𝑡𝑒

Para recordarlo con más claridad, se utiliza la abreviatura


𝑂 𝐴 𝑂
𝑆 𝐶 𝑇 , que sirve para recordar la definición de todas las razones.
𝐻 𝐻 𝐴

Ejemplo

Figura Para cada ángulo dado


Razones
∡𝛼 ∡𝛽

𝑏 𝑎
Seno
𝑐 𝑐
𝑎 𝑏
Coseno
𝑐 𝑐
𝑏 𝑎
Tangente
𝑎 𝑏

121
Las razones trigonométricas se utilizan para determinar las medidas faltantes en
un triángulo, tanto de lados como de ángulos.

Ejemplo Si en la figura adjunta 𝑏 = 25 𝑐𝑚 y 𝑚∡𝛽 = 40°, se debe determinar


los valores de 𝑏, 𝑐 y 𝑚∡𝛼:

Utilizando las razones trigonométricas, se tendría que


25
cos(40°) =
𝑐
De donde se obtiene:
𝑐 ∙ cos(40°) = 25
25 25
𝑐= ≈ ≈ 32,6370757 ≈ 32,6
cos(40°) 0,7660

Con los datos anteriores, se puede utilizar el teorema de Pitágoras para determinar
el valor de 𝑎:
𝑎2 + 252 = 32,62
𝑎2 + 625 = 106,76
𝑎2 = 1062,76 − 625
𝑎2 = 437,76
𝑎 = √437,76 ≈ 20,9227149 ≈ 20,9

De la misma manera, como el ángulo no señalado es recto, su medida es 90°, y,


como la suma de las medidas de los ángulos internos de todo triángulo da como
resultado 180°, se tiene que
180° = 90° + 40° + 𝑚∡𝛼
180° = 130° + 𝑚∡𝛼
180° − 130° = 𝑚∡𝛼
50° = 𝑚∡𝛼

De esta manera, se tiene resuelto el triángulo:

Símbolo Medida
∡𝛼 50°

∡𝛽 40°

𝑎 20,9 𝑐𝑚

𝑏 25 𝑐𝑚

𝑐 32,6 𝑐𝑚

122
Al utilizar la calculadora, los botones útiles para resolver problemas con
trigonometría son el que dice 𝑠𝑖𝑛 para seno, el que dice 𝑐𝑜𝑠 para coseno y el que
dice 𝑡𝑎𝑛 para la tangente. Los cálculos se pueden hacer con las medidas de los
ángulos tanto en grados como en radianes.

Para tener la seguridad de que la calculadora está preparada para hacer los cálculos
en grados, unidad más común en la resolución de la mayor parte de los ejercicios
y problemas de trigonometría, en la parte superior de la pantalla debe aparecer
una letra D, inicial del vocablo inglés degree (grado). Si, por el contrario, se observa
una G (gradientes) o una R (radianes), antes de resolver cualquier operación se
debe oprimir el botón SHIFT, seguido del botón MODE (ubicado a la izquierda del
botón de encendido (ON)) y luego el número 3. Además, si se desea trabajar con
las medidas en radianes, se debe oprimir el botón SHIFT, seguido del botón MODE
y luego el número 4.

De igual manera, los tres botones trigonométricos (SIN, COS y TAN) tienen dos
funciones cada uno. La primera función corresponde al caso en el que se tiene la
medida del ángulo y se desea conocer el valor de su razón trigonométrica. Pero,
cuando se tiene el valor de la razón trigonométrica y se desea conocer la medida
del ángulo, se debe oprimir SHIFT antes de cualquiera de estos botones, con lo cual
se verá el resultado como sin−1, cos−1 o tan−1 .

Ejemplo Si en la figura adjunta 𝑏 = 24 𝑐𝑚 y 𝑎 = 7 𝑐𝑚, se debe determinar


los valores de 𝑐, 𝑚∡𝛽 y 𝑚∡𝛼:

Utilizando las razones trigonométricas, se tendría que


7
tan(𝛽) =
24
De donde se obtiene:
tan(𝛽) ≈ 0,2917
𝛽 = tan−1(0,2917) ≈ 16,262°

Con los datos anteriores, se puede utilizar el teorema de


Pitágoras para determinar el valor de 𝑐:
72 + 242 = 𝑐 2
49 + 576 = 𝑐 2
625 = 𝑐 2
𝑐 = √625 = 25

De la misma manera, como el ángulo no señalado es recto, su medida es 90°, y,


como la suma de las medidas de los ángulos internos de todo triángulo da como
resultado 180°, se tiene que
180° = 90° + 16,262° + 𝑚∡𝛼

123
180° = 106,262° + 𝑚∡𝛼
180° − 106,262° = 𝑚∡𝛼
73,738° = 𝑚∡𝛼

De esta manera, se tiene resuelto el triángulo:

Símbolo Medida
∡𝛼 73,262°
∡𝛽 16,262°
𝑎 7 𝑐𝑚
𝑏 24 𝑐𝑚
𝑐 25 𝑐𝑚

Ejemplo Determine la medida del cateto AB y encuentre las razones


trigonométricas de  .

B

x 8

A 2 C
Solución

Utilizando el teorema de Pitágoras para encontrar la medida del cateto x, se tiene:


x2  22  82
x 2  64  4
x 2  60
x  2 15
Ahora bien, como x es una distancia, entonces x  2 15 .

Por otra parte, las razones trigonométricas de  son las siguientes:

2 1
sen    csc   4
8 4

2 15 15 4 4 15
cos     s e c  
8 4 15 15

124
2 1 15
tan      cot   15
2 15 15 15

Ejemplo

Determine las medidas aproximadas de AB y AC si mABC  29 y d(B,C)= 9.


A

x y

29
C 9 B
Solución
i) ii)
x
tan 29   9 tan 29  x
9
9  0, 554  x
4,99  x

Ejemplo

Si la altura de un triángulo isósceles sobre su lado desigual es 10 cm y la


base es 16 cm. Calcule la medida de sus ángulos internos.
C

Solución

10
10
a) tan      51, 3
8
A 8 8 B
b) Como los ángulos de la base
son congruentes mACB  180  2  51,3  77,4

125
Ejemplos a) Seno de un ángulo:

En cualquier triángulo rectángulo se define el seno del ángulo  como el


cociente del cateto opuesto entre la hipotenusa.

cat. op.
sen  =
hip.


3 5 3 5


4 4

3 4
sen  = sen  =
5 5

b) Coseno de un ángulo:

En cualquier triángulo rectángulo se define el coseno del ángulo  como el


cociente del cateto adyacente entre la hipotenusa.

cat. ady.
cos  =
hip.

 
13 5 13 5 4 5 4

 
12 12 8

12 5 8 4
cos  = cos  = cos  = cos  =
13 13 4 5 4 5

126
C) Tangente de un ángulo:

En cualquier triángulo rectángulo se define la tangente del ángulo  como


el cociente del cateto opuesto entre el cateto adyacente.

cat. op.
tan  =
cat. ady.

Ejemplos:


24 30 24 30 15 3 30

 
18 18 15

24 12 5 3 15
tan  = tan  = tan  = tan  =
18 13 15 5 3

Ejemplos: Con las razones trigonométricas podemos determinar la medida


de los ángulos agudos del triángulo rectángulo conociendo la medida de sus lados
(para este tema es fundamental el uso de la calculadora). Ejemplos De acuerdo
con los datos de las figuras determine la medida de los ángulos q , b y a.

127
Ejercicios

1. Determine las razones trigonométricas de los siguientes triángulos rectángulos

2) Utilice la calculadora y aproxime con tres decimales por exceso el valor de las
seis razones trigonométricas de los ángulos dados en cada caso:

a) 18o b) 73,5o c) 35o d) 85,2o

3) Para cada caso, encuentre el valor del ángulo agudo  , aproximando con dos
decimales por exceso, que cumpla con la condición dada:

a) sen  0,42 b) cos   0,8 c) tan   8, 32

128
4) Los catetos de un triángulo rectángulo miden 5 m y 7 m. Halla la hipotenusa y
los ángulos.

[sol] 8,60 m; 35,53º

5) El cateto menor de un triángulo rectángulo mide 12 m y la hipotenusa 35 m.


Halla el otro cateto y los ángulos.

[sol] 32,88 m; 20,06º

6) En un triángulo rectángulo sabemos que un ángulo mide 37º y el cateto


contiguo 15,4 m. Halla los otros dos lados y el otro ángulo agudo.
[sol] 53º, 19,28 m, 11,60 m

7) En las siguientes figuras, utilizando razones trigonométricas determine el valor aproximado


de las variables " x " "y "

129
7
8) Si se sabe que sen  = , determinar cos . (Sugerencia: dibujar un
25
triángulo rectángulo)

9) En el triángulo siguiente hallar las medidas de tan  y tan . Además hallar


las medidas aproximadas del  y del .

3
10) Si se sabe que cos  = 3 , determinar sen  y tan  . (Sugerencia: dibujar
un triángulo rectángulo)

11)De acuerdo con el criterio LLL, los dos triángulos siguientes son semejantes (
ABC  JFK ). Complete la información que se le pide.

a) sen 1 = ________ a) sen 2 = _________

b) cos 1 = ________ b) cos 2 = __________

c) tan 1 = ________ c) tan 2 = __________

Conclusión:
_______________________________________________________________
_______________________________________________________________

130
4
12) En un triángulo rectángulo, uno de sus ángulos agudos es , y tan  = .
3
El cateto opuesto al  mide 48cm. Hallar las medidas de los otros dos lados
de este triángulo

13) Utilizando razones trigonométricas calcule el valor aproximado de los ángulos


en las siguientes figuras.

14). Dibuje un triángulo que tenga un 17) Resuelva los triángulos rectángulos
5 que tienen las características dadas en
ángulo A, y la razón Sen A = la cada caso. Considere que  es un ángulo
13
interno del triángulo.
contenga también.
a)  = 37° y la hipotenusa 24 cm
15). Calcule las razones b)  = 77° y el cateto opuesto al
trigonométricas de cada uno de los ángulo
ángulos agudos de los siguientes
triángulos rectángulos. Tome en  mide 25 cm
cuenta la información proporcionada en c)  = 39° y el cateto opuesto al
cada figura.
ángulo  mide 34 cm
a) A m(AB) = 10 cm
m(CB) = 8 cm d)  = 18° y el cateto opuesto mide
100 m.
C B e)  = 35° y el cateto opuesto mide 3,8

b) F D m(DE) = 20 cm m.
m(DF) = 24 cm f)  = 70° y el cateto adyacente mide
E 14 m.
g)  = 64° y el cateto adyacente mide
147 m.

131
 = 68° y el cateto adyacente mide 75 cm
16). De acuerdo con los datos de la
19) Calcule el perímetro y el área de cada
figura adjunta, escriba en el espacio el uno de los siguientes triángulos
nombre del vértice del ángulo que hace rectángulos
la igualdad resulte verdadera. A
a)
15 36°
a) Cos ______ = A
17
8
b) Sen ______ = 17cm
17
15 B 27cm C
c) Tan ______ = 8cm
8
8 b) R
d) Tan ______ = B 15cm
15
C
15
e) Sen ______ =
17
8 24°
f) Cos ______ = S 176 m P
17
20)Si la altura de un triángulo isósceles es
18. Determine el valor de las otras razones
trigonométricas para cada uno de los de 16 cm y uno de los ángulos congruentes
siguientes enunciados. mide 35°, calcule el área del triángulo.

4
a) Sen  =
5 8. Determine cada uno de los valores
b) Cos  = 0,6 desconocidos en cada uno de los siguientes
triángulos.
c) Tan  = 1 E b D
a) R b) 68°
d) Cos  = 0,2 y
e) Tan  = 4 x a
12,3cm 70°
1
f) Sen  = S P
2
5,20 cm
g) Tan  = 1,5

2 F
h) Cos  =
2

132
21) Utilice la figura de la derecha para calcular la medida de los lados y de los
ángulos faltantes en cada caso. Trabaje con valores exactos y racionalice
denominadores.

a) c = 12; mA  45o B

b) a = 8; mB  30 o

c) c  3 ; mB  60 o c a

d) c  2 ; b2
b

22) Determina cuál es el cateto opuesto y cuál el cateto adyacente para el ángulo
indicado en cada triángulo.

23) Halla las seis razones trigonométricas para el ángulo  en cada triángulo

133
25) Completa cada espacio con el ángulo que corresponde a cada razón
trigonométrica

Aplicaciones y angulos de elevacion y de deprecion

Ángulos de elevación y de depresión


Estos ángulos se definen cuando hay un objeto observado y un observador
que se encuentran distanciados tanto en altura como a nivel del suelo. Siempre se
definen entre la línea de la vista (distancia entre el objeto y el observador) y la
horizontal (suelo o línea paralela al suelo). La siguiente figura ejemplifica de mejor
manera el dibujo útil para resolver estos ejercicios:

La distancia vertical generalmente se da con referencia a un edificio (torre, faro,


edificio sin especificaciones,…) o alturas (vuelo de un avión o un papalote a una
altura determinada, altura de la ventana, estatura de una persona,…).
Si un observador en un punto 𝑋 ve un objeto, entonces el ángulo que forma la
visual con la horizontal 𝑙 es un ángulo de elevación del objeto, si está arriba del
la horizontal o el ángulo de depresión del mismo, si el objeto que ve está debajo

134
de la horizontal. En la figura adjunta el ángulo de elevación es 𝛼 y el ángulo de
depresión es 𝛽.

𝛼
𝛽

Algunas situaciones que se resuelven con triángulos rectángulos, involucran


ángulos de elevación o ángulos de depresión. Cuando un objeto es observado, la
recta imaginaria que se forma entre el observador y el objeto, se denomina línea
visual. La línea visual forma con la horizontal imaginaria, un ángulo cuyo nombre
depende de la ubicación del objeto con respecto al observador:

• Si el objeto está a un nivel más alto que el


observador, el ángulo se denomina ángulo de elevación.

• Si el objeto está a un nivel más bajo que el


observador, el ángulo se denomina ángulo de depresión.

135
Ejemplos Carlos quiere saber la medida del ancho de un río sin tener que
desplazarse a la otra orilla. Midiendo sus pasos llega a la siguiente situación:

¿Cuánto mide el río de ancho? Primero, como ABC es un triángulo rectángulo del
que se conocen sus catetos, se debe calcular
3
la tangente de a. Así: tan    0, 6 Segundo,
5
como DCE es un triángulo rectángulo del
que se conoce uno de sus catetos y un
ángulo a, entonces se determina a a partir
a
tan  
25
a
de la ecuación 0, 6  n:
25
a  15m

Ejemplo Una colina forma un ángulo de 30° con la base. Si una persona
recorre 2.500 m para llegar a la cima de la colina, ¿cuál es la altura de la colina?

En este problema se conocen la hipotenusa y un ángulo de un triángulo rectángulo,


hay que hallar un cateto. Primero, se plantea una ecuación con la razón que
relaciona la hipotenusa, el ángulo y su cateto opuesto, así:

h
sen30 
2500 Luego, se responde la pregunta del problema: la altura
h  2500.sen  30   1.250m
de la colina es 1.250 m.

136
Ejemplo Una escalera de 2,13 m está apoyada contra una pared. La base
de la escalera está a 1,5 m de la pared. ¿Cuál es la medida del ángulo que forma
la escalera con el piso? ¿Cuál es la altura de la pared?

Primero, se encuentra la medida del ángulo a partir de la razón

1,5
cos    0, 7075 ;  ar cos 0, 7075  450
2,12

h
Luego, se halla la altura de la pared, así: tan 450   h  1,5  tan 450  1,5mts
1,5

La medida del ángulo que forma la escalera con el piso es 45° y la altura de la
pared es 1,5 m

Ejemplo Si deseamos calcular la altura aproximada del Fortín de Heredia,


teniendo algunos datos podemos lograrlo

Del triángulo rectángulo formado, podemos


extraer diversa información: El cateto
correspondiente a la altura del Fortín, es
opuesto al ángulo de 50°. El cateto
horizontal, que mide 11 metros, es
adyacente al ángulo de 50°. Con estos
datos, podemos establecer la siguiente
relación

h
tan 500   h  11  tan 500  13,5mts
11

137
Ejemplo Desde un punto en un plano, se observa, con un ángulo de
elevación un objeto en la parte más alta de un edificio, la distancia del punto de
observación a la base del edificio es de 40m
¿Cuál es la altura aproximada del edificio?

Primero: Construimos una figura que ilustre


lo planteado en el problema. Segundo:
Aplicamos una razón trigonométrica que
satisfaga el problema, en este caso tenemos
que averiguar el segmento AB, por lo que
aplicamos tangente. Tercero: Concluimos
que altura aproximada del edificio es de
64,01m

Ejemplo Desde el punto más alto de una antena, que mide 20m de altura,
se observa, con un ángulo de depresión de 42º un objeto ¿Cuál es la distancia
aproximada entre el punto de observación y el objeto?

Primero: Construimos una figura que ilustre


lo planteado en el problema. Segundo:
Aplicamos una razón trigonométrica que
satisfaga el problema, en este caso tenemos
que averiguar el segmento BC, por lo que
aplicamos seno. Tercero: Concluimos que la
distancia entre el punto de observación y el
objeto es aproximadamente de 29,88m.

Ejemplo

Un alambre de soporte debe ser colocado en la punta de un


poste telefónico de 6 𝑚 de altura y fijado en la tierra. ¿Qué
cantidad de alambre se necesitará para que haga un ángulo de
46° con el nivel del suelo?

138
Al buscar la respuesta, se debe plantear primero la figura que modela la situación:

Al analizar la figura, con respecto al ángulo formado entre el alambre y el suelo, se


tiene que el poste es el cateto opuesto al ángulo y el alambre es la hipotenusa del
triángulo.

Así, recordando las definiciones de las


razones trigonométricas, es claro que la
aplicable es seno, por lo cual:

6
sen(46°) =
𝑥

De donde se obtiene:

𝑥 ∙ sen(46°) = 6

6 6
𝑥= ≈ ≈ 8,341444307 ≈ 8,34
sen(46°) 0,7193

Así, aproximadamente se requieren 8,34 𝑚 de alambre para colocar el poste en el


lugar deseado.

Ejemplo
Desde la orilla del río, un observador de 1,8 𝑚 de altura observa
una piedra rojiza en la otra orilla del río. Con un ángulo de
depresión de 15° ¿Cuál es, en metros, la medida aproximada
del ancho del río?

Al buscar la respuesta, se debe plantear primero la figura que modela la situación:

En el análisis de la figura, con


respecto al ángulo de depresión,
se tiene que la estatura del
observador es el cateto opuesto
al ángulo de 15°, mientras que el
ancho del río corresponde al
cateto adyacente, por lo cual los
cálculos se deben hacer utilizando la razón trigonométrica tangente:
1,8
tan(15°) =
𝑥

139
De donde se obtiene:
𝑥 ∙ tan(15°) = 1,8
1,8 1,8
𝑥= ≈ ≈ 6,78611422 ≈ 6,79
sen(15°) 0,2679

Así, aproximadamente el ancho del río es de 6,79 𝑚.

Ejemplo Un faro construido al nivel del mar tiene su observatorio a 16m


de alto. Si desde él se observa una boya con un ángulo de depresión de 49o, ¿cuál
es la distancia entre la boya y la base del faro?

Un faro construido al nivel del mar tiene su observatorio a 16m de alto. Si desde
él se observa una boya con un ángulo de depresión de 49o, ¿cuál es la distancia
entre la boya y la base del faro?
Solución
Como primer paso conviene hacer un esquema que ilustre la situación. A
49 o

16m
(faro)

B x C (boya)
Sea x la distancia entre la boya y la base del faro.
Observe que la medida del ángulo BAC es 41o, ya que es el complemento de 49o.
Y el ángulo BCA es congruente con el de depresión, por ser ángulos alterno
internos. También se puede ver que BCA y BAC son complementarios.

Se tiene entonces que se puede trabajar con cualquiera de los dos ángulos
agudos del triángulo rectángulo ABC.

Usando la información del BCA se tiene:

16 16
tan 49 o  x
x tan 49 o
16
x  13,91
1,15
Por lo tanto la distancia entre la boya y la base del faro es de aproximadamente
13,91m.

140
Ejemplo
Juan desea estimar la altura del campanario de una iglesia pero, por razones
que no se exponen, no hay manera de llegar directamente a él.

Para lograr su objetivo Juan realiza el siguiente plan: marca un punto B en


el suelo exactamente debajo del campanario y otro punto A a 13m de
distancia, también en el suelo, como se muestra en el dibujo de arriba. El
punto más alto en el campanario lo llama D y C es un punto elegido en la
base del campanario del segmento BD.

Luego desde A mide los ángulos de elevación DAB y CAB y obtiene los
siguientes resultados: mDAB  42 o y mCAB  34 o . Llama x la altura del
campanario y y la distancia desde el suelo a la base del campanario.

Primero calcula el valor aproximado de y:

y
tan 34 o   y  13  tan 34 o 
13
y  13  0,67  8,71

141
Usando que y  8,71 establece esta otra relación:

x  8,71
tan 42 o  
13
x  13  tan 42 o  8,71 
x  13  0,9  8,71  2,99

Juan establece que la altura del campanario es aproximadamente 2,99m.

Ejemplo Desde un punto en el suelo, el ángulo de elevación a la parte


superior de una torre es de 30o. Si un observador ubicado en dicho punto avanza
100 m hacia la torre y vuelve a observar la cima de ésta, encuentra que ahora el
ángulo de elevación es de 60o. Encuentre la altura de la torre.

Solución

Para resolver este problema se puede hacer un esquema como el siguiente:

60o 30o
A y B 100

Se va a llamar h la altura de la torre y y la medida del segmento AB. Usando el


esquema anterior se deducen las siguientes relaciones:
h
tan 60o   h  y  tan 60o  y  3 (1)
y
Por otro lado:
h h
tan 30o   y  100 
y  100 tan 30o
Como h  y  3 , sustituyendo se tiene
y 3
y  100   y  100  3 y 
1
3
2 y  100  y  50

142
Así la altura de la torre es 50m.

Ejemplo La medida del ángulo de depresión desde lo alto de una torre de


34 mide altura hasta un punto K en el suelo es de 800. Calcule la distancia
aproximada del punto K a la base de la torre

Solución

Se dibuja una figura representativa de


la situación.

Se plantea la razón trigonométrica


tangente del ángulo que mide 100
para encontrar el valor de x.

Se da respuesta al problema La distancia aproximada desde el


planteado. punto K a la base de la torre es de 6m.

Ejemplo Un turista observa la parte más alta de un edificio de 15m de


altura, con un ángulo de elevación de 240. Si realiza la observación con unos
binoculares que sostiene a 1,75m del suelo, calcule la distancia aproximada entre
el turista y la parte más alta del edificio.

Se dibuja una figura representativa de


la situación, dividiendo en dos partes la
altura del edificio según el dato de la
altura a la cual se ubican los
binoculares del turista.

143
Se plantea la razón trigonométrica seno
del ángulo que mide 240 para
encontrar el valor de x.

Se da respuesta al problema planteado. La distancia aproximada entre el turista


y la parte más alta del edificio es de
32,6 m

Ejemplo Cuando un avión pasa sobre un punto M ubicado en el suelo, una


estación de observación que está situada a 4kmde M lo observa con un ángulo de
elevación de 190. Calcule la altura aproximada a la que se encuentra el avión en
ese momento.

Se dibuja una figura representativa de


la situación.

Se plantea la razón trigonométrica


tangente del ángulo que mide 190 para
encontrar el valor de x.

Se da respuesta al problema planteado. La altura aproximada del avión en ese


momento es de 1,4km.

144
Ejemplo Una mujer con una estatura de 1,64m proyecta su sombra en
el suelo. Si el ángulo de elevación que se forma desde la punta de la sombra hasta
la mujer es de 420, entonces, calcule la longitud aproximada de la sombra

Se dibuja una figura representativa de


la situación.

Se plantea la razón trigonométrica


tangente del ángulo que mide 420 para
encontrar el valor de x

Se da respuesta al problema La longitud aproximada de la sombra


planteado. es de 1,8m

Ejercicios

1. Un ingeniero coloca un cable desde la parte más alta de una torre


de 45m de altura hasta un punto A en el suelo. Si el ángulo de
elevación que se forma en el punto A es de 380, calcule la longitud aproximada del
cable.

2. Dos edificios A y B están ubicados uno en frente del otro. El edificio A tiene 48
m de altura y el ángulo de depresión que se forma desde su parte más alta hasta
la base del edificio B es de 650. Calcule la distancia aproximada entre ambos
edificios.

145
3. La sombra de un edificio tiene una longitud de 0,15km. Si el ángulo de elevación
que se forma en la punta de la sombra hacia la parte más alta del edificio es de
320, calcule la altura aproximada del edificio.

4. Un avión despega de un punto K en el aeropuerto y asciende con un ángulo


constante de 380con la horizontal. Calcule la altura aproximada del avión después
de volar 1800m.

5. En el suelo se encuentra el objetivo de rescate de un helicóptero que está


volando sobre él, mientras se ubica a 600m de un puesto de observación en tierra,
desde donde es observado con un ángulo de elevación de 550. Calcule la distancia
aproximada entre el objetivo del helicóptero y el puesto de observación.

6. Desde la parte más alta de un faro, con un ángulo de depresión de 540, se


observa un barco en el mar a una distancia de 117m de su base. Calcule la altura
aproximada del faro.

146
5. Resuelva los siguientes problemas

A. Desde la cúspide de una torre de 30m de alto se observa, con un ángulo de


depresión de 29º , un objeto ubicado en el mismo plano horizontal que la base de
la torre. ¿A qué distancia aproximada se encuentra el objeto de la base de la torre?

B. Desde la orilla de un río, un observador de 1,70m de altura observa una piedra


en la otra orilla del río, con un ángulo de depresión de 17º. ¿Cuál es en metros, la
medida aproximada del ancho del río?

C. Un guardabosques ve un fuego con un ángulo de depresión de 7º mientras vigila,


de una torre que mide 825m ¿A qué distancia de la torre, aproximadamente está
el fuego?

D. El ángulo de depresión medido desde lo alto de un faro hasta una embarcación


en el mar es de 15º. Si la altura del faro es 30m entonces ¿Cuál es la distancia
aproximada en metros de la embarcación a la base del faro?

E. Desde un punto ubicado en una torre vertical y con un ángulo de depresión de


37º se observa un objeto en el plano de la base de la torre a 15m de distancia de
la dicha base. ¿A qué altura aproximada se encuentra el punto de observación
ubicado en la torre?

F. ¿Aproximadamente a qué altura en metros se encuentra el punto más alto de


una bandera con respecto al suelo, si el ángulo de elevación a dicho punto es 55º
y la distancia de la base de la bandera y el punto de observación es de 25m?

G. Un árbol de 6m de altura forma con los rayos del sol, un ángulo de 47º ¿Qué
longitud aproximada en metros tendrá la sombra del árbol?

H. Desde el suelo, Juan observa un avión en el cielo con un ángulo de elevación de


50º. Si la distancia entre el avión y Juan es de 800m, entonces ¿Cuál es la altura
aproximada en metros entre el suelo y el avión?

I. ¿Cuál es aproximadamente la altura, en metros, de un edificio si el ángulo de


elevación a la parte más alta del edificio es 37º y la distancia del punto de
observación a la base del edificio es 30m?

J. El ángulo de depresión determinado desde una torre de observación hasta un


auto es de 37º. Si la altura de la torre es de 40m. ¿Cuál es la distancia aproximada,
en metros, del auto a la base de la torre

147
6. Plantea y resuelve las siguientes situaciones:

a) Una persona de 1,60 m observa el asta de una bandera con un ángulo de


elevación de 30º, si se encuentra a 3 m del pie del asta ¿Qué altura tiene
el asta de la bandera?

b) Desde una determinada posición en un camino, una persona observa la


parte más alta de una torre de alta tensión con un ángulo de elevación de 25º. Si
avanza 45 m en línea recta hacia la base de la torre, divisa ahora su parte más alta
con un ángulo de elevación de 55º. Considerando que la vista del observador está
a 1,70 metros del suelo. ¡Cuál es la altura de la torre?

7. Desde un avión que se encuentra a 4500 m de altura se observan dos


autos corriendo en la misma dirección y sentido con un ángulo de
depresión de 62º y 35º. Determina la distancia en que se encuentran
los dos autos.

8. Marca la alternativa correcta:

1) La cumbre de un cerro se ve desde un punto P del llano bajo un ángulo de


elevación de 35º . Al acercarse horizontalmente 2700 m, el ángulo de elevación es
58º. Entonces la altura del cerro es:

A) 3360 m B) 821,7 m C) 2100 m D) 210 m E) 336 m

2) Desde una distancia de 48 metros de una pared se encuentra apoyada una


escala con un ángulo de 30º con respecto al pie de la escalera. A que altura se
encuentra la escalera del suelo.
16
A) 48m B) 48 3m C) 16 m D) 16 3 m E) m
3

9. Se desea calcular la altura de un árbol, del cual no se puede alcanzar su cima.


Con ayuda de instrumentos de medición de ángulo, se obtienen las medidas
indicadas en la figura.

148
10.Suponiendo que se disponen de dispositivos para medir distancias y ángulos
pero no se puede cruzar el río. ¿Cómo calcular el ancho del río?

11. Calcular la altura de la torre con las medidas que muestran la figura:

12. Se sugiere dibujar todas las situaciones que se presentan a continuación.


a) Una escalera de 6,5 m se apoya contra una pared. Cuando el pie de la escalera
dista 1,5 m de la pared. ¿A qué altura llega la escalera?

b) La torre de control del aeropuerto visualiza un avión con un ángulo de elevación


de 15°. Si el avión está a una altura de 2000 metros ¿A qué distancia está de la
torre de control?

c) Se desea construir una rampa para alcanzar una altura de 0,75 metros. Si el
ángulo de inclinación es de 5°. ¿A qué distancia de la entrada debe comenzar la
rampa?

d) Dos amigos remontan barriletes soltando 85 m. de hilo. Uno forma 45° con la
horizontal y el otro 40°. ¿A qué altura está cada barrilete?

149
e) El radar de un barco indica que un objeto buscado se encuentra a 30 metros de
profundidad y que el ángulo de depresión es de 15°. ¿Qué distancia debe recorrer
un buzo hasta alcanzarlo?

f) Desde un edificio de 60 metros de altura se tira un cable hacia otro de 30 metros


de altura. La distancia entre ellos es de 40 metros. ¿Cuántos metros de cable hay
que comprar, como mínimo, para hacerlo?

g) Un avión recorre en línea recta hacia el norte 250 km., luego al este recorre 600
km. ¿A qué distancia se encuentra del punto de partida?

h) La diagonal de un terreno con forma de cuadrado mide 112 metros. ¿Cuánto


cuesta el terreno si el metro cuadrado de tierra vale $ 25?

i) Un barco se encuentra a 800 km. del destino a donde se dirige. Luego de recorrer
200 km hacia el este, sigue su trayectoria hacia el norte. ¿Cuánto recorre en esa
dirección para llegar a destino?

j) Matías maneja un avión a control remoto que él mismo construyó. Parado a 30


metros de su casa, hace volar el avioncito hasta la terraza que está a 16 metros
de altura. ¿A qué distancia está Matías de su avioncito?

13. Para llevar con la carretilla los escombros y tirarlos en un volquete, los albañiles
colocaron un tablón de 3,50 metros formando un ángulo de 25° con la vereda.
¿Cuál es la altura del volquete?

14. Una tormenta en la ciudad quebró un ciprés de la plaza. La punta cayó a 6


metros del tronco formando un ángulo de 27° con el piso. ¿A qué altura se
quebró el árbol? ¿Cuál era la altura del árbol?

150
14. Con un teodolito situado a 54 metros de un edificio, se observa la parte más
alta del mismo con un ángulo de elevación de 25°. El teodolito está colocado
sobre un trípode a 1,50 metros de altura. Calcular la altura del edificio. Dibujar la
situación.

El teodolito es un instrumento óptico de precisión utilizado en geodesia y


topografía. Sirve para medir ángulos. El taquímetro es un teodolito que además
mide distancias.

15. Un observador se encuentra al nivel del suelo a 200 m de la base de una torre
de TV. Desde ahí observa la punta de la torre bajo un ángulo de depresión de 26º
¿A qué altura se encuentra la torre sobre el nivel de los ojos del observador?

16. ¿Cuál es el ángulo de elevación del Sol cuando un objeto de 6 m proyecta una
sombra de 10,3 m? Respuesta: 30º 10´.

17. Un observador visualiza la parte superior de un edificio a 173 m por encima del
nivel de sus ojos a un ángulo de elevación de 27º 50´. ¿A que distancia se
encuentra el observador del edificio? Respuesta: 328 m.

18. Una torre de 40 m. de altura está situada a la orilla de un lago. Desde la punta
de la torre el ángulo de depresión de un objeto en la orilla opuesta del lago es de
30º. ¿Cuál es el ancho del lago? Respuesta: 69,28 m.

19 Un árbol proyecta una sombra de 16,75 metros cuando el ángulo de elevación


es de 32º. Calcular la altura del árbol. Desde un faro colocado a 40 metros sobre
el nivel del mar, el ángulo de depresión de un barco es de 55º. ¿A qué distancia
del faro se halla el barco? Respuesta: 28 m.

20. Un topógrafo determina que desde el punto A en el suelo el ángulo de elevación


hasta la cima de la montaña mide 25º. Cuando él se encuentra en un punto a 200

151
metros más cerca de la base de la montaña, el ángulo de elevación es de 42º.
¿Cuál es la altura de la montaña? Suponer que la base de la montaña y los dos
puntos de observación están sobre la misma recta. Respuesta: 193,44 m.

Ley de Senos
Ya hemos visto como resolver triángulos rectángulos ahora veremos todas
las técnicas para resolver triángulos generales.
A
α Este es un triángulo ABC el ángulo α se escribe en el
c vértice de A, el ángulo β se escribe en el vértice de
b
β γ B y el ángulo γ se escribe en el vértice de C.
B a C Los lados que están opuestos a los vértices ABC y los
escribimos con una letra minúscula abc.

Este tipo de triángulos los podemos resolver utilizando la ley de senos o la ley de
cosenos.

La fórmula para la ley de senos es:

sin  sin  sin  a b c


  no hay diferencia si la tomas así:   pero no
a b c sin  sin  sin 
las puedes mezclar.

Ejemplos

1.El primer caso es de dos ángulos y un lado.


Determina las partes restantes del triángulo si   20 ,  130 y b = 6.
Procedimiento: ordena los datos del problema como se te indica a continuación.
A
c 130° b=6
  130° a = 13.44 20°
B a C
  20° b=6

  30° c = 8.77

152
La suma de los ángulos internos de cualquier triángulo = 180°

  180  130  20  30

1. Observamos que tenemos los valores de  y b por lo que las colocamos en


nuestra fórmula y buscamos el lado a.

sin130 sin 20

a 6 despejamos a
sin130  6
a
sin 20

 13.44  a

Tomamos de nuevo los datos que tenemos seguros del problema que son  y b,
porque pude haberme equivocado en la respuesta anterior y tener esta mala
también.

sin 30 sin 20

c 6
sin 30  6
c
sin 20  8.77  c

2. Determine los lados y ángulos faltantes del triángulo ABC, si


a  16, mBAC  35o y mABC  65o .
B
a=16 65o
c
C
b 35o
Solución A
Como la suma de las medidas de los ángulos internos de un triángulo es 180 o,
para calcular la medida del ángulo faltante
se procede así: 180o  (35o  65o )  80o . De donde mACB  80 o .

153
Luego utilizando la ley senos dos veces se despejan las medidas desconocidas de
los lados.

sen 35o sen65o sen65o  16 0,91  16


 b   25,54
16 b sen 35o 0,57
sen 80o sen35o sen80o  16 0,98  16
 c   27,51
c 16 sen 35o 0,57

3. Determine BC y mABC en el triángulo ABC, si


AB  10, mBAC  37 o y mBCA  45o .

B
Solución
10
37o 45o
A C

Como la suma de los ángulos internos de un triángulo es 180o,


mABC  180 o  (37 o  45o )  98 o .

Hasta este momento no se conoce el significado de sen 98o , aunque usted podría
usar una calculadora para determinar este valor. Por esta razón no se usará este
ángulo en la ley de senos para calcular BC.

sen 37 o sen 45o 10  sen 37 o 10  0,6


Se tiene entonces:   BC    8,57
BC 10 sen 45o 0,7
Así BC es aproximadamente 8,57 y mABC  98 o .
4. Para determinar la distancia entre dos puntos, A y B, en lados opuestos de un
lago, Raquel ubicó un punto C a una distancia de 28m, al mismo lado donde está
marcado el punto A. Luego obtuvo que mBAC  85o y mACB  69o . Como el
triángulo que formó en su estrategia resultó acutángulo decidió utilizar la ley de
senos para despejar la distancia buscada.

154
Primero tuvo que calcular la medida del ángulo ABC:

180o  (85o  69 o )  26 o

Luego planteó y resolvió la ecuación:

sen 69 o sen 23o 28  sen 69 o


 x 
x 28 sen 23o
28  0,93
x  66,77
0, 39

Raquel obtiene que la distancia entre los puntos A y B es de aproximadamente


66,77m.

5. Un poste telegráfico que está inclinado formando un ángulo de 11o con la vertical,
emite una sombra de 8m de largo sobre el suelo horizontal cuando el ángulo de
elevación al sol es de 23o. Determine la longitud del poste.

Solución

Dado que la medida del ángulo ABC es: mABC  90 o  11o  79 o ,


mBAC  180  (79  23 )  78 .
o o o o

Suponga que x es la longitud del poste y utilizando la ley de senos se despeja:

sen 78 o sen 23o 11  sen 23o


 x 
11 x sen 78 o
11  0, 39
x  4, 38
0,98
Finalmente se obtiene que la longitud del poste es aproximadamente 4,38m.

6. Tres amigos se sitúan en un campo de fútbol. Entre Alberto y Berto hay 25


metros, y entre Berto y Camilo, 12 metros. El ángulo formado en la esquina de

155
Camilo es de 20º. Calcula la distancia entre Alberto
y Camilo. El esquema de la situación sería algo así:
tenemos al menos una pareja ángulo-lado opuesto.
Para hallar la medida del lado que nos falta, nos
basta recurrir al teorema del seno. El problema es
que el ángulo opuesto al lado AC tampoco lo
sabemos, algo que tiene fácil solución si primero
aplicamos el teorema del seno para hallar el ángulo
A y después deducir la medida de B. 25/sen20º = 12/senA 73,10 = 12/senA senA
= 12/73,10 sen A = 0,16 A = 9,45º
Como los tres ángulos deben sumar 180º, B debe valer 150,55º. Ahora ya tenemos
todo lo necesario para volver a usar el teorema del seno y hallar la distancia AC:
25/sen20º = AC/sen150, 55º 73,10 = AC/0,49 AC = 73,10·0,49 = 35,94m

Ejercicios
1. Determine en cada caso el valor de la variable " x" y el m q
utilizando la ley de senos.

156
2) PROBLEMAS

1) Dos piedras se encuentran a la orilla de una playa a una distancia uno de otro
de 1.8 Km. en los puntos A y B, y se encuentra una boya situada en un punto
C. Si la piedra A mide un ángulo CAB igual a 79.3° y el que está en B mide
un ángulo CBA igual a 43.6°, ¿a qué distancia está la bolla de la costa?

2) Un poste forma un ángulo de 79° con el piso. El ángulo de elevación del sol
desde el piso es de 69°. Encuentre la longitud del poste si su sombra es de
5.9 m.

3) Si medimos los ángulos de elevación de una montaña desde lo más alto y


desde la base de una torre de 20 metros de alto y éstos son 38.5° y 40.2°
respectivamente ¿Cuál es la altura de la montaña?

4) Un hombre de 5 pies 9 pulgadas de altura se para en un andén que se inclina


hacia abajo con un ángulo constante. Un poste vertical de luz situado
directamente detrás de él proyecta una sombra de 18 pies de largo. El ángulo
de depresión desde la mayor altura del hombre, hasta la punta de su sobra
es de 31° encuentre el ángulo  , como se muestra en la figura, formado por
el andén y la horizontal.

35°

Sombra

5) Si el hombre del problema anterior esta a 22 pies del poste de luz sobre el
andén, encuentre la altura del poste.

157
6) En cada caso determine el valor que se le solicita.

7) Problemas

1. Sea ABC un triángulo rectángulo en A. Si el segmento AB mide 20 cm. y el ángulo


, opuesto a ese lado, mide 42º. Calcula:

a) el lado AC
b) el lado BC
c) el ángulo 

158
2. Si ABC es un triángulo rectángulo en A y los segmentos AB y AC miden 2 m. y 4
m., respectivamente. Calcula:

a) el lado BC
b) el ángulo ABC
c) el ángulo ACB

3. Si MNO es un triángulo rectángulo en M y los lados NO y MO miden 8 m. y 6 m.,


respectivamente. Calcula:

a) el lado MN
b) el ángulo MNO
c) el ángulo MON

4. La sombra que proyecta un árbol de 3,4 m. sobre el piso horizontal mide 4,3 m.
¿Cuál es la medida del ángulo que hace la horizontal con la línea que une los dos
puntos extremos, de la sombra y del árbol?

5. Un avión sale de un aeropuerto y se eleva manteniendo un ángulo constante de


10º hasta que logra una altura de 6 km. Determina a qué distancia horizontal del
aeropuerto se encuentra en ese momento.

6. Una persona se encuentra en la ventana de su apartamento que está situada a


8 metros del suelo y observa el edificio de enfrente de la siguiente manera: la parte
superior, con un ángulo de elevación de 35º y la parte inferior, con un ángulo de
depresión de 43º. Determina la altura del edificio de enfrente.

8) Considere el Δ 𝐴𝐵𝐶. 𝐵
72°

40°
𝐴 35 𝐶

De acuerdo con los datos de la figura, determine la medida de 𝐶𝐵

159
9) Considere el Δ 𝐴𝐵𝐶.
𝐵

58° 54°
Determine 𝐵𝐶 𝐴 8 𝐶

10) Considere el Δ 𝐴𝐵𝐶. 𝐵


45°

95° 𝐷

𝐴 12 𝐶

De acuerdo con los datos de la figura, ¿cuál es aproximadamente la medida de 𝐶𝐷?

𝑃
11. Considere el Δ 𝑃𝑄𝑅.

14

35° 68°
𝑄 𝑅

De acuerdo con los datos de la figura, ¿cuál es aproximadamente la medida de 𝑄𝑅?

12) En un Δ 𝐴𝐵𝐶. Si 𝑚 ∡ 𝐴𝐶𝐵 = 108°, 𝑚 ∡𝐶𝐴𝐵 = 22° y 𝐴𝐵 = 12, entonces, determine


la medida de 𝐴𝐶

160
13) Sea él Δ 𝐴𝐵𝐶. Si 𝑚 ∡𝐴𝐶𝐵 = 80°, 𝑚 ∡𝐶𝐴𝐵 = 42° y 𝐴𝐵 = 10, entonces
determine la medida de 𝐴𝐶

14) Calcule la distancia d de la luz proyectada en el suelo según la imagen

15) Para determinar la distancia entre dos puntos 𝐴 y 𝐵 que se encuentran en las
orillas opuestas de un río, se traza un segmento de recta desde 𝐴 hasta un punto
𝐶 de longitud 240m y se determina que 𝑚 ∡𝐵𝐴𝐶 = 58° y 𝑚 ∡𝐴𝐶𝐵 = 54°. ¿Cuál es
aproximadamente la distancia entre 𝐴 y 𝐵?

16) Para construir un puente sobre un río, es necesario conocer la distancia entre
los puntos 𝐴 y 𝐶 que se encuentran en las orillas opuestas de un río, para ello se
ubica un punto 𝐵 a 25 𝑐𝑚 de 𝐴 de tal manera que 𝑚 ∡ 𝐵𝐴𝐶 = 59° y 𝑚 ∡ 𝐴𝐶𝐵 = 43°.
¿Cuál es aproximadamente la distancia entre 𝐴 y 𝐶?

161
162
Tema 4 Visualización Espacial

Situación Problema

La siguiente imagen representa una tienda de campaña, que cuando


está cerrada, tiene forma de pirámide regular de base cuadrada.

Si la tienda tiene 1,5 m de alto y el lado


de la base mide 1,5 m, entonces,
¿cuánto mide aproximadamente, en
metros cuadrados, la superficie total de
la tienda (incluyendo la base)?

La Clave

Visualización espacial

Pirámides rectas
Sus elementos son:
 Vértice o cúspide: el punto más alto de
la pirámide.
 Arista: es el segmento en el que se
unen dos caras laterales (las
triangulares).
 Altura: es la distancia vertical desde la
base hasta la cúspide.
 Apotema: es la altura del triángulo
formado en cada cara lateral.
Se va a trabajar únicamente con dos casos
de pirámides: con la base cuadrada o cuando
la base es un triángulo equilátero. Pero,
como ambos casos son diferentes, se
estudian por separado:

163
1) Con base cuadrada
Si la medida del lado de un lado de la base se representa por 𝑙 y la medida de la
altura se representa con ℎ, para calcular la apotema de la pirámide se tiene la
siguiente figura:
De esta manera, se puede aplicar el teorema de
Pitágoras para determinar la medida de la apotema:
𝑙 2
ℎ + ( ) = (𝑎𝑝𝑜𝑡𝑒𝑚𝑎)2
2
2
Así, con las medidas del lado de la base y de la
apotema de la pirámide se pueden calcular el área
basal, el área lateral y el área total de la pirámide.
Para el área basal, basta con calcular el área del
cuadrado:
𝐴𝐵 = 𝑙 2
El área lateral se obtiene al calcular el área de los
cuatro triángulos formados en cada uno de los lados
del cuadrado, donde la base corresponde a la
medida del lado de la base y la altura corresponde a la medida de la apotema de
la pirámide:
𝑙 ∙ 𝑎𝑝𝑜𝑡𝑒𝑚𝑎 4 ∙ 𝑙 ∙ 𝑎𝑝𝑜𝑡𝑒𝑚𝑎
𝐴𝐿 = 4 ∙ = = 2 ∙ 𝑙 ∙ 𝑎𝑝𝑜𝑡𝑒𝑚𝑎
2 2
El área total se calcula sumando el área basal con el área lateral:
𝐴𝑇 = 𝑙 2 + 2 ∙ 𝑙 ∙ 𝑎𝑝𝑜𝑡𝑒𝑚𝑎

Ejemplo, se puede resolver el siguiente problema:

Ana le obsequió a su tía un regalo en una envoltura con forma de


pirámide recta de base cuadrada. Si el lado de la base es 5 𝑐𝑚 y la
altura de la pirámide mide 6 𝑐𝑚, entonces, ¿cuál es, en centímetros
cuadrados la cantidad mínima de cartulina que contiene esa
envoltura?

Como el problema se trata sobre la cartulina necesaria para la envoltura del


regalo, se debe determinar el área total de la pirámide. Para esto se tiene que la
medida del lado de la base es 5 y la de la altura es 6, por lo que se tiene:
5 2
6 + ( ) = (𝑎𝑝𝑜𝑡𝑒𝑚𝑎)2
2
2
𝑎𝑝𝑜𝑡𝑒𝑚𝑎 = √42,25 = 6,5
Así, ya se pueden calcular las áreas solicitadas:
𝐴𝐵 = 52 = 25
𝐴𝐿 = 2 ∙ 𝑙 ∙ 𝑎𝑝𝑜𝑡𝑒𝑚𝑎 = 2 ∙ 5 ∙ 6,5 = 65
𝐴𝑇 = 25 + 65 = 90
Finalmente, la respuesta solicitada es 90 centímetros cuadrados.

164
2)Con base triangular
Si la medida del lado de un lado de la base se representa por 𝑙 y la medida de la
altura se representa con ℎ, para calcular la apotema de la pirámide se tiene la
siguiente figura:
De esta manera, se puede aplicar el teorema de
Pitágoras para determinar la medida de la
apotema:
2
2
𝑙√3
ℎ +( ) = (𝑎𝑝𝑜𝑡𝑒𝑚𝑎)2
6
Así, con las medidas del lado de la base y de la
apotema de la pirámide se pueden calcular el área
basal, el área lateral y el área total de la pirámide.
Para el área basal, basta con calcular el área del
triángulo:
𝑙 2 √3
𝐴𝐵 =
4
El área lateral se obtiene al calcular el área de los
tres triángulos formados en cada uno de los lados del triángulo equilátero, donde
la base corresponde a la medida del lado de la base y la altura corresponde a la
medida de la apotema de la pirámide:
𝑙 ∙ 𝑎𝑝𝑜𝑡𝑒𝑚𝑎 3 ∙ 𝑙 ∙ 𝑎𝑝𝑜𝑡𝑒𝑚𝑎
𝐴𝐿 = 3 ∙ =
2 2
El área total se calcula sumando el área basal con el área lateral:
𝑙2 3 3 ∙ 𝑙 ∙ 𝑎𝑝𝑜𝑡𝑒𝑚𝑎

𝐴𝑇 = +
4 2
Ejemplo, se puede resolver el siguiente problema:

¿Cuál es el área total de una pirámide de base triangular si la


medida del lado de su base es 6 y la medida de su altura es 2√6?

Como se tiene que la medida del lado de la base es 6 y la medida de la altura es


2√6, se obtiene que:
2
2 6√3
(2√6) + ( ) = (𝑎𝑝𝑜𝑡𝑒𝑚𝑎)2
6
24 + 3 = (𝑎𝑝𝑜𝑡𝑒𝑚𝑎)2
27 = (𝑎𝑝𝑜𝑡𝑒𝑚𝑎)2
𝑎𝑝𝑜𝑡𝑒𝑚𝑎 = √27 = 3√3

165
Así, ya se pueden calcular las áreas solicitadas:
62 √3 36√3
𝐴𝐵 = = = 9√3
4 4
3 ∙ 6 ∙ 3√3 54√3
𝐴𝐿 == = = 27√3
2 2
𝐴𝑇 = 9√3 + 27√3 = 36√3
Finalmente, la respuesta solicitada es 36√3.

En la Pirámide es importante considerar la siguiente información

Nótese que internamente se forman dos triángulos rectángulos

Con este triángulo se pueden obtener


cualquiera de los datos indicados. Obsérvese
que la arista lateral corresponde a la
hipotenusa.

Con este triángulo se pueden obtener cualquiera de los


datos indicados. Obsérvese que la altura de la cara
corresponde a la hipotenusa.

166
Ejemplos
1. Calcule la apotema de una pirámide de base cuadrada si la altura de la
pirámide mide y el lado de la base mide 12cm

Recordar

Se tiene

2. Calcula el área lateral, total de una pirámide cuadrangular de 10 cm de


arista básica y 12 cm de altura.

167
Nota

Área Total Área lateral Área Basal


Se averigua el área AB = l. l
AT = AB + AL de uno de los (cuando la base es un
triángulos que polígono de 5 o más lados
forman la pirámide, (en esta fórmula la “a” es
y se multiplica por la apotema de la base, no
la cantidad de de la pirámide)
triángulos que hay.

3. Hallar el área total de una pirámide cuadrangular cuya arista de la base mide
10, la altura de 12 cm y un Apotema del poliedro de 13 cm.

Nos enfocamos en la forma de la base de la pirámide para despejar estas fórmulas.


El problema indica que es una pirámide cuadrangular con las siguientes medidas

168
Área de la base es Área Lateral

10  13
A  10  Al  4
2

2
Ab  100cm
Al  260cm 2

Área total 100 + 260 = 360cm

4.Hallar el área total y de una pirámide regular triangular cuyas medidas son las
siguientes:

Obtengamos primero el área lateral (el de las tres caras triangulares, sin la base),
coloreadas en la figura de abajo.
Recuerda que en una pirámide regular la altura de cada uno de los triángulos
laterales (caras), llamada apotema del poliedro (Ap.), es igual a la altura del
triángulo lateral.

lado de la base  apotema


Al  3
2
6  12
Al   3  108cm2
2

169
Y ahora el área de la base. Para ello en la fórmula general vamos a sustituir por
la fórmula para obtener el área de un triángulo, ya que la base es un triángulo
equilátero.

l2 3
Ab 
4
Área del triángulo Equilátero
 6
2
3
Ab   15,57cm 2

A total = 15,57 + 108 =123,57

Ejemplos

170
Tipos de pirámide rectangular

Existen dos tipos de pirámide rectangular:


 Pirámide rectangular recta: la pirámide es recta cuando todas sus caras
laterales son triángulos isósceles. En este caso, la altura o recta
perpendicular al plano de la base que pasa por el vértice (o ápice) de la
pirámide corta a la base por el centro del rectángulo.
 Pirámide rectangular oblicua: la pirámide es oblicua cuando no todos
los triángulos laterales son isósceles y la altura o recta perpendicular al plano
de la base no corta por el centro del rectángulo.

Área de la pirámide rectangular


El cálculo del área de la pirámide rectangular cambia según si la pirámide
es recta u oblicua.
Área de la pirámide rectangular recta

puede calcularse sabiendo los lados diferentes de la base (a y b) y la altura de


la pirámide (h). Su fórmula es:

171
Ejercicios

1) Si la altura de una pirámide es de 20 𝑐𝑚 y su base es un cuadrado de 18 𝑐𝑚


de lado, entonces determine el área lateral de la pirámide.

2) La base de una pirámide es un cuadrado cuyo lado mide 10m y la altura de


la pirámide es 21m ¿Cuál es el área total de la pirámide?

3) ¿Cuál es el área lateral de una pirámide recta de base cuadrada, si el lado de


la base mide 20cm y la altura de la pirámide es 24cm?

4) De acuerdo con los datos de la figura, determine el área lateral (en cm2) de
la pirámide cuadrangular.

172
5) Hallar el área total de una pirámide cuadrangular regular cuyo lado de la base
mide 4 cm. y cuya altura tiene 10 cm.

6) Calcula la superficie total de una pirámide cuadrangular recta, sabiendo que el


lado de la base es 4 m y la altura es 6 m.

7) La Pirámide de Keops tiene base cuadrada con un lado de 232,805 m y altura


148,208 m. Quedándote sólo con las unidades en metros, calcula su superficie
lateral.

173
8) Determine el área total ( área de la base más área lateral) de una pirámide, si
la base es un triángulo equilátero cuyo lado mide 10cm y la apotema de la pirámide
mide 8cm

9) Calcule el área total de una pirámide de base, un triángulo equilátero, si la


apotema de la pirámide mide 12cm y el lado de la base mide 8cm

10) Calcule el área total de una pirámide, si la base es un triángulo equilátero


cuyo lado mide 18cm, la apotema de la base mide 3 3cm y la altura de la
pirámide 12cm

11) Identifique la base, las caras laterales, la altura, las apotemas y el


ápice o cúspide de las siguientes pirámides rectas

174
12)Una pirámide triangular cuya base es un triángulo equilátero de lado 1.5 cm,
tiene una altura de 3.6 cm y la apotema de la base mide 0.43 cm. Calcula el área
de dicha pirámide redondeando a dos cifras decimales.

Situación Problema

Considere los datos de la siguiente figura, que


representa la caja en la que se empacan pequeños frascos de tinta
negra:

6 cm

Tinta
negra

4 cm

De acuerdo con los datos de la figura anterior, si la caja tiene forma de prisma
recto de base cuadrada, entonces, ¿cuánto mide la superficie lateral de la caja, en
centímetros cuadrados?

175
La Clave

Prismas rectos

Sus elementos son:

 Dos bases iguales que no


necesariamente son cuadriláteros.
 Caras laterales: rectángulos que se
forman en cada uno de los lados de
cualquier base.
 Arista: segmento en el que se unen
dos caras laterales.
 Altura: distancia vertical u horizontal
que separa las dos bases.
Se va a trabajar con tres tipos de prismas: cuadrados, rectangulares y cuando las
bases son triángulos equiláteros.

1) Prismas cuadrados
Estos prismas se caracterizan porque cada una de sus bases es cuadrada. El área
basal, a diferencia de las pirámides, considera las dos bases, por lo cual se
diferencia el área de una base (𝐴𝑏 ) del área basal (𝐴𝐵 ), que incluye el área de las
dos bases.

Así, se tiene que:

𝐴𝑏 = 𝑙 2

𝐴𝐵 = 2 ∙ 𝐴𝑏 = 2 ∙ 𝑙 2

Para el área lateral, se tienen cuatro rectángulos cuya base corresponde a un lado
del cuadrado y cuya altura corresponde a la altura del prisma, por lo que:

𝐴𝐿 = 4 ∙ 𝑙 ∙ ℎ

Y así, el área total del prisma corresponde a la suma del área basal y el área lateral:

𝐴𝑇 = 2 ∙ 𝑙2 + 4 ∙ 𝑙 ∙ ℎ

176
Ejemplo, se puede resolver el siguiente problema:

En un prisma recto de base cuadrada, el área de una de sus bases es


32 𝑐𝑚2, y la medida de la altura del prisma es el doble de la longitud
de la diagonal de la base, entonces, ¿cuál es el área total del prisma?

Antes de presentar la respuesta, es necesario aclarar que 32 𝑐𝑚2 corresponde al


área de una de las bases, es decir, 𝐴𝑏 . Así, se debe despejar la medida del lado de
la base:

𝐴𝑏 = 𝑙 2 = 32

𝑙 = √32 = 4√2

De igual manera, es necesario calcular la medida de la diagonal del cuadrado. Pero,


para calcularla, se tiene la siguiente figura:

Aplicando el teorema de Pitágoras, se obtiene que


2 2
(4√2) + (4√2) = (𝑑𝑖𝑎𝑔𝑜𝑛𝑎𝑙)2

32 + 32 = (𝑑𝑖𝑎𝑔𝑜𝑛𝑎𝑙)2

64 = (𝑑𝑖𝑎𝑔𝑜𝑛𝑎𝑙)2

𝑑𝑖𝑎𝑔𝑜𝑛𝑎𝑙 = √64 = 8

Y, como la altura del prisma mide el doble de la


diagonal de la base,

ℎ = 2 ∙ 𝑑𝑖𝑎𝑔𝑜𝑛𝑎𝑙 = 2 ∙ 8 = 16

De esta manera, se pueden sacar las áreas faltantes:

𝐴𝐵 = 2 ∙ 𝐴𝑏 = 2 ∙ 32 = 64

𝐴𝐿 = 4 ∙ 4√2 ∙ 16 = 256√2

𝐴𝑇 = 64 + 256√2

177
2) Prismas rectangulares
Como su nombre lo indica, cada base de este tipo de prismas corresponde a un
rectángulo. Como sus lados son diferentes (largo 𝑙 y ancho 𝑎), esto se debe
considerar a la hora de calcular las áreas. Así:

𝐴𝑏 = 𝑙 ∙ 𝑎

𝐴𝐵 = 2 ∙ 𝐴𝑏 = 2 ∙ 𝑙 ∙ 𝑎

Para el área lateral, se tienen dos rectángulos cuya base es el largo del rectángulo
basal y dos rectángulos cuya base corresponde al ancho del rectángulo basal, y los
cuatro rectángulos laterales tienen como altura a la altura del prisma, por lo que:

𝐴𝐿 = 2 ∙ 𝑙 ∙ ℎ + 2 ∙ 𝑎 ∙ ℎ

Y así, el área total del prisma corresponde a la suma del área basal y el área lateral:

𝐴𝑇 = 2 ∙ 𝑙 ∙ 𝑎 + (2 ∙ 𝑙 ∙ ℎ + 2 ∙ 𝑎 ∙ ℎ)

Ejemplo, se tiene el siguiente problema:

Determine la cantidad de plástico, en centímetros cuadrados, necesaria para


envolver una candela en forma de prisma rectangular con las siguientes
dimensiones: 2 𝑐𝑚 de ancho, 4 𝑐𝑚 de largo y 6 𝑐𝑚 de altura.

Como las medidas de las dimensiones están claramente dadas en el problema, se


puede hacer el cálculo directo de las áreas:

𝐴𝑏 = 4 ∙ 2 = 8

𝐴𝐵 = 2 ∙ 𝐴𝑏 = 2 ∙ 8 = 16

𝐴𝐿 = 2 ∙ 4 ∙ 6 + 2 ∙ 2 ∙ 6 = 48 + 24 = 72

𝐴𝑇 = 16 + 72 = 88

Así, se tiene que, en total, se necesita 88 𝑐𝑚2 de plástico para cubrir la candela.

3) Prismas triangulares
Este tipo de prismas tiene en cada base un triángulo equilátero. Así, se tiene que:

𝑙 2 √3
𝐴𝑏 =
4

𝑙 2 √3 𝑙 2 √3
𝐴𝐵 = 2 ∙ 𝐴𝑏 = 2 ∙ =
4 2

178
Para el área lateral, se tienen tres rectángulos cuya base corresponde a un lado
del triángulo equilátero y cuya altura corresponde a la altura del prisma, por lo
que:

𝐴𝐿 = 3 ∙ 𝑙 ∙ ℎ

Y así, el área total del prisma corresponde a la suma del área basal y el área lateral:

𝑙 2 √3
𝐴𝑇 = +3∙𝑙∙ℎ
2

Ejemplos, se tiene el siguiente problema:

Determine la cantidad de cartón, en centímetros cuadrados, necesaria para


elaborar la envoltura de un chocolate, que tiene forma de prisma triangular
en el que la medida del lado de la base es de 2 𝑐𝑚 y que tiene 30 𝑐𝑚 de altura.

Aplicando las fórmulas, se tiene que:

22 √3 4√3
𝐴𝑏 = = = √3
4 4

𝐴𝐵 = 2 ∙ 𝐴𝑏 = 2 ∙ √3 = 2√3

𝐴𝐿 = 3 ∙ 2 ∙ 30 = 180

𝐴𝑇 = 2√3 + 180 ≈ 183,464102 ≈ 183,5

De esta manera, es claro que, aproximadamente, se necesita un total de 183,5 𝑐𝑚2


de cartón para envolver el chocolate.

179
1) Paralelepípedo (Prisma recto de base rectangular)

Área Total Área lateral Área Basal Área de una


base
Se suman las Se obtiene Se suman Se averigua
áreas de los 6 el área de las áreas de el área del
rectángulos los 4 los rectángulo
que forman el rectángulos rectángulos de la base.
paralelepípedo que están a de la base
los lados del inferior y la
prisma. base
Luego se superior
suman (ambas son
iguales)

Ejemplos

1. De acuerdo con los datos de la figura, ¿cuál es el área del prisma rectangular
recto?

6
3
3
Solución
𝐴𝑇 = 𝐴𝐵 + 𝐴𝐿 = 2 ∙ 3 ∙ 3 + 4 ∙ 3 ∙ 6 = 90. Luego, el área total mide 90. ∎

2. El área de un prisma rectangular se calcula sabiendo los lados de la


base rectangular (a y b) y su altura (h).
Un prisma rectangular (u ortoedro) es un poliedro cuya superficie está
formada por dos rectángulos iguales y paralelos llamados bases y por cuatro
caras laterales que son también rectángulos paralelos e iguales dos a dos.
Su área se calcula por la siguiente fórmula:

180
Sea un prisma rectangular de dimensiones conocidas, siendo los lados contiguos
de la base a=3 cm y b=1,5 cm y la altura h=4 cm.

¿Cuál es su área?

Su área se calcula mediante la suma de los seis


rectángulos de su superficie, que al seria guales dos a dos,
será el doble de la suma de los tres rectángulos diferentes.
Ejemplo del cálculo del área del prisma rectangular
Y se obtiene que el área de este prisma rectangular es de
45 cm2.

La fórmula del área del prisma rectangular se obtiene como resultado de


sumar el área de los seis rectángulos de la superficie del prisma. És decir,
el área del prisma rectangular es el sumatorio del área de las dos
bases B (rectángulos PTWR y QUZS),los
dos rectángulos laterales R1 (PQSR y TUZW) y los dos R2 (RSZW y PQUT).

181
Aplicando la fórmula del área del rectángulo, el área de
los rectángulos B, R1 y R2son:

El área del prisma rectangular es la suma del área de los seis rectángulos (dos
bases B, dos R1 y dos R2). Por tanto, será la suma del área de los tres rectángulos
multiplicada por dos, obteniendo la fórmula.

3. Hallar el área total de un prisma triangular cuya base mide 10 x 43 y con una
altura de 42 cm; si la altura el prisma mide 60 cm.

Nos enfocamos en la forma de las bases del prisma para despejar estas fórmulas.
El problema indica que es un prisma triangular con las siguientes medidas.

182
Obtengamos primero el área lateral (el de las tres caras) (para recordar cómo se
obtiene el área de un rectángulo).

Al  perimetro de la base  altura


Al  (10  43  43)  60
Al  96  60
Al  5760cm 2

Y ahora el área de las bases. Para ello en la fórmula general vamos a


sustituir por la fórmula para obtener el área de un triángulo, ya que la base
es triangular; y después el resultado se multiplicará por 2 (ya que el prisma tiene
dos bases iguales, en este caso, triángulos isósceles). Es el área coloreada. (Para
recordar cómo se obtiene el área de un triángulo).

AB  2
 base  altura 
2

AB  2
10  42 
2

AB  2
 420   420cm2
2
AT  AL  AB
AT  5760  420
At  6180cm 2

4. Calcula el área lateral y el área total de un paralelepípedo de 25 cm de alto,


15 cm de ancho y 10 cm de largo. Área lateral:

Hay dos rectángulos de 25 por 15: A=25·15=375 cm2 Hay dos rectángulos de 25
por 10: A=25·10=250 cm2 El área lateral es:
Al = 2 · 375 + 2 · 250 = 1250 cm2
Área total: Las bases son dos rectángulos de 15 por 10:
A = 25 · 15 = 375 cm2 El área total es: At = 1250 + 2 · 150 = 1550 cm2

183
5. Para calcular el área total o parcial de un prisma se puede realizar calculando
por separado cada área de las figuras que forman el prisma o utilizar fórmulas
como se indica a continuación.

6. Calcular el área lateral y el área total de un prisma triangular de 40


centímetros de altura y 25 centímetros de arista de la base.
Área lateral: hay tres rectángulos iguales:
Al = 3 · 40 · 25 = 3000 cm2

Área de la base: un triángulo equilátero. Se aplica el Teorema de Pitágoras

184
h  25  12,5  21, 65
2 2

25  21, 65
Ab   270, 63cm2
2
At  3000  2  270, 63  3541, 27cm2

Ejercicios

1. Halla el área de un prisma triangular de altura 6 cm y base


un triángulo equilátero de lado 5 cm. Redondea a dos
cifras decimales.

2) María regala a su padre un best seller por su cumpleaños. Elige la


encuadernación de tapas duras que tiene forma de prisma rectangular, siendo sus
medidas 18 cm de largo, 12 cm de ancho y 6 cm de grosor. Si sabemos que al
envolverlo un 10% del envoltorio queda oculto por sí mismo, ¿cuál es la cantidad
de papel de regalo gastada?

185
3) De acuerdo con los datos de la figura ¿Cuál es el área total del prisma
rectangular recto?

3cm 8cm

4cm

4) Una madre compra a su hija una caja de sus bombones favoritos. La caja
tiene forma de prisma triangular de 21 cm de larga y 12 cm de lado de la
base. ¿Cuál es la cantidad de papel mínima que se necesita para envolverla?

5) Estoy construyendo una piscina de 5,7 metros de largo, 4 metros de ancho y 1,9
metros de alto. Quiero cubrir las paredes y el fondo con azulejos de forma cuadrada
de 20 cm de lado. ¿Cuántos azulejos necesitaré si aproximadamente se desperdicia
un 10%?

6) Calcula el área total de un prisma triangular de 55 metros de altura y 30


metros de arista de la base

7)Determine el área de la base, el área basal, el área lateral y el área total de un


prisma, si la base es un triángulo equilátero cuyo lado mide 7cm y la altura 11cm

186
8)Determine el área de la base, el área basal, el área lateral y el área total de un
prisma, si la base es un cuadrado cuyo lado mide 13cm y la altura 17cm

9) Calcule el área de la base, el área basal, el área lateral y el área total de un


prisma, si la base es un rectángulo que mide de ancho 15cm, de largo 32cm y la
altura 12cm

10) Determine el área de la base, el área basal, el área lateral y el área total de un
prisma, si la base es un rectángulo que mide de ancho 17cm,de largo 36cm y de
altura 10c

11) Hallar el área total de un prisma triangular regular cuya base tiene 8 pulgadas
de lado y la altura del prisma es igual a 17 pulgadas.

Ejercicios de la Unidad II Geometría


Selección Única

1). De acuerdo con los datos de la figura, si el ABCE es un cuadrado,

entonces el perímetro del trapecio ABDE es:

A) 36 C) 42
B) 26  2 7 D) 32  2 7

187
2) De acuerdo con los datos de la figura, ¿cuál es la distancia del punto “ A ” al
punto “ B ”?

Cada representa un cuadrado de 1 cm de lado.

A) 4 cm C) 8 cm

B) 4 2 cm D) 2 10 cm

3) Una persona camina 6 km hacia el este y 8 km hacia el norte, de manera

consecutiva. ¿A qué distancia está la persona del punto de partida?

A) 2 km C) 10 km

B) 2 7 km D) 2 19 km

188
4) Considere el siguiente contexto:

La casa de Pedro
Pedro desea construir una casa cuyas dimensiones se muestran en la
figura. Para construir el techo utilizará láminas de zinc de 3,66 m de

largo AD , de manera que el punto más alto del techo esté centrado
con respecto al frente de la casa.

Si Pedro desea que el sobrante CD del techo tenga una longitud de 0,45 m , entonces,

¿cuál debe ser aproximadamente la medida de la altura AB ?

A) 1,14 m C) 1,25 m

B) 1,30 m D) 2,10 m
2
5) La cantidad radianes equivale a:
3

A) 60 C) 120
B) 240 D) 270

189
6) De acuerdo con los datos de la figura, el valor de sen  es:

3
A) cm
5

4
B) cm
5

5
C) cm
4
4
D) cm
3

7) Considere las siguientes proposiciones referidas a los ángulos agudos  y 

de un triángulo rectángulo escaleno:

I. sen   cos 

1
II. tan  
tan 

¿Cuáles de ellas son verdaderas?


A) Ambas C)Ninguna
B) Solo la I C)Solo la II

8) De acuerdo con los datos de la figura, la medida aproximada del AB es:

A) 6,8 cm

B) 8,3 cm

C) 9,8 cm

D) 17,1 cm

190
9) De acuerdo con los datos de la figuras, si la m AD  100 m , entonces, ¿cuál es

aproximadamente la medida del DC ?

A) 160, 00 m
B) 260, 00 m
C) 276,98 m
D) 376,98 m

10) En un rombo, la medida de uno de los ángulos internos es 80 , y la


medida de cada lado es 10 cm , ¿cuál es aproximadamente la medida de la
diagonal menor del rombo?

A) 7, 66 cm c) 9,84 cm
B) 12,86 cm D) 19, 70 cm

11) Una escalera de 4 m de longitud está apoyada a una pared de tal manera
que forma con el suelo un ángulo de 60 . ¿Cuál es aproximadamente la altura
que alcanza la escalera con respecto al suelo?

A) 15 m C) 2, 31 m
B) 36, 46 m D) 4, 62 m

191
12) De acuerdo con los datos de la figura, si el ángulo de elevación con que
observa la persona el punto más alto del árbol es de 60 y el ángulo de
depresión con que observa la base del árbol es de 25 , entonces la altura del
árbol es aproximadamente:

A) 1,10 m
B) 2,58 m
C) 2,81 m
D) 4,40 m

13) En la siguiente figura se muestra el recorrido que realiza un teleférico


para transportar personas desde el punto A hasta la cima de una montaña
en el punto C . De acuerdo con los datos de la figura, la distancia que recorre
el teleférico es aproximadamente:

A) 663,1 m

B) 993,0 m

C) 1358,8 m

D) 1844,3 m

14) Sea una pirámide recta de base cuadrada, si el área de su base es 9 cm 2

y cada cara lateral posee un área de 7,5 cm 2 , entonces, ¿cuál es la medida de

la apotema de la pirámide?

A) 1,50 cm C) 4, 00 cm

B) 5,00 cm D) 5, 22 cm

192
15) Sea una pirámide recta cuya base es un triángulo equilátero. Si la
medida del lado de la base es 12 cm y la medida de la apotema de la pirámide

es 2 5 cm , entonces, ¿cuál es el área lateral de la pirámide?

A) 12 5 cm2 C) 36 2 cm2

B) 36 3 cm2 D) 36 5 cm2

16) En una pirámide recta de base rectangular, la medida del largo de la


base es 8 m , la medida del ancho de la base es 6 m . la medida de la apotema

de la pirámide que corresponde al ancho de la base es 41 m , la medida de la

apotema de la pirámide que corresponde al largo de la base es 34 m y la

medida de cada arista lateral es 5 2 m . ¿Cuál es el área total de esa pirámide?

A)  48  34  41  m 2 C)  48  15 2  20 2  m 2

B)  48  8 34  6 41  m 2 D)  48  30 2  40 2  m2

17) ¿Cuál es el área lateral de un prisma recto de base cuadrada, si la


medida de cada lado de la base es 14 cm y la medida de la altura del prisma

es 16 cm ?

A) 224 cm 2 C) 448 cm 2

B) 896 cm2 D) 1344 cm2

18) Feliciano quiere forrar con papel de regalo la parte externa de una caja
con forma de cubo. Si la medida de cada arista es 8 cm , entonces, ¿cuánto

papel necesitará Feliciano?

A) 80 cm2 B) 128 cm2

B) 320 cm 2 D) 384 cm 2

193
19) De acuerdo con los datos del DEF , el área “ A ” del triángulo en términos
de “ x ” corresponde a:

A) A  2x  2
B) A  x2  2x
x2  2
C) A 
2

x2  2x
D) A 
2

20) Considere los datos de la siguiente figura:


¿Cuál es la distancia entre los puntos A y B ?
A) 3, 60

B) 5,38

C) 8,54

D) 9, 43

21) Dados los puntos A  2,3 , B  4,6  y C  0, 1 ubicados en un plano

cartesiano. ¿Cuál es la medida del BC ?


A) 22

B) 33

C) 41

D) 65

194
22) Considere la siguiente figura que ilustra una escalera apoyada en una
pared ¿Cuántos metros de largo mide la escalera?
A) 5

B) 7

C) 12

D) 16

23) Considere los datos de la siguiente figura, en la cual ABCD corresponde


a un cuadrado:

Si BC  8 y DE  14 , entonces, el perímetro del cuadrilátero ABED corresponde a:

A) 38 C) 40
B) 44 D) 48
2
24) Si la medida de un ángulo es radianes, entonces, la medida en grados
3

de dicho ángulo corresponde a:

A) 120
120
B)

1
C)
270


D)
270

195
25) Considere las siguientes proposiciones:

I. sen 30  cos  90  30

II. Si m   45 , entonces, sen   cos 

De ellas, ¿cuál o cuáles son verdaderas?


A) Ambas
B) Ninguna
C) Solo la I
D) Solo la II
3 4
25)Si sen     y cos     entonces, el valor de tan   corresponde a:
5 5

3
A)
4
4
B)
3

5
C)
3

7
D)
10

196
Considere el siguiente contexto para responder las preguntas 26 y 27;

Un niño amarra su cometa, a nivel del suelo (punto A ), con una cuerda. La cuerda
mide 50 metros desde el punto A hasta la cometa y " x " representa la altura a la
que se encuentra la cometa, tal y como se ¡lustra en la siguiente imagen:

26)La longitud desde el punto A al B , corresponde aproximadamente a:


A) 25, 00
B) 30,14
C) 37, 68
D) 39,93

27)La altura a la que se encuentra la cometa, corresponde aproximadamente a:

A) 23,96
B) 24, 07
C) 30, 09
D) 31,94

197
28) Considere la siguiente figura sobre un triángulo ABC :

¿Cuál es aproximadamente la medida de AC ?


A) 7, 00
B) 7, 20
C) 7,93
D) 8,17

Considere la siguiente figura para responder las preguntas 28 y 29:

28)¿Cuál es, aproximadamente, la medida de AB ?

E) 9, 27
F) 10, 00
G) 10,39
H) 12, 00

198
29)¿Cuál es aproximadamente la medida de BC ?
I) 9, 00

J) 9, 27

K) 15,59
L) 18,54
29) Desde la cúspide de una torre de 10 metros de altura se observa con
un ángulo de depresión de 23 , a un joven a nivel del plano de la base de
dicha torre. ¿A qué distancia está el joven de la base de la torre?
a. 4, 24
b. 9, 21
c. 23,56

d. 25,59

30) La altura de un prisma recto de base cuadrada es 10 . Si el lado de la base es 6


, entonces, el área lateral de ese prisma es:
a. 112
b. 120
c. 240

d. 276

31) La altura de un prisma recto de base rectangular es 8 . Si las dimensiones de la


base son 4 de ancho y 5 de largo, entonces, ¿cuál es el área total de dicho prisma?
a. 146
b. 160
c. 184

d. 200

199
Considere el siguiente contexto para responder las preguntas 28 y 29:
La altura de cada cara lateral de una pirámide recta de base cuadrada es 5 , y la
longitud del lado de la base es 6 .
32) La longitud de la apotema de la pirámide corresponde a:
a. 3
b. 4
c. 5

d. 7
33) El área total de la pirámide corresponde a:
a. 44
b. 96
c. 156
d. 180

34) La base de una pirámide recta es un triángulo equilátero. Si la altura de cada


una de las tres caras laterales de la pirámide es 3 y la longitud del lado de la base
es 8 , entonces, ¿cuál es el área lateral de la pirámide?
a. 24
b. 36
c. 72

d. 73

200
201
202
Capítulo III Nuestro primer
desafío matemático,

Relaciones Y Algebra un paso más para


aprender

Habilidades Específicas

Al finalizar el capítulo el estudiante deberá estar en capacidad de:

1. Identifica casos en la que se aplica función cuadrática

2. Efectuar operaciones con expresiones algebraicas

3... Utilizar distintas representaciones para las funciones cuadráticas


4. Utilizar las ecuaciones de primer y segundo grado para resolver problemas

5. Plantear problemas a partir de una situación dada

Conceptos clave
1. Función Cuadrática 4.Discriminante 7.Vertice

2. Ecuaciones 5.Racionalizacion 8.Eje de simetría


Cuadrática
6.Fracciones 9.Intersecciones
3. Factorización algebraicas

203
 Introducción
Al ingresar a este ciclo cada estudiante tiene la habilidad para resolver ecuaciones
sencillas de primer grado, reconocer relaciones de dependencia
entre dos cantidades variables, aplicar la regla de tres,
porcentajes y proporcionalidad directa con el fin de solucionar
problemas, calcular la distancia entre puntos ubicados en un
mapa con escala y realizar operaciones que involucran suma,
resta, multiplicación y división. También, puede comprender el
concepto de variable e identificar cuantitativamente cambios en
la variable. El Tercer ciclo ampliará estas habilidades e incluirá
otras que tienen que ver con el estudio de relaciones de diversos tipos (lineal,
cuadrática, proporcionalidad inversa), así como el uso de distintas representaciones
para las relaciones mencionadas (verbal, tabular, algebraica, gráfica). Las
funciones cuadráticas, que también son casos particulares de relaciones, se
introducen en 9º Año. El enfoque con el que se abordan las funciones lineales y
cuadráticas privilegia la relación entre variables (dependientes e independientes) y
sus distintas representaciones por medio de tablas, expresiones algebraicas y
gráficas. Es un enfoque consistente con toda la preparación recibida desde la
educación Primaria.

Tema 1 Función Cuadrática

Situación Problema
Después de varias mediciones e intentos, una estudiante encontró
una fórmula para averiguar a qué altura (h), en metros, se encuentra
una bola cuando ha transcurrido una determinada cantidad de
segundos (t) después de haberla lanzado al aire con cierta fuerza.

F(x) = –3t2 + 30 t

 Calcule la altura (h), en metros, según la cantidad de segundos indicada.

t=0 t=1 t=2 t=3 t=4

h= h= h= h= h=

204
 Escriba en dónde está la bola cuando han transcurrido 5 segundos.
R/ _________________________

Las funciones cuadráticas son utilizadas en algunas disciplinas como, por


ejemplo, Física, Economía, Biología, Arquitectura. Son útiles para describir
movimientos con aceleración constante, trayectoria de proyectiles, ganancias y
costos de empresas, variación de la población de una determinada especie que
responde a este tipo de función, y obtener así información sin necesidad de recurrir
a la experimentación.

Haciendo clic en la imagen podrás evidenciar


algunas aplicaciones de la función cuadrática en el
deporte, en la naturaleza, en el espectáculo y en
creaciones de hombre.

205
Es con el físico y astrónomo italiano Galileo Galilei (1564-1642) que el concepto de
función cuadrática adquiere relevancia
cuando el descubrió́ la trayectoria que
describían los cuerpos en caída libre y
demostró́ que la velocidad aumentaba
en razón al cuadrado del tiempo, es
decir, V = k . t2. Sus ideas fueron tan
avanza- das para su tiempo que fue
perseguido por la Inquisición, ya que
estas iban en contra de lo establecido
por la Iglesia Católica. Gracias a
Galileo se pudo analizar el recorrido
que des- cribe el lanzamiento de un
proyectil, pues debido a la fuerza de
gravedad, estos no se desplazan en
forma rectilínea, sino que su
movimiento lo que describe es una
parábola. Muchos son los fenómenos
de la naturaleza y de la vida cotidiana
donde las funciones cuadráticas están
presentes, por ejemplo, la trayectoria
que describe el balón cuando un
jugador de voleibol hace un saque, el
movimiento de un conejo cuando este
se desplaza de un lugar a otro, el nado
de los delfines y el salto de un motociclista en un deporte extremo, representan
tras- rectorías parabólicas.

LA Clave
Función cuadrática
La función cuadrática tiene como forma general 𝑓(𝑥) = 𝑎𝑥 2 + 𝑏𝑥 + 𝑐, con
𝑎, 𝑏, 𝑐 números reales y con la condición de que 𝑎 ≠ 0.

Ejemplo,
 𝑚(𝑥) = (2𝑥 − 5) + 𝑥 2 + 3 = 2𝑥 − 5 + 𝑥 2 + 3 = 𝑥 2 + 2𝑥 − 2 tiene la forma 𝑚(𝑥) =
𝑎𝑥 2 + 𝑏𝑥 + 𝑐, por lo que se cataloga como cuadrática.
 𝑝(𝑥) = (𝑡 2 − 5)2 = 𝑡 4 − 10𝑡 2 + 25 no tiene la forma 𝑝(𝑡) = 𝑎𝑡 2 + 𝑏𝑡 + 𝑐, por lo que
no es una función cuadrática.

206
 Situaciones modeladas con funciones cuadráticas
Las ganancias de una empresa están modeladas por la fórmula 𝑔(𝑥) = −𝑥 2 +
4𝑥 − 1, donde 𝑥 corresponde a la cantidad de unidades producidas de cien mil en
cien mil y 𝑔(𝑥) son las ganancias que van de un millón de colones en un millón.
Represente la función cuadrática presentada en el ejemplo.

Ejemplo
1. Como 𝑥 representa unidades producidas, solo se pueden representar valores
naturales, para lo cual se elabora la siguiente tabla:
Valor de 𝒙 Cálculo para 𝒈(𝒙) Par ordenado
2 (0, −1)
0 𝑔(0) = −(0) + 4 ∙ 0 − 1 = −1
2 (1,2)
1 𝑔(1) = −(1) + 4 ∙ 1 − 1 = 2
2 (2,3)
2 𝑔(2) = −(2) + 4 ∙ 2 − 1 = 3
2 (3,2)
3 𝑔(3) = −(3) + 4 ∙ 3 − 1 = 2
2 (4, −1)
4 𝑔(4) = −(4) + 4 ∙ 4 − 1 = −1
2 (5, −6)
5 𝑔(5) = −(5) + 4 ∙ 5 − 1 = −6

Representación tabular de 𝑔(𝑥):


𝑥 0 1 2 3 4 5
𝑔(𝑥) −1 2 3 2 −1 −6

Con estos valores, se puede graficar la función dada, por medio de la figura
adjunta:

207
Interprete Bajo ciertas condiciones la distancia d en metros a la que se
encuentra un objeto por encima del suelo viene dada por la fórmula
d (t) = –2t2 + 20t, donde t es el tiempo en segundos.

2) Sea f  x    x 2  4 x  3 .

Una representación tabular de esta función es la siguiente:

x f(x)
-1 -8
0 -3
1 0
2 1
3 0
4 -3
5 -8
3) Supongamos una ecuación de 2º grado (el exponente de x debe ser 2): y=x2
Vamos a dar valores a la variable independiente x y conseguiremos que la
variable dependiente y tome los suyos:
En primer lugar damos a x el valor 3, luego 2, después 0, seguidamente – 2 y por
fin, – 3. La variable dependiente y recibirá los valores: 9, 4,0, 4 y 9
Podemos escribir:

Colocamos en el eje de coordenadas los puntos:

y luego, unimos esos puntos tal como lo ves en la figura siguiente:

208
4) Representa gráficamente la ecuación de 2º grado: y=2x2

Respuesta:

Solución

Dando valores a x: 2, 1, 0, -1 y -2 obtenemos los de y en la ecuación de 2º


grado: y  2 x 2

Fijados los puntos, los unimos y obtendremos la parábola

209
5) Graficar f(x) = −2x2 + 3x – 3.

Crea la tabla de valores. Observa que en esta tabla, los valores de x aumentan.
Los valores de y aumentan y luego comienzan a disminuir. Esto indica una parábola

Usa la tabla de pares para graficar los puntos

Conecta los puntos lo mejor que puedas, usando una curva suave. Recuerda que
la parábola está compuesta de dos imágenes como en un espejo, entonces si tus
puntos no tienen pares con el mismo valor, querrás incluir puntos adicionales

210
EjerciciosComplete la información que se le pide y trace la gráfica
para la función f(x) = x2 – 2x – 8
Y

1) Tabla de valores:

x f(x)

X | | | | | | | |
x

2.Complete la información que se le pide y trace la gráfica para la función


f(x) = 4x2 + 12x + 9
y
1) Tabla de valores:
x f(x)

| | | | | | | | x

211
3: Complete la información que se le pide y trace la gráfica para la función
f(x) = x2 – 4x + 7
y

2) Tabla de valores:
x f(x)

| | | | | | | | x

4: Complete la información que se le pide y trace la gráfica para la función


f(x) = –4x2 – 8x + 5
y

2) Tabla de valores:
x f(x)

| | | | | | | | x

212
5: Complete la información que se le pide y trace la gráfica para la función
f(x) = –x2 + 4x – 4
y

2) Tabla de valores:
x f(x)

| | | | | | | | x

6Complete la información que se le pide y trace la gráfica para la función


f(x) = –x2 – 6x – 10

y
2) Tabla de valores:

x f(x)

| | | | | | | | x

213
6) Interprete
Bajo ciertas condiciones la distancia d en metros a la que se encuentra un objeto
por encima del suelo viene dada por la fórmula d (t) = –2t2 + 20t, donde t es el
tiempo en segundos.

¿Qué relación observa entre el tiempo y la distancia?


______________________________

¿Cuál es la razón por la que los valores correspondientes a la distancia se


repiten?

_________________________________________________________________

¿Cuál es la altura máxima que alcanza el objeto? ¿En qué tiempo se da?

_________________________________________________________________

214
8) En la figura adjunta se muestra la representación gráfica de los valores
obtenidos en la tabla anterior utilizando el software Graph.

¿Será posible con la gráfica determinar


la altura del objeto a los 3,5 segundos?
¿Cuál será́ la altura máxima? Si su
respuesta es afirmativa, entonces
determine la
altura.____________________

¿Cuál es el tiempo que tarda el objeto


en el aire? ¿Se podrá́ determinar la
altura del objeto a los 20 segundos?
Justifique su
respuesta._______________________
_

¿Cuál de las dos representaciones


brinda más información sobre la
situación planteada, la tabla de valores
o la gráfica? Justifique su respuesta.

215
216
Tema 2 Expresiones algebraicas

Situación Problema
Calcule el área de las siguientes figuras

Análisis de problema

La Clave

FACTORIZACIÓN
Factorizar una expresión algebraica consiste en escribirla como un
producto.
El objetivo de este método es aplicar la siguiente propiedad:

∀𝐚, 𝐛, 𝐜 ∈ ℝ, 𝐚𝐛 + 𝐚𝐜 = 𝐚(𝐛 + 𝐜).

Este método se aplica cuando los términos de un polinomio poseen letras en común
o cuando los números de cada término no son coprimos (máximo común divisor
diferente de1).
Cuando realizamos las multiplicaciones:

1. 2x(x2 – 3x + 2) = 2x3 – 6x2 + 4x


2. (x + 7)(x + 5) = x2 + 12x + 35

entonces vemos que las expresiones de la izquierda son los factores y las de la
derecha son las expresiones a factorizar, es decir, la factorización es el proceso
inverso de la multiplicación.

217
La factorización es de extrema importancia en la Matemática, así es que debes
tratar de entender lo más que puedas sobre lo que vamos a trabajar.

Existen varios casos de factorización:

1. FACTOR COMUN MONOMIO:

Factor común monomio: es el factor que está presente en cada término del
polinomio

Ejemplos

1) ¿cuál es el factor común monomio en 12x + 18y - 24z?

Entre los coeficientes es el 6, o sea, 6·2x + 6·3y - 6· 4z = 6(2x + 3y - 4z)

2) ¿Cuál es el factor común monomio en: 5a2 - 15ab - 10 ac


El factor común entre los coeficientes es 5 y entre los factores literales es a,
por lo tanto
5a2 - 15ab - 10 ac = 5a·a - 5a·3b - 5a · 2c = 5a(a - 3b - 2c)

3) ¿Cuál es el factor común en 6x2y - 30xy2 + 12x2y2


El factor común es “6xy “porque
6x2y - 30xy2 + 12x2y2 = 6xy(x - 5y + 2xy)

4) Factorizar a 5  a 2 b 2 

a) Me pregunto ¿qué letra tiene igual? a

b) ¿Cuál es el exponente más pequeño de la a? 2

c) Entonces escojo el a 2
d) Coloco la a 2 y abro un paréntesis

a 5  a 2b 2  a2 (

e) Divido cada término entre a 2

a 5 a 2b 2
2
 2  a2 (
a a

f) Coloco la respuesta dentro del paréntesis restando los exponentes así:


“No se te olvide que para dividir se copia la base igual y se restan los

218
exponentes”
“No se te olvide cualquier base elevada a la cero es igual a 1.”

Se copia el signo

a 5 a 2b 2
    
 2  a 2 a 5 2  a 2 2 b 2 = a 2 a 3  a 0 b 2 = a 2 a 3  b 2 
a2 a

5) Factorice 10𝑥 2 𝑦 4 − 15𝑥 7 𝑦 3 .

Solución
10𝑥 2 𝑦 4 − 15𝑥 7 𝑦 3
= 5𝑥 2 𝑦 3 (2𝑦 − 3𝑥 5 ).
6) Por ejemplo, al buscar la factorización de 4𝑦𝑧 3 + 6𝑦𝑧 − 8𝑦, se debe proceder de la
siguiente manera:
1. El máximo común divisor de4, 6 y 8 es 2, por lo que este es el valor del
coeficiente numérico del factor común.
2. Para cada variable, se debe tomar el menor de sus exponentes y éste será el
exponente de la variable en el factor común:
Exponente de la variable en cada término
Variable
Primero Segundo Tercero Factor común
𝒚 𝟏 𝟏 𝟏 𝟏
𝒛 𝟑 𝟏 𝟎 𝟎

Por lo tanto, el factor común es 2𝑦 1 𝑧 0 = 2𝑦

3. Se escribe el factor común y a su lado un paréntesis con el otro factor. Cada


término del otro factor corresponde a dividir el original por el factor común.
Se debe recordar que los números se dividen, mientras que los exponentes
se restan, por lo que la factorización del ejemplo que da de la siguiente
manera:
4𝑦𝑧 3 + 6𝑦𝑧 − 8𝑦 = 2𝑦(4 ÷ 2𝑦1−1 𝑧 3−0 + 6 ÷ 2𝑦1−1 𝑧1−0 − 8 ÷ 2𝑦1−1 𝑧 0−0 ) =

2𝑦(2𝑦 0 𝑧 3 + 3𝑦 0 𝑧1 − 4𝑦 0 𝑧 0 ) = 2𝑦(2𝑧 3 + 3𝑧 − 4)

7. Otros ejemplos:

219
Expresión Factor común Factorización
𝟐𝟒𝒙 𝒚 + 𝟏𝟎𝒙𝟐 𝒚 + 𝟏𝟎𝟎𝒙
𝟐 𝟒 𝟐𝒙 𝟐𝒙(𝟏𝟐𝒙𝒚𝟒 + 𝟓𝒙𝒚 + 𝟓𝟎)
𝟐𝟓𝒙𝟓 𝒚𝟒 − 𝟑𝟓𝒙𝟐 𝒚𝟑 − 𝟏𝟎𝒚𝟑 𝒙𝟑 𝟓𝒙𝟐 𝒚𝟑 𝟓𝒙𝟐 𝒚𝟑 (𝟓𝒙𝟑 𝒚 − 𝟕 − 𝟐𝒙)

𝒑(𝒛 − 𝟏) (𝒛 − 𝟏) (𝒛 − 𝟏) 𝒑 𝟏
− (𝒛 − 𝟏) ( − )
𝟏𝟓 𝟐𝟓 𝟓 𝟑 𝟓

3 2 5
8. Determine el máximo factor común de 40x y z y 100x2 y4 .

Solución

El mayor divisor común entre 40 y 100 es 20. El exponente menor de x es


2, el de y es 2 y z no es un factor común en los monomios. Luego el
2 2
máximo factor común es 20x y .

9. Determine el máximo factor común de 25xy z ,  15xy z


2 3 2
y 50x2 y 3 z .

Solución

El mayor divisor común entre 25, 15 y 50 es 5. La potencia menor de x es


2
1, la de y es 2 y la de z es 1. Luego el máximo factor común es 5xy .

Recuerde:

El mayor divisor común


de 25, 15 y 50.

15  25  50 5
es 5
3  5  10

4 2 3 2 7
10. Determine un factor común para x y z ,  x2 y y x4 y 3 .
3 3 3

220
Solución
1
El coeficiente numérico del factor común podría ser o bien cualquiera de los
3
4 2 7
números , o . Por otro lado, la potencia menor de x es 2, la de y es 1 y
3 3 3
1 2
la variable z no es común en lo monomios, luego son factores en común x yo
3
4 2 7 2 1 2
x y 0 x y , etc. Aunque también lo son xy o x.
3 3 3 3
1 2 3 1
Determine un factor común para x y ,  b3 x 3 y 5 y 5x 4 y 3 .
3 2

Solución
El coeficiente numérico del factor común podría ser cualquiera de los números
1 1 1
, , 5 o entre otros. La potencia menor de x es 2, la de y es 3 y la
3 2 6
1 2 3 1 2 3
variable b no es común en lo monomios. Son factores en común x y o x y o
3 2
1
5x2 y3 o x 2 y 3 .
6

Determine un factor común para 8 2a b ,  8abc y  6 32a2 c 5 .


2
11.

Solución
Como 8 = 2 3 =2 2 y 6 32  6  2 5  6  2 2 2  24 2
Un factor numérico en común es 2 2 o -2 2 . Por otro lado, la potencia menor de
a es 1 y las variables b y c no son comunes en lo monomios, luego son factores en
común 2 2a o 2 2a entre otros.

Ejercicios
1) Factorice m 6 n 4  m 4 n 2 

221
a) Me pregunto ¿qué letras tiene igual? _________________ .
b) De las letras que escogí ¿Cuáles son los exponentes más pequeños?
_____________.
c) Entonces escojo las letras con exponentes más pequeños y abro paréntesis.
m 6 n 4  m 4 n 2  _________ (
d) Divido cada término entre las letras con menor exponente:

m 6 n4 m 4 n2
  _________ (
____ _____
e) Divido y coloco la respuesta dentro del paréntesis.
m 6 n4 m 4 n2
  _________
____ _____

2) Halla el factor común de los siguientes ejercicios:

1. 6x - 12 = 2. 4x - 8y =
3. 24a - 12ab = 4. 10x - 15x2 =
5. 14m2n + 7mn = 6. 4m2 -20 am =
7. 8a3 - 6a2 = 8. ax + bx + cx =
9. b4-b3 = 10. 4a3bx - 4bx =
11. 14a - 21b + 35 = 12. 3ab + 6ac - 9ad =
13. 20x - 12xy + 4xz = 14. 6x4 - 30x3 + 2x2 =
15. 10x2y - 15xy2 + 25xy = 16. 12m2n + 24m3n2 - 36m4n3
=
17. 2 3 4
2x + 6x + 8x - 12x = 18. 10p2q3 + 14p3q2 - 18p4q3
- 16p5q4 =
19. m n p + m n p - m n p + m2n4p3 =
3 2 4 4 3 5 6 4 4

20. 3 2 8
x y  xy 2 
4 9
21. 1 2 3 1 3 4 1 2 5
a b  a b  a b 
1 4 2
a b 
2 4 8 16
22. 4
a 2b 
12
ab 
8 2 3
a b 
16 3
a b
35 5 15 25

3)Factoriza:
1) 4 x  8 y 

2) x 3  4 x 2  x 
3)  5 x 3  10 x 2 
4) 2 x 3  3 x 2 
5) 5 x  10xy  5 x 2 y 

6) 3x 5 y 4  9 x 2 y 3 3xy  3 y 

222
7) 5 y 2 x  15 yx 2  y 3 x 4 

8) 6 x 2 y 2  9 x 3 y 6  27 xy 3 
1
9) 3a 2  a 
2
x x2
10)  
6 5
x 1
11)  
6 12
1 3 1
12) x  x2  
8 4 2
13) 8 x  16 y  12 z 

14) x 4 y 3  2x3 y 4 

15) 2 x 3  5 x 2  3x 4 

16) 6b 4  12b 3 
17) 3abc  5bc  7abcd  2bcd 
18) 5 x 2 y 3 z 7  3x 8 y 4 z  11x 5 y 3 z 4 

19) 14x 3 y 2 z 4  7 x 5 y 3 z 7  21x 2 y 5 z 3 

20) 54n 4 m 6  18n 3 m 5  27n 7 m 3 


4. Determine el máximo factor común de:

a) 2 xy ,  10 x b) 6m n ,  12m n, 3m
3 2 2

c) 7 m n , 35m n ,  21m n d) 6a b , 18abc ,


3 2 7 5 3 13 2 5
32b2c3

5. Determine un factor común para:


1 2 2 1 4 2 2 5 3 7
a) b x ,  ab 3 x b) r s t ,  r 4 s6 ,  rst
2 2 5 5 5

223
b) factor común polinomio
1. Se identifica el factor común
2 Se divide cada término del polinomio por el factor común
3. Se abren dos paréntesis, en el primero se escribe el factor común y
en el segundo los cocientes hallados en el paso anterior (cada uno con su respectivo
signo)

Ejemplos Factorar o descomponer en dos factores:


1

224
4

225
7

17

8. Factorice5𝑥(𝑥 − 𝑦) − 7𝑎(𝑥 − 𝑦).

Solución
5𝑥(𝑥 − 𝑦) − 7𝑎(𝑥 − 𝑦)
= (𝑥 − 𝑦)(5𝑥 − 7𝑎).

9) Factorice5𝑥(𝑥 − 𝑦) − 7𝑎(𝑦 − 𝑥).

Solución
5𝑥(𝑥 − 𝑦) − 7𝑎(𝑦 − 𝑥)
= 5𝑥(𝑥 − 𝑦) + 7𝑎(𝑥 − 𝑦)
= (𝑥 − 𝑦)(5𝑥 + 7𝑎).

226
10)Factoriza x(a + b ) + y( a + b ) =
Existe un factor común que es (a + b ) = x(a + b ) + y( a + b ) =
= ( a + b )( x + y )

11)Factoriza 2a(m - 2n) - b (m - 2n) = 2a(m - 2n) - b (m - 2n)


= (m - 2n )( 2a - b )

Ejercicios

1) Factorizar las siguientes expresiones algebraicas por el método de factor común.

1) x(a + b) + m(a + b) 2) 2x(a – 1) – y(a – 1)

3) m(x + 2) + m + 2 4) a(x + 1) – x – 1

5) 2x(x + y+ z) – x – y – z 6) (x – a)(y + 2) + b(y + 2)

7) (x + 2)(x – 1) – (x – 1)(x – 3) 8) x(a – 1) + y(a – 1) – a + 1

227
2) Factorizar las siguientes expresiones algebraicas por el método de factor
común

1. a(x + 1) + b ( x + 1 ) = 2. m(2a + b ) + p ( 2a + b ) =
3. x2( p + q ) + y2( p + q ) = 4. ( a2 + 1 ) - b (a2 + 1 ) =
5. ( 1 - x ) + 5c( 1 - x ) = 6. a(2 + x ) - ( 2 + x ) =
7. (x + y )(n + 1 ) - 3 (n + 1 ) = 8. (a + 1 )(a - 1 ) - 2 ( a - 1 ) =

9. (a( a + b ) - b ( a + b ) = 10. (2x + 3 )( 3 - r ) - (2x - 5 )( 3 - r ) =

FACTOR COMUN POR AGRUPACION

P r o c e d i m i e n t o:
1. Se agrupan los términos convenientemente, utilizando paréntesis
2. Se saca factor común de cada uno de los paréntesis
3. Se realiza una segunda factorización (el factor común será, en este caso, el
paréntesis).
Características:
- Tienen más de tres términos
Forma de factorizar: ax  bx  ay  by

Primero:
Observa cuidadosamente y notaras que los dos primero términos tienen
factor común x y los dos últimos términos tienen factor común y ahora lo
operamos:
ax  bx  ay  by 
xa  b   ya  b  

Los paréntesis me deben de quedar iguales si no es así entonces busco otras


parejas o tríos.

Segundo:
Observamos que nos queda como factor común el paréntesis a  b 
Factorizamos

ax  bx  ay  by 
xa  b   y a  b  
a  b  Es el factor común para saber cuál es el otro paréntesis
tapa con tu dedo los paréntesis a  b  lo que te queda
es el otro paréntesis

228
x a  b   y a  b  

x a  b   y a  b  

Lo que te queda al tapar los paréntesis a  b  es


x + y esto lo colocas en el otro paréntesis y
terminaste.

x a  b   y a  b  

x a  b   y a  b  

a  b  x  y 

Ejemplos
1. Determinar la factorización completa del polinomio −12𝑥 2 + 10𝑤 − 15 +
8𝑥 2 𝑤, se recurre directamente a agrupar los términos, pues no hay un factor común
que se pueda obtener. Un posible criterio para agrupar es ubicar juntos los términos
con coeficientes numéricos pares y, por otro lado, los de coeficientes impares. Así,
al resolver la factorización se tiene:
−12𝑥 2 + 10𝑤 − 15 + 8𝑥 2 𝑤 = (−12𝑥 2 + 8𝑥 2 𝑤) + (10𝑤 − 15) =

4𝑥 2 (−3 + 2𝑤) + 5(2𝑤 − 3) = 4𝑥 2 (2𝑤 − 3) + 5(2𝑤 − 3) = (2𝑤 − 3)(4𝑥 2 + 5)

Por lo tanto, se tiene que −12𝑥 2 + 10𝑤 − 15 + 8𝑥 2 𝑤 = (2𝑤 − 3)(4𝑥 2 + 5)

2.No obstante, en este método a veces sucede que se debe incluir un signo negativo
en el factor común para poder hacer un cambio de signos. Por ejemplo, al factorizar
7𝑏 − 𝑎 − 49𝑏𝑎2 + 7𝑎3 , se podría colocar juntos a los términos que contienen a la
variable 𝑏 y a los que no la tienen. En este caso, se tendría que

7𝑏 − 𝑎 − 49𝑏𝑎2 + 7𝑎3 = (7𝑏 − 49𝑏𝑎2 ) + (−𝑎 + 7𝑎3 ) =

7𝑏(1 − 7𝑎2 ) + 𝑎(−1 + 7𝑎2 ) = 7𝑏(1 − 7𝑎2 ) − 𝒂(𝟏 − 𝟕𝒂𝟐 ) = (1 − 7𝑎2 )(7𝑏 − 𝑎)

Por lo tanto, 7𝑏 − 𝑎 − 49𝑏𝑎2 + 7𝑎3 = (1 − 7𝑎2 )(7𝑏 − 𝑎).

229
3. Factorice

2𝑥 − 3𝑥𝑦 − 4𝑦 + 6𝑦 2 = (2𝑥 − 4𝑦) + (−3𝑥𝑦 + 6𝑦 2 ) = 2(𝑥 − 2𝑦) + 3𝑦(−𝑥 + 2𝑦) =

2(𝑥 − 2𝑦) − 𝟑𝒚(𝒙 − 𝟐𝒚) = (𝑥 − 2𝑦)(2 − 3𝑦)

Por lo tanto, 2𝑥 − 3𝑥𝑦 − 4𝑦 + 6𝑦 2 = (𝑥 − 2𝑦)(2 − 3𝑦).

4. Factorice𝑏𝑥 − 𝑏𝑦 + 𝑎𝑥 − 𝑎𝑦.
Solución
𝑏𝑥 − 𝑏𝑦 + 𝑎𝑥 − 𝑎𝑦
= 𝑏(𝑥 − 𝑦) + 𝑎(𝑥 − 𝑦)
= (𝑥 − 𝑦)(𝑏 + 𝑎).
5.Factorice 𝑝𝑥 − 𝑞𝑥 − 𝑝𝑦 + 𝑞𝑦.

Solución
𝑝𝑥 − 𝑞𝑥 − 𝑝𝑦 + 𝑞𝑦
= 𝑝𝑥 − 𝑝𝑦 − 𝑞𝑥 + 𝑞𝑦
= 𝑝(𝑥 − 𝑦) − 𝑞(𝑥 − 𝑦)
= (𝑥 − 𝑦)(𝑝 − 𝑞).
Recuerde:
6. Factorice 𝑥 4 − 𝑥 + 2𝑥 3 − 2.
Usando la distributivita
Solución
  10m  5n   10m  5n
= 𝑥 4 − 𝑥 + 2𝑥 3 − 2.
= 𝑥 4 + 2𝑥 3 − 𝑥 − 2. 5  2m  n   10m  5n
= 𝑥 3 (𝑥 + 2) − (𝑥 + 2)
= (𝑥 + 2)(𝑥 3 − 1) 5  2m  n   10m  5n

7. Factorice el polinomio 3x  3y  ax  ay usando el método de agrupación.

Solución
Observe que, entre los cuatro monomios que forman el polinomio, no hay un
factor común. Sin embargo, si se agrupa como  3x  3 y    ax  ay  , en el primer
grupo hay un 3 de común y en el segundo una a. Al factorizar cada uno de estos
factores, se obtiene: 3  x  y   a  x  y  . La expresión que resulta aún no está
factorizada ya que es una suma con dos sumandos. Pero entre ellos hay un factor
común:  x  y  .
Luego la factorización pedida es  x  y   3  a 

230
9. Factorice el polinomio 2mb  nb  10m  5n usando el método de agrupación.

Solución
Para este polinomio, entre los cuatro monomios que lo forman, tampoco hay un
factor común. Sin embargo, si se agrupa como  2mb  nb    10m  5n  , en el
primer grupo hay una b de común y en el segundo un 5. Al factorizar cada uno de
estos factores, se obtiene: b  2m  n   5  2m  n  . La expresión que resulta aún
no está factorizada ya que es una suma con dos sumandos. Pero no se puede
seguir con el proceso de factorización ya que no hay factores en común entre
éstos sumandos.

Observe que en el segundo sumando también se puede factorizar un -5 y se


obtiene: b  2m  n   5  2m  n  , con lo que se logra un factor común entre los
sumandos.
Recuerde:
Luego la factorización es:  2m  n   b  5 
y2  1  p  p  1  y2

10.Factorice el polinomio 4xy  3zp  4x  3z  4xp  3zy usando el método de


2 2

agrupación.

Solución
Para este polinomio tampoco hay un factor común a todos los monomios. Usando
las propiedades conmutativa y asociativa de la adición se obtiene la agrupación:
 4xy 2
 4x  4xp    3zp  3z  3zy 2  . En el primer grupo hay 4x de factor común y
en el segundo 3z. Al factorizar cada uno de estos términos, se obtiene:
4 x  y 2  1  p   3z  p  1  y 2  y y 2
 1  p  es un factor común. La factorización es
y 2
 1  p   4 x  3z 

231
Ejercicios
1Factorizar por agrupamiento

1. a2 + ab + ax + bx = 2. ab + 3a + 2b + 6 =
3. ab - 2a - 5b + 10 = 4. 2ab + 2a - b - 1 =
5. am - bm + an - bn = 6. 3x3 - 9ax2 - x + 3a =
7. 3x2 - 3bx + xy - by = 8. 6ab + 4a - 15b - 10 =
9. 3a - b2 + 2b2x - 6ax = 10. a3 + a2 + a + 1 =
11. ac - a - bc + b + c2 - c =

12. 6ac - 4ad - 9bc + 6bd + 15c2 - 10cd =


13. ax - ay - bx + by - cx + cy =
14. 3am - 8bp - 2bm + 12 ap =
15. 18x - 12 - 3xy + 2y + 15xz - 10z =
16. 15 2 21
x  xz 
10
xy 
143
yz  5 x  7 z 
4 4 3 3
17. 2 8 4 16
am  am  bm  bn 
3 3 5 5

2) Factorice cada uno de los siguientes polinomios utilizando el método de


agrupación.

a) 8xz  4xy  14z  7 y b) y  3y  9 y  27


3 2

c) a  7 ab  3ac  21bc d) x  xy  6x  6 y
2 2

 
e) z x  y  y x  y
2 2
  f) 9  10  x   x  x  10 

3) Factorizar las siguientes expresiones algebraicas por el método agrupamiento.

1) ax + bx + ay + by 2) 3m2 – 6mn + 4m – 8n

3) 2x2 – 3xy – 4x + 6y 4) x + z – 2ax – 2az2

5) 3ax – 3x + 4y – 4ay 6) ax – ay + az + x – y + z

7) a2x – ax2 – 2a2y + 2axy + x3 – 2x2y

232
III Método Trinomio Cuadrado Perfecto

Definición: Una cantidad es un cuadrado perfecto cuando es el


producto de multiplicar dos factores iguales.

Procedimiento:
1. Se ordena el trinomio
2. Se extrae la raíz cuadrada del primer y tercer términos
3. Se halla el doble producto de las raíces obtenidas en el paso anterior
4. Si el producto hallado en el paso anterior es igual al segundo término del
trinomio y si el primero y tercer términos tienen igual signo, se trata de un trinomio
cuadrado perfecto y se factoriza como tal.
5. Se escribe dentro de un paréntesis las raíces cuadradas del primero y tercer
términos, separadas por el signo del segundo término, y el paréntesis elevado al
cuadrado.
Características:
- Tienen tres términos (ordenarlo en forma descendente)
- El primer término y el tercero tienen raíz cuadrada exacta.
- El segundo término es la multiplicación de la raíz cuadrada del primer término
por la raíz cuadrada del tercer término multiplicada siempre por 2, si da como
resultado el segundo término entonces es un trinomio cuadrado perfecto.

Forma de factorizar: 4y4 1 4y2 

Primero: ordeno el trinomio en forma descendente

4y4 1 2y2 


4y4  4y2 1 

Segundo: saco raíz cuadrada del primer término tanto a al


número como la letra.
4y4 1 4y2 
4y4  4y2 1 

4
2
2y

2y2

233
Tercero: saco raíz cuadrada del segundo término tanto a al
número como la letra.
4y4 1 4y2 
4y4  4y2 1 

4
2
2y 1

2y2 1

Cuarto: realizo la prueba para ver si es un trinomio cuadrado perfecto.


Multiplico el primer término ( 2 y 2 ) de mi respuesta con el segundo ( 1 )y
luego multiplico siempre por 2 para ver si me da el segundo término (
4y2 )

4y4  4y2 1  Siempre se


Como son iguales si es multiplica por 2
un trinomio cuadrado
perfecto 2 y 
2
  1  2  4 y 2

Quinto: opero los signos del ejercicio y lo coloco al centro

   

4y4  4y2 1 

4
2
2y 1

2y2  1
Sexto: lo encierro entre paréntesis y lo elevo al cuadrado.

4y4  4y2 1 

4
2
2y 1

2 y 2
 1 
2

234
Ejemplos

1. Factorice el trinomio x  8 x  16 .
2

Solución
Observe que este trinomio contiene dos términos que son cuadrados perfectos:
x2 y 16 ; cuyas raíces cuadradas son x y 4 respectivamente. El doble producto de
estas raíces es 2  x  4  8x que coincide con el término restante del trinomio dado.
x2  8 x  16   x  4 
2
Luego la factorización es

2. Factorice el trinomio a  2 3a  3 .
2

Solución
2
En este trinomio se reconoce que a es un cuadrado. Por otro lado, recuerde que

 3
2
3 , luego el doble producto de las raíces cuadradas de estas expresiones es

2  a  3  2 3a , que coincide con el término restante del trinomio dado. Como este
 
2
término tiene signo negativo, la factorización es a  2 3a  3  a  3
2
.

3. Factorice el trinomio  x  10 x  25 .
2

Solución
Este trinomio no responde exactamente al patrón de las fórmulas dadas para el
cuadrado de un binomio, pero observe que se puede factorizar un -1, o sea un
signo negativo.

 x 2  10 x  25  1  x 2  10 x  25 
   x  5
2

4. Factorice el trinomio  x  26 x  169 .


2

Solución
Nuevamente este trinomio no responde exactamente al patrón de las fórmulas
dadas para el cuadrado de un binomio, pero se procede:
 x 2  26 x  169  1  x 2  26 x  169 
   x  13 
2

235
Ejercicios

1. Factorizar por medio de trinomios cuadrados perfectos


2. b2 - 12b + 36 = 3. 25x2 + 70xy + 49y2 =
4. m2 - 2m + 1 = 5. x2 + 10x + 25 =
6. 16m - 40mn + 25n =
2 2
7. 49x2 - 14x + 1 =
8. 36x2 - 84xy + 49y2 = 9. 4a2 + 4a + 1 =
10.1 + 6ª + 9a =
2
11.25m2 - 70 mn + 49n2 =
12.25a2c2 + 20acd + 4d2 = 13.289a2 + 68abc + 4b2c2 =
14.16x y - 8 x y z + z =
6 8 3 4 7 14

2. Factorizar las siguientes expresiones algebraicas por el método trinomio


cuadrado perfecto.
1) m2 + 2m + 1 2) 4x2 + 25y2 – 20xy 3) 1 – 16ax2 + 64a2x4

b2 1 b b2
4) x 2  bx  5)  
4 4 3 9

IV Método Diferencia de cuadrados

Procedimiento:
1. Se extrae la raíz cuadrada al minuendo y al sustraendo
2. Se abren dos paréntesis
3. En el primer paréntesis se escribe la suma y en el segundo la diferencia de las
raíces halladas en el paso 1

Características:
- Tienen dos términos ( es un binomio = bi significa 2)
- El signo que los separa siempre es menos
- Las potencias de letras están elevadas con números pares 2, 4, 6…
- Tiene raíz cuadrada exacta el primer término
- Tiene raíz cuadrada exacta el segundo término

236
Forma de factorizar:
Primero abro paréntesis (

Segundo saco raíz cuadrada al número si no la se, le saco los factores primos al
número así: Por cada pareja de 2
sale un dos

16 2
8 2 2 x 2 = 4 y esta
2 Multiplico los números
4 2 es la raíz
circulados
2 2 2 cuadrada 16

1
Coloco la respuesta así: ( 4

Tercero saco la raíz cuadrada de la letra asi:


2
a 2 divido siempre la potencia entre dos a 2
a
y la respuesta es la raíz de 3) la letra.

Coloco la respuesta
(4a así

Cuarto copio el signo.

Coloco así ( 4a 

Quinto saco la raíz cuadrada del segundo término siguiendo los pasos
segundo y tercero.

Coloco la respuesta así ( 4a  b

Sexto cierro paréntesis.


así ( 4a  b)

Séptimo copio el primer paréntesis solamente que le cambio el signo a +.


Así
(4a  b) (4a  b)

237
Ejemplos

1. Factorice 4𝑥 2 − 9𝑦 2 .

Solución
Siguiendo el esquema
4𝑥 2 − 9𝑦 2
2𝑥
−3𝑦
2𝑥
+3𝑦

se obtiene que 4𝑥 2 − 9𝑦 2 = (2𝑥 − 3𝑦)(2𝑥 + 3𝑦).


2. Factorice el binomio x  36 .
2

Solución

Como 36  6 , x  36 se expresa como x  6 , luego utilizando la fórmula anterior


2 2 2 2

x2  62   x  6  x  6  .

3. Factorice el binomio a  16b .


4 4

Solución

  y 16b4   4b2  , el binomio a 4  16b4 se puede expresar como:


2 2
Como a  a
4 2

a4  16b4   a2    4b2    a2  4b2  a2  4b2  .


2 2

Ya que a y 4b   2b  son cuadrados, la diferencia a  4b


2 2 2 2 2
se escribe como
 a  2b  a  2b  . Por otro lado, observe que  a2  4b2  es una suma de cuadrados y
no responde al patrón de la fórmula dada anteriormente. En cursos posteriores se
va a justificar que no hay manera de expresar una suma de la forma a  b como
2 2

el producto de polinomios.

Por lo tanto la factorización completa de a  16b es:


4 4

a4  16b4   a  2b  a  2b   a2  4b2 

238
4. Factorice el binomio x  5 .
2

Solución
Aunque pareciera que el binomio dado no se puede expresar como una diferencia
de cuadrados, ya que 5 no es un cuadrado perfecto, se puede hacer lo siguiente:
 5    x  5  x  5 
2
x2  5  x2 

1
5. Factorice el binomio  0, 49 x2 .
4
Solución
2
1 1
Como    y 0, 49   0,7  , se tiene:
2

4 2
2

 0, 49 x2    -  0,7 x     0,7 x  
1 1 1 1
  0,7 x  
2

4 2 2  2 
 1  7 x  1  7 x  .
  
 2 10  2 10 

6. Factorice el binomio x
6m
 9 y2n .
  y 9 y2 n   3yn  , luego
2 2
Solución Observe que x  x
6m 3m

x6 m  9 y 2 n   x 3 m    3 y n  
2 2

x 3m
 3 y n  x3 m  3y n 

Ejercicios
a. Factorizar por medio de diferencia de cuadrados
1. 9a - 25b =
2 2
2. 16x2 - 100 =
3. 4x2 - 1 = 4. 9p2 - 40q2 =
5. 36m2n2 - 25 = 6. 49x2 - 64t2 =
7. 169m2 - 196 n2 = 8. 121 x2 - 144 k2 =
9. 9 2 49 2
a  b  10. 1 4 9 4
x  y 
25 36 25 16
11. 3x2 - 12 = 12. 5 - 180f2 =
13. 8y2 - 18 = 14. 3x2 - 75y2 =
15. 45m3n - 20mn = 16. 2a5 - 162 a3 =

239
b.Factorizar las siguientes expresiones algebraicas por el método diferencia de
cuadrados.

1) 1 – a2 2) 16x2 – 25y2 3) 49x2y6z10 – a12

a2 b4
4)  5) a2n – 9b4m
4 9

V Método Combinación de los métodos anteriores


En esta sección, se aplicará una combinación de métodos para factorizar
polinomios.

Ejemplos
1. Factorice (𝑥 − 2) − (𝑥 2 − 4𝑥 + 4).

Solución
(𝑥 − 2) − (𝑥 2 − 4𝑥 + 4)
= (𝑥 − 2) − (𝑥 − 2)2
= (𝑥 − 2)[1 − (𝑥 − 2)]
= (𝑥 − 2)(3 − 𝑥).
2. Factorice𝑥 2 (2
− 𝑦) − 2𝑥(2 − 𝑦) + 2 − 𝑦.

Solución
𝑥 2 (2 − 𝑦) − 2𝑥(2 − 𝑦) + 2 − 𝑦
= (2 − 𝑦)(𝑥 2 − 2𝑥 + 1)
= (2 − 𝑦)(𝑥 − 1)2 .
3. Factorice2𝑥 2 (𝑥 − 3) + 𝑥 2 − 6𝑥 + 9.

Solución
2𝑥 2 (𝑥 − 3) + 𝑥 2 − 6𝑥 + 9
= 2𝑥 2 (𝑥 − 3) + (𝑥 − 3)2
= (𝑥 − 3)(2𝑥 2 + 𝑥 − 3)
= (𝑥 − 3)(2𝑥 − 3)(𝑥 − 1).

240
4. Factorice 2𝑥 − 2𝑥𝑦 + 𝑦 2 − 1
Solución
2𝑥 − 2𝑥𝑦 + 𝑦 2 − 1
= −2𝑥(𝑦 − 1) + (𝑦 − 1)(𝑦 + 1)
= (𝑦 − 1)(−2𝑥 + 𝑦 + 1).

5. Factorice 𝑥 2 − 16𝑦 2 + 10𝑥 + 25.


Solución
𝑥 2 − 16𝑦 2 + 10𝑥 + 25
= 𝑥 2 + 10𝑥 + 25 − 16𝑦 2
= (𝑥 + 5)2 − 16𝑦 2
= (𝑥 + 5 − 4𝑦)(𝑥 + 5 + 4𝑦)

6. Factorice 16𝑥 2 − 𝑦 2 − 8𝑥 + 1.

Solución
16𝑥 2 − 𝑦 2 − 8𝑥 + 1
= 16𝑥 2 − 8𝑥 + 1 − 𝑦 2
= (4𝑥 − 1)2 − 𝑦 2
= (4𝑥 − 1 + 𝑦)(4𝑥 − 1 − 𝑦).
7. Factorice 𝑎(𝑎 − 1) − 𝑏(𝑏 + 1)

Solución
𝑎(𝑎 − 1) − 𝑏(𝑏 + 1)
= 𝑎2 − 𝑎 − 𝑏 2 − 𝑏
= 𝑎2 − 𝑏 2 − 𝑎 − 𝑏
= (𝑎 − 𝑏)(𝑎 + 𝑏) − (𝑎 + 𝑏)
= (𝑎 + 𝑏)(𝑎 − 𝑏 − 1).

8. Factorice el binomio 5 x  80 x .
5 3

Solución
El binomio dado no responde al patrón de una diferencia de cuadrados, pero se
reconoce un factor común. Entonces:

5x 5  80 x 3  5x 3  x 2  16 
diferencia de
cuadrados

 5x 3  x  4  x  4 

241
9. factorice la expresión a
2
a 2
 9   25  a2  9  .

Solución
Observe que la expresión no está factorizada, es una suma con dos sumandos,
que tienen como factor común a a 2
 9  , así:

a2  a2  9   25  a2  9    a2  25   a2  9 
diferencia de diferencia de
cuadrados cuadrados

  a  5  a  5  a  3  a  3 

10. Factorice el polinomio b  a  6 a  6b .


2 2

Solución
En esta expresión no hay un factor común para todos lo monomios ni tampoco
responde a la fórmula de diferencia de dos cuadrados, pero agrupando se logra
factorizar. Entonces:

b 2  a 2  6 a  6 b   b 2  a 2    6 a  6b 
diferencia de el 6 es un factor
cuadrados común

  b  a  b  a   6  a  b 
 1  a  b  b  a   6  a  b 
  a  b   1  b  a   6 
  a  b  b  a  6 
11. Factorice el polinomio y  x  8 y  16 .
2 2

Solución
Observe que la forma de este polinomio no responde a las fórmulas dadas
anteriormente, pero si se acomoda usando la conmutatividad de la adición y se
agrupa usando la asociatividad, se logra factorizar.

242
y 2  x 2  8 y  16  y 2  8 y  16  x 2
cuadrado perfecto

  y  4   x2
2

diferencia de
cuadrados

  y  4  x  y  4  x 
12. Factorice el polinomio 2x2  6 y  y2  9
Solución
Este polinomio tampoco responde al patrón de las fórmulas, pero acomodando y
agrupando, se tiene

2 x 2  6 y  y 2  9  y 2  6 y  9  2 x 2
cuadrado perfecto

  y  3   2x2
2

.
diferencia de cuadrados


 y  3  2x y  3  2x 
Ejercicios
A) Factorice por medio de combinación de métodos
1. 2ab + 4a2b - 6ab2 = 2. 2xy2 - 5xy + 10x2y - 5x2y2 =
3. b2 - 3b - 28 = 4. a2 + 6a + 8 =
5. 5a + 25ab = 6. bx - ab + x2 - ax =
7. 6x2 - 4ax - 9bx + 6ab = 8. ax + ay + x + y =
9. 8x2 - 128 = 10.4 - 12y + 9y2 =
11.x4 - y2 = 12.x2 + 2x + 1 - y2 =
13.(a + b )2 - ( c + d)2 = 14.a2 + 12ab + 36b2 =
15.36m2 - 12mn + n2 = 16.x16 - y16 =

243
B) Descomponga en varios factores las siguientes expresiones algebraicas.

1. 5a2 – 5 2. 3x3 – 18x2y + 27xy2

3. x4 – y4 4. 6ax2 + 12ax – 90x

5. 3x4 – 26x2 – 9 6. x3 – 4x – x2 + 4

7. 2x4 – 32 8. x4 – 13x2 + 36

VI Método por Inspección


Este método se aplica para factorizar polinomios cuadráticos donde ∆ es
un número cuadrado perfecto. Se presentan dos casos, donde el
polinomio es mónico (tiene coeficiente principal igual a 1) o si no es mónico.

Caso 1

Este caso se aplica cuando el polinomio es mónico, esto es, cuando tiene la forma
𝑥 2 + 𝑏𝑥 + 𝑐. Para factorizar dicho polinomio, se deben buscar dos números reales
𝑚 y 𝑛 tales que 𝑚 ⋅ 𝑛 = 𝑐 y 𝑚 + 𝑛 = 𝑏, con lo que se obtiene que:

𝑥 2 + 𝑏𝑥 + 𝑐 = (𝑥 + 𝑚)(𝑥 + 𝑛).

 Si 𝑐 > 0, 𝑏 > 0 los dos números 𝑚, 𝑛 son positivos.


 Si 𝑐 > 0, 𝑏 < 0 los dos números 𝑚, 𝑛 son negativos.
 Si 𝑐 < 0, los dos números 𝑚, 𝑛 tienen signos opuestos.
Como nemotecnia, a veces se ponen los números 𝑚 y 𝑛 como lo muestra el
siguiente esquema

𝑥 2 + 𝑏𝑥 + 𝑐
𝑥 𝑚
𝑥 𝑛

244
entendiendo que se deben buscar dos números 𝑚 y 𝑛, tales que multiplicados den
𝑐 y sumados resulten igual a 𝑏. Luego, se copian los factores en línea recta por lo
que se obtiene:

𝑥 2 + 𝑏𝑥 + 𝑐 = (𝑥 + 𝑚)(𝑥 + 𝑛).
Caso 2
Este caso se aplica cuando el polinomio no es mónico, esto es, cuando tiene la
forma 𝑎𝑥 2 + 𝑏𝑥 + 𝑐. Para factorizar dicho polinomio, se deben buscar cuatro
números reales 𝑚, 𝑛, 𝑝 y 𝑞 tales que 𝑚 ⋅ 𝑛 = 𝑐, 𝑝 ⋅ 𝑞 = 𝑎 y que cumpla 𝑝 ⋅ 𝑛 + 𝑞 ⋅ 𝑚 =
𝑏, con lo que se obtiene que:
𝑎𝑥 2 + 𝑏𝑥 + 𝑐 = (𝑝𝑥 + 𝑚)(𝑞𝑥 + 𝑛).
 Si 𝑐 > 0, 𝑏 > 0 los dos números 𝑚, 𝑛 son positivos.
 Si 𝑐 > 0, 𝑏 < 0 los dos números 𝑚, 𝑛 son negativos.
 Si 𝑐 < 0, los dos números 𝑚, 𝑛 tienen signos opuestos.
Como nemotecnia, a veces se ponen los números 𝑚, 𝑛, 𝑝 y 𝑞 como lo muestra el
siguiente esquema

𝑎𝑥2 + 𝑏𝑥 + 𝑐

𝑝𝑥 𝑚

𝑞𝑥 𝑛

entendiendo que se deben buscar dos números 𝑚 y 𝑛, tales que multiplicados den
𝑐; además que se deben buscar dos números 𝑝 y 𝑞 tales que 𝑝 ⋅ 𝑞 = 𝑎, y también
que al multiplicar los números 𝑝, 𝑛 y 𝑚, 𝑞 en cruz y luego sumarlos se obtenga 𝑏, ie
𝑝 ⋅ 𝑛 + 𝑞 ⋅ 𝑚 = 𝑏.
Luego, se copian los factores en línea recta por lo que se obtiene

Ejemplos
1. Factorice 2𝑥 2 − 11𝑥 + 5.

Solución
El discriminante del trinomio 2𝑥 2 − 11𝑥 + 5 es ∆= (−11)2 − 4 ⋅ 2 ⋅ 5 = 81, por lo que el
trinomio se puede factorizar por inspección. Siguiendo el esquema de esta sección,
con −5 ∙ −1 = −5, 2 ∙ 1 = 2 y 2 ∙ −5 + −1 ∙ 1 = −11

2𝑥 2 − 11𝑥 + 5
2𝑥 −1
1𝑥 −5

245
se obtiene que 2𝑥 2 − 11𝑥 + 5 = (2𝑥 − 1)(𝑥 − 5).

2. Factorice −6𝑥 2 + 7𝑥 − 2.

Solución
El discriminante del trinomio −6𝑥 2 + 7𝑥 − 2 es ∆= (7)2 − 4 ⋅ −6 ⋅ −2 = 1, por lo que el
trinomio se puede factorizar por inspección. Se debe observar que

−6𝑥 2 + 7𝑥 − 2 = −(6𝑥 2 − 7𝑥 + 2).

Siguiendo el esquema de esta sección, con 3 ∙ 2 = 6, −2 ∙ −1 = 2 y 3 ∙ −1 + 2 ∙ −2 = −7

6𝑥 2 − 7𝑥 + 2
3𝑥 −2
2𝑥 −1

se obtiene que −6𝑥 2 + 7𝑥 − 2 = −(3𝑥 − 2)(2𝑥 − 1) = (2 − 3𝑥)(2𝑥 − 1) .

3. Factorice el polinomio 6 x 2  23 x  10

Solución Se buscan los factores para -6 y - 10

Se expresa la factorización

4. Factorice el polinomio 2x2  5x  3

Se buscan los factores para 2 y - 3

Se expresa la factorización

246
5. Factorice en forma completa 𝑥 2 + 6𝑥 + 5.
Solución
El discriminante del trinomio 𝑥 2 + 6𝑥 + 5 es ∆= 62 − 4 ⋅ 1 ⋅ 5 = 16, por lo que el
trinomio se puede factorizar por inspección. Siguiendo el esquema de esta sección,
con 5 ∙ 1 = 5 y 5 + 1 = 6

𝑥2 + 6𝑥 + 5

𝑥 5

𝑥 1

se tiene que 𝑥 2 + 6𝑥 + 5 = (𝑥 + 5)(𝑥 + 1) .


6. Factorice en forma completa 𝑥 2 − 𝑥 − 6.
Solución El discriminante del trinomio 𝑥 2 − 𝑥 − 6 es ∆= (−1)2 − 4 ⋅ 1 ⋅ −6 = 25, por
lo que el trinomio se puede factorizar por inspección. Siguiendo el esquema de
esta sección, con −3 ∙ 2 = −6 y −3 + 2 = −1
𝑥2 − 𝑥 − 6

𝑥 2

𝑥 −3

se tiene que 𝑥 2 − 𝑥 − 6 = (𝑥 − 3)(𝑥 + 2).

7. Factorice en forma completa 𝑥 2 − 6𝑥 + 9.


Solución
El discriminante del trinomio 𝑥 2 − 6𝑥 + 9 es ∆= (−6)2 − 4 ⋅ 1 ⋅ 9 = 0, por lo que el
trinomio se puede factorizar por inspección. Siguiendo el esquema de esta sección,
con −3 ∙ −3 = 9 y −3 − 3 = −6

𝑥2 − 6𝑥 + 9

𝑥 −3

𝑥 −3

se tiene que 𝑥 2 − 6𝑥 + 9 = (𝑥 − 3)(𝑥 − 3) = (𝑥 − 3)2

247
Ejercicios

Factorice los siguientes polinomios cuadráticos:

a. 𝑥 2 + 2𝑥 + 1
b. 𝑥 2 − 4𝑥 + 4
c. 𝑥 2 + 4
d. 𝑥 2 − 4
e. 𝑥 2 − 3𝑥 + 10
f. 𝑥 2 + 𝑥 + 1
g. 𝑥 2 + 𝑥 − 1
h. 𝑥 2 + 4𝑥 + 4
i. 3𝑥 2 − 𝑥 − 2
j. 𝑥 2 − 9𝑥 + 8
k. 2𝑥 2 − 5𝑥 + 3
l. 3𝑥(𝑥 − 2) + 3
m. 3𝑥 2 − 13𝑥 − 10
1
n. −9𝑥 2 + 3𝑥 −
4
2
o. 2𝑥 − 3𝑥 − 1
𝑥 1
p. 𝑥 2 + −
2 2
2
q. −6+𝑥 − 𝑥
r. −𝑦 2 + 2𝑦 − 15
s. 𝑥 2 − 𝑥𝑦 − 30𝑦 2
t. 6𝑥 2 − 11𝑥𝑦 + 4𝑦 2
25 1 1
u. 𝑥4 + 𝑥2 +
36 3 25

VII Completar cuadrados

Completar cuadrados en una expresión algebraica

Como se mencionó anteriormente, los primeros dos productos notables son


(𝑥 + 𝑦)2 = 𝑥 2 + 2 ∙ 𝑥 ∙ 𝑦 + 𝑦 2 y (𝑎 − 𝑏)2 = 𝑎2 − 2 ∙ 𝑎 ∙ 𝑏 + 𝑏 2 . Pero, cualquier trinomio de la
forma 𝑥 2 + 𝑏𝑥 + 𝑐 se puede transformar en una expresión de la forma 𝑥 2 + 2ℎ𝑥 + ℎ2 +
𝑐 − ℎ2 = (𝑥 + ℎ)2 + 𝑘. Para lograrlo, se debe asumir que 𝑏𝑥 corresponde al término
del medio del producto notable, es decir, 𝑏𝑥 = 2ℎ𝑥. Al resolver esta ecuación, se

248
obtiene el valor de ℎ, para luego sumar y restar su cuadrado. Finalmente, el valor
de 𝑘 se obtiene al resolver 𝑐 − ℎ2 .

Por ejemplo, al completar los cuadrados de 𝑥 2 + 8𝑥 + 14, se tendría que

8𝑥
8𝑥 = 2ℎ𝑥 ⟹ =ℎ⟹4=ℎ
2𝑥

De esta manera, se tiene que ℎ2 = 42 = 16, con lo que

𝑥 2 + 8𝑥 + 14 = 𝑥 2 + 8𝑥 + 16 + 14 − 16 = (𝑥 + 4)2 − 2

Otro ejemplo es completar los cuadrados de 𝑥 2 + 6𝑥 + 7, donde

6𝑥
6𝑥 = 2ℎ𝑥 ⟹ =ℎ⟹3=ℎ
2𝑥

De esta manera, se tiene que ℎ2 = 32 = 9, con lo que

𝑥 2 + 6𝑥 + 7 = 𝑥 2 + 8𝑥 + 9 + 7 − 9 = (𝑥 + 3)2 − 2

Como último ejemplo, se tiene completar los cuadrados de la expresión 𝑥 2 + 3𝑥 + 2,


donde

3𝑥 3
3𝑥 = 2ℎ𝑥 ⟹ =ℎ⟹ =ℎ
2𝑥 2
3 2 9
De esta manera, se tiene que ℎ2 = (2) = 4, con lo que la respuesta es

2 2
9 9 3 2 1
𝑥 + 3𝑥 + 2 = 𝑥 + 3𝑥 + + 2 − = (𝑥 + ) −
4 4 2 4

249
Ejemplos

1 Exprese de la forma (x+h)2 + k la expresión x 2  10 x  28

x 2  10 x  28
2 2
 10   10 
x  10 x        28
2

 2  2
x 2
 10 x  25   25  28

 x  5 3
2

2. Expresar el trinomio x 2  6 x  10 de la forma (x+h)2 + k

a. Se identifican los valores de p y q donde p= 6 y q=10

b. Se busca una expresión algebraica de la forma

a. Se sustituye y obtenemos

b. Se concluye que

250
3.Expresar el trinomio y 2  5 y  1 de la forma (x+h)2 + k

a. Se identifican los valores de p y q, p=-5 y q=1


b. Se busca una expresión algebraica de la forma

c. Se sustituye y obtenemos

c. Se concluye que

4. Factorizar x 2  4 x  5 se procede de la siguiente manera:

251
Ejercicios

1. Exprese los siguientes trinomios de la forma (x+h)2 + k

252
División de polinomios

Para dividir polinomios, dependiendo de la forma de cada polinomio, se


trabaja de dos maneras, cada una será explicada por separado. Se debe
recordar que los exponentes de las variables se restan y el resultado se deja en el
numerador o en el denominador, dependiendo de la posición en la que se encuentre
originalmente el exponente mayor; mientras que los coeficientes numéricos se
dividen.

Se puede decir que hay 3 reglas básicas usadas para simplificar fracciones o
cocientes de monomios. La primera de ellas es la propiedad para multiplicar
fracciones. Esta propiedad permite expresar una fracción como un producto.

Propiedad para multiplicar fracciones Recuerde:

Si a, b, x, y son números reales con b  0 y y  0 , entonces Si a, b, c , d son números


ax a x reales con b  0 y d  0
 
by b y
a c ac
 
b d bd
Ejemplos

1. Utilice la propiedad anterior para expresar cada una de las


expresiones como un producto de fracciones

15 3  5 3 5
   o bien Recuerde:
28 4  7 4 7
Si a es un número real
a
a0 1
15 5  3 5 3 a
   o bien
28 4  7 4 7

15 3  5 3 5
  
28 2  14 2 14

Si se asume x  y en la propiedad para multiplicar fracciones, se obtiene una regla


para simplificar fracciones.

253
Propiedad de cancelación, conocida como ley de
cancelación.

Si a, b, x son números reales con b  0 y x  0 , entonces


ax a x a a
   1 
bx b x b b

Esta propiedad permite dividir el numerador y el denominador de una fracción por


cualquier número diferente de 0.

Como usted observó siempre que se trabaja con fracciones se hace énfasis en que
los denominadores deben ser diferentes de 0. De aquí en adelante, en esta lección,
se asumirá que todo denominador es distinto de cero

Ejemplos

1) Utilizando la propiedad de cancelación y los axiomas de campo, se


simplifican fracciones o lo que es equivalente se efectúa la división de monomios.

9 xy 3  3  x  y 3 3 x y 3 y 3 y 3y
a) 15 x 2
      1   1    
35 x x 3 5 x x 5 x 5 x 5x
porque la multiplicación es con-
mutativa y asociaciativa en

x8 y 2 x5  x3  y  y x5 y x 3  y
b) 5   5   1  1  x3 y  x3 y
xy x y
5
x y 1
porque la multiplicación es conmutativa
y asociativa en

a2 a2 a2 1 1 1
c)  2  2  5  1 5  5
a 7
a a 5
a a a a

254
2) Utilice las propiedades de cancelación y de división de potencias, y los axiomas de campo
para efectuar la división de monomios. Asuma que los denominadores son distintos de 0.

Solución.

27 x6 z 3  9  x 2  x 4  z 9 x 4 z 9 x 4 z
  
a) 6 x 2 y 2 3  2  x 2  y 2 2 y 2 2y2
porque la multiplicación es con-
mutativa y asociaciativa en

42ab12 7  6ab12 7 6 1 1
b)    7 1  b129  
49a b c 7  7 a b c 7 7 a
7 9 7 9
c

6 1 3 1 6b 3
1 6  b   6
7a c 7a c
Observe que al dividir un monomio por un monomio el resultado no siempre es un
monomio.

3) Utilice las propiedades de cancelación y de división de potencias, y los axiomas


de campo para efectuar la división de monomios. Asuma que los denominadores
son distintos de 0.

Solución.

27 x6 z 3  9  x 2  x 4  z 9 x 4 z 9 x 4 z
  
a) 6 x 2 y 2 3  2  x 2  y 2 2 y 2 2y2
porque la multiplicación es con-
mutativa y asociaciativa en

255
42ab12 7  6ab12 7 6 1 12  9 1
b)     b  
49a7 b9 c 7  7 a7 b9 c 7 7 a7 1 c

6 1 3 1 6b 3
1 6  b   6
7a c 7a c

Observe que al dividir un monomio por un monomio el resultado no siempre es


un monomio.

 a  b 5
5) Simplifique la siguiente fracción .
 a  b 10
Solución

Observe que las expresiones del numerador y el denominador no son monomios,


sin embargo se usa la propiedad de división de potencias.

 a  b 5 1 1
10  10  5 
 a  b  a  b  a  b 5

También se puede realizar de la siguiente manera

En este caso, se divide cada monomio del polinomio por el monomio dado como
divisor y el resultado es la suma o resta de estas divisiones.

Como ejemplos se tienen los siguientes:

256
(25𝑝7 𝑞 3 − 30𝑝𝑞) ÷ (15𝑝3 𝑞 4 ) = (−5𝑥 3 𝑦 3 − 10𝑥 2 𝑦 4 + 15𝑥𝑦 2 ) ÷ (−5𝑥𝑦 3 ) =
25𝑝7 𝑞 3 − 30𝑝𝑞 −5𝑥 3 𝑦 3 − 10𝑥 2 𝑦 4 + 15𝑥𝑦 2
= =
15𝑝3 𝑞 4 −5𝑥𝑦 3
25𝑝7 𝑞 3 30𝑝𝑞 −5𝑥 3 𝑦 3 10𝑥 2 𝑦 4 15𝑥𝑦 2
3 4
− = − + =
15𝑝 𝑞 15𝑝3 𝑞 4 −5𝑥𝑦 3 −5𝑥𝑦 3 −5𝑥𝑦 3
5𝑝4 2 3
− 2 3 𝑥 2 + 2𝑥𝑦 −
3𝑞 𝑝 𝑞 𝑦

(10𝑡 4 𝑤 5 − 8𝑡 3 𝑤 3 ) ÷ (12𝑡 2 𝑤 2 ) = (6𝑎8 𝑏 8 − 3𝑎6 𝑏 6 − 𝑎2 𝑏 3 ) ÷ (3𝑎3 𝑏 2 ) =


10𝑡 4 𝑤 5 − 8𝑡 3 𝑤 3 6𝑎8 𝑏 8 − 3𝑎6 𝑏 6 − 𝑎3 𝑏 2
= =
12𝑡 2 𝑤 2 3𝑎2 𝑏 3
10𝑡 4 𝑤 5 8𝑡 3 𝑤 3 6𝑎8 𝑏 8 3𝑎6 𝑏 6 𝑎3 𝑏 2
− = − − =
12𝑡 2 𝑤 2 12𝑡 2 𝑤 2 3𝑎2 𝑏 3 3𝑎2 𝑏 3 3𝑎2 𝑏 3
5𝑡 2 𝑤 3 2𝑡𝑤 𝑎
− 2𝑎6 𝑏 5 − 𝑎4 𝑏 3 −
6 3 3𝑏

Ejercicios Simplifique cada expresión. Asuma que los denominadores son


distintos de 0.

8a2
c)   5
7
5bc
d)  a  b 
st 3
a) b)
2a 10 b 6  st   a  b 12

e)
 a  b 15
f)
2, 4  10 5
g)
 3a    2 a 
5 4

 a  b 3 4, 8  10 2  6a  3 2

13c d    8x2 y 
4 2 4

h) i)
 26cd   4x y 
2 3 2 5

257
División de un polinomio por un monomio
Para efectuar la división de un polinomio por un monomio se utiliza la
propiedad distributiva de la multiplicación respecto de la adición y se
convierte el procedimiento en una división de monomios.

Ejemplo

Para dividir el binomio x 5  3 x 3 por el monomio 3x 2 se puede expresar como una


fracción y se utiliza la distributividad de la multiplicación respecto de la adición:
x5  3x3
  x5  3x3  
1
2
 2
3x 3x
5
1 x 1 3x3
   
3x 2 1 3x2 1
x5 3x3 x3
  x
3x 2 3x2 3
Ejemplo

Para dividir el polinomio 4x y  6x yz  2xyz  3 y


5 2 3 4 3 2

2
por 4x yz se procede de manera similar:

4 x 5 y 2  6 x 3 yz 4  2 xyz 3  3 y 2

4 x 2 yz
1
2
4 x yz
 4 x 5 y 2  6 x 3 yz 4  2 xyz 3  3 y 2  

1 1
2
 4x5 y 2  2
 6 x 3 yz 4 
4 x yz 4 x yz
1 1
2
 2 xyz 3  2
 3y2 
4 x yz 4 x yz
4x5 y 2 6 x 3 yz 4 2 xyz 3 3y 2
   
4 x 2 yz 4 x 2 yz 4 x 2 yz 4 x 2 yz
x3 y 3 xz 3 z2 3y
  
z 2 2x 4x2 z

258
Ejercicios

1) Efectúe cada una de las siguientes divisiones:

a) (8 x5 y  12 x3 yz 3  16 xyz  3 y 2 )   4 x 3 y 

b) (32 x4 z 2  96 x2 y 4  48 x6 y 8 )  16 x 2 y 2 

c) (45a4 b2  75a3b  30a4 )  15a2 

División de un polinomio entre polinomio

En este caso se utiliza el algoritmo de la división, tal como se conoce con


los números enteros. Para recordar, se resuelve el siguiente ejemplo:

𝟏 𝟐 𝟕 5 45 Operaciones

− 9 0 𝟐𝟖 45 ∙ 2 = 90

3 7 𝟓

− 3 6 0 45 ∙ 8 = 360

𝟏 𝟓

Como ya se sabe, en el dividendo (1275) se busca la cifra más pequeña que sea
divisible por el divisor (45). Luego, se realiza la división, cuyo resultado se anota
debajo del divisor. Cada vez que esto se repite, se multiplica el último número
anotado por el divisor y este producto se resta al número que se utilizó como
dividendo. La división termina cuando el posible dividendo es menor que el divisor.
A ese último dividendo se le llama residuo (15) y al número que se escribió debajo
del divisor se le llama cociente (28).

Y, recordando, en el sistema de numeración decimal, cuando en una casilla no se


tiene un valor determinado, se coloca un 0. De manera similar se trabaja la división
de polinomios. Para realizar esta operación es necesario que, tanto, el divisor como
el dividendo, estén ordenados por el exponente de la variable, de mayor a menor.

La diferencia es que se divide el término de mayor grado (exponente) del dividendo


por el término de mayor grado del divisor. Se anota el resultado y, de acuerdo con
el algoritmo, se multiplica y se resta para obtener un nuevo dividendo. El final de

259
la división se da cuando el mayor grado del dividendo es menor que el mayor del
divisor.

Ejemplos

Para explicar con más claridad el proceso, se resuelven con detalle los siguientes
ejemplos:

1. La división de polinomios  6x 2
 8x  8    3x  1 se realiza de la siguiente
manera:

Paso1: En este caso tanto el dividendo como el divisor son polinomios en una
variable. Se debe verificar que estén ordenados, de acuerdo con el grado de la
variable, de mayor a menor. Si no lo están se ordenan.

Paso 2. Se busca un monomio que al multiplicarlo por 3x resulte 6x 2 , esto es, 2x .


Ya que 3 x  2 x  6 x 2

6x2  8x  8
3x  1
2x
Paso3. Se multiplica el divisor por 2x y se coloca el resultado debajo del dividendo
y luego se resta.

6x2  8x  8 3x  1
  6x2  2x  2x
0  6x  8
Paso 4. Se busca un monomio que al multiplicarlo por 3x resulte 6x y se procede
como en el paso 3.

6x2  8x  8 3x  1
  6x2  2x  2x  2
0  6x  8
  6x  2 
06
Paso 5. Se continúa el proceso hasta que en el residuo se tiene un polinomio cuyo
grado es estrictamente menor que el grado del polinomio divisor. En este caso el
grado del divisor es 1 y el grado del residuo es 0, por lo que se termina la división.

Se tiene entonces que el cociente de la división es  3 x  1 y el residuo es 6.

260
De este procedimiento se desprende que:

 6x 2
 8x  8    3x  1 2x  2   6

o también se escribe:

6 x2  8 x  8 6
  2x  2   .
3x  1 3x  1

En este caso, como el residuo es diferente de 0, el trinomio  6x 2


 8 x  8  no es
dividido en forma exacta por el binomio  3 x  1 . Es decir el trinomio  6 x2  8 x  8 
no es divisible por el binomio  3 x  1 .

Cuando se efectúa una división, es posible expresar

dividendo = cociente  divisor + residuo

o bien
dividendo residuo
 cociente  .
divisor divisor

2. La división de polinomios, 12x2  19 x  21   3x  7  se realiza de la siguiente


manera:

Paso 1: Primero se verifica que estén ordenados en potencias decrecientes. Si no


lo están se ordenan.

Paso 2. Se busca un monomio que al multiplicarlo por 3x resulte 12x 2 .


12x2  19 x  21 3x  7
4x
Paso 3. Se multiplica el divisor por 4x , se coloca el resultado debajo del dividendo
y se resta.

12 x2  19 x  21 3x  7
  12 x2  28 x  4x
0  9 x  21
Paso 4. Se busca un monomio que al multiplicarlo por 3x resulte 9x y se procede
como en el paso 3.

261
12 x 2  19 x  21 3x  7
  12 x 2  28 x  4x  3
0  9 x  21
  9 x  21
0

Se tiene entonces:

12x 2
 19x  21   3x  7  4x  3   0   3x  7  4x  3  .

Se desprende que:

12 x2  19 x  21 0
  4x  3    4x  3 .
3x  7 3x  1

En este caso, como el residuo es 0, el trinomio 12 x2  19 x  21 es dividido en forma


exacta por el binomio  3x  7  .

Cuando el residuo es igual a 0 se dice que el divisor divide al dividendo en forma


exacta y tanto el cociente como el divisor son factores del dividendo, esto es, el
polinomio 12x2  19x  21 es divisible por 3x + 7. Así  4 x  3  y  3x  7  son factores
de 12 x2  19 x  21 y una factorización de 12 x2  19 x  21 es  4 x  3  (3x  7) .

3. Enseguida se hace la división de polinomios 4x  10x  3x  4 x  5x :


3 5 4 2
   
Paso 1: Primero verificar que estén ordenados en potencias decrecientes de x. Si
no lo están se ordenan.

 10x 5
 3x4  4 x3    5x2  4 x 

Paso 2. Se busca un monomio que al multiplicarlo por 5x 2 resulte 10x 5 .


 10 x 5  3 x 4  4 x 3 5 x 2  4 x
2x3
Paso 3. Se multiplica el divisor por 2x , se coloca el resultado debajo del dividendo
3

y se restan los polinomios.

262
 10 x 5  3 x 4  4 x 3 5 x 2  4 x
  10 x 5  8 x 4  2x3
0  5x4  4 x 3
Paso 4. Se busca un monomio que al multiplicarlo por 5x 2 resulte 5x 4 y se
procede como en el paso 3.

 10 x 5  3x 4  4 x 3 5x 2  4 x
  10 x 5  8 x 4  2x3  x2
0  5x 4  4 x 3
  5x 4  4 x 3 
0

Se tiene entonces:

 10x 5
 3x4  4x3    5x2  4x  2x3  x2   0   5x2  4x  2x3  x2 
10 x5  3x4  4 x3
de donde:  2x3  x2
5x  4 x
2

En este caso, como el residuo es 0, el trinomio 10 x  3 x  4 x es dividido en


5 4 3

forma exacta por el binomio 5 x 2  4 x y tanto 5 x  4 x como 2x  x son factores


2 3 2

de 10 x  3 x  4 x . El trinomio 10 x  3 x  4 x es divisible por 5 x  4 x .


5 4 3 5 4 3 2

División de polinomios cuando hay más de una variable

4. Se va a realizar la división:  2 x2  11y 2  3xy    x  2 y 

Paso 1: Se ordena el dividendo y el divisor respecto de una de las variables, según


potencias decrecientes. En este caso se elige respecto de x .

 2x 2
 3xy  11y 2    x  2 y 

2x2
Paso 2. Se busca un monomio que al multiplicarlo por x resulte 2x 2 :  2x
x

2 x 2  3xy  11y 2 x  2 y
2x

263
Paso 3. Se multiplica el divisor por 2x , se coloca el resultado debajo del dividendo
y se resta.

2 x 2  3xy  11y 2 x  2y
  2 x 2  4 xy  2x
0  7 xy  11y 2

Paso 4. Se busca un monomio que al multiplicarlo por x resulte 7xy y se procede


como en el paso 3.
Recuerde:
2 x 2  3 xy  11y 2 x  2y
7 xy
  2 x 2  4 xy  2x  7 y  7y
x
0  7 xy  11y 2
  7 xy  14 y 2 
0  3y2

Paso 5. El grado de la variable x en el divisor es 1 y en el residuo es 0


( 0  x 0  0  1  0 ), por lo que se termina la división.

Se tiene así que:

 2x 2
 3xy  11y 2    x  2 y  2x  7 y   3y 2
de donde:

2 x2  3xy  11y 2 3y2


  2x  7 y  
x  2y x  2y

Si para realizar esta misma división se ordena el dividendo y divisor según


potencias decrecientes de la variable y se obtiene:
Paso 1:
 11y 2
 3xy  2x2    2 y  x 

Paso 2. Se busca un monomio que al multiplicarlo por 2y resulte 11y :


2

11y 2 11
 y.
2 y 2

264
 11y 2  3xy  2 x 2 2 y  x
11
y
2

11
Paso 3. Se multiplica el divisor por y , se coloca el resultado debajo del dividendo
2
y se restan los polinomios.

 11y 2  3xy  2 x 2 2 y  x

  11y 2  xy 
11 11
y
 2  2
5
0  xy  2 x 2
2

5
Paso 4. Se busca un monomio que al multiplicarlo por 2y resulte xy y se
2
5 5
xy xy 5  1 x 5 x 5
procede como en el paso 3: 2  2      x.
2 y  2 y 2  2 1 4 4
1

 11y 2  3xy  2 x 2 2 y  x

  11y 2  xy 
11 11 5
y x
 2  2 4
5
0  xy  2 x 2
2
5
  xy  x 2 
5
 2 4 
3
0  x2
4
Paso 5. El grado de la variable y en el divisor es 1 y en el residuo es 0, por lo que
se termina la división.
Se tiene entonces:

 11y  3xy  2 x2    2 y  x   y  x   x 2 de donde:


2 11 5 3
 2 4  4

265
3 2
11y  3xy  2 x
2 2 x
 11 5 
  y  x  4
2 y  x  2 4  2 y  x

5) (3𝑥 2 + 2𝑥 − 8) ÷ (𝑥 + 2)

3𝑥 2 +2𝑥 −8 𝑥+2

−3𝑥 2 −6𝑥 3𝑥 − 4

−4𝑥 −8

+4𝑥 +8

Así se tiene que 3𝑥 2 + 2𝑥 − 8 = (𝑥 + 2)(3𝑥 − 4) + 0, además de que el cociente es 𝑥 +


2 y el residuo es0.

6) (𝑥 3 − 125) ÷ (𝑥 − 5) = (𝑥 3 + 0𝑥 2 + 0𝑥 − 25) ÷ (𝑥 − 5)

𝑥3 +0𝑥 2 +0𝑥 −25 𝑥−5

−𝑥 3 +5𝑥 2 𝑥 2 + 5𝑥 + 25

+5𝑥 2 +0𝑥 −25

−5𝑥 2 +25𝑥

25𝑥 −25

−25𝑥 +25

Así se tiene que 𝑥 3 − 125 = (𝑥 − 5)(𝑥 2 + 5𝑥 + 25) + 0, además de que el cociente es


𝑥 2 + 5𝑥 + 25 y el residuo es0.

266
7) (5𝑥 3 + 3𝑥 2 + 2) ÷ (𝑥 + 𝑥 2 + 3) = (5𝑥 3 + 3𝑥 2 + 0𝑥 + 2) ÷ (𝑥 2 + 𝑥 + 3)

5𝑥 3 +3𝑥 2 +0𝑥 +2 𝑥2 + 𝑥 + 3

−5𝑥 3 −5𝑥 2 −15𝑥 5𝑥 − 2

−2𝑥 2 −15𝑥 +2

+2𝑥 2 +2𝑥 +6

−13𝑥 +8

Así se tiene que 5𝑥 3 + 3𝑥 2 + 2 = (𝑥 2 + 𝑥 + 3)(5𝑥 − 2) + (−13𝑥 + 8), además de que el


cociente es 5𝑥 − 2 y el residuo es −13𝑥 + 8.

Ejercicios

1) Compruebe que:

a) 2 x  3xy  11y
2 2
   x  2y  2x  7 y   3y 2

 11 y  5 x   3 x2  11y 2  3xy  2x 2
b)  2 y  x   
 2 4  4
2.) Efectúe las divisiones.

a) x  9x  20   x  5 
2

b)  a  2a2  3a  5    a  3 
3

c)  3x2  6 y 2  8xy    x  3y 
d)  2x 3
 3x 2  4    x 2  3x 

3) ¿Es x  3 un factor de x 2  x  12 ? Justifique su respuesta.

4.División de monomio entre monomio

4a 3 b 2  5a 4 b 3 c
1.  3) 
 2ab  a 2b

267
 20mx 2 y 3  9d 7
2)  4) 6

4 xy 3 3d

 5a 2  a 3b 4 c
 
5)
a 8) 3 4
a b

 a 2b 54x 2 y 2 z 3
 
6)
 ab 9)
 6 xy z
2 3

 8a 2 x 3  xy 2
 
7)
 8a x
2 3 10)
2y

5x 4 y 5 16m 6 n 4
 
12)
 6x y
4 11)
 5n 7

 108a 7 b 3 c 8  2m 2 n 3
 
13)
 20b 5 c 8 14)
 3m 5 n 6

268
5a m b n c

15)
 6a b c
3 4

5.División polinomio entre monomio

a 2  ab
1) 
a

3a 3  5ab 2  6a 2 b 3
2) 
 2a

3a 3  6a 2 b  9ab 2
3) 
3a

6m 3  8m 2 n  20mn 2
4) 
 2m

28x 4  14x 3
5) 
7x 2

20x 3  5 x 2  10x
6) 
10x 2

10 x  5 y
7) 
5

269
8 y 3  32 y 2  16 y
8) 
 4y2

10x 2  8 x
9) 
6 x 1

15x 3  10x 2
10) 
5x 2

8. División entre polinomio entre polinomio

1) Dividir x2 + 2x – 3 entre x+3

2) Dividir x2 – 20 + x entre x+5

3) Dividir m2 – 11m + 30 entre m–6

4) Dividir x2 + 15 – 8x entre 3–x

5) Dividir x2 + 2x – 17 entre x–3

6) Dividir 2x3 – 2 – 4x entre 2 + 2x

270
7) Dividir x4 + x3 – 3x2 + 1 entre x2 – 3

8) Dividir 3x2 + 2x – 8 entre x+2

9) Dividir x4 – x2 – 2x + 2 entre x2 – x – 1

10) Dividir x5 – x4 + 6x2 – 5x + 3 entre x2 – 2x + 3

Operaciones con Expresiones algebraicas en ℝ

Se llama expresión algebraica al número real que resulta al operar números y letras
que representan números reales, por medio de las operaciones fundamentales
(suma, la resta, la multiplicación, la división, las potencias y las raíces).

A los números se les llama constantes y a las letras se les llama variables. Las
expresiones algebraicas separadas por el signo más o el signo menos se llaman
términos algebraicos.

Un monomio es una expresión algebraica constituida por un solo término donde:


1. No aparecen letras en el denominador.
2. Los exponentes de las variables del numerador son números naturales o
números enteros positivos.
Una expresión algebraica formada por la suma o resta de monomios con diferentes
factores literales se llama polinomio o expresión algebraica polinómica. Los
polinomios se clasifican en:
1. monomio.
2. binomio: Consiste en la suma o resta de dos monomios.
3. trinomio: Consiste en la suma o resta de tres monomios.
4. polinomio de más de tres términos: Consiste en la suma o resta de cuatro o
más monomios.

271
Se llama expresión algebraica polinómica fraccionaria a aquella donde el numerador
y el denominador son polinomios.
Las fracciones algebraicas son aquellas en las que tanto el numerador como el
denominador son polinomios. El proceso de simplificarlas implica factorizar por
separado cada polinomio y “cancelar” únicamente aquellos factores que se repiten
tanto en el numerador como en el denominador.

Simplificación de expresiones algebraicas polinómicas


Para simplificar expresiones algebraicas fraccionarias polinómicas se factoriza el
numerador y el denominador y luego se aplica la ley de la cancelación de la
multiplicación que expresa lo siguiente:

∀𝑎, 𝑏, 𝑐 ∈ ℝ,
𝑎𝑏 𝑏
𝑎≠0 = .1
𝑎𝑐 𝑐

Ejemplos

2(𝑥−7)
1. Simplifique al máximo .
4(𝑥−7)(3𝑥−2)
Solución
2(𝑥 − 7)
4(𝑥 − 7)(3𝑥 − 2)
1
= .
2(3𝑥 − 2)

𝑥 2 +3𝑥−10
2. Simplifique al máximo .
2𝑥 2 −3𝑥−2

Solución
Se factoriza tanto el numerador como el denominador
𝑥 2 + 3𝑥 − 10
2𝑥 2 − 3𝑥 − 2
(𝑥 − 2)(𝑥 + 5)
=
(2𝑥 + 1)(𝑥 − 2)

𝑥+5
= .
2𝑥 + 1

272
𝑥−𝑥 3
3. Simplifique al máximo .
𝑥 2 −4𝑥+3

Solución
𝑥 − 𝑥3
𝑥 2 − 4𝑥 + 3
𝑥(1 + 𝑥)(1 − 𝑥)
=
(𝑥 − 3)(𝑥 − 1)
𝑥(1 + 𝑥)(𝑥 − 1)
=−
(𝑥 − 3)(𝑥 − 1)
𝑥(1 + 𝑥)
=− .
(𝑥 − 3)
𝑥(2−3𝑥)−4+6𝑥
4. Simplifique al máximo .
3𝑥𝑦−2𝑦+6𝑥−4

Solución

𝑥(2 − 3𝑥) − 4 + 6𝑥
3𝑥𝑦 − 2𝑦 + 6𝑥 − 4
𝑥(2 − 3𝑥) − 2(2 − 3𝑥)
=
3𝑥(𝑦 + 2) − 2(𝑦 + 2)
(2 − 3𝑥)(𝑥 − 2)
=
(𝑦 + 2)(3𝑥 − 2)
−𝑥 + 2
= .
𝑦+2
5. Simplifique

𝑥2 − 1 2𝑚2 − 4𝑚 𝑎𝑏 − 2𝑎𝑐 − 2𝑑𝑐 + 𝑑𝑏


= = =
𝑥 2 + 2𝑥 + 1 𝑚2 − 4 2𝑎𝑏 − 𝑎𝑐 − 𝑑𝑐 + 2𝑑𝑏
(𝑥 + 1)(𝑥 − 1) 2𝑚(𝑚 − 2) (𝑎 + 𝑑)(𝑏 − 2𝑐)
= = =
(𝑥 + 1)2 (𝑚 + 2)(𝑚 − 2) (2𝑏 − 𝑐)(𝑎 + 𝑑)
𝑥−1 2𝑚 𝑏 − 2𝑐
𝑥+1 𝑚+2 2𝑏 − 𝑐

273
𝑎2 𝑛2 − 36𝑎2 𝑥 2 − 𝑥 − 20 2𝑥 3 − 6𝑥 2 − 36𝑥
= = =
𝑎𝑛2 + 𝑎𝑛 − 30𝑎 𝑥 2 − 10𝑥 + 25 𝑥 2 − 13𝑥 + 42
𝑎2 (𝑛 + 6)(𝑛 − 6) (𝑥 − 5)(𝑥 + 4) 2𝑥(𝑥 + 3)(𝑥 − 6)
= = =
𝑎(𝑛 + 6)(𝑛 − 5) (𝑥 − 5)2 (𝑥 − 6)(𝑥 − 7)
𝑎(𝑛 − 6) 𝑥+4 2𝑥(𝑥 + 3)
𝑛−5 𝑥−5 𝑥−7

Ejercicios

1. Simplifique al máximo:

𝑥 2 −1
a. 𝑥 2 −2𝑥+1

4𝑥 2 𝑦−9𝑦
b.4𝑥 2 +12𝑥+9

𝑥 2 −4
c. 𝑥−2

𝑎2 −𝑏 2
d.𝑎2 −2𝑎𝑏+𝑏2

3𝑥 2 −5𝑥−2
e. 𝑥 2 −4

2−𝑥−3𝑥 2
f. 6𝑥 2 −𝑥−2

𝑥 2 −25
g.𝑥 2 −8𝑥+15

4𝑥 2 𝑦−9𝑦
h.4𝑥 2 +12𝑥+9

274
12𝑥 2 +3𝑥
i. 20𝑥 2 +9𝑥+1

6𝑥−3𝑥𝑦−6𝑦+3𝑦 3
j. 6𝑥−3𝑥𝑦+6𝑦−3𝑦 3

Operaciones con expresiones algebraicas polinómicas

Suma y resta de expresiones algebraicas polinómicas

Para sumar y restar expresiones algebraicas polinómicas se debe aplicar la


siguiente propiedad:
𝑎 𝑐 𝑎⋅𝑑±𝑏⋅𝑐
∀𝑎, 𝑏, 𝑐 ∈ ℝ, 𝑏𝑑 ≠ 0 ±𝑑 = .
𝑏 𝑏⋅𝑑

Antes de resolver estas operaciones, es necesario calcular el mínimo común


múltiplo de los denominadores. Por ejemplo, con números enteros se tiene que
30 = 2 ∙ 3 ∙ 5 y que 24 = 23 ∙ 3 y el resultado que se obtiene es 23 ∙ 3 ∙ 5 = 120. Como se
puede observar en el ejemplo, se toma de cada factor su máxima potencia.

De la misma manera se trabaja con polinomios. Por ejemplo, al determinar el


mínimo común múltiplo de los polinomios 12𝑡 3 + 12𝑡 2 𝑠 + 3𝑡𝑠 2 y 4𝑡 4 − 𝑠 2 𝑡 2 ,
se tiene que sus factorizaciones son:

12𝑡 3 + 12𝑡 2 𝑠 + 3𝑡𝑠 2 = 3𝑡(2𝑡 + 𝑠)2

4𝑡 4 − 𝑠 2 𝑡 2 = 𝑡 2 (2𝑡 + 𝑠)(2𝑡 − 𝑠)

Finalmente, el mínimo común múltiplo corresponde a 3𝑡 2 (2𝑡 + 𝑠)2 (2𝑡 − 𝑠).

Para resolver las sumas y restas con fracciones algebraicas se debe hacer el
procedimiento de homogeneizarlas, es decir, conseguir que tengan un denominador
común, para lo cual se debe determinar el mínimo común múltiplo de sus
denominadores. Luego, con cada fracción por separado, se divide el nuevo
denominador por el anterior y ese resultado se multiplica por el numerador que
tenía la fracción originalmente. Finalmente, se resuelve la suma o resta y, de ser
posible, se simplifica el resultado final.

275
Ejemplos
𝑥 2
1. Simplifique al máximo + 𝑥−1.
𝑥+1
Solución
𝑥 2
+
𝑥+1 𝑥−1

𝑥(𝑥 − 1) + 2(𝑥 + 1)
=
(𝑥 + 1)(𝑥 − 1)
𝑥 2 − 𝑥 + 2𝑥 + 2
=
(𝑥 − 1)(𝑥 + 1)
𝑥2 + 𝑥 + 2
= .
(𝑥 − 1)(𝑥 + 1)

3𝑥 2 6𝑥
2. Simplifique al máximo + 4−𝑥 2
𝑥 2 −4

Solución
3𝑥 2 6𝑥
+
𝑥 − 4 4 − 𝑥2
2

3𝑥 2 6𝑥
= 2 − 2
𝑥 −4 𝑥 −4
3𝑥 2 − 6𝑥
=
(𝑥 − 2)(𝑥 + 2)
3𝑥(𝑥 − 2)
=
(𝑥 − 2)(𝑥 + 2)
3𝑥
= .
𝑥+2

3. Simplifique al máximo
2 𝑥+4
− 2
𝑥−2 𝑥 −𝑥−2
Solución

2 𝑥+4
− 2
𝑥−2 𝑥 −𝑥−2
2 𝑥+4
= −
𝑥 − 2 (𝑥 − 2)(𝑥 + 1)
2(𝑥 + 1) − (𝑥 + 4)
=
(𝑥 − 2)(𝑥 + 1)
2𝑥 + 2 − 𝑥 − 4
=
(𝑥 − 2)(𝑥 + 1)

276
𝑥−2
=
(𝑥 − 2)(𝑥 + 1)
1
= .
𝑥+1

2 1
4. Simplifique al máximo −
𝑥 2 −4 2𝑥−4

Solución
2 1

𝑥 2 − 4 2𝑥 − 4
2 1
= −
(𝑥 − 2)(𝑥 + 2) 2(𝑥 − 2)
2 ⋅ 2 − (𝑥 + 2)
=
2(𝑥 − 2)(𝑥 + 2)
4−𝑥−2
=
2(𝑥 − 2)(𝑥 + 2)
2−𝑥
=
2(𝑥 − 2)(𝑥 + 2)
−1
=
2(𝑥 + 2)
−1
= .
2𝑥 + 4

2𝑥 2 −6𝑥+1 2𝑥
5. Simplifique al máximo − .
2𝑥 2 −7𝑥−4 2𝑥+1

Solución
2𝑥 2 − 6𝑥 + 1 2𝑥
2

2𝑥 − 7𝑥 − 4 2𝑥 + 1
2𝑥 2 − 6𝑥 + 1 2𝑥
= −
(2𝑥 + 1)(𝑥 − 4) 2𝑥 + 1
2𝑥 2 − 6𝑥 + 1 − 2𝑥(𝑥 − 4)
=
(2𝑥 + 1)(𝑥 − 4)
2𝑥 − 6𝑥 + 1 − 2𝑥 2 + 8𝑥
2
=
(2𝑥 + 1)(𝑥 − 4)
2𝑥 + 1
=
(2𝑥 + 1)(𝑥 − 4)
1
= .
(𝑥 − 4)

277
6. Simplifique
20 15𝑎 20 15𝑎
− = −
𝑎3 − 25𝑎 𝑎2 + 2𝑎 − 15 𝑎(𝑎 + 5)(𝑎 − 5) (𝑎 + 5)(𝑎 − 3)

El mínimo común múltiplo de los denominadores es 𝑎(𝑎 + 5)(𝑎 − 5)(𝑎 − 3).

Al dividir el nuevo denominador por cada uno de los originales se tiene:

𝑎(𝑎 + 5)(𝑎 − 5)(𝑎 − 3)


=𝑎−3
𝑎(𝑎 + 5)(𝑎 − 5)

𝑎(𝑎 + 5)(𝑎 − 5)(𝑎 − 3)


= (𝑎 − 5)(𝑎 − 3)
(𝑎 + 5)(𝑎 − 3)

Así, se tiene que

20 15𝑎 20 15𝑎
− 2 = − =
𝑎3 − 25𝑎 𝑎 + 2𝑎 − 15 𝑎(𝑎 + 5)(𝑎 − 5) (𝑎 + 5)(𝑎 − 3)

20(𝑎 − 3) − 15𝑎(𝑎 − 5)(𝑎 − 3) 20𝑎 − 60 − 15𝑎(𝑎2 + 2𝑎 − 15)


= =
𝑎(𝑎 + 5)(𝑎 − 5)(𝑎 − 3) 𝑎(𝑎 + 5)(𝑎 − 5)(𝑎 − 3)

20𝑎 − 60 − 15𝑎3 − 30𝑎2 + 225𝑎 −15𝑎3 − 30𝑎2 + 245𝑎 − 60


= =
𝑎(𝑎 + 5)(𝑎 − 5)(𝑎 − 3) 𝑎(𝑎 + 5)(𝑎 − 5)(𝑎 − 3)

−5(𝑎 − 3)(3𝑎2 + 15𝑎 − 4) −5(3𝑎2 + 15𝑎 − 4)


=
𝑎(𝑎 + 5)(𝑎 − 5)(𝑎 − 3) 𝑎(𝑎 + 5)(𝑎 − 5)

Así, el resultado final es

20 15𝑎 −5(3𝑎2 + 15𝑎 − 4)


− =
𝑎3 − 25𝑎 𝑎2 + 2𝑎 − 15 𝑎(𝑎 + 5)(𝑎 − 5)

7. Como ejemplo, se puede resolver la siguiente operación:


𝑚+1 2 (𝑚 + 1)2 + 2(𝑚2 − 𝑚) 𝑚2 + 2𝑚 + 1 + 2𝑚2 − 2𝑚
+ = = =
𝑚2 − 𝑚 𝑚 + 1 𝑚(𝑚 − 1)(𝑚 + 1) 𝑚(𝑚 − 1)(𝑚 + 1)

3𝑚2 + 1
𝑚(𝑚 − 1)(𝑚 + 1)

8. Otro ejemplo consiste en resolver la operación


3𝑥 − 1 2𝑥 3𝑥 − 1 2𝑥
2
− 2 = − =
𝑥 − 10𝑥 − 24 𝑥 + 8𝑥 + 12 (𝑥 − 12)(𝑥 + 2) (𝑥 + 2)(𝑥 + 6)

(3𝑥 − 1)(𝑥 + 6) − 2𝑥(𝑥 − 12) (3𝑥 2 + 17𝑥 − 6) − (2𝑥 2 − 24𝑥)


= =
(𝑥 + 6)(𝑥 − 12)(𝑥 + 2) (𝑥 + 6)(𝑥 − 12)(𝑥 + 2)

278
𝑥 2 + 41𝑥 − 6
(𝑥 + 6)(𝑥 − 12)(𝑥 + 2)

9. Como ejemplo, se puede resolver la siguiente operación:


3𝑥 − 12 2 (3𝑥 − 12) + 2(𝑥 − 1) 3𝑥 − 12 + 2𝑥 − 2 5𝑥 − 14
2
+ = = =
𝑥 +𝑥−2 𝑥+2 (𝑥 + 2)(𝑥 − 1) (𝑥 + 2)(𝑥 − 1) (𝑥 + 2)(𝑥 − 1)

10. ejemplo es la siguiente operación:


𝑎−2 𝑎 − 2 7 (𝑎 − 2) − 7 ∙ (𝑎 + 2) 𝑎 − 2 − 7𝑎 − 14
−7= − = + =
𝑎+2 𝑎+2 1 𝑎+2 𝑎+2
−6𝑎 − 16 −2(3𝑎 + 8)
=
𝑎+2 𝑎+2

11. Un último ejemplo es la siguiente operación:


1 𝑥−𝑦 1 𝑥−𝑦 1 ∙ (3𝑥 + 2𝑦) + (𝑥 − 𝑦)
+ 2 = + = =
3𝑥 − 2𝑦 9𝑥 − 4𝑦 2 3𝑥 − 2𝑦 (3𝑥 + 2𝑦)(3𝑥 − 2𝑦) (3𝑥 + 2𝑦)(3𝑥 − 2𝑦)

3𝑥 + 2𝑦 + 𝑥 − 𝑦 4𝑥 + 𝑦
=
(3𝑥 + 2𝑦)(3𝑥 − 2𝑦) (3𝑥 + 2𝑦)(3𝑥 − 2𝑦)

Ejercicios

1. Simplifique al máximo:

4𝑥+5 1−3𝑥
a. −
2𝑥−1 2𝑥−1

𝑥−1 𝑥
b.𝑥 2 −4 − 𝑥 2 +4𝑥+4

1 𝑥+3
c. 𝑥+1 − 𝑥 2 −2𝑥+1

𝑥 𝑥+1
d.𝑥−2 − 𝑥 2 −3𝑥+2

279
2 𝑥−4
e. 𝑥+3 − 𝑥 2 −9

𝑎 𝑎𝑥+𝑥 2
f. 𝑥−𝑎 − 𝑥 2 −𝑎2

𝑎 𝑎
g.𝑥 − 𝑥 2 +𝑥

2 9
h. −
3𝑥+1 9𝑥 2 +6𝑥+1

1 𝑥+2 3
i. − + 𝑥3
𝑥 𝑥2

5 8 3
j. 𝑥−1 − (𝑥−1)2 − (𝑥−1)3

3𝑥 3
k. 𝑥 2 −9 + 3−𝑥

𝑥 1
l. − 𝑥+1
𝑥 2 −1

𝑥−2𝑦 2𝑥−𝑦
m. − 2𝑥+𝑦
𝑥+2𝑦

𝑚−𝑛 𝑚2 +6𝑚𝑛+𝑛2
n.𝑚+𝑛 + 𝑚2 −𝑛2

280
2. Resuelva en su cuaderno las siguientes operaciones y simplifique el resultado si es
posible:

a b 2a b b 4a x 2 y xy
1) 4)
12 15 30 15x 2xy

4x 2 2x 3 2 3y 1 y2 2y
2) 5)
8x 4x y y2 y 3

x 3y x 2 y 4xy 2 2 4y 7 1
3) 6) 2
3xy 5x 2 y 2 y 3 y y 6 y 2

a b a b 4ab a b b a 2b
7) 12)
a b b a a b2
2
ab a 2
ab ab b2

x 3 4x 1 2x 6 a b a2 b2
8) 13)
20x 10 60x 30 40x 20 a b a b a2 b2

a 1 x 1 1
9) 2
14) 2
ab b b y xy y x

5 10y x y x 2y 1
10) 2
15) 2
y 5 y 25 x x2 xy y x

6 3 3 16)
11)
x2 1 x x2 x 1 1 a 1 1
(a 1)(a 2) (a 1)(a 2)(a 3) a 1

281
Respuestas:

5a b 2(a b) 12) 0
1) 7)
60 a b

1 1 2b 2b
2) 8) 13) o
2x 30 a b b a

8x 3y 1 y
3) 9) 14)
15xy a b x(y x)

13x 11 11 13x 5 (y x)
4) o 10) 15)
30x 30x y 5 x2

4y 3 3 a 5
5) 11) 16)
y2 x (a 1)(a 3)

3y 3y
6) o
(y 3)(y 2) (3 y)(y 2)

282
Multiplicación de expresiones algebraicas polinómicas

Para multiplicar expresiones algebraicas polinómicas se debe aplicar la


siguiente propiedad:

𝑎 𝑐 𝑎⋅𝑐
∀𝑎, 𝑏, 𝑐 ∈ ℝ, 𝑏𝑑 ≠ 0 ⋅ = .
𝑏 𝑑 𝑏⋅𝑑

El procedimiento para resolver las multiplicaciones de fracciones algebraicas


consiste en factorizar el numerador y el denominador de cada fracción y, de ser
posible, simplificar cada una de ellas. Luego, se multiplican entre sí los
numeradores para obtener el numerador del resultado y, de la misma forma, se
multiplican entre sí los denominadores. Finalmente, se simplifica el resultado al
máximo.

Ejemplos
𝑥 2 −6𝑥+9 −2𝑥+2
1. Simplifique al máximo ⋅ .
𝑥 2 −1 𝑥−3

Solución
𝑥 2 − 6𝑥 + 9 −2𝑥 + 2

𝑥2 − 1 𝑥−3
(𝑥 − 3)(𝑥 − 3) −2(𝑥 − 1)
=− ⋅
(𝑥 − 1)(𝑥 + 1) (𝑥 − 3)
−2(𝑥 − 3)
= .
𝑥+1

𝑥 2 −3𝑥 3−2𝑥−𝑥 2
2. Simplifique al máximo ⋅
𝑥 2 −9 𝑥 2 −𝑥

Solución
𝑥 2 − 3𝑥 3 − 2𝑥 − 𝑥 2

𝑥2 − 9 𝑥2 − 𝑥
𝑥(𝑥 − 3) −(𝑥 2 + 2𝑥 − 3)
= ⋅
(𝑥 − 3)(𝑥 + 3) 𝑥(𝑥 − 1)
𝑥(𝑥 − 3) −(𝑥 − 1)(𝑥 + 3)
= ⋅
(𝑥 − 3)(𝑥 + 3) 𝑥(𝑥 − 1)
= −1.

283
𝑥 2 −𝑦 2 𝑥−𝑦
3. Simplifique al máximo ∙ .
𝑥 2 −2𝑥𝑦+𝑦 2 3𝑥+3𝑦

Solución

𝑥2 − 𝑦2 𝑥−𝑦
2 2

𝑥 − 2𝑥𝑦 + 𝑦 3𝑥 + 3𝑦
(𝑥 + 𝑦)(𝑥 − 𝑦) (𝑥 − 𝑦)
= ⋅
(𝑥 − 𝑦)2 3(𝑥 + 𝑦)
1
= .
3

5𝑥 2 𝑥 2 −𝑥−12
3. Simplifique al máximo
𝑥 3 +3𝑥 2 ⋅ 5𝑥 2 −20𝑥

Solución
5𝑥 2 𝑥 2 − 𝑥 − 12

𝑥 3 + 3𝑥 2 5𝑥 2 − 20𝑥
5𝑥 2 (𝑥 − 4)(𝑥 + 3)
= 2 ⋅
𝑥 (𝑥 + 3) 5𝑥(𝑥 − 4)
1
= .
𝑥

4. Por ejemplo, se puede resolver la siguiente operación:


𝑦2 𝑦 3 − 2𝑦 2 − 8𝑦 𝑦2 𝑦(𝑦 − 4)(𝑦 + 2) 𝑦 3 (𝑦 − 4)(𝑦 + 2)
∙ = ∙ = =
𝑦2 + 2 𝑦2 − 4 𝑦 2 + 2 (𝑦 + 2)(𝑦 − 2) (𝑦 2 + 2)(𝑦 + 2)(𝑦 − 2)

𝑦 3 (𝑦 − 4)
(𝑦 2 + 2)(𝑦 + 2)

5. El ejemplo consiste en resolver la operación


𝑘 3 − 3𝑘 2 + 2𝑘 𝑘 2 + 𝑘 − 12 𝑘(𝑘 − 2)(𝑘 − 1) (𝑘 + 4)(𝑘 − 3)
∙ 2 = ∙ =
4𝑘 2 − 6𝑘 𝑘 − 5𝑘 + 6 2𝑘(2𝑘 − 3) (𝑘 − 3)(𝑘 − 2)

(𝑘 − 2)(𝑘 − 1) (𝑘 + 4) (𝑘 − 2)(𝑘 − 1)(𝑘 + 4) (𝑘 − 1)(𝑘 + 4)


∙ = =
2(2𝑘 − 3) (𝑘 − 2) 2(2𝑘 − 3)(𝑘 − 2) 2(2𝑘 − 3)

284
6. Como último ejemplo consiste en resolver la operación
𝑎 + 𝑏 2𝑦 − 𝑥 𝑎 + 𝑏 −(𝑥 − 2𝑦) −(𝑎 + 𝑏)(𝑥 − 2𝑦) −1
∙ = ∙ = =
𝑥 − 2𝑦 2𝑏 + 2𝑎 𝑥 − 2𝑦 2(𝑎 + 𝑏) 2(𝑥 − 2𝑦)(𝑎 + 𝑏) 2

División de expresiones algebraicas polinómicas

Para dividir expresiones algebraicas polinómicas se debe aplicar la


siguiente propiedad:
El algoritmo es parecido al de las multiplicaciones, con la diferencia de que se debe
invertir la segunda fracción antes de iniciar
𝑎
𝑎 𝑐 𝑎⋅𝑑
∀𝑎, 𝑏, 𝑐 ∈ ℝ, 𝑏𝑑 ≠ 0 ÷ = 𝑏
𝑐 = .
𝑏 𝑑 𝑏⋅𝑐
𝑑

Ejemplos
𝑥−1 2𝑥−2
1. Simplifique al máximo ÷ 𝑥 2 +2𝑥+1.
𝑥+1

Solución
𝑥−1 2𝑥 − 2
÷ 2
𝑥 + 1 𝑥 + 2𝑥 + 1
𝑥 − 1 𝑥 2 + 2𝑥 + 1
= ⋅
𝑥+1 2𝑥 − 2
(𝑥 − 1)(𝑥 + 1)2
=
(𝑥 + 1)2(𝑥 − 1)
𝑥+1
= .
2

5𝑥 2 5𝑥 2 −20𝑥
2. Simplifique al máximo ÷ 𝑥 2 −𝑥−12.
𝑥 3 +3𝑥 2

Solución
5𝑥 2 5𝑥 2 − 20𝑥
÷
𝑥 3 + 3𝑥 2 𝑥 2 − 𝑥 − 12
5𝑥 2 (𝑥 − 4)(𝑥 + 3)
= 2 ⋅
𝑥 (𝑥 + 3) 5𝑥(𝑥 − 4)
1
= .
𝑥

285
𝑥+𝑦 𝑥 2 +𝑥𝑦
3. Simplifique al máximo ÷ 𝑥 2 −𝑦 2 .
𝑥2

Solución
𝑥 + 𝑦 𝑥 2 + 𝑥𝑦
÷ 2
𝑥2 𝑥 − 𝑦2
𝑥 2 (𝑥 + 𝑦)(𝑥 − 𝑦)
= ⋅
𝑥+𝑦 𝑥(𝑥 + 𝑦)
𝑥(𝑥 − 𝑦)
= .
𝑥+𝑦
4. Por ejemplo:
𝑎𝑥 2 + 5 𝑎2 𝑥 2 + 5𝑎 𝑎𝑥 2 + 5 2𝑎 − 1 𝑎𝑥 2 + 5 2𝑎 − 1
2
÷ = ∙ = ∙ =
4𝑎 − 1 2𝑎 − 1 4𝑎 − 1 𝑎 𝑥 + 5𝑎 (2𝑎 + 1)(2𝑎 − 1) 𝑎(𝑎𝑥 2 + 5)
2 2 2

(𝑎𝑥 2 + 5)(2𝑎 − 1) 1
2
=
(2𝑎 + 1)(2𝑎 − 1) ∙ 𝑎(𝑎𝑥 + 5) 𝑎(2𝑎 + 1)

5. El ejemplo consiste en resolver la operación


𝑦 2 − 121 𝑦 2 − 𝑦 − 132 𝑦 2 − 121 𝑦 2 − 144
÷ = ∙ =
𝑦 2 + 22𝑦 + 120 𝑦 2 − 144 𝑦 2 + 22𝑦 + 120 𝑦 2 − 𝑦 − 132

(𝑦 − 11)(𝑦 + 11) (𝑦 + 12)(𝑦 − 12) (𝑦 − 11)(𝑦 + 11) (𝑦 + 12)


∙ = ∙ =
(𝑦 + 12)(𝑦 + 10) (𝑦 + 11)(𝑦 − 12) (𝑦 + 12)(𝑦 + 10) (𝑦 + 11)

(𝑦 − 11)(𝑦 + 11)(𝑦 + 12) 𝑦 − 11


=
(𝑦 + 12)(𝑦 + 10)(𝑦 + 11) 𝑦 + 10

6. Como último ejemplo se resuelve la operación


𝑥 2 − 5𝑥 − 24 𝑥2 + 𝑥 − 6 𝑥 2 − 5𝑥 − 24 𝑥 2 − 10𝑥 + 16
÷ = ∙ =
𝑥 2 − 𝑥 − 12 𝑥 2 − 10𝑥 + 16 𝑥 2 − 𝑥 − 12 𝑥2 + 𝑥 − 6
(𝑥 − 8)(𝑥 + 3) (𝑥 − 2)(𝑥 − 8) (𝑥 − 8) (𝑥 − 8) (𝑥 − 8)(𝑥 − 8) (𝑥 − 8)2
∙ = ∙ = =
(𝑥 − 4)(𝑥 + 3) (𝑥 − 2)(𝑥 + 3) (𝑥 − 4) (𝑥 + 3) (𝑥 − 4)(𝑥 + 3) (𝑥 − 4)(𝑥 + 3)

286
Ejercicios

1. Simplifique al máximo:

(𝑥−5)2 3𝑥+15
a. ∙ 𝑥 2 −25
9

−3𝑥 2 +4𝑥−1 𝑥 2 −4
b. ∙ 𝑥 2 +𝑥−2
2−𝑥

𝑎2 +2𝑎𝑏+𝑏 2 6𝑎
c. ⋅
𝑎2 −𝑏 2 3𝑎+3𝑏

4𝑥 2 +8 𝑥 2 −1
d. ⋅ 𝑥 2 +2
𝑥 2 −2𝑥+1

𝑥−1 (𝑥+1)2
e. ⋅
2𝑥 2 +4𝑥+2 𝑥−1

𝑥 2 −6𝑥+9 2𝑥−2
f. ⋅
𝑥 2 −1 𝑥−3

2𝑦−1 𝑦
g. ⋅
2𝑥𝑦+2𝑦 𝑥 3 +𝑥 2

𝑥 2 +2𝑥−3 2𝑥 2 +6𝑥
h. ⋅ 𝑥 2 +6𝑥+9
2𝑥

𝑥 2 +𝑥−2 2𝑥−𝑥 2
i. ⋅ 𝑥 2 −2𝑥+1
4−𝑥 2

𝑥𝑦−2𝑦 𝑥 2 𝑦+4𝑥𝑦+4𝑦
j. ⋅
𝑥 2 𝑦 2 −4𝑦 2 𝑥𝑦+2𝑦

287
−3𝑥 2 −9𝑥+30 −8𝑥+2𝑥 2 +8
k. ⋅
𝑥 2 −4𝑥+4 𝑥 2 −4

2. Simplifique al máximo:

𝑥−2 1
a. 𝑥 2 +4𝑥+4 ÷ 𝑥 2 −4

2𝑥−1 𝑥+1
b. 𝑥 3 +3𝑥 ÷ 𝑥 2 +3

𝑥 3 −4𝑥 2 +3𝑥
c. ÷ (𝑥 − 3)
𝑥+2

6𝑥+12 8𝑥−12
d. 6𝑥 2 +6𝑥−24 ÷ 15𝑥−20

𝑥 2 −𝑥−12 𝑥 2 +4𝑥+3
e. ÷
𝑥 2 +𝑥−2 𝑥 2 −1

𝑥 2 −𝑥−12 𝑥 2 +4𝑥+3
f. ÷
𝑥 2 +𝑥−2 𝑥 2 −1

3𝑥 3 −75𝑥 𝑥 2 −10𝑥+25
g. ÷
𝑥3 𝑥2

𝑚+3 𝑚
h. ÷ 𝑚−3
𝑚

𝑥 3 +𝑥 𝑥 3 −𝑥 2
i. ÷ 𝑥 2 −2𝑥+1
𝑥 2 −𝑥

𝑥 4 −𝑎4 𝑥−𝑎
j. ÷ 𝑥 2 +𝑎𝑥
𝑥 2 +2𝑎𝑥+𝑎2

288
𝑥 2 −𝑦 2
k. 𝑦 2 −𝑥𝑦 ÷ (𝑥 − 𝑦)2

2𝑥 2 −18 (𝑥−3)2
l. ÷
𝑥 𝑥2

𝑥−5 𝑥+5
m. ( ) ÷ (5(𝑥+5)−25)
𝑥

𝑥−𝑦 𝑥+𝑦
n. ( ) ÷ (𝑦(𝑦+𝑥)−𝑦 2 )
𝑥

3. Resuelva en su cuaderno las siguientes operaciones y simplifique el


resultado si es posible:

6y2 2x2
1) =
4x 3y2

12abc ab
2)  =
33 66

x2 y
3) 6x2y3  =
5

3x 2 y
4)  –x2y3 =
5z2

5x 2x 5z
5)  =
y y2 4x 2

2x 2 x2 4x 5
6) =
50 2x 2 3x 3

2b2 ab a2 2ab b2
7) =
a2 ab a2
2ab

9y 4 6y3 4y2 4 9y 2
8) =
3y2 5y 2 27y 4 8y

289
16a2 24ab 9b2 16a 12b
9) =
64a3 27b3 32a2
24ab 18b2

a2 a 12 a2 5a 24 a2 a 56
10)  =
a2 49 a 5 a2 a 20

9y 2 30y 25 9y 2 25 21y 35
11) 3
 2
 2 =
y 1 14 14y y y 1

x2 1 x3 x 4x 8
12)  =
3x 6 6x 12 x2 4

a2 4b2 7a2 15ab 2b2 42a 12b


13) 2
 =
49a 28ab 4b 2 49a2 4b2 7a2
13ab 2b2

Respuestas:

1) x (x 1) 1 x 1 1
6) o 10) o
3(5 x) 3(5 x) a 7 7 a

2) –24c

3) 30y2 b(a b) 2(1 y)(3y 5) 2(1 y)(5 3y)


7) 11) o
a2 (3y 5)2 (3y 5)2
3
4)
5z2 y 2 y y x 2
8) o 12)
y 1 1 y 2x

2x 2 y
5)
z 1 (a 2b)2 (a 2b)2
9) 13) o
2 6(2b 7a) 6(7a 2b)

290
Racionalización

3 1 a 5
Expresiones como , …, tienen en común que sus
, ,3
2 2  3 2x 2
denominadores son irracionales o al menos aparecen en ellos alguna raíz.
La operatoria con tales expresiones no es sencilla y resulta muy práctico
transformar los denominadores en expresiones racionales. En otras palabras, se
trata de ‘’hacer que desaparezcan’’ las raíces que hayan en el denominador.
El procedimiento a emplear consiste en amplificar por un factor adecuado. Es decir,
se multiplica el numerador y el denominador por una misma cantidad, con lo cual
la expresión original no cambia.

A
I. RACIONALIZACIÓN DE EXPRESIONES DE LA FORMA:
a
2
¿Cómo racionalizar la fracción ? En los casos como éste, el factor adecuado para
3
amplificar es la raíz que aparece en el denominador, o sea 3 .

Ejemplo: 2 2 3 2 3 2 3
  
3 3 3 3² 3  
( se amplifica por 3 )

Se puede observar que el denominador original 3 (irracional) se ha transformado


en 3 (racional).
Además si bien la expresión inicial ha cambiado su ‘’forma’’, sigue siendo la misma,
ya que al amplificar una fracción su valor no se altera.
2 2 3
Por lo tanto 
3 3

Denominador Denominador
Irracional Racional

En general, cuando el denominador es una raíz cuadrada, ella misma es el factor


de amplificación.

291
Ejercicios: Racionaliza los denominadores. (Desarrolla en tu cuaderno)

A
II. RACIONALIZACIÓN DE EXPRESIONES DE LA FORMA: n
a
3
Para racionalizar, por ejemplo, la fracción 3
es necesario amplificar por 3 2² , por
2
lo cual se consigue que el radicando sea 2³
3 3  3 2² 33 4 33 4
Ejemplo: 3   
2 3 2  3 2² 3 2  2² 3 2³
33 4

2
En general, si en el denominador aparece a es necesario amplificar por
n k n
ank
con el objeto de igualar el índice de la raíz con el exponente del radicando.

Ejercicios: Racionaliza los denominadores. (En tu cuaderno)

292
A
III. RACIONALIZACIÓN DE EXPRESIONES DE LA FORMA: 
a b
Si el denominador es un binomio, se amplifica la fracción por su conjugado. Si se
trata, por ejemplo, de 3  2 se amplifica por 3  2 . La idea es formar el
producto de la suma por la diferencia que es igual a la diferencia de los cuadrados,
con lo cual se consigue eliminar las raíces.

Ejemplo

3

3  3  2  3 3  3 2  3 3 3 2 3 3 3 2
  3 3 3 2
3 2  3 2   3  2   3 ²   2 ² 32 1

Ejercicios: Racionaliza los denominadores


2
. 
5 2
7
2. 
5 3
10
3. 
7 2
12
4. 
7 5
3
5. 
3 2
2
6. 
6 2
3 2
7. 
11  2
5 2
8. 
7 2
7 10
9. 
10  3
3
10. 
2 3 2
9
11. 
52 2
3
12. 
3 2 2 3
3 2
13. 
52 3

293
2 3
14. 
2 7 3 2
2 3
15. 
3 2
1 5
16. 
5 3
6 2
17. 
2 3 2
3 1
18. 
3 6 2 3
3 2
19. 
5 2 1

. Racionalizar los siguientes denominadores

294
En resumen
Racionalización
Consiste en eliminar las expresiones radicales en uno de los términos de
una fracción, puede ser del numerador o del denominador. Se trabajan dos casos
diferentes: raíz de un monomio y sumas o restas con al menos una expresión con
una raíz cuadrada.
 Raíces de monomios
En este caso, únicamente se trabaja con la racionalización de los denominadores.
La esencia del procedimiento es extraer cada uno de los elementos del radical,
multiplicando tanto el numerador como el denominador por una expresión radical,
donde cada número y cada variable tengan como exponente la resta del índice del
radical menos el exponente original que tiene dentro del radical. Por ejemplo:

Factor
Expresión Multiplicación Simplificación
racionalizador
2𝑥√3𝑦 ∙ √2𝑥 2𝑥√6𝑥𝑦
= =
2𝑥√3𝑦 2𝑥√3𝑦 √2𝑥 3𝑦√2𝑥 ∙ √2𝑥 3𝑦√22 𝑥 2
√22−1 𝑥 2−1 = √2𝑥 ∙
3𝑦√2𝑥 3𝑦√2𝑥 √2𝑥 2𝑥√6𝑥𝑦 √6𝑥𝑦
=
3𝑦 ∙ 2𝑥 3𝑦
5 5
3𝑥 2 ∙ √𝑥 3𝑥 2 √𝑥
3𝑥 2 3𝑥 2 5 = 5 =
5 5
3𝑥 2 √𝑥 5 5
2√𝑥 4 ∙ √𝑥 2√𝑥 5
5 = 5 √𝑥 5−4 = √𝑥 5 ∙5 5 5
√32𝑥 4 2 √𝑥 4 2√𝑥 4 √𝑥 3𝑥 2 √𝑥 3𝑥 √𝑥
=
2𝑥 2
3 3
21 ∙ √7 21√7
21 21 3 = 3 =
3 21 √7 3 3
√72 ∙ √7 √73
3 = 3 √73−2 = 3√7 3 ∙3 3
√49 √72 √72 √7 21 √7 3
= 3 √7
7
4 4
10𝑥 2 ∙ √52 𝑥 10𝑥 2 √52 𝑥
4 4 = 4 =
10𝑥 2 10𝑥 2 10𝑥 2
4
√52 𝑥 √52 𝑥 3 ∙ √52 𝑥 √54 𝑥 4
4 4 4
4 = 4 √54−2 𝑥 4−3 = √52 𝑥 4 ∙4 10𝑥 2 √52 𝑥 4
√25𝑥 3 √52 𝑥 3 √52 𝑥 3 √52 𝑥 = 2𝑥 √52 𝑥 =
5𝑥
4
2𝑥 √25𝑥

295
Factor
Expresión Multiplicación Simplificación
racionalizador
(√𝑥 − √𝑦) ∙ √𝑥𝑦
=
√𝑥𝑦 ∙ √𝑥𝑦
√𝑥 − √𝑦 √𝑥 − √𝑦 √𝑥𝑦 √𝑥 ∙ 𝑥𝑦 − √𝑦 ∙ 𝑥𝑦
√𝑥 2−1 𝑦 2−1 = √𝑥𝑦 ∙ =
√𝑥𝑦 √𝑥𝑦 √𝑥𝑦 √𝑥 2 𝑦 2
√𝑥 2 𝑦 − √𝑥𝑦 2 𝑥√𝑦 − 𝑦√𝑥
=
𝑥𝑦 𝑥𝑦

(𝑥√𝑥 − 𝑥 3 √𝑥) ∙ √𝑥
=
√𝑥 3 − √𝑥 7 √𝑥 ∙ √𝑥
=
𝑥 √𝑥 − 𝑥 3 √𝑥 √𝑥 𝑥 √𝑥 ∙ 𝑥 − 𝑥 3 √𝑥 ∙ 𝑥
√𝑥 √𝑥 2−1 = √𝑥 ∙ =
𝑥 √𝑥 − 𝑥 3 √𝑥 √𝑥 √𝑥 √𝑥 2
𝑥 ∙ 𝑥 − 𝑥3 ∙ 𝑥 𝑥2 − 𝑥4
√𝑥 = =
𝑥 𝑥
3
𝑥−𝑥

 Sumas o restas con al menos una expresión con una raíz cuadrada
En este caso se debe indicar si la racionalización se hace en el numerador o en el
denominador. En ambos casos se trabaja igual. Se busca que en el término
racionalizado se obtenga la tercera fórmula notable, para eliminar las expresiones
radicales. La fórmula utilizada es (𝑎 + 𝑏)(𝑎 − 𝑏) = 𝑎2 − 𝑏 2 , por lo que el término para
racionalizar se deduce de la siguiente manera:

Término a Factor Fórmula


racionalizar racionalizador obtenida
𝑎+𝑏 𝑎−𝑏 𝑎2 − 𝑏 2
𝑎−𝑏 𝑎+𝑏 𝑎2 − 𝑏 2

296
Término a Factor
Expresión Resultado
racionalizar racionalizador

(√𝑥 − √𝑦)(√𝑥 + √𝑦)


=
(𝑥 √𝑦 − 𝑦√𝑥)(√𝑥 + √𝑦)

√𝑥 − √𝑦 √𝑥 2 − √𝑦 2
Numerador √𝑥 + √𝑦 =
𝑥 √𝑦 − 𝑦√𝑥 (𝑥 √𝑦 − 𝑦√𝑥)(√𝑥 + √𝑦)
𝑥−𝑦
(𝑥√𝑦 − 𝑦√𝑥)(√𝑥 + √𝑦)

(√𝑥 + √𝑦)(2√𝑥 − √𝑦)


=
(2√𝑥 + √𝑦)(2√𝑥 − √𝑦)

√𝑥 + √𝑦 (√𝑥 + √𝑦)(2√𝑥 − √𝑦)


Denominador 2√𝑥 − √𝑦 2 =
2√𝑥 + √𝑦 (2√𝑥) − √𝑦 2

(√𝑥 + √𝑦)(2√𝑥 − √𝑦)


4𝑥 − 𝑦

(√𝑎 − √3)(√𝑎 − √3)


=
(√𝑎 + √3)(√𝑎 − √3)
2
√𝑎 − √3 (√𝑎 − √3)
Denominador √𝑎 − √3 =
√𝑎 + √3 √𝑎2 − √32
2
(√𝑎 − √3)
𝑎−3

297
Término a Factor
Expresión Resultado
racionalizar racionalizador

(√ℎ + 9 − 3)(√ℎ + 9 + 3)
=
ℎ(√ℎ + 9 + 3)

√(ℎ + 9)2 − 32
=
ℎ(√ℎ + 9 + 3)

√ℎ + 9 − 3 ℎ+9−9
Numerador √ℎ + 9 + 3 =
ℎ ℎ(√ℎ + 9 + 3)

ℎ+9−9
=
ℎ(√ℎ + 9 + 3)

1
√ℎ + 9 + 3

(√𝑥 − √𝑥 − 1)(√𝑥 + √𝑥 − 1)
=
√𝑥(√𝑥 + √𝑥 − 1)

√𝑥 2 − √(𝑥 − 1)2
=
√𝑥(√𝑥 + √𝑥 − 1)

√𝑥 − √𝑥 − 1 𝑥 − (𝑥 − 1)
Numerador √𝑥 + √𝑥 − 1 =
√𝑥 √𝑥(√𝑥 + √𝑥 − 1)

𝑥−𝑥+1
=
√𝑥(√𝑥 + √𝑥 − 1)

1
√𝑥(√𝑥 + √𝑥 − 1)

298
Ejercicios
1. Racionaliza las siguientes expresiones:

1 1 a
  
3 2 a

2 3 a
  
3 2 2 3 b a

2. Racionaliza las siguientes expresiones:

a 5 a 2 a b
    
m 3 a b 3 1 a b

2 a b 5 3 a 2
    
5 3 b a 5 3 7
a 3 7
23

m 3 a x 2 x 2 y 4
    
q m 5 2 5
5 3 2
x a 2 y 2 x 5 1

3 3 5 3 7
    
3 6 4  13 3
2 5 2 4
3 3
3 74 6

3 3 2 2 2
    
2 8
35
6 5 2 2 1 2 1

3 2 1 3 2 a b
  
3 2 1 3 5
5 b 3
a b

299
300
Tema 3 Ecuaciones Cuadráticas

Situación Problema
Se cortan las esquinas de una lámina de cartón que mide 20 cm de
largo por 10 cm de ancho, para hacer una caja rectangular sin tapa.
¿Cuáles son los valores posibles para la altura x de la caja para que
su volumen sea igual a 24?

Análisis de la actividad En este caso tenemos

V = x(10 – 2x) (20 – 2x) = 24x.

Al multiplicar se obtiene la ecuación 4x2 – 60x + 200 = 24, que simplificada


queda x2–15x + 44 = 0.

La Clave

Ecuaciones cuadráticas

Las ecuaciones de segundo grado son del tipo: ax2 + bx + c = 0 con a


 0.
Ejemplos:

1. x2  5x  6 =0 2. 4 x 2  4 x  1=0 3.  x2  3x  4 =0 1 2 3
4. x  x 1  0
2 4

301
2. Determine los valores de a, b,c, de la ecuación 3x2 + 3x - 18 = 0
Resolución: Como todos los coeficientes son múltiplos de 3, dividiendo todos
los términos entre este número, se obtiene una ecuación equivalente más
sencilla: x2 + x - 6 = 0 a = 1; b = 1; c = -6

3. Determine los valores de a,b,c x2 + 2x + 1 = 0 a = 1, b = 2, c = 1

Ejercicios
1. Indica el valor de los coeficientes a, b y c de las siguientes ecuaciones
de segundo grado.

a b c

a) 3x2 + 2x – 3 = 0

b) x2 + x = 0
3
c) x – x2 + 5 = 0
2

d) 6 – x2 = 0

e) 2x – 3 = x2
3
f) 3 + 5x2 =
2

2. Reconoce el tipo de ecuación de 2º grado, indicando los valores de los


coeficientes a, b y c:

1. 5x2  3x  2  0 2. 9x2  1  0 3. 4x2  4x  1  0


4. 3x2  2x  0 5.  12x2  6x  9  0 6. 36 x2  25  0
7.  11x2  110 x 8. 16 x2  24 x  9 9. 49  x2
10. 121x2  45 x  15 11. 23x  4x2 12. 50 x2  87

Ecuación de segundo grado incompleta


Una ecuación de segundo grado es incompleta cuando los términos b
o c, o ambos, son cero. (Si a = 0, la ecuación resultante sería bx + c
= 0, que no es una ecuación de segundo grado.)

302
La expresión de una ecuación de segundo grado incompleta es:

ax2 = 0; si b=0 y c = 0.

I Caso Ecuación de la forma ax 2  c  0

Para resolver ecuaciones incompletas en las que falta el término en x, esto es,
cuando b = 0, se procede de la siguiente manera:
1. Se escribe la ecuación en la forma

ax 2  c  0

2. Se identifican los valores numéricos de los coeficientes a y c


3. Se sustituyen los valores numéricos de los coeficientes en la fórmula

Deducción de la fórmula (*):

303
Ejemplos

304
305
Ejercicios
1. Resuelve las siguientes ecuaciones de segundo grado incompletas,
explicando el proceso seguido.

a) x2 - 16 = 0

b) 3x2 - 147 = 0

c) x2 - 144 = 0

d) 7x2 = 343

e) 3x2 = 243

f) x2 - 24 = 120

g) 3x2 + 12 = 0

h) 7x2 - 28 = 0

II Caso Ecuación de la forma ax 2  bx  0

Procedimiento
Para resolver ecuaciones incompletas en las que falta el término c, esto es, cuando
c = 0, se procede de la siguiente manera:
1. Se escribe la ecuación en la forma

2. Se factoriza la ecuación anterior obteniendo la ecuación equivalente

3. Se iguala cada uno de los factores anteriores a cero

306
4. Como se puede deducir una de las soluciones siempre es cero; la otra
solución se calcula sustituyendo los valores numéricos de los coeficientes a y b
en (*)

Ejemplos Resolver las ecuaciones:

a) 2x2 + 4x = 0; b) -3x2 + 2x = 0; c) x2 - x = 0

Resolución:

a) 2x2 + 4x = 0
Sacando factor común x, resulta:


x  0

x (2x  4)  0ó
 4
2x  4  0  x   2
 2

La ecuación tiene dos soluciones: x = 0 y x = -2.

b) -3x2 + 2x = 0
Sacando factor común x, resulta:


x  0

x ( 3 x  2)  0  ó
 2 2
 3 x  2  0  x  
 3 3

2
La ecuación tiene dos soluciones x1  0  x 2 
3

307
c) x2 - x = 0
Sacando factor común x, resulta:

x  0

x ( x  1)  0  ó
x  1  0  x  1

La ecuación tiene dos soluciones: x = 0 y x = 1.

Ejercicios
1. Resuelve las siguientes ecuaciones de segundo grado incompletas, explicando el
proceso seguido:

a) 2x2 + 7x = 0

b) x2 - 64x = 0

c) 5x2 - 40x = 0

d) 4x2 – 9x = 0

x2
e) =x
5

f) 3x2 + 27x = 0

g) 7x2 = 3x

h) 6x2 + 2x = 0

2. ¿Cuáles de las siguientes ecuaciones de segundo grado son incompletas? ¿Por


qué?

a) 6x2 + 3x - 1 = 0 d) 2x - 4x2 = 0

b) 4x - x2 = 0 e) 3 - x = x2

c) 2x - 1 = x2 f) x2 - 3 + x = 0

308
3. Comprueba si los siguientes valores de x son las soluciones de las siguientes
ecuaciones de segundo grado:

a) x = 1, x = -1 de x2- 1 = 0

b) x = 4, x = -4 de 80 = 20x2

c) x = 9, x = -9 de 3x2 - 27 = 0

d) x = 1, x = -1 de -x2 + 1 = 0

4Resuelve las siguientes ecuaciones de segundo grado incompletas, explicando el


proceso seguido.

a) x2 - 16 = 0

b) 3x2 - 147 = 0

c) x2 - 144 = 0

d) 7x2 = 343

e) 3x2 = 243

f) x2 - 24 = 120

g) 3x2 + 12 = 0

h) 7x2 - 28 = 0

5. Comprueba si los siguientes valores de x son las soluciones de las siguientes


ecuaciones de segundo grado.

a) x = 1, x = -1 de x - x2 = 0

b) x = 0, x = 1 de x2 = x

c) x = 1, x = -10 de 3x2 = 30x

d) x = 0, x = 12 de 3x2 - 39x = 0

e) x = 0, x = -5 de 4x2 + 20x = 0

f) x = 0, x = 1 de 6x2 - 6x = 0

309
Ecuación de segundo grado completa
Una ecuación de segundo grado es completa cuando los tres
coeficientes a, b, y c son distintos de cero. La expresión de una
ecuación de segundo grado completa es: ax2 + bx + c = 0. ax2 + bx
= 0; si c = 0. ax2 + c = 0; si b = 0.
Las ecuaciones cuadráticas tienen la forma 𝑎𝑥 2 + 𝑏𝑥 + 𝑐 = 0. Si la ecuación no tiene
esta forma, se debe manipular algebraicamente para reducirla. A partir de los
valores de 𝑎, 𝑏 y 𝑐 se pueden obtener los valores de 𝑥 que satisfacen la ecuación.
Primero es necesario calcular el discriminante de la ecuación (∆= 𝑏 2 − 4𝑎𝑐) para
saber cuántas soluciones reales (llamadas raíces) tiene:

Cantidad de Conjunto solución


Signo del discriminante
soluciones reales
∆> 0 2 𝑆 = {𝑥1 , 𝑥2 }

∆= 0 1 𝑆 = {𝑥1 }

∆< 0 0 𝑆 =∅ o 𝑆 ={}

De igual manera, se tienen dos fórmulas definidas para calcular el valor exacto de
cada raíz:

−𝒃 + √∆ −𝒃 − √∆
𝒙𝟏 = 𝒙𝟐 =
𝟐𝒂 𝟐𝒂
Ejemplos
1. Sea la ecuación 𝑥 2 + 2𝑥 = −1. Determine si 1 es una solución de dicha
ecuación.

Solución
12 + 2 ∙ 1 = 3 ≠ −1. Por tanto, 1 no es solución de tal ecuación. ∎

2. Determine el número de soluciones de la ecuación 2𝑥 2 − 𝑥 = 6 sin necesidad


de resolverla.
Solución
La ecuación 2𝑥 2 − 𝑥 = 6 se transforma en 2𝑥 2 − 𝑥 − 6 = 0. Además, ∆= (−1)2 − 4 ∙ 2 ∙
−6 = 49, por lo que tal ecuación posee dos soluciones

310
3. Determine el conjunto solución de 𝑥 2 + 2𝑥 − 3 = 0.
Solución

𝑥2 + 2𝑥 − 3 = 0

−2+ √22 −4∙1∙−3 −2− √22 −4∙1∙−3


𝑥= o𝑥 =
2⋅1 2⋅1
𝑥 = 1 o 𝑥 = −3

Por lo tanto, 𝑆 = {1, −3}. ∎

4.Determine el conjunto solución de 2𝑥 − 6𝑥 2 = −1 + 3𝑥

Solución

2𝑥 − 6𝑥 2 = −1 + 3𝑥
−6𝑥 2 + 2𝑥 − 3𝑥 + 1 = 0
−6𝑥 2 − 𝑥 + 1 = 0

1+√(−1)2 −4⋅−6⋅1 1−√(−1)2 −4⋅−6⋅1


𝑥= o𝑥 =
2⋅−6 2⋅−6
−1 1
𝑥= o𝑥 = .
2 3

−1 1
Luego, 𝑆 = { , }.∎
2 3

5. Determine el conjunto solución de 3𝑥 2 − 9𝑥 = (𝑥 − 3)2 .


Solución
3𝑥 2 − 9𝑥 = (𝑥 − 3)2
3𝑥 2 − 9𝑥 = 𝑥 2 − 2 ∙ 𝑥 ∙ 3 + 32
3𝑥 2 − 9𝑥 = 𝑥 2 − 6𝑥 + 9
3𝑥 2 − 9𝑥 − 𝑥 2 + 6𝑥 − 9 = 0
2𝑥 2 − 3𝑥 − 9 = 0

3+√(−3)2 −4⋅2⋅−9 3−√(−3)2 −4⋅2⋅−9


𝑥= o𝑥 =
2⋅2 2⋅2

311
−3
𝑥= o 𝑥 = 3.
2

−3
Luego, 𝑆={ , 3} . ∎
2

6. Determine el conjunto solución de 𝑥(𝑥 − 1) − 2(𝑥 − 1) = 1.


Solución

𝑥(𝑥 − 1) − 2(𝑥 − 1) = 1
𝑥 2 − 𝑥 − 2𝑥 + 2 = 1
𝑥 2 − 3𝑥 + 1 = 0
3+√(−3)2 −4⋅1⋅1 3−√(−3)2 −4⋅1⋅1
𝑥= o𝑥=
2⋅1 2⋅1

3+√5 3−√5
𝑥= O𝑥= .
2 2

3+√5 3−√5
Por tanto, 𝑆 = { , }.
2 2

7. Resuelva 𝑥(𝑥 − 2) − 4(𝑥 − 3) = 2

Solución

𝑥(𝑥 − 2) − 4(𝑥 − 3) = 2
𝑥 2 − 2𝑥 − 4𝑥 + 12 = 2
𝑥 2 − 2𝑥 − 4𝑥 + 12 − 2 = 0
𝑥 2 − 6𝑥 + 10 = 0.

Asimismo, ∆= (−6)2 − 4 ⋅ 1 ⋅ 10 = −4. Luego, 𝑆 = { }. ∎

312
𝑥
8. Determine el conjunto solución de 2 + = (𝑥 + 1)(𝑥 − 1).
2

Solución
𝑥
2+ = (𝑥 + 1)(𝑥 − 1)
2
4 + 𝑥 2(𝑥 2 − 1)
=
2 2
4 + 𝑥 = 2(𝑥 2 − 1)
4 + 𝑥 = 2𝑥 2 − 2
4 + 𝑥 − 2𝑥 2 + 2 = 0
−2𝑥 2 + 𝑥 + 6 = 0
−1+√12 −4⋅−2⋅6 −1−√12 −4⋅−2⋅6
𝑥= O𝑥=
2⋅−2 2⋅−2
−3
𝑥=2o𝑥= .
2
3
Por tanto, 𝑆 = {2, − }. ∎
2
3+𝑥 𝑥(𝑥−1)
9. Determine el conjunto solución de − = 1.
2 3

Solución

3 + 𝑥 𝑥(𝑥 − 1)
− =1
2 3
3(3 + 𝑥) − 2𝑥(𝑥 − 1) 6
=
6 6
3(3 + 𝑥) − 2𝑥(𝑥 − 1) = 6
9 + 3𝑥 − 2𝑥 2 + 2𝑥 = 6
−2𝑥 2 + 3𝑥 + 2𝑥 + 9 − 6 = 0
−2𝑥 2 + 5𝑥 + 3 = 0
−5+√52 −4⋅−2⋅3 −5−√52 −4⋅−2⋅3
𝑥= o𝑥=
2⋅−2 2⋅−2
−1
𝑥= o 𝑥 = 3.
2

313
−1
Por lo tanto, 𝑆 = { , 3}. ∎
2

10. Resolver la ecuación x2 - 5x + 6 = 0.


Resolución:
a = 1; b = -5; c = 6.

 b  b2  4ac  ( 5)  ( 5)2  4  1 6 5  25  24 5  1 5  1


x    
2a 2 1 2 2 2

5 1 6 5 1 4
x1    3 y x2   2
2 2 2 2
La ecuación tiene dos soluciones: x = 3 y x = 2.
11. Resolver la ecuación 3x2 + 3x - 18 = 0
Resolución:
Como todos los coeficientes son múltiplos de 3, dividiendo todos los términos
entre este número, se obtiene una ecuación equivalente más sencilla:

x2 + x - 6 = 0
a = 1; b = 1; c = -6

 1  12  4  1 ( 6)  1  25  1  5
x  
2 1 2 2

 1 5 4  1 5  6
Las soluciones son: x1    2 y x2    3
2 2 2 2

11. Resolver la ecuación x2 + x + 1 = 0


Resolución:
En esta ecuación a = 1; b = 1; c = 1.
Aplicando la fórmula:

 1  1  4  1 1  1  1  4  1   3
x  
2 1 2 2
La ecuación no tiene solución, ya que el discriminante es negativo.
12. Por ejemplo, para determinar el conjunto solución de la ecuación 6 + (𝑦 + 1)2 −
𝑦 2 = −(𝑦 + 2)2 , se debe reducir a la forma general. Así:

314
6 + (𝑦 + 1)2 − 𝑦 2 = −(𝑦 + 2)2
6 + (𝑦 2 + 2𝑦 + 1) − 𝑦 2 = −(𝑦 2 + 4𝑦 + 4)
6 + 𝑦 2 + 2𝑦 + 1 − 𝑦 2 = −𝑦 2 − 4𝑦 − 4
6 + 𝑦 2 + 2𝑦 + 1 − 𝑦 2 + 𝑦 2 + 4𝑦 + 4 = 0
𝑦 2 + 6𝑦 + 11 = 0
∆= 𝑏 2 − 4𝑎𝑐 = (6)2 − 4(1)(11) = 36 − 44 = −8
Como ∆< 0 , entonces la ecuación no tiene soluciones reales, por lo que 𝑆 = ∅.

13. Otro ejemplo es determinar el conjunto solución de la ecuación


𝑥(3𝑥 − 1) + 2𝑥 2 − 3𝑥 − 1 = (𝑥 + 1)2
3𝑥 2 − 𝑥 + 2𝑥 2 − 3𝑥 − 1 = 𝑥 2 + 2𝑥 + 1
3𝑥 2 − 𝑥 + 2𝑥 2 − 3𝑥 − 1 − 𝑥 2 − 2𝑥 − 1 = 0
4𝑥 2 − 6𝑥 − 2 = 0
∆= 𝑏 2 − 4𝑎𝑐 = (−6)2 − 4(4)(−2) = 36 + 32 = 68
Como ∆> 0 , entonces la ecuación tiene dos soluciones reales, por lo que

−𝑏 + √∆ −𝑏 − √∆
𝑥1 = = 𝑥2 = =
2𝑎 2𝑎
−(−6) + √68 −(−6) − √68
= =
2(4) 2(4)
6 + 2√17 6 − 2√17
= =
8 8
2(3 + √17) 2(3 − √17)
= =
8 8
3 + √17 3 − √17
4 4

3 + √17 3 − √17
𝑆={ , }
4 4

315
14. Otro ejemplo es determinar el conjunto solución de la ecuación
3𝑥 2 − 12𝑥 + 12 = 0
∆= 𝑏 2 − 4𝑎𝑐 = (−12)2 − 4(3)(12) = 144 − 144 = 0
Como ∆= 0 , entonces la ecuación tiene una solución real, por lo que

−𝑏 + √∆ −(−12) + √0 12
𝑥= = = = 2 ⟹ 𝑆 = {2}
2𝑎 2(3) 6

15. El último ejemplo es determinar el conjunto solución de la ecuación


(2𝑥)2 + (2𝑥 + 2)2 = (2𝑥 + 4)2
4𝑥 2 + 4𝑥 2 + 8𝑥 + 4 = 4𝑥 2 + 16𝑥 + 16
4𝑥 2 + 4𝑥 2 + 8𝑥 + 4 − 4𝑥 2 − 16𝑥 − 16 = 0
4𝑥 2 − 8𝑥 − 12 = 0
∆= 𝑏 2 − 4𝑎𝑐 = (−8)2 − 4(4)(−12) = 64 + 192 = 256
Como ∆> 0 , entonces la ecuación tiene dos soluciones reales, por lo que

−𝑏 + √∆ −𝑏 − √∆
𝑥1 = = 𝑥2 = =
2𝑎 2𝑎
−(−8) + √256 −(−8) − √256
= =
2(4) 2(4)
8 + 16 24 8 − 16 −8
= =3 = = −1
8 8 8 8

𝑆 = {3, −1}
Ejercicios
1. Determinar el discriminante y cuantas soluciones reales posee las
siguientes ecuaciones cuadráticas
1) x2 + 3x + 2 = 0 2) 4x2 + 4x + 1 = 0 3) x2 + 4x + 5 = 0

4) – x2 + 9 = 0 5) 7x2 – 6x = 0 6) 5x2 – 10x + 5 = 0

316
2. Comprueba si los siguientes valores de x son las soluciones de las siguientes
ecuaciones de segundo grado:

a) x = 1, x = -1 de 2x2 - x - 1 = 0

b) x = 3, x = 4 de x2 - 7x + 12 = 0

c) x = -6, x = 0 de x2 + 5x - 6 = 0

d) x = -2, x = 4 de x2 - 2x - 8 = 0

e) x = 0, x = 1 de x2 - 2x - 3 = 0
3. .Resuelve las siguientes ecuaciones de segundo grado completas:

a) x2 - 5x + 6 = 0 g) 12 = x2 + x m) x2 - 6x + 10 = 0

5
b) x2 + 5x + 6 = 0 h) 3x + 10 = x2 n) x2 + x+1=0
2

9
c) x2 – x - 6 = 0 i) x2 - 4x + 4 = 0 ñ) x2 + 3x + =0
4

d) x2 + x - 6 = 0 j) 9x2 - 6x + 1 = 0 o) -2x2 - x - 1 = 0

e) 8x2 - 10x + 3 = 0 k) 100x2 + 20x = -1 p) -x2 + 2x - 3 = 0

f) 4x + 1 = -4x2 l) x2 + x + 1 = 0

317
4. ¿Cuánto vale el discriminante en las siguientes ecuaciones?

a) 3x2 + 2x - 9 = 0 = c) x - 1 = 3x2 =

b) 5x - 6x2 + 10 = 0 = d) x2 = 2x – 8 =

5. Indica cuáles de las siguientes ecuaciones de segundo grado tienen dos


soluciones distintas, cuáles dos soluciones iguales y las que no tienen solución,
además grafica 2 de las ecuaciones dadas, marcando en el plano vértice y eje de
simetría.
a) x2 - 2x + 1 = 0 = d) x2 + 4x + 4 = 0 =

b) 3x2 - 2x + 1 = 0 = e) 2x2 - x + 3 = 0 =

6. Sea la ecuación 𝑥 2 + 2𝑥 − 3 = 0. Determine las soluciones de la ecuación y


explique el significado del conjunto solución de tal ecuación.
7. Sea la ecuación 𝑥 2 − 3 = 0. Determine las soluciones de la ecuación y explique
el significado del conjunto solución de tal ecuación.
8. Sea la ecuación 2𝑥 2 − 1 = 0. Determine si 3 es una solución de tal ecuación.
9. Sea la ecuación 𝑥 2 − 25 = 0. Determine si 5 es una solución de tal ecuación.
10. Determine el número de soluciones de la ecuación 2𝑥 2 − 𝑥 = 6 sin
necesidad de resolverla.
11. Determine el número de soluciones de la ecuación 𝑥 2 + 6 = 0 sin
necesidad de resolverla.
12. Determine el número de soluciones de la ecuación 𝑥 2 + 2𝑥 + 1 = 0 sin
necesidad de resolverla.
13. Una raíz de la ecuación 3𝑥 2 − 10𝑥 + 𝑘 = 0 es3. Determinar la otra raíz y
hallar el valor de 𝑘.

14. Resolver las siguientes ecuaciones en ℝ:


I. 2
𝑥 = 100
II. 𝑥 2 − 64 = 0
III. 𝑥 2 + 100 = 0
IV. 3𝑥 2 = 9
V. 𝑥 2 − 9 = 160
VI. 𝑥 2 − 25 = 0
VII. 2𝑥 2 − 3 = 5
VIII. 𝑥2 + 𝑥 − 6 = 0

318
IX. 2𝑥 2 − 𝑥 = 6
X. 9𝑥 2 − 6𝑥 + 1 = 0
XI. 𝑥2 + 𝑥 − 6 = 0
XII. 𝑥 2 − 6𝑥 + 12 = 2
XIII. 𝑥(10 − 𝑥) = 54
XIV. 𝑥 2 = 𝑥(1 − 3𝑥)
XV. 2𝑥 − 6𝑥 2 = −1 + 3𝑥
XVI. 𝑥(𝑥 + 4) = 2𝑥(𝑥 − 4)
XVII. 6(𝑥 − 5) − (25 − 𝑥 2 ) = 0
XVIII. (𝑥 − 1)2 + (𝑥 − 32)2 = 𝑥 2
XIX. (𝑥 − 2)(𝑥 + 2) = −(3𝑥 − 2)2
XX. 𝑥(𝑥 − 1) − 2(𝑥 − 1) = 1
XXI. 𝑥(2 − 3𝑥) = 𝑥 2
XXII. (4𝑥 − 1)(2𝑥 + 3) = (𝑥 + 3)(𝑥 − 1)
XXIII. 2(𝑥 − 4)2 = 338
XXIV. (2𝑥 + 1)2 = −6𝑥 + 2
XXV. 10𝑥(𝑥 − 2) + 3(𝑥 + 1) = 0
15. Resolver las siguientes ecuaciones en ℝ:
𝑥
1) 𝑥 ( + 6) = 80
5
(𝑥−1)2
2) = (𝑥 − 1)
2
𝑥−1 2
3) ( ) =𝑥−1
2
𝑥+1
4) (𝑥 − 1)2 = 3 −
2
𝑥
5) (3𝑥 − 1) = 1
3
𝑥 2 𝑥 2
6) ( ) + (5 − ) = 13
4 4
(4𝑥+1) −1
7) − (𝑥 − 3)(𝑥 + 3) =
2 2
𝑥(𝑥+3)
8) 2𝑥 = 4 −
2
2
𝑥2 𝑥−4
9) − =2
4 2
𝑥 2 −1 𝑥+1 3𝑥−5
10) − =
2 8 8

RESOLUCIÓN DE PROBLEMAS
Para resolver un problema mediante una ecuación de 20 grado con una
incógnita, conviene seguir las siguientes fases:

319
A) Leer el problema.
1) Lea con atención el enunciado del problema. Hágalo primero en forma
general, y luego parte por parte.
B) Identificar datos.
1) Escriba los datos conocidos.
2) Seleccione una letra para representar los datos desconocidos. Si hay
varios, represente uno de ellos con una letra. Exprese los demás
términos con la misma letra.
3) Reconozca y anote la pregunta o las preguntas.
C) Planear una estrategia.
1) Establezca, en el enunciado, una relación entre los datos que se conoce
y los que no se conoce.
2) Escriba la ecuación que representa la relación planteada en el punto
anterior.
D) Aplicar la estrategia.
1) Resuelva la ecuación para obtener el valor numérico de la incógnita.
2) Verifique el resultado responda a la pregunta o las preguntas del
problema.
3) Anote la respuesta o las respuestas.
E) Comprobar el resultado.
1) Compruebe si la solución es la correcta.
2) Verifique si el resultado sea coherente con el enunciado del problema.

Ejemplos
1. La suma de dos números es 5 y su producto es −84. Hallar dichos
números.
Solución
Sea 𝑥 un número y 5 − 𝑥 el otro número. De acuerdo con las condiciones del
problema se construye la siguiente ecuación: 𝑥(5 − 𝑥) = −84. Al resolverla se
obtiene:
𝑥(5 − 𝑥) = −84

5𝑥 − 𝑥 2 = −84

−𝑥 2 + 5𝑥 + 84 = 0.

𝑥 = −7 o 𝑥 = 12.

Luego, los números son 12 y −7

320
2. La suma de dos números es 10 y la suma de sus cuadrados es 58. Halle ambos
números.
Solución
Sea 𝑥 un número y 10 − 𝑥 el otro número. De acuerdo con las condiciones del
problema se construye la siguiente ecuación: 𝑥 2 + (10 − 𝑥)2 = 58. Al resolverla se
obtiene:
𝑥 2 + (10 − 𝑥)2 = 58
𝑥 2 + 100 − 20𝑥 + 𝑥 2 = 58
𝑥 2 + 100 − 20𝑥 + 𝑥 2 − 58 = 0
2𝑥 2 − 20𝑥 + 42 = 0
𝑥 2 − 10𝑥 + 21 = 0.
𝑥 =7o𝑥 =3.

Luego, los números son 7 y3. ∎


3. El producto de dos naturales consecutivos equivale a la suma de esos números
aumentada en 19. De ellos, ¿cuál es el número mayor?
Solución

Sea 𝑥 el número natural menor. De acuerdo con las condiciones del problema se
construye la siguiente ecuación: 𝑥(𝑥 + 1) = 𝑥 + 𝑥 + 1 + 19. Al resolverla se obtiene:
𝑥(𝑥 + 1) = 𝑥 + 𝑥 + 1 + 19
𝑥 2 + 𝑥 = 2𝑥 + 20
𝑥 2 + 𝑥 − 2𝑥 − 20 = 0
𝑥 2 − 𝑥 − 20 = 0
𝑥 = −4 o 𝑥 = 5.
La solución 𝑥 = −4 se descarta por ser 𝑥 un número natural. Así, el número mayor
es6.
4. El perímetro de un jardín rectangular es de 30 𝑐𝑚 y su área es de 54 𝑐𝑚2.
¿Cuáles son sus dimensiones?
Solución
Sea 𝑥 la medida del ancho y 15 − 𝑥 la medida del largo. De acuerdo con las
condiciones del problema se construye la siguiente ecuación 𝑥(15 − 𝑥) = 54. Al
resolverla se obtiene:

321
𝑥(15 − 𝑥) = 54
15𝑥 − 𝑥 2 = 54
−𝑥 2 + 15𝑥 − 54 = 0.
𝑥 = 6 o 𝑥 = 9.
Por lo tanto, la medida del ancho es 6 y la medida del largo es9.
5. El producto de dos números negativos es 80. El número mayor excede en seis a la
quinta parte del número menor. ¿Cuál es el número menor?

Solución

Sea 𝑥 el número menor. Luego si 𝑦 representa el número mayor entonces 𝑦 = 6 +


𝑥
. De acuerdo con las condiciones del problema se construye la siguiente ecuación:
5
𝑥
𝑥 (6 + 5) = 80. Al resolverla se obtiene:

𝑥
𝑥 (6 + ) = 80
5
𝑥2
6𝑥 + = 80
5
𝑥2
+ 6𝑥 − 80 = 0
5
𝑥 2 + 30𝑥 − 400
=0
5
𝑥 = 10 o 𝑥 = −40.

La solución 𝑥 = 10 se descarta por ser 𝑥 un número negativo. Así, el número menor


es −40.
6. La medida del largo de un rectángulo excede en 4 a la medida del ancho. Si
la medida del ancho y la del largo se aumentan en4, entonces el área
resultante es el doble que el área del rectángulo original. ¿Cuál es la medida
del largo del rectángulo original?
Solución
Sea 𝑥 la medida del largo y 𝑥 − 4 la medida del ancho del rectángulo original.
Entonces, por las condiciones del problema se forma la ecuación (𝑥 + 4)(𝑥 − 4 + 4) =
2𝑥(𝑥 − 4). Al resolverla se obtiene:

322
(𝑥 + 4)(𝑥 − 4 + 4) = 2𝑥(𝑥 − 4)
𝑥(𝑥 + 4) = 2𝑥(𝑥 − 4)
𝑥 2 + 4𝑥 = 2𝑥 2 − 8𝑥
𝑥 2 + 4𝑥 − 2𝑥 2 + 8𝑥 = 0
−𝑥 2 + 12𝑥 = 0
𝑥 = 0 o 𝑥 = 12 .
La solución 𝑥 = 0 se desecha porque el largo es mayor que0. Por lo tanto, la medida
del largo del rectángulo original es 12.
7. La hipotenusa de un triángulo rectángulo es una unidad más larga que uno
de los catetos y 32 unidades más larga que el otro ¿cuánto mide el cateto
menor?
Solución
Sea 𝑥: la longitud de la hipotenusa; 𝑥 − 1: la longitud de un cateto mayor y 𝑥 − 32:
la longitud del cateto menor. Luego, por el teorema de Pitágoras se forma la
ecuación (𝑥 − 1)2 + (𝑥 − 32)2 = 𝑥 2 . Al resolverla se obtiene:

(𝑥 − 1)2 + (𝑥 − 32)2 = 𝑥 2
𝑥 2 − 2𝑥 + 1 + 𝑥 2 − 64𝑥 + 1024 = 𝑥 2
𝑥 2 − 66𝑥 + 1025 = 0.
𝑥 = 25 o 𝑥 = 41 .

La solución 25 se desecha porque al sustituir 𝑥 = 25 en la medida del cateto menor


se obtiene 25 − 32 = −7 < 0. Por lo tanto, la medida del cateto menor es 41 − 32 =
9.
8. Un padre tiene 54 años y Su hijo 12años. ¿Cuántos años hace que la edad
del padre fue el cuadrado de la edad del hijo?
Solución: Llamemos X a la pregunta ¿Cuántos años hace? Luego planteamos que
la edad del padre hace X años era de 54 - X, pero entonces el hijo tenía 12 - X. Por
tanto el modelo según la condición que da el problema es:

323
54  x  12  x  con este mod elo buscamos la solución resolvamos por fórmula
2

54  x  122  212x  x 2  54  x  144  24x  x 2  54  144  x 2  24x  x


de donde x 2  23x  90  0 tenemos que a  1 b  23 c  90
  23  232  4190 23  529  360 23  169 23  13
x   
21 2 2 2
23  13 36 23  13
tenemos dos valores para x a saber x1    18 x 2  5
2 2 2
Si nos remitimos a Las condiciones del problema, la primera solución de la ecuación
que es 18 no nos cumpliría, pues 12-18 da -6 años lo que no sería posible en esta
solución, pues tendríamos una edad negativa, por tanto la solución para este
problema es X=5 años, es decir hace 5 años el padre tenía el cuadrado de la edad
del hijo o sea edad del padre hace cinco años igual 49 años y edad del hijo hace
también cinco años igual a 7, lo que nos satisface Las condiciones del problema.

9. Descomponer el número 20 en dos partes tales que al efectuar el producto,


sea 96.
Solución:
Llamemos X a uno de los número, por lo tanto el otro número será 20 - X
Condición que el producto de los dos sea igual a 96
Modelo matemático X (20 - X) = 96. De donde
20x  x  96  x  20x  96  0 resolvamos por factorización
2 2

x  12x  8  0  x  12  0 ó x  8  0  x  12 ó x  8

Observemos que los dos valores de la solución cuadrática cumplen La condición,


pues al escoger alguno de ellos inmediatamente aparece el otro, cuando lo llevamos
a la condición pedida. Los dos números suman veinte y Su producto da 96.

10. Hallar los números que sumados consigo mismo, sea igual al producto
de él por sí mismo.
Solución: Llamemos a ese número X, por lo tanto podemos formar el modelo
matemático con la condición que no da el problema.

x  x  x  x de donde 2 x  x 2  x 2  2 x  0 resolvemos por factorización


Modelo a resolver:
x x  2   0  x  0 ó x  2

Solución, el 0 y el 2 cumplen la condición dada por el problema.

324
Nota: Cuando se invierten p pesos a UN interés compuesto del r por ciento anual,
C  p1  r 
n
al final de n años el capital C será: Teniendo en cuenta esta nota
resolver el siguiente problema:
11. ¿A qué interés $ 100.000 aumentará a $ 144.000, después de dos
años?
Solución:
C = 144.000. p = 100.000. r =?

Modelo a resolver:

144.00  0  100.0001  r   1  r   1  r 2
144.000 144 12
  1 r  
2 2

100.000 100 10
6 6 1 11
1 r    r   1  r  ó r  
5 5 5 5
Escogemos el interés como 1/5 por 100 igual al 20% anual.

I  800 p  7 p 2
12. El ingreso mensual de una compañía está dado por el modelo

Donde p es el precio del producto en pesos del, producto que fabrica.


¿ A qué precio el ingreso será de $10.000, sí el precio debe ser mayor de $ 50 ?
Solución:
10.000  800 p  7 p 2  7 p 2  800 p  10.000  0 resolvamos para p
  800   8002  47 10.000 800  640000 280000
por fórmula cuadrática : p   En
27  14
800  360000 800  600 800  600 1400 800  600 200
de p      100 p    14.29.
14 14 14 14 14 14
este problema escogemos el valor 100, puesto que este valor cumple la condición
que dá el problema, el cual dice que el valor debe ser mayor de $ 50.

Lo que podemos observar pues al resolver `problemas con modelos de segundo


grado, es que no siempre los dos valores sirven para solucionar un problema, es
decir la solución de
UN modelo no siempre conduce a la solución del problema, sino, que hay que
escoger cual es la solución que me resuelve el problema, a continuación
encontrarás una serie de problemas para construyas el modelo matemático, del
cual puedes obtener la solución.

325
13. Si, por ejemplo, se debe resolver el siguiente problema:
La medida del largo de un rectángulo excede en 4 a la medida del
ancho. Si tanto la medida del ancho como la del largo se aumentan en
4, se obtiene otro rectángulo con un área equivalente al doble del área
del rectángulo original. ¿Cuál es la medida del largo del rectángulo
original?
Para determinar la ecuación, se debe diferenciar los dos rectángulos de los que
habla el enunciado:

Característica Primero Segundo


Ancho 𝑥 𝑥+4

Largo 𝑥+4 𝑥+4+4=𝑥+8

Área (ancho por largo) 𝑥(𝑥 + 4) (𝑥 + 4)(𝑥 + 8)

Área según el problema 𝑥(𝑥 + 4) 2 ∙ 𝑥(𝑥 + 4)

De esta manera, para plantear la ecuación se toman las dos formas de representar
el área del segundo, teniendo siempre presente que el resultado que se obtiene es
el ancho del primer rectángulo. Así:
(𝑥 + 4)(𝑥 + 8) = 2 ∙ 𝑥(𝑥 + 4)
𝑥 2 + 12𝑥 + 32 = 2𝑥 2 + 8𝑥
𝑥 2 + 12𝑥 + 32 − 2𝑥 2 − 8𝑥 = 0
−𝑥 2 + 4𝑥 + 32 = 0
∆= 𝑏 2 − 4𝑎𝑐 = (4)2 − 4(−1)(32) = 16 + 128 = 144

Como ∆> 0 , entonces la ecuación tiene dos soluciones reales, por lo que

−𝑏 + √∆ −𝑏 − √∆
𝑥1 = = 𝑥2 = =
2𝑎 2𝑎
−(4) + √144 −(4) − √144
= =
2(−1) 2(−1)
−4 + 12 8 −4 − 12 −16
= = −4 = =8
−2 −2 −2 −2

𝑆 = {−4,8}

326
Luego, como el ancho del rectángulo original debe ser positivo, su medida es8. Sin
embargo, esta no es la respuesta solicitada, pues se pregunta por el largo. Así, la
respuesta final, que corresponde a la medida del largo es 𝑥 + 4 = 8 + 4 = 12.
14. Como ejemplo, se resuelve el siguiente problema:

El cuadrado del mayor de dos números naturales consecutivos excede


en 57 al triple del menor. ¿Cuál es el número mayor?

Para determinar la ecuación, se debe diferenciar los dos números naturales de los
que habla el enunciado:

Número Representación
Menor 𝑥

Mayor 𝑥+1

Luego, el cuadrado se refiere a la segunda potencia y el triple hace referencia a la


multiplicación por3. De este modo, para plantear la ecuación se toma la redacción
del enunciado. Así:
(𝑥 + 1)2 = 3𝑥 + 57
𝑥 2 + 2𝑥 + 1 = 3𝑥 + 57
𝑥 2 + 2𝑥 + 1 − 3𝑥 − 57 = 0
𝑥 2 − 𝑥 − 56 = 0
∆= 𝑏 2 − 4𝑎𝑐 = (−1)2 − 4(1)(−56) = 1 + 224 = 225

Como ∆> 0 , entonces la ecuación tiene dos soluciones reales, por lo que

−𝑏 + √∆ −𝑏 − √∆
𝑥1 = = 𝑥2 = =
2𝑎 2𝑎
−(−1) + √225 −(−1) − √225
= =
2(1) 2(1)
1 + 15 16 1 − 15 −14
= =8 = = −7
2 2 2 2

𝑆 = {8, −7}

327
Luego, como todo número natural debe ser positivo, el número menor es8. Sin
embargo, esta no es la respuesta solicitada, pues se pregunta por el número mayor.
Así, la respuesta final, que corresponde al valor del número mayor es 𝑥 + 1 = 8 +
1 = 9.
15. Como ejemplo, se presenta el siguiente problema:

El cuadrado de la edad de A hace 6 años es igual a la edad que


tendrá dentro de 6 años. ¿Cuál es la edad actual de A?

Para determinar la ecuación, se debe diferenciar las tres edades de las que habla
el enunciado:

Edad Representación
Hace 6 años 𝑥−6

Actual 𝑥

Dentro de 6 años 𝑥+6

Luego, el cuadrado se refiere a la segunda potencia. De este modo, para plantear


la ecuación se toma la redacción del enunciado. Así:
(𝑥 − 6)2 = 𝑥 + 6
𝑥 2 − 12𝑥 + 36 = 𝑥 + 6
𝑥 2 − 12𝑥 + 36 − 𝑥 − 6 = 0
𝑥 2 − 13 + 30 = 0
∆= 𝑏 2 − 4𝑎𝑐 = (−13)2 − 4(1)(30) = 169 − 120 = 49
Como ∆> 0 , entonces la ecuación tiene dos soluciones reales, que corresponden a

−𝑏 + √∆ −(−13) + √49 −𝑏 − √∆ −(−13) − √49


𝑥1 = = = 𝑥2 = = =
2𝑎 2(1) 2𝑎 2(1)
13 + 7 20 13 − 7 6
= = 10 = =3
2 2 2 2

𝑆 = {3,10}

328
Luego, como ambas soluciones son positivas, se deben comprobar las edades, para
ver cuál es la edad actual:

Edad 𝒙=𝟑 𝒙 = 𝟏𝟎

Hace 6 años 𝑥 − 6 = 3 − 6 = −3 𝑥 − 6 = 10 − 6 = 4

No se puede determinar para


Cuadrado de 6 años 42 = 16
la edad negativa
No se puede determinar para
Actual 𝑥 = 10
la edad negativa
No se puede determinar para
Dentro de 6 años 𝑥 + 6 = 10 + 6 = 16
la edad negativa

Así, la respuesta final, que corresponde a la edad actual de 𝐴, es 𝑥 = 10 años.

16. Como ejemplo, se resuelve el siguiente problema:

La hipotenusa de un triángulo rectángulo es una unidad más larga que


uno de los catetos y 32 unidades más larga que el otro ¿cuánto mide
el cateto menor?

Para determinar la ecuación, se debe diferenciar los tres lados del triángulo
rectángulo del que habla el enunciado:

Lado Representación de su medida


Hipotenusa 𝑥

Primer cateto 𝑥−1

Segundo cateto 𝑥 − 32

Luego, como el problema se refiere a un triángulo rectángulo, se debe aplicar el


teorema de Pitágoras:
(𝑥 − 1)2 + (𝑥 − 32)2 = 𝑥 2
𝑥 2 − 2𝑥 + 1 + 𝑥 2 − 64𝑥 + 1024 = 𝑥 2
𝑥 2 − 2𝑥 + 1 + 𝑥 2 − 64𝑥 + 1024 − 𝑥 2 = 0
𝑥 2 − 66𝑥 + 1025 = 0
∆= 𝑏 2 − 4𝑎𝑐 = (−66)2 − 4(1)(1025) = 4356 − 4100 = 256

329
Como ∆> 0 , entonces la ecuación tiene dos soluciones reales, por lo que

−𝑏 + √∆ −𝑏 − √∆
𝑥1 = = 𝑥2 = =
2𝑎 2𝑎
−(−66) + √256 −(−66) − √256
= =
2(1) 2(1)
66 + 16 82 66 − 16 50
= = 41 = = 25
2 2 2 2

𝑆 = {25,41}
Luego, como ambas soluciones son positivas, se deben comprobar las medidas
para determinar cuál corresponde a la medida de la hipotenusa del triángulo
rectángulo:

Edad 𝒙 = 𝟐𝟓 𝒙 = 𝟒𝟏

Hipotenusa 𝑥 = 25 𝑥 = 41

Primer cateto 𝑥 − 1 = 25 − 1 = 24 𝑥 − 1 = 41 − 1 = 40

Segundo cateto 𝑥 − 32 = 25 − 32 = −7 𝑥 − 32 = 41 − 32 = 9

Así, como con 𝑥 = 25 la medida de un cateto da negativa, la respuesta final es que


la medida del cateto menor es9.
Ejercicios

1) Hallar el valor de la constante 𝑘 de tal manera que las raíces de la ecuación


3𝑥 2 − 4𝑥 + 𝑘 = 0 sean iguales.
2) Determine el valor de 𝑘 de tal manera que 3 sea una raíz de la ecuación 𝑥 2 −
2𝑥 + 𝑘 = 0 .
3) Determinar el valor de 𝑘 de tal forma que la ecuación 𝑥 2 + 𝑥 + 𝑘 = 0 tenga una
raíz nula.
4) Determinar el intervalo al que pertenece 𝑘 de tal forma que la ecuación 𝑘𝑥 2 −
6𝑥 + 5 = 0 tenga sus raíces reales.
5) Determinar el valor de la constante 𝑘 de tal manera que las raíces de la
ecuación 3𝑥 2 − 3𝑥 + 𝑘 = 0 no sean reales.
6) Determinar el intervalo al que pertenece 𝑘 de tal forma que la ecuación 𝑥 2 +
5𝑥 + 𝑘 = 0 no tenga sus raíces reales.

330
7) Toda ecuación cuadrática se puede expresar en la forma 𝑥 2 − 𝑠𝑥 + 𝑝 = 0,
donde 𝑠 es la suma de las raíces y 𝑝 es el producto de ellas. Determine, sin
resolver la ecuación 2𝑥 2 + 𝑥 − 6 = 0 el valor del producto de las raíces.
8) Determine el producto y la suma de las raíces de la ecuación 4𝑥 2 + 𝑥 − 3 = 0,
sin necesidad de resolverla.
9) La suma de dos números es 23 y el producto es 102. ¿Cuáles son esos
números?
10) El producto de dos números negativos es 90. El número mayor excede
en 7 a un tercio del número menor. ¿Cuál es el número menor?
11) El cuadrado del mayor de dos números positivos y consecutivos excede
en 100 al cuádruplo del mayor de ellos. ¿Cuál es el menor de esos números?
12) El producto de dos números positivos es 2. Si el número mayor excede
17
en 11 al menor, entonces, ¿cuál es el número mayor?
1
13) La diferencia de dos números es . Si el doble del número menor
3
excede en 3 al número mayor, entonces determine el número menor.
14) Divida 40 en dos partes de modo que la suma de los cuadrados de estas
partes sea igual a 800.
15) La suma de los cuadrados de tres números es 549. Si el segundo es
dos tercios del primero y el tercero es la mitad del primero, entonces ¿cuáles
son los números? Si 𝑥 denota al primer número, determine una ecuación que
permite resolver el problema.
16) El producto de dos números enteros positivos es 2160 y el número
menor es las tres quintas partes del número mayor. ¿Cuál es el número
mayor?
3
17) Si la medida del radio de un círculo se aumente en 4, entonces su área
361𝜋
es . ¿Cuál es la medida del radio del círculo original?
16
18) Las longitudes de los lados de un triángulo rectángulo son 6 − 𝑥, 13 − 𝑥
y 14 − 𝑥. ¿Cuál es la longitud de la hipotenusa?
19) Si la edad de 𝑀 excede en 6 años a la edad de 𝑁 y la suma de los
cuadrados de ambas es 260 años, entonces ¿Cuál es la edad en años de 𝑀?
20) El perímetro de un rectángulo es 76 y su área es de 360. ¿Cuál es la
medida del ancho del rectángulo?
21) La diferencia entre el largo y el ancho de un rectángulo es 7. Si el área
del rectángulo es 78, entonces, ¿cuál es la medida del largo?
22) El área de un rectángulo es 15. Si la medida del largo es igual a 4
aumentado en el triple de la medida del ancho, entonces ¿cuál es la medida
del largo del rectángulo?
23) La medida del ancho de un rectángulo equivale a las tres cuartas partes
de la medida del largo. Si el área es 108, entonces, ¿cuál es la medida del
ancho del rectángulo?

331
24) La medida del largo de un rectángulo excede a la medida del ancho en
seis unidades. Si la medida del ancho se aumente en dos unidades y la del
largo se disminuye en tres unidades, el área será de 30 unidades cuadradas.
¿Cuál es la medida del largo del rectángulo original?
25) Un trozo de alambre de 20 𝑚 de largo se corta en dos y cada pedazo se
dobla para que tome la forma de un cuadrado. Si la suma de las áreas
formadas por los cuadrados es de 13 𝑚2 . ¿Cuánto mide cada pedazo?
26) Un trozo de alambre de 100 𝑐𝑚 de largo se corta en dos y cada pedazo
se dobla para que uno forme un cuadrado y otro forme un círculo. Si la suma
de las áreas formadas es 397 𝑐𝑚2 , entonces, ¿Cuánto mide cada pedazo?
27) Si la longitud de cada lado de un cuadrado se aumenta en 6, entonces
el área del cuadrado que se forma es cuatro veces el área del cuadrado
original. ¿Cuál es el perímetro del cuadrado original?
28) Si cada una de las longitudes de dos lados opuestos de un cuadrado se
duplica y cada una de las longitudes de los otros lados se disminuye en 2 𝑐𝑚,
entonces el área del rectángulo resultante es de 32 𝑐𝑚2 mayor que el área del
cuadrado original.
29) Si la longitud de cada lado de un cuadrado aumenta en 12, y se obtiene
otro cuadrado con un área igual, nueve veces al área del cuadrado inicial,
entonces, ¿cuál es el área del cuadrado inicial?
30) Un terreno rectangular de 50 𝑚 por 24 𝑚 se cubre completamente de
zacate y se rodea por una acera de cemento de ancho uniforme. Si el área
cubierta por dicha acera es de 480 𝑚2 , entonces ¿cuál es la medida del ancho
de la acera?

31. Resuelva los siguientes problemas de ecuaciones cuadráticas


1) A es dos años mayor que B y la suma de los cuadrados de ambas edades es
130 años. Hallar ambas edades.

Datos Planteo y Operación Respuesta

332
2) La suma de dos números es 9 y la suma de sus cuadrados es 53. Hallar los
números

Datos Planteo y Operación Respuesta

3) Si el perímetro de un rectángulo es 40cm. y su área 96cm2, entones hallar la


medida del largo del rectángulo.

Datos Planteo y Operación Respuesta

4) La medida de la base de un triángulo es tres unidades mayor que la medida de


la altura. Si el área del triángulo es 119, entonces ¿Cuáles son las dimensiones del
triángulo?

Datos Planteo y Operación Respuesta

5) El producto de dos números enteros negativos es 90. El número mayor excede


en siete a un tercio del número menor. ¿Cuál es el número menor?

Datos Planteo y Operación Respuesta

333
334
Tema 4 Funciones Cuadráticas

Situación Problema
Un lanzador de peso puede ser modelado usando la
ecuación y  0, 0241x 2  x  5.5 , donde x es la
distancia recorrida (en pies) y yes la altura (también
en pies). ¿Qué tan largo es el tiro?

Análisis de la actividad
El lanzamiento termina cuando el tiro cae a tierra. La altura y en esa posición es 0,
entonces igualamos la ecuación a 0. 0  0, 0241x 2  x  5.5 Esta ecuación es difícil de
factorizar o de completar el cuadrado, por lo que la resolveremos usando la fórmula
cuadrática ¿Tienen sentido las raíces? La parábola descrita por la función cuadrática tiene dos
intersecciones en x. Pero el tiro sólo viajó sobre parte de esa curva.

Una solución, -4.9, no puede ser la distancia recorrida porque es un número negativo

La otra solución, 46.4 pies, debe ser la distancia del lanzamiento

La Clave
Se llama función cuadrática o parábola a la función 𝑓 tal que
𝑓: 𝐴 ⟶ ℝ
𝑓(𝑥) = 𝑎𝑥 2 + 𝑏𝑥 + 𝑐
donde 𝑎, 𝑏, 𝑐 ∈ ℝ , 𝑎 ≠ 0 y 𝑥 es una variable.
Para graficar una parábola se necesitan tres elementos: la(s) intersección(es) con
el eje 𝑥, la intersección con el eje 𝑦 y el vértice.
Una parábola es cóncava hacia arriba (tiene la forma⨄) si 𝑎 > 0. Una parábola
es cóncava hacia abajo (tiene la forma⋂) si 𝑎 < 0.
La intersección con el eje 𝒙 se determina resolviendo la ecuación 𝑓(𝑥) = 0. (0, 𝑐)
es el punto de intersección con el eje 𝒚.
Toda parábola tiene o un punto máximo o un punto mínimo. Este punto
corresponde al vértice:

335
−𝑏 −Δ
𝑉=( , ) , con Δ = 𝑏 2 − 4𝑎𝑐.
2𝑎 4𝑎

Si 𝒂 < 𝟎 el vértice es un punto máximo y si 𝒂 > 𝟎 el vértice es un punto


mínimo.
−𝑏
La recta 𝑥 = se llama eje de simetría
2𝑎

. Ejemplos

f x   x 2  5 x  6 hx   4 x 2  25

k x    x 2  2 x g x    x 2

Se Tiene que f(x) = ax2 + bx + c , donde f:

Caso I: Cuando a > 0. La parábola será cóncava


hacia arriba. y
Eje de

Características: Simetría

1) Discriminante:  = b2 – 4ac

b
2) Eje de Simetría: x=
2a
x x1 x2
b
b
3) Vértice: , 2a
2a 4a

4) Intersecciones con el eje “x”: Se


c
obtienen por medio de la calculadora.
Vértice (punto
(Mode 5- 3 4a
mínimo)
5) Intersección con el eje “y”: (0, c)

336
x
Notas:
y
1) Si  > 0 la función intersecará al eje “x” dos veces.

2) Si  = 0 la función intersecará al eje “x” solo una


vez.

3) Si  < 0 la función NO intersecará al eje “x”. y

Caso II: Cuando a < 0. La parábola será cóncava


hacia abajo. y

Vértice (punto
Características:
máximo)

4a
c
1) Discriminante:  = b2 – 4ac

x x1 x2
b
6) Eje de Simetría: x= b
2a 2a

b
7) Vértice: , Eje de
2a 4a
simetría

8) Intersecciones con el eje “x”: Se


obtienen por medio de la calculadora.

9) Intersección con el eje “y”: (0,c)

337
Notas:
y
1) Si  > 0 la función intersecará al eje “x” dos veces.

2) Si  = 0 la función intersecará al eje “x” solo una vez.

3)
x
Si  < 0 la función NO intersecará al eje “x”.

Ejemplos

2.Efectúe la gráfica de 𝑓(𝑥) = 2𝑥 2 +


1. Efectúe la gráfica de 𝑓(𝑥) = 𝑥 2 + 2𝑥 + 2𝑥 − 4.
1. Solución
2𝑥 2 + 2𝑥 − 4 = 2(𝑥 + 2)(𝑥 − 1)
Solución
Intersección con el eje 𝑥: (−1,0). y Δ = 𝑏 2 − 4𝑎𝑐 = 4 − 4 ⋅ 2 ⋅ −4 = 36 .
Intersección con el eje 𝑦: (0,1).
−𝑏 −Δ −2 0 Intersección con el eje 𝑥: (−2,0) y
Vertices: ( , ) = ( 2 , 4) = (−1,0).
2𝑎 4𝑎 (1,0).
La gráfica de la función 𝑓 es: Intersección con el eje 𝑦: (0, −4).
−𝑏 −Δ −2 −36
𝑦 Vertices: ( , )=(4 , )=
2𝑎 4𝑎 8
(−0.5, −4.5)
La gráfica de la función 𝑓 es:
𝑦

1
𝑥
−1 −2 −0.5 1
𝑥

−4
−4.5

3. Efectúe la gráfica de 𝑓(𝑥) = −𝑥 2 − 𝑥 + 6.

Solución
−𝑥 2 − 𝑥 + 6 = −(𝑥 − 2)(𝑥 + 3) y Δ = 𝑏 2 − 4𝑎𝑐 = 1 − 4 ⋅ −1 ⋅ 6 = 25 .
Intersección con el eje 𝑥: (2,0) y (−3,0).
Intersección con el eje 𝑦: (0,6).

338
−𝑏 −Δ 1 −25
Vertices: ( , )=( , ) = (−0.5,6.25)
2𝑎 4𝑎 −2 −4
La gráfica de la función 𝑓 es:
𝑦
6.25
6

−3 −0.5 2 𝑥

4. Efectúe la gráfica de 𝑓(𝑥) = 𝑥 2 + 𝑥 + 5.Determine la intersección con el eje 𝑥


1. para 𝑓(𝑥) = 𝑥 2 − 𝑥 + 12
Solución
Δ = 𝑏 2 − 4𝑎𝑐 = 1 − 4 ⋅ 1 ⋅ 1 = −3 . Solución
Δ = 𝑏 2 − 4𝑎𝑐 = 1 − 4 ⋅ 1 ⋅ 12 = −47 Por lo
Intersección con el eje 𝑥: No tiene. que la gráfica de 𝑓 no corta al eje 𝑥.
Intersección con el eje 𝑦: (0,1).
−𝑏 −Δ −1 3
Vértice: ( 2𝑎 , 4𝑎 ) = ( 2 , 4) = (−0.5,0.75)
6. Sea 𝑓(𝑥) = 𝑎𝑥 2 + 𝑏𝑥 + 𝑐 con 𝑎 ≠ 0. Si
𝑓(0) = −1 y el vértice de la gráfica de 𝑓
La gráfica de la función 𝑓 es:
es (1,0), determine el valor de 𝑎 y
𝑦
concluya si la gráfica es cóncava hacia
abajo o hacia arriba.

Solución
−𝑏 −Δ −Δ
1 𝑉 = ( 2𝑎 , 4𝑎 ) = (4,0). Luego , 4𝑎 = 0 ⇒ Δ =
0.75
𝑥 −𝑏
−0.5 0. 2𝑎
= 1 ⇒ 𝑏 = −2𝑎.
𝑓(0) = −1 ⇒ 𝑐 = −1. Así 𝑓(𝑥) = 𝑎𝑥 2 −
2𝑎𝑥 − 1 y Δ = 𝑏 2 − 4𝑎𝑐 = 4𝑎2 + 4𝑎 = 0 ⇒
𝑎 = 0 o 𝑎 = −1. Puesto que 𝑎 ≠ 0
entonces 𝑎 = −1 y la gráfica es cóncava
hacia abajo
7. Determine el vértice de la función 𝑓(𝑥) = −3(𝑥 − 2)(𝑥 + 3) y concluya si la
función posee un máximo o un mínimo.
Solución
𝑓(𝑥) = −3𝑥 2 − 3𝑥 + 18.
−𝑏 −Δ 3 −225 −1 225
𝑉=( , )=( , )=( , )
2𝑎 4𝑎 −6 −12 2 12
−1 75
= ( , ).
2 4

339
Como la gráfica de 𝑓 es cóncava hacia abajo entonces la función posee un
máximo absoluto
8. Determine la intersección con el eje 𝑦 para 𝑓(𝑥) = 3𝑥 2 − 𝑥 + 5.

Solución
La intersección con el eje 𝑦 es (0,5). ∎
9.Por ejemplo, se puede analizar y graficar las siguientes funciones:
Característica 𝒇(𝒙) = 𝟑𝒙𝟐 + 𝟑𝟎𝒙 + 𝟏𝟎𝟎 𝒈(𝒙) = −𝟓𝒙𝟐 − 𝟓𝒙 + 𝟑𝟎

Valor de 𝑎 3 −5

Valor de 𝑏 30 −5

Valor de 𝑐 100 30

Concavidad Hacia arriba Hacia abajo

∆= 302 − 4 ∙ 3 ∙ 100 = ∆= (−5)2 − 4 ∙ −5 ∙ 30 =


Discriminante
−300 625

−(−5) −625
−30 −(−300) ( , )=
( , )= 2 ∙ −5 4 ∙ −5
Vértice 2∙3 4∙3
−1 125
(−5, 25) ( , )
2 4
−1
Eje de simetría 𝑥 = −5 𝑥=
2

Intersecciones con el eje X No hay (𝑆 = ∅) (2,0) y (−3,0)

Intersección con el eje Y (0,100) (0,30)

340
10. Sea f  x    x 2  4 x  3 .
Una representación tabular de esta función es la siguiente:

x f(x)

-1 -8

0 -3

1 0

2 1

3 0

4 -3

5 -8

En este caso las constantes son: a = -1, b = 4, c = -3. Esta parábola abre hacia
abajo dado que a  1; su vértice es el punto máximo, cuya coordenada x es:

b 4 4
Vx     2
2a 2  1 2

En la representación tabular vemos que a este valor de x le corresponde

f  2    2  4  2  3 
2

 4  8  3  1
Por lo que el vértice de la parábola es el punto ( 2, 1 ).
Al igual que en la recta, el término independiente indica el punto donde la parábola
intersecta al eje y. En esta función es el punto (0,-3).
La representación gráfica de esta función, obtenida de la tabla es:

341
Como se puede ver en la figura, esta parábola cruza el eje x en dos puntos, esto
es, tiene dos raíces. Al igual que con la función lineal, para encontrar las raíces
se resuelve la ecuación f ( x)  0 :

f  x    x2  4x  3
0   x2  4 x  3

A diferencia de las funciones lineales, no se puede despejar directamente; por lo


tanto, se factoriza cuando es posible, o se utiliza la fórmula general

b  b 2  4ac
x
2a
En este caso:

4   4  4  1 3
2

x 
2  1

 4  2
 1
2
4  16  12 4  2 
  
2 2  4  2
 3
 2

Las raíces son x=1 y x = 3. Note que b2  4ac  (4)2  4(1)(3)  16  12  4  0


, conocido como discriminante, en este caso es positivo.
Ejercicios
1. En las siguientes funciones identificar la concavidad e indicar el número
de intersecciones que hay con el eje X.
2 2
1) y  x  5 x  3 2) y  2 x  5 x  4
2 2
3) y  x  2 x  4 4) y   x  x  3
2 2
5) y  3 x  4 6) y  2 x  3 x  1

342
2) En las siguientes funciones determinar las intersecciones con ambos ejes, e
indicar el eje de simetría en cada una de ellas.

1) y  x 2  4 x  3 2) y  4 x 2  9

3) y  x 2  5 x 4) y  2 x 2  5 x  4

5) y  3 x 2  5 x  6 6) y   x 2  2 x  24

3. Complete y realice la grafica  y

a) f x    x 2  4 x  5 


x

                  


















 y

f x   x 2  6 x  5

b) 


x

                  


















343
4. Realice el análisis para las siguientes funciones cuadráticas

a) f(x) = x2 – 2x – 8 b) f(x) = 4x2 + 12x + 9

c) f(x) = –4x2 – 8x + 5 d) f(x) = –x2 + 4x – 4

5. Ejercicios Relacionados con Funciones Cuadráticas

a) El eje de simetría para la función f(x) = 9x2 – 18x + 3 corresponde a:


_______________

b) El punto máximo de la función f(x) = x – x2 – 12 corresponde a:


_________________

x2 2x
c) El vértice de la parábola dada por f(x) = es: __________________
2

d) La función f(x) = –3x2 + 4x – 5 interseca al eje “y” en el punto:


_______________

e) La función f(x) = 4x2 + 5x – 6 interseca al eje “x” en los puntos:


____________ y ____________

f) La función f(x) = –9x2 + 25 interseca al eje “x” en los puntos:


____________ y ____________

g) La función f(x) = –6x2 – 30x interseca al eje “x” en los puntos: ___________
y ____________.

h) La función f(x) = 36x2 – 12x + 1 interseca al eje “x” en el punto:


______________

344
i) Para la función f(x) = 3x2 + 6x + 5 se cumple que:

( ) interseca dos veces el eje “x” ( ) no interseca el eje “x”


( ) interseca el eje “x” una vez

Análisis de los elementos y características de la función cuadrática


Recuerde que una función f definida por f : IR  IR con f x   ax 2  bx  c,
con a, b, c  IR , a  0 recibe el nombre de función cuadrática, entonces:
o Signo de a:

a0 a0
o Signo de  :

0 0 0


No toca al eje x Toca 1 vez al eje x Toca dos veces al eje x

o Signo de c:

c0 c0 c0

b
o Signo de :
2a

b b b
0 0 0
2a 2a 2a

345

o Signo de :
4a

  
0 0 0
4a 4a 4a

b
o Signo de b: Se resuelve la inecuación formada al analizar el eje de simetría ó bien se puede
2a
aplicar el siguiente cuadro de signos:

b b b
0 0 0
Si 2a 2a 2a
a0 b0 b0 b0
a0 b0 b0 b0

Ejercicios:
De acuerdo a las gráficas adjuntas de funciones cuadráticas, complete en
el espacio indicado el símbolo >, < ó =, según corresponda.

346
Aplicaciones de la función cuadrática
Se trata de problemas en los que hay que utilizar características de la
función cuadrática para determinar las respuestas.
Ejemplo

Un ambientalista descubre que cierto tipo de árbol después de ser


plantado crece de tal forma que después de 𝑡 años su altura ℎ(𝑡) está
dada por ℎ(𝑡) = 0,34 + 5,36𝑡 − 1,5𝑡 2 en metros
a) ¿Con cuántos años de plantado el árbol alcanza la máxima altura?
b) ¿Cuál es la máxima altura del árbol?
c) ¿En el momento de ser plantado cuánto medía el árbol?
d) ¿Cuántos años después de plantado el árbol deja de crecer?
e) ¿Cuántos años deben pasar para que el árbol alcance una altura de
5,005 metros?
f) ¿Cuál es la altura del árbol después de un año de plantado?

Para resolver este problema, se debe identificar a qué aspecto se refiere cada
pregunta:
a) Primera coordenada del vértice
b) Segunda coordenada del vértice
c) Altura cuando 𝑡 = 0
d) Eje de simetría
e) Solución de la ecuación 0,34 + 5,36𝑡 − 1,5𝑡 2 = 5,005
f) Altura cuando 𝑡 = 1

347
Para responder cada una de las preguntas, es necesario determinar que como
ℎ(𝑡) = 0,34 + 5,36𝑡 − 1,5𝑡 2 , entonces ℎ(𝑡) = −1,5𝑡 2 + 5,36𝑡 + 0,34, por lo que 𝑎 = −1,5,
𝑏 = 5,36 y 𝑐 = 0,34.

Pregunta Proceso Solución

−𝑏 −(5,36)
a 𝑡= = ≈ 1,787 1,787 años
2𝑎 2 ∙ −1,5

−∆ −(30,7696)
b 𝑡= = ≈ 5,12827 5,12827 metros
4𝑎 4 ∙ −1,5

c ℎ(0) = −1,5 ∙ (0)2 + 5,36 ∙ 0 + 0,34 = 0,34 0,34 metros

−𝑏 −(5,36)
d 𝑡= = ≈ 1,787 1,787 años
2𝑎 2 ∙ −1,5

0,34 + 5,36𝑡 − 1,5𝑡 2 = 5,005 0,06 años, por ser la


e
𝑆 = {3.64,0.06} solución menor a 1,787.

f ℎ(1) = −1,5 ∙ (1)2 + 5,36 ∙ 1 + 0,34 = 4,2 4,2 metros

Para resolver otros problemas similares es necesario recordar que:


 Un objeto lanzado al aire toca el suelo cuando la altura equivale a 0.
 Cuando en la pregunta se incluyan las palabras máximo o mínimo, se hace
referencia a las coordenadas del vértice.
 La altura desde la que es lanzado un objeto se obtiene cuando el tiempo
equivale a 0.
 Cuando en una empresa o en un negocio no se obtienen ganancias es por
una de dos razones: las ganancias son iguales a los costos o las ganancias
son iguales a 0.

Ejemplos

1. Un objeto se lanza desde un edificio de 15 𝑚 de altura a una velocidad de 10 𝑚 ⁄


𝑠. La altura del nivel del suelo en un instante 𝑡 viene dada por:

ℎ(𝑡) = −5𝑡 2 + 10𝑡 + 15


a. Determinar la altura máxima.
b. Determine el número de segundos t para que el objeto choque contra el suelo.

348
Solución
−𝑏 −△ −10 −400
a. 𝑉 = ( 2𝑎 , 4𝑎 ) = (−10 , ) = (1,20) . Luego, la altura máxima es de 20 𝑚.
−20

b.
0 = −5𝑡 2 + 10𝑡 + 15
0 = −5𝑡 2 + 10𝑡 + 15
(𝑡 + 1)(𝑡 − 3) = 0
𝑡 = 3 o 𝑡 = −1.

Luego, objeto chocará contra el suelo en 3 𝑠

2. Sea 𝐴(𝑥) = 𝑥 2 − 10𝑥 + 50 el área de un cuadrado donde 𝑥 es la medida de uno de


los lados. Determine el valor del área mínima.

Solución

−𝑏 −𝑏 10
𝑉=( , 𝑓 ( )) = ( , 25) = (5,25).
2𝑎 2𝑎 2

El valor mínimo es 𝐴 = 25

Ejercicios

Marque con x

1) En una tienda donde se venden calculadoras se ha encontrado que


cuando las calculadoras se venden a un precio “𝑥” dólares por unidad,
el ingreso “𝑟” como una función del precio está dada por 𝑟(𝑥) = −750𝑥 2 +
15000𝑥. ¿Cuál debe ser el precio unitario en dólares para que el ingreso
sea máximo?

a. 5
b. 10
c. 20
d. 80

349
2) Considere las siguientes proposiciones referidas a la función 𝑓 dada por
𝑓(𝑥) = −4,9𝑥 2 + 20𝑥 + 30, que describe la trayectoria a los "𝑥" segundos
de haberse lanzado un proyectil hacia arriba, desde el techo de un
edificio:

I. La altura del edificio desde donde se lanza el proyectil es de 20.


II. En su trayectoria, la altura máxima que alcanza el proyectil, respecto al plano
de donde se lanzó es de aproximadamente 50,41.

De ellas, ¿cuáles son verdaderas?

a. Ambas
b. Ninguna
c. Solo la I
d. Solo la II

3. Si la productividad "𝑝" de una empresa con "𝑥" cantidad de empleados está


dada por 𝑝(𝑥) = −𝑥 2 + 160𝑥 entonces, ¿cuántos empleados garantizan la
productividad máxima de la empresa?

a. 40
b. 80
c. 160
d. 6400

4. La producción “𝑃” en kilogramos de manzanas de una finca está dada por


𝑃(𝑥) = 500𝑥 − 5𝑥 2 , donde “𝑥” es el número de árboles por hectárea. ¿Cuál es
el número de árboles por hectárea que hace que la producción total sea
máxima?

a. 50
b. 100
c.9375
d.12500

5. El fabricante de un artículo ha determinado que el ingreso en dólares "𝐼" en


−𝑥 2
términos del precio de venta "𝑥" está dado por𝐼(𝑥) = + 190𝑥. ¿Cuál es el ingreso
2
máximo que puede obtener el fabricante?

a. 95
b. 190
c. 18 050
d. 36 100

350
6. Se tiene 60 𝑚 de alambre para hacer una cerca de una sola vuelta de un
jardín rectangular sin que sobre alambre. Si la cerca se debe colocar
únicamente en tres lados porque el otro lado limita con una pared, entonces
¿cuál es el área máxima que se puede cercar?

a. 225 𝑚2
b. 300 𝑚2
c. 450 𝑚2
d. 3600 𝑚2

7. Sea 𝑓 una función dada por 𝑓(𝑡) = 20𝑡 − 4,9𝑡 2 + 50 que describe la trayectoria a
los "𝑡" segundos de una piedra lanzada hacia arriba desde el techo de un edificio.
¿Cuál es aproximadamente el tiempo en segundos necesario para que la piedra
alcance su máxima altura con respecto al suelo?

a. 0,12
b. 0,25
c. 2,04
d. 4,08

Práctica de la Unidad III

1) De acuerdo con los datos del DEF , el área “ A ” del triángulo en


términos de “ x ” corresponde a:

A) A  2x  2

B) A  x2  2x

x2  2
A
C) 2

x2  2x
A
D) 2

351
2) De acuerdo con los datos de la gráfica de la función cuadrática f , ¿cuál es un posible criterio de f ?

A) f  x   x2  4 x  3

B) f  x   x2  4x  3

C) f  x   x2  4 x  3

D) f  x    x  4 x  3
2

3) Considere el siguiente contexto:

La altura de un objeto
En la clase de Matemáticas, el profesor junto con sus estudiantes realizan un experimento que consiste en
lanzar un objeto hacia arriba en forma vertical y medir la altura " h " , en metros, que alcanza en un
determinado tiempo " t " en segundos. Con la ayuda de un celular graban el experimento y logran obtener
los datos que se presentan en la siguiente tabla:

1 3 5
t 1 2 3
2 2 2
5 9 5
h 2 2 0
4 4 4

¿Cuál criterio corresponde a una función cuadrática que relaciona la altura del
objeto en términos del tiempo?

A) h  t   3t  t 2

B) h  t   t 2  3t

C) t  h   3h  h2

D) t  h   h2  3h

352
4) La factorización completa de la expresión 8x3  8x es:

A) 8x  x  1 x  1

B) 8x  x  1 x  1

C) 8x2  x  1 x  1

D) 8x2  x  1 x  1

5) Un factor de la expresión x2  2x  3 es:


A) x 3
B) x3
C) x2
D) x2

6) La expresión  63 b6  2a2b2    2a2b  es equivalente a:

A) 3ab5  b
B) 8ab5  4b
C) 3ab5  2a2b2
D) 3a5 b7  a4b3
7) Considere las siguientes proposiciones, referidas a la operación  x2  6x  7    x 1 :

I. El cociente es x  5 .
II. El residuo es 0 .

¿Cuáles de ellas son verdaderas?


A) Ambas
B) Ninguna
C) Solo la I
D) Solo la II

353
2x 5x  2
8) La expresión  es equivalente a:
3x  1 3x  1

7x  2
A)
6x

102  4 x
B)
9 x2  1
21x 2  3x  2
C)
9x2  1
21x 2  3x  2
D)
9x2  1

9) ¿Cuál es una solución de la ecuación 18x2  21x  5  0 ?


A) 3
1
B)
3

6
C)
5

5
D)
6

10) El conjunto solución de la ecuación  x  5  2  x  5 es:


2

A) 7

B) 5,7

C) 3,5

D) 6  6, 6  6 

11) Considere las Siguientes ecuaciones:


I. x2  10x  25  0

II. x2  4  0

¿Cuáles de ellas tienen discriminante igual a cero?


A) Ambas
B) Ninguna
C) Solo la I
D) Solo la II

354
12) Considere el siguiente enunciado:

Doña Marta tiene un lote en forma de rectángulo cuyo largo es el doble


del ancho y su área es de 242 m 2 . Ella desea construir un muro de 1 m de

alto en el borde del lote y dejar una entrada al lote de 3 m de ancho. De

acuerdo con los materiales que requiere, más la mano de obra, cada
metro cuadrado del muro tendrá un costo de ¢15 000 .

De acuerdo con la información anterior, si la entrada al lote no lleva muro,


entonces, ¿cuál es el costo total de muro?
A) ¢450 000

B) ¢945 000

C) ¢990 000

D) ¢3 630 000

Considere el siguiente contexto para responder las preguntas 13, 14 y 15:

El lanzador de pelotas
Pedro lanza una pelota hacia arriba. La altura “ h  t  ” en metros que

alcanza la pelota está dada por h t   4,9t 2  v0t , donde “ t ” es el tiempo en

segundos y “ v0 ” es la velocidad inicial, en metros por segundo, con que

lanza la pelota.

13) Si v0  29, 4 m / s , entonces, ¿cuánto tiempo tarda la pelota en caer al

suelo?

A) 3 s

B) 6 s

C) 24,5 s

D) 34,3 s

355
14) Si v0  29, 4 m / s , entonces, ¿cuál es la altura máxima que alcanza la pelota?

A) 3,0 m

B) 44,1 m

C) 58,8 m

D) 88, 2 m

15) Si la pelota cae al suelo a los 10 s , entonces, ¿cuál fue la velocidad inicial con

que Pedro lanzó la pelota?

A) 4,90 m / s

B) 9,51 m / s

C) 9,80 m / s

D) 49, 00 m / s

Considere el siguiente contexto para responder las preguntas 16 y 17:


La siguiente tabla proporciona algunos datos sobre cierto medicamento, el cual
varía su nivel de efectividad al transcurrir el tiempo de suministrado al paciente.
Esta relación está modelada por C  t   t 2  6t , donde " C  t  " representa el nivel de

efectividad del medicamento a las " t " horas de aplicado:


t 0 2 4 6

C t  0 8 8 0

16) A las tres horas exactas de aplicado el medicamento, el nivel de


efectividad corresponde a:
a. 5
b. 6
c. 8

d. 9

356
17) Si se le suministra el medicamento al paciente a la 5 : 00 a.m. , entonces,
inmediatamente después de desaparecido el efecto de este. ¿A qué hora debe
aplicarse la siguiente dosis?
a. 7 : 00 a.m.
b. 9 : 00 a.m.
c. 10 : 00 a.m.
d. 11: 00 a.m.

18) En la siguiente tabla se representa algunos pares ordenados que


pertenecen al gráfico de la función cuadrática h :
x. -1 0 1 3

h  x 0 -3 -4 0

De acuerdo con la información anterior, la representación gráfica de la función h


corresponde a:

a.
b.

c. d.

357
1
19) Al racionalizar se obtiene como resultado:
x 5 n3

xn
A) 2
x n

n xn
B)
x

x xn
C)
n

xn
D)
x n23

20) Si x2  6x 10 se expresa de la forma  x  h 2  k , entonces, k corresponde

a:
a. 1
b. 3
c. 6

d. 10

21) Al factorizar completamente x102  x100 y 2 , uno de los factores corresponde a:

a. x100

b. x102
c. x2  y

d. x  y 2

22) El residuo de  x2  2   x  1 , corresponde a:

a. 1
b. 2
c. 1

d.  2

358
x 3
23) Al efectuar y simplificar al máximo  , el numerador de la expresión
x  2 3x  6

resultante corresponde a:
a. x 1

b. x 1
c. x3

d. x  3
x 1 2x  4
24) Al efectuar y simplificar al máximo  , el denominador de la
x2  x x  2

expresión resultante corresponde a:

a. x

b.  x
c. x 1

d. x  2

25) Considere las siguientes proposiciones:

I. La ecuación x2  4 tiene una única solución real.


II. La ecuación x2  5x  4 tiene dos soluciones reales distintas.

De ellas, ¿cuál o cuáles son verdaderas?

A) Ambas
B) Ninguna
C) Solo la I
D) Solo la II

359
26) La siguiente figura representa el cuadrado ABCD :

Una ecuación que permite calcular la longitud " x " del lado del cuadrado,
corresponde a:

a. x2  5

b. 2x2  5
c. x2  25

d. 2 x2  25
27) La siguiente figura representa el rectángulo ABCD , donde " x " es la
longitud del ancho:

Si el área del rectángulo es 15 , entonces, el perímetro corresponde a:


a. 13
b. 16
c. 17
d. 30

360
28) La siguiente figura representa el triángulo rectángulo ABC :

¿Cuál es el área de ABC ?


a. 6
b. 9
c. 10
d. 20
Considere el siguiente contexto, para contestar las preguntas 29 y 30:
La función f dada por f  x   5t 2  20t , describe el recorrido de un objeto a los " t "

segundos de haberse lanzado hacia arriba desde el suelo (la altura que alcanza el
objeto se mide en metros y el roce de este con el aire no se considera).
29) ¿Cuántos segundos transcurren desde el momento en que se lanza el
objeto hasta que este regresa al suelo?
a. 4
b. 5
c. 15
d. 20
30) ¿Cuál es la altura máxima, en metros, que alcanza el objeto?
a. 20
b. 25
c. 35

d. 45

361
362
Nuestro primer

Capítulo III Estadística desafío matemático,


un paso más para
aprender
Y Probabilidad
Habilidades Específicas

Al finalizar el capítulo el estudiante deberá estar en capacidad de:

1. Identifica variables cuantitativas y cualitativas (continuas y discretas)

2. Interpreta la información proporcionada por una tabla de distribución de


frecuencia
3. Interpreta Histogramas y polígonos de distribuciones.
4. utiliza el concepto de frecuencia relativa como concepto de aproximación de la
probabilidad.

5. Introduce la ley de los grandes números

Conceptos clave
1.Cualitativas 4.Continua 7.Distribuciones de
frecuencia
2.Cuantitativas 5.Poligono de
Frecuencia 8.Muestras aleatorias
3. Discreta
6.Histograma 9.Ley de los grandes
números

363
 Introducción
Los diversos medios de comunicación juegan un rol cada vez más importante en
la sociedad, aquel ciudadano que posea dominio sobre la información que le
rodea tendrá una mejor oportunidad de desarrollarse dentro
de la misma. Los temas de Estadística y Probabilidad se
incluyen desde hace varios años en el currículo tanto de
primaria como de secundaria de muchos países. Debido al
carácter instrumental de la Estadística para otras disciplinas
y la importancia de su razonamiento dentro de una sociedad
caracterizada por la disposición de información, la necesidad
de analizarla y tomar decisiones basadas en un conjunto de
datos, es que se le ha dado importancia a su enseñanza. Este material está
dirigido a docentes de Matemática de secundaria, pretende ser un apoyo para el
educador en cuanto a conceptos y tipos de actividades que puede realizar al
abordar el tema de Estadística y Probabilidad

Tema 1 Variables

Situación Problema
El director requiere recopilar información del nivel socioeconómico de
las familias que tienen a sus hijos en la institución que él dirige.
Sabemos que en dicha actividad se estableció que las características
del estudio son:
• Número de personas asalariadas pertenecientes al núcleo familiar.
• El salario neto mensual de las personas que conforman el núcleo familiar.
• El nivel de escolaridad que tiene el padre, la madre o el encargado del estudiante.
• El número de personas en primaria, secundaria o en educación superior que
habitan en la vivienda.
• La condición de si la vivienda donde habitan es alquilada, prestada o propia.
• Área de construcción de la vivienda.
• El número de personas que viven en la vivienda.

1. Para cada una de estas variables determine si los datos obtenidos son cantidades
o cualidades. 2. En relación con las variables numéricas, indique si los datos
correspondientes se obtienen por conteo o por medición. 3. En relación con las
variables cualitativas indique si los datos correspondientes pueden ser ordenados
siguiendo algún patrón u orden natural entre ellos

364
Análisis de la actividad
En el cuadro 2 se encuentra la clasificación de las variables según la condición de
numéricas y de cualidad

Cuadro 2. Clasificación de las variables según el tipo de datos que se obtuvieron

En el cuadro 3 se encuentra la clasificación de las variables numéricas.

Cuadro 3. Clasificación de las variables numéricas

365
En el cuadro 4 se encuentra la clasificación de las variables cualitativas.
Cuadro 4. Clasificación de las variables cualitativas

Repaso
Estadística
Es la disciplina que permite recopilar, organizar, sintetizar y analizar datos o hacer
inferencias de ellos.
Población y muestra
La población de un estudio corresponde a la totalidad de los individuos que se
pretende estudiar. Cuando la población es muy grande como para trabajar con
todos sus miembros, se selecciona una parte de ella para hacer el estudio. Esa
selección recibe el nombre de muestra.
Unidad estadística
La unidad estadística es cada uno de los individuos que participan en el estudio.
La Clave
Variables estadísticas
Son las características estudiadas en un trabajo estadístico. Se clasifican
de la siguiente manera:
Tipo de variable estadística Concepto

Son atributos que no se pueden expresar


con cantidades numéricas. Por ejemplo:
Cualitativas
color de cabello, materia abarcada en un
libro, etc.

Los valores de un intervalo real que se le


pueden asignar son determinados. Por
Discretas
ejemplo: la talla de los zapatos o la
cantidad de hijos de una persona.
Cuantitativas
Pueden tomar cualquiera de los valores
de un intervalo real. Por ejemplo: la
Continuas
estatura en metros, el tiempo en
minutos, la masa en kilogramos, etc.

366
Nota Variables cualitativas nominales y ordinales En relación con las características
no numéricas de las unidades estadísticas, se presentan las variables cualitativas,
que también se pueden su clasificar en relación con el tipo de dato que generan.
Como puede notarse, en la variable sexo del jefe del núcleo familiar, los datos
pueden variar en dos categorías: hombre o mujer; pero estas dos categorías
pueden ser colocadas en cualquier orden debido a que no existe prioridad clara
entre ellas, a estas variables se les denomina variables cualitativas nominales.
Por otro lado, cuando se analiza la variable nivel de escolaridad de la persona
encargada del estudiante, se tiene claro que las categorías de escolaridad aunque
son cualidades pueden ser ordenadas naturalmente de menos escolaridad a más
escolaridad o viceversa. En los casos en que se presenta esta situación se dice que
la característica es una variable cualitativa ordinal. Algunos ejemplos de
variables cualitativas nominales son: color de los ojos, religión, preferencia política,
entre otros. Por su parte, algunos ejemplos de variables cualitativas ordinales son:
tallas de las camisas (S, M, L), condición socioeconómica (baja, media, alta), entre
otras.

Nota: A las variables cualitativas se les acostumbra llamar variables categóricas, pues los
datos que generan pertenecen a diferentes categorías. Entonces cuando una variable
cuantitativa se agrupa en clases puede ser analizada como una variable categórica

Ejemplo 1

Clasificar las variables indicadas en cada caso:


Variable

1 Calidad de un producto (bueno, regular o malo)

2 Número de años laborados en una empres

3 Cantidad de artículos diarios vendidos en un negocio

Tipo de programa televisivo favorito (noticieros, telenovelas, reality


4
show, etc)

5 Cantidad de habitantes de una casa

6 Cantidad de litros de gasolina vendidos por un pisteros

367
Solución
Variable Tipo

1 Cualitativa

2 Cuantitativa continua

3 Cuantitativa discreta

4 Cualitativa

5 Cuantitativa discreta

6 Cuantitativa continua

Ejemplo 2
Se desea estudiar, entre otras variables, el peso y la estatura de los
estudiantes de un colegio, con el propósito de evaluar su condición de salud. Una
vez recolectada la información, con los datos se calculó el Índice de Masa Corporal
(IMC) que viene dado por la fórmula:
Peso(kg )
IMC 
Estatura(m)

Una vez determinados los valores numéricos de este índice, tomando en cuenta la
edad y el sexo de los estudiantes, se realizó la siguiente clasificación:
• Peso insuficiente
• Peso normal
• Sobrepeso
• Preobesidad
• Obesidad I
• Obesidad II
• Obesidad extrema
En este caso se puede notar que la variable IMC que originalmente es numérica
se reclasifica en una variable categórica; pero no pierde su sentido pues la
condición de que un estudiante pertenezca a una categoría está en función del
valor numérico de su IMC.
A manera de resumen, el concepto de característica o variable es lo que deseamos
medir o evaluar de la unidad estadística, se puede entonces realizar una
clasificación de las mismas como se presenta en el siguiente esquema:

368
Ejemplo 3 Determine para cada variable contextualizada su clasificación,
a saber: cualitativa (nominal u ordinal) y cuantitativa (continua y discreta).
Variable contextualizada Clasificación de la variable

Estaturas de los integrantes del


equipo de baloncesto de un colegio.

Número de hermanos de cada uno de


los estudiantes de sétimo grado de un
colegio en particular.

Deporte preferido de los estudiantes


de undécimo año.

Color de zapatos de los asistentes a


una fiesta

Nivel de escolaridad de los padres de


familia de los estudiantes de un grupo
guía del docente.

Longitud que hay entre el dedo anular


y el codo de los alumnos del aula.

Cantidad de padres de familia


asistentes a la primera reunión del
año lectivo por nivel, en una
institución en particular.

Temperatura de una persona o su


estado febril utilizando la clasificación
Subfebril o febrícula: Menos de

369
37,5ºC; fiebre ligera: Menos de 38ºC;
fiebre moderada: 38 – 39ºC; fiebre
alta: 40ºC y hiperpirexia: 41ºC.

Solución
Variable contextualizada Clasificación de la variable

Estaturas de los integrantes del Cuantitativa continua


equipo de baloncesto de un colegio.

Número de hermanos de cada uno de Cuantitativa discreta


los estudiantes de sétimo grado de un
colegio en particular.

Deporte preferido de los estudiantes Cualitativa nominal


de undécimo año.

Color de zapatos de los asistentes a Cualitativa nominal


una fiesta

Nivel de escolaridad de los padres de Cualitativa ordinal


familia de los estudiantes de un grupo
guía del docente.

Longitud que hay entre el dedo anular Cuantitativa continua


y el codo de los alumnos del aula.

Cantidad de padres de familia Cuantitativa discreta


asistentes a la primera reunión del
año lectivo por nivel, en una
institución en particular.

Temperatura de una persona o su Cualitativa ordinal


estado febril utilizando la clasificación
Subfebril o febrícula: Menos de
37,5ºC; fiebre ligera: Menos de 38ºC;
fiebre moderada: 38 – 39ºC; fiebre
alta: 40ºC y hiperpirexia: 41ºC.

370
Ejemplos

1) Salario: ¢100 000, ¢85 000,...

2) Edad: 21 años, 14 años,... Variables cuantitativas

3) Sexo: Masculino, Femenino.


Variables cualitativas
4) Estado Civil: soltero(a), casado(a),...

Ejemplo

1) Estatura: 1,70m; 1,64m; 5m...


Variables cuantitativas continuas
2) Peso: 64,5Kg, 60Kg, 72,8Kg,...

3) Edad: 20 años, 18 años,... Variables cuantitativas discretas


4) Cantidad de estudiantes por sección: 30, 34, 28,...

Ejemplo Clasifique las siguientes variables en cualitativa, cuantitativa


discreta o cuantitativa continua

Cuantitativa
Característica Cualitativa Discreta Continua
Raza de perros X
Gastos semanales de una X
compañía
(excesivos, moderados, bajos)
La temperatura de un X
estudiante
La condición socioeconómica X
de una familia
(alta, regular, baja)
Duración de una llamada X
telefónica
Extensión, en 𝑚2 de un terreno X
Sueldo mensual de una X
persona
Clases de insectos X

371
Cantidad de personas de una X
familia
El número de camisas de un X
armario
El mes en que cumple años X
los estudiantes de un
grupo de noveno
El equipo de fútbol preferido X
de un grupo
Especies de árboles X
Distancia recorrida por un X
carro
Velocidad de un avión X
Número de perros en una X
familia
Edad de una persona(niño, X
joven, adulto, anciano)
Cantidad de materias que X
aprueba un joven
en un trimestre
Ingresos recibidos X
anualmente por una empresa
Religión que se profesa X

Ejercicios 1.Clasifique las variables estadísticas siguientes en


cualitativas y cuantitativas

a) Llamadas telefónicas de una familia en un día.

b) Duración de llamadas telefónicas.

c) Precios de los libros que se venden en una librería.

d) Saques de esquina de los partidos de fútbol.

e) Intención del voto en las próximas elecciones.

f) Personas que conviven en un domicilio.

g) Longitud de las palabras en un texto.

372
h) Número de estaciones de radio.

i) Categorías profesionales.

j) Áreas territoriales.

k) Clases de insectos.

l) Calidad del producto.

2. Clasifique las variables estadísticas siguientes en discretas y continuas

a) El número de tomates producidos por cada planta en un invernadero de 100


m2 de superficie.

b) Las temperaturas registradas cada 12 horas en un invernadero.

c) El número de goles marcados cada semana en primera división.

d) La presión atmosférica registrada cada día en le observatorio del volcán


Irazú.

e) La velocidad de un automóvil durante un viaje.

f) Los tamaños de los recién nacidos registrados en la sección maternidad del


Hospital De La Mujer.

g) Los puntos obtenidos en la tirada de un par de dados.

h) El número de litros de gasolina en el tanque de su carro.

i) El número de pantalones de su armario.

j) El peso de un niño durante el primer año de su vida.

373
k) La longitud de cable eléctrico producidos por una fábrica

3. Clasifique la siguiente lista de características como variables cualitativa o


cuantitativa ya sea discreta o continua.

Características Cualitativa Cuantitativa


1) Razas de perros
2) Gastos semanales de una compañía
(excesivos, moderados o bajos)
3) La temperatura de un estudiante.
4) El color de los ojos de una mujer.
5) Puesto que desempeña un trabajador.
6) La condición socioeconómica de un trabajador
(alta, baja, regular).
7) Edad de una persona (niño, joven, adulto,
anciano).
8) Duración de una llamada telefónica.
9) Estado civil de los profesores.
10) Clases de insectos.
11) Religión que profesa.
12) Cantidad de gasolina requerida por un
automóvil (en litros).
13) El número de camisas que hay en un
armario.
14) Consumo de electricidad por vivienda.
15) El área del cantón de Escazú.
16) Peso de los estudiantes.
17) Estatura de un jugador de baloncesto.
18) La velocidad que viaja un avión.
19) Tipos de papel.
20) La altitud de un cerro.
21) Cantidad de carros que posee una
familia.
22) Ingresos familiares en colones.
23) Distancia recorrida por un automóvil.
24) Estatura de una persona en
centímetros.
25) Densidad poblacional de Costa Rica.
26) La escolaridad de una persona.
27) Salario de un trabajador.
28) Número de hijos de una familia.
29) Edad de una persona en años.
30) Mi equipo de fútbol preferido.

374
31) El número de años trabajados por los
empleados de una empresa.
32) El mes en que cumplo años.
33) El número de turistas que vienen al país
cada año.
34) Especies de árboles.
35) El número de personas que van de
romería a Cartago.
36) El consumo de agua por vivienda en m3.
37) Calidad de un producto.
38) La longitud de una faja.
39) La cantidad de pintura necesaria para
pintar una casa.
40) El número de lapiceros que hay en una
cartuchera.

Distribuciones de frecuencias

La frecuencia de una variable estadística se refiere a cuántas veces se


repite cada valor en una lista. Se pueden identificar tres tipos:
Tipo de frecuencia Concepto

Cantidad exacta de veces que se repite el valor o de


Absoluta
valores del intervalo.

Corresponde a la frecuencia absoluta dividida por el


Relativa
total de datos.

Es la frecuencia relativa multiplicada por 100. Se


Relativa porcentual
expresa con la notación de porcentaje.

Frecuencias acumuladas
Se trata de sumar las frecuencias indicadas (cualquiera de los tres tipos existentes)
hacia arriba (más de) y hacia abajo (menos de). Si el cálculo ha sido correcto, los
resultados de la última frecuencia acumulada deben ser los siguientes:
 Frecuencia absoluta: el total de los datos obtenidos.
 Frecuencia relativa: 1.
 Frecuencia relativa porcentual: 100%.
Dependiendo del tipo de variable, este tipo de distribuciones se trabaja de forma
diferente.

375
Ejemplo
De acuerdo con los datos que se le presentan, correspondientes a las
edades de los empleados de un Call Center, en años cumplidos, elabore una
distribución de frecuencias:

24 23 21 24 23 21 23 24 25 23

22 22 24 25 23 21 22 21 23 24

21 21 21 23 22 22 22 24 21 22

24 25 23 22 21 23 24 25 23 21

22 23 24 21 22 22 24 23 25 21
Solución

Frecuencia
Frecuencia Frecuencia
Dato relativa
absoluta relativa
porcentual

21 12 0,24 24%

22 11 0,22 22%

23 12 0,24 24%

24 10 0,20 20%

25 5 0,10 10%

Total 50 1 100%

Ejemplo 2
En un grupo 25 niños de kínder se le preguntó a cada niño por la cantidad de
mascotas que hay en su casa. Las respuestas obtenidas fueron las siguientes:
Elabore 1 0 2 3 4 0 4 3 1 1 0 4 3 4 2 una

1 2 3 1 2 2 3 2 1 1

distribución de frecuencias simple y otras dos incluyendo las frecuencias


acumuladas más de y menos de.

376
Solución
La distribución con las frecuencias simples es la siguiente:

Frecuencia
Frecuencia Frecuencia
Dato relativa
absoluta relativa
porcentual

0 3 0,12 12%

1 7 0,28 28%

2 6 0,24 24%

3 5 0,20 20%

4 4 0,16 16%

Total 25 1 100%

Calculando las frecuencias acumuladas con los datos anteriores, se obtienen las
siguientes distribuciones de frecuencias:

 Más de (hacia arriba):

Frecuencia
Frecuencia Más Frecuencia Más Más
Dato relativa
absoluta de relativa de de
porcentual

0 3 25 0,12 1 12% 100%

1 7 22 0,28 0,88 28% 88%

2 6 15 0,24 0,60 24% 60%

3 5 9 0,20 0,36 20% 36%

4 4 4 0,16 0,16 16% 16%

377
Menos de(hacia abajo):

Frecuencia
Frecuencia Menos Frecuencia Menos Menos
Dato relativa
absoluta de relativa de de
porcentual

0 3 3 0,12 0,12 12% 12%

1 7 10 0,28 0,40 28% 40%

2 6 16 0,24 0,64 24% 64%

3 5 21 0,20 0,84 20% 84%

4 4 25 0,16 1 16% 100%

Como se puede ver en las dos distribuciones, la última frecuencia acumulada


cumple las condiciones dadas para la última en calcularse. En este caso, por ser 25
niños, la última frecuencia absoluta acumulada da 25, la última relativa acumulada
da como resultado 1 y la última relativa porcentual acumulada da como resultado
100%.
La frecuencia absoluta para las variables cuantitativas continuas se trabaja por
intervalos, pues todos los valores son diferentes. Antes de introducir cómo se
trabajan, es necesario aclarar algunos conceptos:
Concepto Descripción

Amplitud general, rango Es la diferencia entre el mayor valor y el


o recorrido valor menor que toma la variable. Para
poder determinarlo, es necesario ordenar los
datos.

Clase o intervalo Es un conjunto que contiene al menos un


dato estadístico. Se sugiere calcular la
cantidad de clases como √𝑛, donde 𝑛
corresponde a la totalidad de datos. Siempre
se redondea. Se recomienda que sean entre
5 y 10.

Amplitud de clase Es el tamaño de cada clase. Se obtiene al


dividir el rango por la cantidad de clases.
Siempre se redondea.

378
Concepto Descripción

Límites de clase Son los valores que delimitan las clases. El


límite menor (o inferior) es el mínimo valor
de la clase, mientras que el límite mayor (o
superior) corresponde al máximo valor de la
clase. Generalmente se utiliza una menor
unidad para determinarlos. Por ejemplo, si
los datos son números enteros, los límites se
determinan con números que contengan una
posición decimal (décimas), tales que el
primer límite menor sea más pequeño que el
menor dato y que el último límite superior
sea más grande que el dato mayor. De ser
necesario, se agrega una clase adicional, de
modo que todos los datos estén dentro de
una.

Marca de clase Es el valor medio de la clase se obtiene al


sumar el límite superior y el límite inferior y
luego dividir ese resultado por 2.

Frecuencia absoluta de Corresponde a la cantidad de datos que


la clase pertenecen a la clase, por lo que éstos
pueden ser diferentes.

Frecuencia relativa de la Corresponde a la frecuencia absoluta de la


clase clase dividida por el total de datos.

Frecuencia relativa Es la frecuencia relativa multiplicada por 100.


porcentual Se expresa con la notación de porcentaje.

Ejemplo
Elabore una tabla de frecuencias que permita agrupar los datos del
siguiente contexto:

379
Las notas de 40 estudiantes en un examen de Francés fueron las siguientes:

84 70 81 63 90 91 81 35 85 84

89 73 48 73 56 79 78 43 86 74

93 63 59 96 76 13 75 86 95 30

73 31 79 53 71 68 55 83 81 18

SoluciónPrimero se debe determinar el dato mayor (96) y el dato menor (13).


Así, se tienen los siguientes cálculos:

Concepto Descripción

Amplitud general, rango


96 − 13 = 83
o recorrido

Clase o intervalo La cantidad de clases es √40 ≈ 6.

Amplitud de clase 83 ÷ 6 ≈ 14

Como el menor valor es 13, se comenzará la primera clase en 13 −


0,5 = 12,5. Los siguientes límites son:

12,5 + 14 = 26,5

26,5 + 14 = 40,5
Límites de clase
40,5 + 14 = 54,5

54,5 + 14 = 68,5

68,5 + 14 = 82,5

82,5 + 14 = 96,5

Las marcas de las seis clases son:

(12,5 + 26,5) ÷ 2 = 19,5

(26,5 + 40,5) ÷ 2 = 33,5

Marca de clase (40,5 + 54,5) ÷ 2 = 47,5

(54,5 + 68,5) ÷ 2 = 61,5

(68,5 + 82,5) ÷ 2 = 75,5

(82,5 + 96,5) ÷ 2 = 89,5

380
De esta manera, la distribución de frecuencias solicitada quedaría como sigue:

Marca de Frecuencia Frecuencia Frecuencia relativa


Clase
la clase absoluta relativa porcentual

[12.5,26.5[ 19,5 2 0,05 5%

[26.5,40.5[ 33,5 3 0,075 7,5%

[40.5,54.5[ 47,5 3 0,075 7,5%

[54.5,68.5[ 61,5 6 0,15 15%

[68.5,82.5[ 75,5 14 0,35 35%

[82.5,96.5[ 89,5 12 0,30 30%

Total 40 1 100%

Ejemplo
Elabore una tabla de frecuencias que permita agrupar los datos del siguiente
contexto:

Se efectúa un estudio de acuerdo a una muestra de 50 automóviles de una


determinada marca, con respecto al consumo de combustible en litros
redondeados a la unidad más próxima, cuando el vehículo abarca una
distancia de 350 kilómetros:

27 34 31 29 27 24 23 36 37 31

38 36 36 36 34 32 38 25 38 26

22 32 36 27 27 32 28 31 28 39

18 20 25 27 33 37 40 37 26 36

20 26 40 21 35 35 22 24 28 19

381
Solución
Primero se debe determinar el dato mayor (40) y el dato menor (18). Así, se tienen
los siguientes cálculos:

Concepto Descripción

Amplitud general, rango o recorrido 40 − 18 = 22

Clase o intervalo La cantidad de clases es √50 ≈ 7.

Amplitud de clase 22 ÷ 7 ≈ 3

Límites de clase Como el menor valor es 18, se comenzará la


primera clase en 18 − 0,5 = 17,5.

Los siguientes límites son: 17,5 + 3 = 20,5

20,5 + 3 = 23,5

23,5 + 3 = 26,5

26,5 + 3 = 29,5

29,5 + 3 = 32,5

32,5 + 3 = 35,5

35,5 + 3 = 38,5

38,5 + 3 = 41,5

Debido a que la sétima clase no llega al


máximo valor, se añade una clase más.

382
Concepto Descripción

Marca de clase Las marcas de las ocho clases son:

(17,5 + 20,5) ÷ 2 = 19

(20,5 + 23,5) ÷ 2 = 22

(23,5 + 26,5) ÷ 2 = 25

(26,5 + 29,5) ÷ 2 = 28

(29,5 + 32,5) ÷ 2 = 31

(32,5 + 35,5) ÷ 2 = 34

(35,5 + 38,5) ÷ 2 = 37

(38,5 + 41,5) ÷ 2 = 40

De esta manera, la distribución de frecuencias solicitada quedaría como sigue:

Marca de Frecuencia Frecuencia Frecuencia relativa


Clase
la clase absoluta relativa porcentual

[17.5,20.5[ 19 4 0,08 8%

[20.5,23.5[ 22 4 0,08 8%

[23.5,26.5[ 25 7 0,14 14%

[26.5,29.5[ 28 9 0,18 18%

[29.5,32.5[ 31 6 0,12 12%

[32.5,35.5[ 34 5 0,10 10%

[35.5,38.5[ 37 12 0,24 24%

[38.5,41.5[ 40 3 0,06 6%

Total 50 1 100%

383
Ejercicios
A) Don David, por su experiencia en Mercadeo y por conocer algo de
Psicología del Consumidor le piden que evalué la aceptación de un
Perfume en cada dama el lo realiza a través de une encuesta de 40
damas, donde mide la opinión de éstas con punteos de una a cien.
El cuadro siguiente registra la información obtenida.

PUNTEOS SOBRE LA OPINIÓN DEL PERFUME

68 73 62 66 96 79 65 86 84 79

65 78 78 62 80 67 75 88 75 82

89 67 73 74 82 73 87 75 61 97

57 81 68 60 74 94 75 78 88 72

Punteos Mujeres Frecuencia Frecuencia Frecuencia Frecuencia Frecuencia


Absoluta Relativa Acumulada Acumulada

Relativa

56 – 60

61 – 65

66 – 70

71 – 75

76 – 80

81 – 85

86 – 90

91 – 95

96 – 100

384
2Estaturas en centímetros de una muestra de 35 varones adultos

156 166 173 178 183 159 168

160 170 174 179 185 163 171

164 175 175 181 187 165 172

165 172 177 182 188 178 173

187 185 184 182 180 176 175

Punteos Mujeres Frecuencia Frecuencia Frecuencia Frecuencia Frecuencia


Absoluta Relativa Acumulada Acumulada

Relativa

156 – 163

164 – 171

172 – 179

180 – 187

188 – 195

3. Construya una tabla de frecuencias absolutas y frecuencias relativas para cada


uno de los siguientes problemas. (Sugerencia: para aquellos estudiantes que
tengan acceso a una computadora, utilizar Microsoft Excel para la construcción de
las distribuciones de frecuencia)

a) Los 40 alumnos de una clase han obtenido las siguientes calificaciones.


3, 15, 24, 28, 33, 35, 38, 42, 23, 38, 36, 34, 29, 25, 17, 7, 34, 36, 39, 44, 31, 26,
20, 11, 13, 22, 27, 47, 39, 37, 34, 32, 35, 28, 38, 41, 48, 15, 32, 13

b) Los miembros de una comunidad pequeña tienen las siguientes edades:


42 60 60 38 60 63 21 66 56 57 51 57 44 45 35 30 35 47 53 49 50 49 38 45 28 41
47 42 53 32 54 38 40 63 48 33 35 61 47 41 55 53 27 20 21 42 21 39 39 34 45 39
28 54 33 35 43 48 48 27 53 30 29 53 38 52 54 27 27 43 28 63 41 23 58 56 59 60
40 24

385
c) Investigados los precios de un mismo artículo en diferentes locales comerciales
de una ciudad se han obtenido los siguientes resultados

700 300 500 400 500 700 400 750 800 500 500 750 300 700 1000 1500 500 750
1200 800 400 500 300 500 1000 300 400 500 700 500 300 400 700 400 700 500
400 700 1000 750 700 800 750 700 750 800 700 700 1200 800

d) Puntajes obtenidos por los 100 candidatos que se presentaron a un concurso:


38 51 32 65 25 28 34 12 29 43 71 62 50 37 8 24 19 47 81 53 16 62 50 37 4 17
75 94 6 25 55 38 46 16 72 64 61 33 59 21 13 92 37 43 58 52 88 27 74 66 63 28
36 19 56 84 38 6 42 50 98 51 62 3 17 43 47 54 58 26 12 42 34 68 77 45 60 31
72 23 18 22 70 34 5 59 20 68 55 49 33 52 14 40 38 54 50 11 41 76

4. Considere la siguiente tabla de frecuencias y determine

a) ¿Cuántos estudiantes tiene una nota


menor a 60?
b) ¿Qué porcentaje de estudiantes tiene
una nota menor a 70?
c) ¿Cuál es el intervalo de clase que tiene
mayor frecuencia?
d) ¿Cuál es el intervalo de clase que tiene
menor frecuencia?
e) ¿Qué porcentaje de estudiantes tiene
una nota de 75o más?
f) ¿Cuál es el límite inferior de la primera
clase?
g) ¿Cuál es el límite superior de la última
clase?

386
5. Considere la siguiente tabla de frecuencias y determine

1) ¿Cuántos empleados tiene una edad menor a 40?


2) ¿Qué porcentaje de empleados tiene una edad menor a 60?
3) ¿Cuál es el intervalo de clase que tiene mayor frecuencia?
4) ¿Cuál es el intervalo de clase que tiene menor frecuencia?
5) ¿Qué porcentaje de empleados tiene una edad de 50o más?
6) ¿Cuál es el límite inferior de la segunda clase?
7) ¿Cuál es el límite superior de la cuarta clase

Gráficos estadísticos
Con los datos ordenados en distribuciones de frecuencias, y utilizando
para rotular las clases los límites o las marcas, es posible resumir los
resultados por medio de la elaboración de dos tipos de gráficos:
histogramas y polígonos de frecuencias. Como cada uno tiene sus características
particulares, se presentan por separado.

 Histograma: es un gráfico de barras verticales que representa las frecuencias


absolutas o relativas de cada clase, donde cada clase se representa con sus
límites o con su marca. Las barras, a diferencia de los gráficas para variables
cuantitativas discretas, no tiene separaciones entre la barras.

Para el ejemplo anterior, si se elabora un histograma que represente los datos


obtenidos, rotulando las clases con sus límites, se tendría el siguiente:

387
Al rotular las clases con la marca de cada una, se resumen los datos en el siguiente
histograma:

 Polígono de frecuencias: es un gráfico en el que se representa las frecuencias


absolutas o relativas de cada clase, donde cada clase se representa con su
marca. Para establecer una conexión se ubica un punto que una cada marca
o representación con los límites de la clase con su frecuencia (absoluta o
relativa). Y, finalmente, se unen los puntos, de dos en dos, con una línea
recta.

388
Así, si para el ejemplo anterior, si se elabora un polígono de frecuencias que
represente los datos obtenidos, se tendría el siguiente:

Al rotular las clases con los límites de cada una, se resumen los datos en el siguiente
polígono de frecuencias:

389
Interpretación de distribuciones de frecuencias y gráficos estadísticos con
variables cuantitativas continuas
Siendo realistas, se debe admitir que todos los días interpretamos distribuciones
de frecuencias y gráficos estadísticos, aunque no necesariamente somos
conscientes de ello. Constantemente vemos resultados de elecciones en algún país
del mundo, resultados de aprobación en bachillerato, tablas de posiciones de los
equipos de alguna competencia deportiva, entre otros. Sin embargo, en ocasiones
se interpretan los datos de forma errónea.

Ejemplo
Se tiene los datos del Censo realizado en el 2011 en Costa Rica, relativos a la
asistencia a la educación regular en Costa Rica, de acuerdo con la edad de los
costarricenses mayores de 5 años:

Total que Kínder o


Población
Grupo de asiste a la preparatoria, Parauniversitaria Enseñanza
de 5 años y
edad educación escuela o o universidad especial
más
regular colegio
Total 3 962 995 1 194 587 891 100 300 447 3 040
5 a 9 años 342 057 308 936 307 972 - 964
10 a 14 años 387 056 352 522 351 424 - 1 098
15 a 19 años 405 176 239 722 187 654 51 304 764
20 a 24 años 410 480 136 999 20 958 115 827 214
25 a 29 años 378 424 69 624 8 244 61 380 -
30 a 34 años 332 897 35 426 4 750 30 676 -
35 a 39 años 288 071 18 183 3 036 15 147 -
40 a 44 años 282 914 11 850 2 096 9 754 -
45 a 49 años 267 747 8 648 1 463 7 185 -
50 a 54 años 235 256 5 549 1 103 4 446 -
55 a 59 años 183 581 3 246 793 2 453 -
60 a 64 años 137 624 1 934 640 1 294 -
65 a 69 años 103 528 872 353 519 -
70 a 74 años 78 054 502 262 240 -
75 años y más 130 130 574 352 222 -
Fuente: http://www.inec.go.cr/Web/Home/GeneradorPagina.aspx

390
De acuerdo con este cuadro, responda las siguientes preguntas:

a. ¿Qué porcentaje de las personas que tenían entre 20 y 24 años se


encontraban inscritas en el sistema educativo regular?
b. ¿Qué porcentaje de las personas que asistían a las instituciones
parauniversitarias o universitarias tenían entre 45 y 49 años?
c. ¿Cuál es el grupo de edad que tenía más personas inscritas en los
centros de enseñanza especial?
d. ¿Cuántas personas mayores de 50 años asistían a la educación
regular?
e. ¿Cuántas personas que estudiaban en kínder, escuela o colegio
tienen entre 25 y 29 años?

Solución
Para responder, se deben considerar ciertos parámetros. Así:
1. El total de personas inscritas en el sistema educativo regular que tenían entre
20 y 24 años era de 136 999, mientras que el total de habitantes con esas
edades era de 410 480, por lo que el porcentaje se obtiene al resolver
(136 999 ÷ 410 480) × 100 ≈ 33,4%.

2. El total de personas inscritas en las instituciones universitarias o


parauniversitarias era de 300 447, mientras que, de ese grupo, 7185 tenían
entre 45 y 49 años. De esta manera, el porcentaje se obtiene al resolver la
operación (7185 ÷ 300 447) × 100 ≈ 2,4%.

3. Entre los datos de las instituciones de enseñanza especial, el mayor es 10989


personas, que corresponde a las personas que tenían entre 10 y 14 años.

4. Para la educación regular en general se tienen los siguientes datos: de 50 a


54 años, habían 5549 personas inscritas, de 55 a 59 años, 3246 personas,
de 60 a 64 años, 1934 personas, de 65 a 69 años, habían 872 personas, de
70 a 74 años, 502 personas y mayores de 75 se tenía un total de 574
personas estudiando en el sistema educativo regular. Así, el total de personas
mayores de 50 años que estaban inscritas en el sistema educativo regular
corresponde a la frecuencia acumulada más de hasta la clase que inicia en
50, es decir, 5549 + 3246 + 1934 + 872 + 502 + 574 = 12677.

5. Para las personas que tenían entre 25 y 29 años, el total era de 8244.

391
Ejemplo
Con respecto al histograma adjunto, responda las siguientes preguntas:

1. ¿Cuál es la clase con la mayor frecuencia de longitudes de los caminos


asfaltados?
2. ¿Cuál es el porcentaje de los caminos asfaltados cuya longitud está entre los
5,5 y los 7,5 kilómetros?
3. ¿Cuántos caminos asfaltados tienen su longitud entre los 7,5 y los 11,5
kilómetros?

Solución

Respetando los parámetros a utilizar, las respuestas son las siguientes:

1. Como la columna de la primera clase es la más alta, entonces la clase con


la mayor frecuencia es la de las longitudes entre 2,5 y 3,5 kilómetros.
2. El total de caminos corresponde a 6 + 3 + 4 + 2 = 15 y los caminos cuya
longitud está entre 5,5 y 7,5 kilómetros son 4, por lo que el porcentaje se
calcula al resolver (4 ÷ 15) × 100 ≈ 26,7%.
3. La respuesta corresponde a la frecuencia absoluta de esa clase, que
corresponde a 2 caminos asfaltados.

392
Ejemplo 10

Interprete la información del siguiente polígono de frecuencias:

1. ¿Cuál es la clase donde se ubican los kilogramos rebajados por la mayor


cantidad de personas?
2. ¿Cuántas personas rebajaron entre 1,5 y 4,5 kilogramos?
3. ¿Qué porcentaje de las personas perdió entre 4,5 y 5,5 kilogramos?

Solución
1. La respuesta corresponde a la clase con la mayor frecuencia absoluta, que es
la tercera, es decir la clase que va de 3,5 a 4,5 kilogramos.
2. Para responder a esta pregunta, es necesario identificar las clases
involucradas: de 1,5 a 2,5, hay un total de 6 personas. De 2,5 a 3,5, hay un
total de 8 personas y, finalmente, de 3,5 a 4,5 hay un total de 10 personas.
Así, el total de personas que perdieron entre 1,5 y 4,5 kilogramos
corresponde a 6 + 8 + 10 = 24.
3. El título del gráfico indica que el total de personas fue 30. Y, de acuerdo con
los datos del gráfico, un total de 6 personas perdió entre 4,5 y 5,5 kilogramos.
Así, el porcentaje se obtiene al resolver (6 ÷ 30) × 100 = 20%.

393
Muestras aleatorias
La muestra en un trabajo estadístico se utiliza cuando la población es muy
grande como para estudiar a todos sus miembros uno por uno. El
muestreo es la técnica utilizada para determinar la muestra a partir de una
población. Existen tres tipos de muestreo, que determinan los tres tipos de muestra
utilizados:
 Aleatoria: se determina la muestra utilizando el azar.
 Determinista: la muestra se determina a conveniencia de las personas que
llevan a cabo el estudio, por ejemplo, por facilidad de acceso a los
informantes, entre otras razones.
 Censo: la población no es muy grande, por lo que se realiza el estudio con la
totalidad de los individuos que la conforman.

Ejemplo
Clasifique el muestreo de las siguientes situaciones, definiendo si la muestra
utilizada es aleatoria, determinista o si corresponde a un censo:
1. En un grupo de estudio hay 8 miembros y se quiere saber cuál es su color
favorito, se les pregunta uno por uno.
2. Para estudiar la ingesta de frutas en los estudiantes de primaria, se estudia
a todos los estudiantes de un grupo en varias escuelas cercanas a la casa de
los que realizan el estudio.
3. Se hace una encuesta para saber cuál emisora de radio es la más escuchada,
para ello se escoge una parte de la población y se le consulta por medio de
una llamada telefónica.
4. Al repartir los premios de una rifa, se asigna un número a cada participante
y se van seleccionando números al sacarlos de una tómbola.
5. En un equipo de fútbol se toma las medidas a cada uno de los jugadores para
elaborar el nuevo uniforme.
6. Para estudiar la presencia de las caries en los estudiantes de secundaria, se
estudia a todos los estudiantes de un solo cantón.
Solución
1. En el estudio se utiliza un censo.
2. En el estudio se utiliza una muestra determinista.
3. En el estudio se utiliza una muestra aleatoria.
4. En el estudio se utiliza una muestra aleatoria.
5. En el estudio se utiliza un censo.
6. En el estudio se utiliza una muestra determinista.

394
Ejercicios
1. Clasifique las siguientes variables como cualitativas o
cuantitativas (discretas o continuas):
a) Estado civil de una persona
b) Estatura en centímetros
c) Estatura en metros
d) Edad en años cumplidos
e) Color de ojos
f) Precio de un producto en dólares
g) Cantidad de masa en kilogramos
h) Altura de un edificio en metros
i) Temperatura en grados Celsius
j) Equipo favorito de fútbol

2. Elabore una distribución de frecuencias, incluyendo las acumuladas “más de”,


para las calificaciones obtenidas por un grupo de 33 estudiantes en un
examen de Filosofía:

85 53 83 83 100 100 98 98 75 17 67

83 90 67 100 82 58 98 67 98 100 68

98 100 77 100 90 82 84 100 95 62 22

3. En una tarea de Química, 32 estudiantes obtuvieron las siguientes


calificaciones:

91 85 59 79 91 79 96 94

46 59 91 75 60 96 79 78

75 79 93 91 98 83 51 98

59 79 83 83 90 91 36 93

Elabore una distribución de frecuencias simples (absoluta, relativa y relativa


porcentual) y responda las siguientes preguntas:

a) Si se aprueba con 70 como nota mínima, ¿cuántas personas aprobaron?


b) ¿Cuál es la clase donde se ubica la mayor cantidad de estudiantes?

395
4. El histograma adjunto representa la distribución de las calificaciones de un
grupo de estudiantes en un examen de Psicología. Con base en sus datos,
responda las siguientes preguntas:

Fuente: https://lauracaleroncaballero.files.wordpress.com/2010/03/dibujo.jpg

a) ¿Cuántos estudiantes tiene el grupo?


b) Si para aprobar la prueba la nota mínima es 70, ¿cuántos estudiantes
reprobaron?
c) ¿En cuál clase hubo menos estudiantes?
d) ¿Cuáles son las dos clases donde hubo más estudiantes?
e) ¿Cuál es el límite superior de la tercera clase?
f) ¿Cuál es el límite inferior de la sétima clase?
g) ¿Cuántas clases hay?

5. Considere los datos del polígono de frecuencias adjunto, donde se presentan


las masas, en kilogramos, de los niños de una escuela, para responder las
siguientes preguntas:

396
Fuente: https://sites.google.com/site/excelestadistico1/unidades/continuacion-unidad-iv

a) ¿Cuántas clases hay?


b) ¿Cuántos niños hay en la escuela?
c) ¿Cuántos niños tienen una masa de más de 50 kg?
d) ¿Cuántos niños tienen una masa de menos de 48 kg?
e) ¿Cuál es el límite superior de la sexta clase?
f) ¿Cuál es el límite inferior de la tercera clase?
g) ¿Cuál es la clase que representa más niños?

6. Determine el tipo de muestreo que se utiliza en cada caso (censo, aleatorio


o determinista):
a) Se pregunta a los estudiantes de sétimo año por su bebida favorita. El estudio
se hace para ver las preferencias de los estudiantes de todo el colegio.
b) Se elige a los representantes de una sección para una competencia deportiva
asignando un número a cada estudiante y se saca de una bolsa cinco papeles
con los números de los elegidos.
c) Se hace un estudio donde se quiere saber el programa favorito de los
seguidores de una emisora. Se hace la pregunta a cada aficionado que llama
a sus líneas durante un día.
d) Se utilizan los números de tres páginas de un directorio telefónico para
investigar si una marca nueva de chocolates es conocida por los
consumidores.
e) De cada cinco llamadas que ingresan a un programa televisivo en vivo se
elige una para que el televidente interactúe con los presentadores.
f) Para determinar la estatura promedio de un grupo de niños de primer grado,
la maestra mide la estatura de todos los niños.
g) Para determinar la materia favorita de los estudiantes de noveno año, se le
hace la pregunta a los estudiantes del 9-1.

397
7. El histograma adjunto, rotulado con las marcas de cada clase, representa las
masas, en kilogramos, de los miembros de una familia. Responda con base
en sus datos las siguientes preguntas:

Fuente: http://cbpacedi1.blogspot.com/2011/03/

a) ¿Cuántos intervalos de clase hay?


b) ¿Cuál es la marca de la cuarta clase?
c) ¿En cuál clase hay más miembros de la familia?
d) ¿En cuál clase hay menos miembros de la familia?
e) ¿Cuál es la marca de la primera clase?
f) ¿Cuál es la marca de la última clase?

8. Elabore un histograma y un polígono de frecuencias para la siguiente


distribución de frecuencias:

Edades de los integrantes de un grupo de baile


Edades Integrantes

[14.5, 19.5[ 7

[19.5, 24.5[ 9

[24.5, 29.5[ 10

[29.5, 34.5[ 5

[34.5, 39.5[ 3

[39.5, 44.5[ 6

Total 40

398
399
400
Tema 2 Probabilidad

Situación Problema
a) Nos presentamos a un supermercado a comprar un litro de aceite, y
aunque el contenido del recipiente dice 1000 ml, tenemos la duda de
si efectivamente estos recipientes contienen esta cantidad, por lo que estamos
interesados en determinar la probabilidad que el recipiente tenga menos de un litro
tal como dice la etiqueta. ¿Cómo se podría enfrentar este problema? b) Existe la
creencia entre la población que en un embarazo cualquiera es igualmente probable
que la criatura sea niño o niña; no obstante, los datos indican que en una población
hay más mujeres que hombres. ¿Cómo comprobar si son igualmente probables? c)
Se ha afirmado que la probabilidad de que una persona fumadora muera de una
enfermedad asociada con el consumo del cigarrillo es aproximadamente de un
medio. ¿Cómo creen que ha sido posible estimar esta relación?

Análisis del Problema


En relación con las interrogantes, queda claro que es imposible conocer el espacio
muestral, por ello se requiere un análisis particular para cada caso:
a) Ante la imposibilidad de analizar todos los recipientes con aceite que se
producen, si se toma una muestra aleatoria de 50 de éstos y se mide su contenido
se puede tener una aproximación a la probabilidad de que un recipiente contenga
menos de un litro, por medio de la frecuencia relativa del subconjunto de la muestra
que contiene menos de un litro de aceite.
b) Del mismo modo, para estimar la probabilidad de que nazca un niño como
producto de un embarazo cualquiera, se puede tomar una muestra aleatoria de
varios nacimientos y determinar la frecuencia relativa de nacimientos varones con
respecto al total de nacimientos considerados.
c) Finalmente, haciendo un análisis de defunciones para las cuales se sabe que en
vida la persona era fumadora y determinando la causa de la muerte, sería posible
aproximar la probabilidad de que una persona fumadora muera por causa de una
enfermedad vinculada con el fumado.

401
La Clave
Probabilidad
Dentro del estudio de la probabilidad se utilizan los siguientes conceptos:

1) Espacio muestral: Corresponde al conjunto de resultados que se puede


obtener al realizar un experimento.
2) Punto muestral: Es cada uno de los elementos del espacio muestral.
3) Casos favorables: Puntos muestrales que satisfacen la o las características
que indica un evento.
4) Probabilidad clásica o laplaciana:
 Se utiliza cuando el espacio muestral es relativamente pequeño.
 Debe su nombre a Pierre-Simon Laplace
 Corresponde a dividir el total de casos favorables por el total de casos
posibles:
𝑐𝑎𝑠𝑜𝑠 𝑓𝑎𝑣𝑜𝑟𝑎𝑏𝑙𝑒𝑠
𝑃𝑟𝑜𝑏𝑎𝑏𝑖𝑙𝑖𝑑𝑎𝑑 =
𝑡𝑜𝑡𝑎𝑙 𝑑𝑒 𝑐𝑎𝑠𝑜𝑠 𝑝𝑜𝑠𝑖𝑏𝑙𝑒𝑠
 Reglas:
1. La probabilidad de un evento imposible es igual a 0.
2. La probabilidad de un evento probable corresponde a un número
decimal, ubicado entre 0 y 1.
3. La probabilidad de un evento seguro es 1.

5) Evento: conjunto de puntos muéstrales de un experimento.


6) Tipos de eventos:
 Clasificación por la cantidad de casos favorables:
a. Simple: solo tiene un caso favorable.
b. Compuesto: tiene más de un caso favorable.
c. Imposible o improbable: no tiene casos favorables.
d. Posible o probable: tiene al menos un caso favorable y uno no
favorable.
e. Seguro: todos los puntos muéstrales son casos favorables.
 Clasificación por comparación de dos eventos A y B:
a. Más probable: A es más probable que B si tiene más casos favorables.
b. Menos probable: A es menos probable que B si tiene menos casos
favorables.
c. Igualmente probable: A es igualmente probable que B si tienen la
misma cantidad de casos favorables.
d. Mutuamente excluyentes: un evento es probable o seguro mientras
que el otro es imposible

402
Ejemplo

Determinar la probabilidad de obtener un escudo al lanzar una moneda.

Solución
Se tiene un total de dos eventos posibles: se obtiene un escudo o se obtiene una
corona. Sin embargo, solo se tiene un evento favorable: obtener un escudo. La
probabilidad en este caso sería el resultado de 1 ÷ 2 = 0,5.

Ejemplo

Determinar la probabilidad de obtener un número mayor que 4 cuando se lanza un


dado.

Solución
El total de eventos posibles es de seis: obtener un 1, un 2, un 3, un 4, un 5 o un
6. No obstante, el total de eventos favorables es de dos: se obtiene un 5 o se
obtiene un 6. La probabilidad en este caso corresponde a 2 ÷ 6 = 0,33.
La Clave

Probabilidad frecuencial Se utiliza cuando el espacio muestral es


muy grande. En este tipo de casos, se elige una muestra y frecuencia
relativa porcentual de los casos favorables de esa muestra se utilizan
como la probabilidad dada.

Ejemplo
Una fábrica de llantas en Costa Rica produce como promedio diario 11 000 llantas.
En el departamento de control de calidad realizan un muestreo de los lotes
producidos para determinarlos defectos. Según algunos registros, tomando en
cuenta 33 000 llantas de tres días diferentes, se dieron los siguientes resultados
de las llantas que presentaron defectos:
Defecto Cantidad de llantas

Separación de banda rodadora 207

Bombas o erupciones 170

Pérdida de presión 130

Otros 142

Total 649

403
Determine:
a. ¿Cuál es la probabilidad de que una llanta presente como defecto la existencia
de erupciones?
b. ¿Cuál es la probabilidad de que una llanta en perfecto estado presente
pérdida de presión?
c. ¿Cuál es la probabilidad de que una llanta presente algún defecto?
Solución
Para determinar la primera respuesta, es necesario considerar que el total de
eventos posibles es de 649, pues esa es la cantidad de llantas defectuosas que se
obtuvo en la muestra. Así, la frecuencia relativa corresponde a 170 ÷ 649 ≈ 0,26 y
este valor se asigna a la probabilidad solicitada.
Para responder a la segunda preguntase debe considerar que el total de eventos
posibles corresponde a las 33 000 llantas que se produjeron en los tres días. De la
misma forma, es necesario notar que el total de eventos favorables es de 130, por
lo que la probabilidad solicitada correspondería a la frecuencia relativa, calculada
por 130 ÷ 33 000 ≈ 0,0039.
Finalmente, en la tercera pregunta se toma como total de eventos posibles las 33
000 llantas producidas y como total de eventos favorables las 649 llantas
defectuosas. Así, la frecuencia relativa correspondiente a la probabilidad solicitada
es 649 ÷ 33 000 ≈ 0,0197.
Ejemplo

Al lanzar una tachuela existen dos eventos posibles: A: que la tachuela caiga
sobre su cabeza (con la punta hacia arriba) B: que la tachuela caiga acostada

Debido a que no se conoce con exactitud la probabilidad de que la tachuela caiga


con la punta hacia arriba, proponga una estrategia que permita aproximar esta
probabilidad.

Solución

Se podría pensar que la probabilidad de ambos eventos es 1/ 2 (un caso favorable


entre dos posibles). Sin embargo, la forma irregular de la tachuela afecta el
resultado de forma incierta, lo que hace dudar que los eventos tengan la misma

404
posibilidad de ocurrir. Por lo que al dudar de una de las premisas de la definición
clásica de probabilidad (eventos simples equiprobables), no se debería aplicar esta
definición. Entonces, ¿cómo se podría encontrar la probabilidad de que la tachuela
caiga con la punta hacia arriba? Esta situación presenta una dificultad adicional a
la implementación de la definición clásica o laplaciana, ya que no todos los puntos
muestrales son equiprobables o por lo menos no se está seguro de que esto ocurra.
Ante esta situación, se puede aproximar la probabilidad del evento A por medio de
una muestra aleatoria, estrategia que se utilizó anteriormente y que también es
válida para enfrentar el problema. Es decir se puede repetir el experimento una
cantidad grande de veces y determinar la proporción de veces en que la tachuela
cae con la punta hacia arriba. Evidentemente es apenas una aproximación, pues el
valor encontrado podría estar muy lejos del valor real, entre más repeticiones se
realicen mejor será la estimación. Por ejemplo, suponga que se lanza 100 veces de
las cuales en 35 la tachuela cayó con la punta hacia arriba, entonces una
aproximación de la probabilidad de este evento es 35/ 100 = 0,35.

Tipos de eventos
Existen tres tipos de eventos: seguros, probables e imposibles.
 Un evento seguro es aquel cuya probabilidad de ocurrencia es igual a 1. Es
decir, sin importar las condiciones que se presenten, siempre va a suceder
este evento.
 Un evento probable es aquel que tiene casos favorables y casos no
favorables. Por esto, siempre su probabilidad va a dar un número decimal
ubicado entre 0 y el 1.
 Un evento imposible es un evento que no tiene ningún caso favorable en todo
el espacio muestral. Debido a esto, su probabilidad corresponde a 0.

Ejemplo
Una empresa de venta de vehículos en Costa Rica desea realizar un estudio con
respecto al grado de conformidad que tienen sus clientes, de acuerdo con su
género, luego de 6 meses de tener el vehículo, para tomar medidas que acerquen
más a los clientes a la compañía. Los resultados, luego de entrevistar por teléfono
a 2 540 clientes, fueron:

405
Grado de conformidad Hombres Mujeres Total

Muy conforme 288 658 946

Conforme 320 422 742

Poco conforme 256 225 481

Nada conforme 156 215 371

Total 1020 1520 2540

Para cada uno de los siguientes eventos, determine la probabilidad e indique su


clasificación:
1) Que un cliente entrevistado sea hombre o mujer.
2) Que la persona entrevistada no haya comprado un vehículo en la
empresa.
3) Que un cliente se sienta poco conforme con el vehículo adquirido.

Solución
En el primer caso, todos los clientes entrevistados eran hombres o mujeres, por lo
que la probabilidad de este evento se obtiene al resolver 2540 ÷ 2540 = 1. Como la
probabilidad obtenida es 1, el evento se clasifica como seguro.
En el segundo caso, se aclara en el enunciado que se entrevistó únicamente a
personas que eran clientes de la empresa, por lo que los eventos favorables son 0
y la probabilidad del evento es 0 ÷ 2540 = 0. Así, este evento se clasifica como
imposible.
Finalmente, en el último caso se tiene un total de 481 clientes poco conformes con
el vehículo, por lo que la probabilidad de este evento se obtiene al resolver 481 ÷
2540 ≈ 0,19. Como la probabilidad es un número entre 0 y 1, el evento se clasifica
como probable.
La ley de los grandes números
Esta ley indica que, al aumentar la cantidad de veces que se realiza un
experimento, la probabilidad frecuencial de los eventos tiende a acercarse a la
probabilidad clásica o Laplaciana.

Ejemplo
Al lanzar una moneda al aire y anotar si se obtuvo escudo o corona en cada
lanzamiento, se obtuvo el siguiente resultado:

406
Número de Cara
lanzamientos Escudo Corona

60 28 32

100 47 53

400 195 205

1000 499 501

¿Se cumple la probabilidad clásica?


Solución
Al determinar las frecuencias relativas, se puede completar la tabla de la siguiente
manera:

Número de Cara Frecuencia relativa


lanzamientos Escudo Corona Escudo Corona

60 28 32 0,4667 0,5333

100 47 53 0,4700 0,5300

400 195 205 0,4875 0,5125

1000 499 501 0,4990 0,5010

Al comparar las frecuencias relativas, se puede observar cómo la probabilidad de


obtener una corona va disminuyendo al mismo tiempo que va aumentando la
probabilidad de obtener un escudo. Al llegar a los mil lanzamientos, las
probabilidades de obtener escudo o corona son casi iguales.
Si se comparan estos resultados con la probabilidad de obtener escudo o corona al
lanzar una vez una moneda, se tiene que hay un evento favorable para cada caso
y en total son dos casos posibles, por lo que ambas probabilidades obtenidas
serían: 1 ÷ 2 = 0,5.
Como se puede observar, ambos casos se cumple la ley de los grandes números,
pues ambas probabilidades tienden a acercarse a 0,5.

407
Ejemplo
Un grupo de estudiantes afirma que el candidato A obtendrá cerca de un 60% de
los votos en las próximas elecciones. Para probarlo, cada uno realizó una encuesta
a diferente cantidad de personas, preguntando por quién votaría. Luego
presentaron los datos en una tabla como la siguiente:

Número de encuestados Votos por el candidato A

100 56

150 89

200 123

250 142

300 178

¿Es posible llegar a la conclusión de los estudiantes a partir de la información que


recolectaron?, ¿cómo?

Solución
De acuerdo con los datos de la tabla, se podría completar una tercera columna que
contenga la frecuencia relativa de los votos por el candidato A en cada encuesta:

Número de Votos por el Frecuencia


encuestados candidato A relativa

100 56 0,560

150 89 0,593

200 123 0,615

250 142 0,568

300 178 0,593

De acuerdo con las frecuencias relativas calculadas, la respuesta sería que sí es


posible concluir que el candidato obtendrá cerca del 60% de los votos, pues la

408
frecuencia relativa es cada vez más cercana a 0,60, que sería la probabilidad
correspondiente a ese 60%.

Ejemplo
Durante un juego de mesa, se lanzó un dado legal, de seis caras, un total
de 1500 veces. Se fueron haciendo anotaciones de los resultados cada 300
lanzamientos, de manera que los datos obtenidos se resumieron en la siguiente
tabla:

Lanzamientos 1 2 3 4 5 6

300 47 58 42 52 53 48

600 97 105 99 102 98 99

900 138 155 143 157 149 158

1200 195 201 204 197 201 202

1500 249 253 248 251 247 252

Determine:
1. La probabilidad de obtener un número par en cada caso.
2. La probabilidad de obtener un 5 en cada caso.
3. Marco afirmó que no importa la cantidad de veces que se lance el dado, pues
siempre se va a obtener la probabilidad que existe si se lanzara una única
vez. ¿Es esto cierto? Justifique su respuesta.

Solución
Para responder cada punto, se hace uso de la probabilidad frecuencial, por ser
tantos lanzamientos en cada caso.

1. Las probabilidades se obtienen sumando, en cada caso, las probabilidades de


obtener un 2, un 4 o un 6, de modo que se obtiene la siguiente tabla:

Lanzamientos 2 4 6 Probabilidad

300 58 52 48 158/300=0,5267

600 105 102 99 306/600=0,5100

409
900 155 157 158 470/900=0,5222

1200 201 197 202 600/1200=0,5000

1500 253 251 252 756/1500=0,5040

2. La probabilidad de obtener un 5 en cada caso, se puede construir una tabla


como la siguiente

Lanzamientos 5 Probabilidad

300 53 53/300=0,1767

600 98 98/600=0,1633

900 149 149/900=0,1656

1200 201 201/1200=0,1675

1500 247 247/1500=0,1647

3. Para responder la pregunta, hay que calcular la probabilidad de cada número


en cada cantidad de lanzamientos y comparar esos resultados con la
probabilidad de un solo lanzamiento:

Probabilidades
Lanzamientos
1 2 3 4 5 6

300 0,1567 0,1933 0,1400 0,1733 0,1767 0,1600

600 0,1617 0,1750 0,1650 0,1700 0,1633 0,1650

900 0,1533 0,1722 0,1589 0,1744 0,1656 0,1756

1200 0,1625 0,1675 0,1700 0,1642 0,1675 0,1683

1500 0,1660 0,1687 0,1653 0,1673 0,1647 0,1680

Al calcular la probabilidad de cada resultado en un solo lanzamiento, se utiliza la


probabilidad Laplaciana, por lo que para cada uno de los seis posibles resultados
se tiene como probabilidad 1/6=0,1667. Analizando los resultados obtenidos, todos

410
tienen cercanía con ese valor, por lo cual, aplicando la Ley de los grandes números,
la afirmación de Marco es correcta.

Práctica
1. Determine en cuáles casos se utiliza la probabilidad
frecuencial y en cuáles se puede utilizar la probabilidad
frecuencial:
a) Se lanza un dado de seis caras 5 veces.
b) Se lanzan cinco monedas al aire.
c) Se busca saber la probabilidad de que una llanta se produzca con defectos.
La producción diaria es de 2500 llantas.
d) Se hace una encuesta sobre la preferencia televisiva de los adultos mayores.
Se entrevistó a 1500 personas.
e) Se lanza una moneda tres veces.
f) Se lanza un dado de 20 caras 10 veces.
g) Se sacan 5 bolas de color de una urna que contiene 40 bolas de diferentes
colores.

2. Resolver los siguientes problemas vinculados con fenómenos aleatorios


en diversos contextos:
a) Calcula la probabilidad de aprobar un examen de matemáticas si se sabe
que hay una probabilidad de 0, 4 de no aprobar.

b) Se lanza un dado 400 veces y 60 veces ha salido 3. ¿Cuál es la frecuencia


relativa de este suceso?

c) Se lanza una moneda 200 veces y 98 veces ha salido corona. ¿Cuál es la


frecuencia relativa del suceso corona?

d) Se ha lanzado una moneda 1461 veces y en 817 ocasiones ha salido


escudo. ¿cuál es la probabilidad de que en el lanzamiento 1462 salga
escudo?

e) En una urna hay 3 bolas blancas, 2 rojas y 4 azules. Calcula la probabilidad


de que al extraer una bola al azar, salga roja.

f) Un dado está trucado para que el 6 tenga una probabilidad de salir de 0,25.
¿Cuál es la probabilidad de no obtener un 6?

411
3. Si se elige una persona de forma aleatoria, dada la siguiente tabla:

Ocupación Bajo Medio Alto Total

Ama de casa 8 26 6 40

Obreros 16 40 14 70

Ejecutivos 6 62 12 80

Profesionales 0 2 8 10

Total 30 130 40 200

 Determinar la probabilidad de que la persona elegida tenga las


siguientes ocupaciones:
 Ama de casa
 Obrero
 Ejecutivo
 Profesional

 Determinar la probabilidad de que el ingreso familiar de la persona


elegida sea:
 Bajo
 Medio
 Alto

 Determinar la probabilidad de que la persona elegida se clasifique dentro


del grupo:
 ejecutivo con ingreso alto.
 ama de casa con ingreso bajo.
 profesional con ingreso medio.

4. La probabilidad a priori de que exista petróleo en una determinada zona se


estima en 0, 2. Antes de decidir la perforación se realiza un estudio del
terreno. El 70 % de las veces en que el informe fue positivo se encontró
petróleo, mientras que el 95 % de las veces en que el informe fue
negativo el terreno no tenía petróleo. Determinar la probabilidad de que se
encuentre petróleo si el resultado del informe es positivo.

5. La Sociedad Mundial de Gemelos ha observado que de cada mil


personas con antecedentes familiares de gemelos, 58 tienen
descendencia de gemelos ¿Cuál es la probabilidad de que una persona
que tiene antecedentes familiares de gemelos, tenga descendencia de
gemelos?

412
6. Se lanza un dado de ocho caras, en total 200 veces. Determine para
cada evento si se clasifica como seguro, probable o imposible, basado
en la siguiente tabla:

Resultado 1 2 3 4 5 6 7 8

Repeticiones 23 27 24 26 25 22 29 24

 Obtener un 10 en el lanzamiento.
 Obtener un número menor que 4.
 Obtener un 0 en alguno de los lanzamientos.
 Obtener un número mayor o igual que 7.
 Obtener un número menor o igual que 8.
7. La división de control de calidad de una empresa que se dedica a la
producción de calculadoras realiza un muestreo para determinar la
probabilidad de que una calculadora falle. Se estudia una producción de
1000 calculadoras empacadas en cajas de 100. Clasifique cada muestra
como aleatoria, censo o determinista:
a) Se toma una calculadora al azar de cada caja.
b) Se verifica la calidad de las calculadoras que los clientes reportaron con algún
problema.
c) Se verifica cada calculadora de cada pedido de una sola librería.
d) De cada 100 calculadoras se revisa una al azar.
e) Se enumeran las calculadoras del 000 al 999. Se verifica la calculadora cuyo
número coincide con la serie del premio mayor de la lotería nacional.
f) En cada caja se enumeran las calculadoras del 00 al 99. Se verifica en cada
caja la calculadora que tiene el número del premio mayor de la lotería
nacional.
g) Se revisan las primeras diez calculadoras de la producción.
h) De cada cinco calculadoras, se escoge una al azar.
i) De cada cinco calculadoras, se revisa la segunda.
j) Se verifica la calidad de todas las calculadoras.

413
8. Considere que un dado legal se lanzó 1000 veces y se obtuvo los siguientes
resultados:

Cara Frecuencia absoluta Frecuencia relativa

1 125

2 166

3 171

4 0,18

5 164

a) Complete la distribución de frecuencias.


b) Determine la probabilidad de obtener un 2.
c) Determine la probabilidad de obtener un número menor que 3.
d) Determine la probabilidad de obtener un número mayor que 4.
9. Determine la probabilidad de cada uno de los siguientes eventos, con base en la
tabla adjunta:
Total de galletas Vainilla Chocolate Naranja Limón

50 12 13 11 14

100 22 27 30 21

150 35 40 37 38

200 45 44 52 49

250 65 64 60 61

a) Al fabricar 100 galletas, ¿cuál es la probabilidad de que una galleta sea de


chocolate?
b) Al fabricar 250 galletas, ¿cuál es la probabilidad de que una galleta sea de
limón?
c) Al fabricar 50 galletas, ¿cuál es la probabilidad de que una galleta sea de
vainilla?

414
d) Al fabricar 150 galletas, ¿cuál es la probabilidad de que una galleta sea de
naranja?
e) ¿Cuál es la probabilidad de fabricar una galleta de vainilla cuando se producen
200 galletas?
f) Sin importar la cantidad de galletas producidas, ¿cuál es la probabilidad de
que una galleta sea de naranja? Justifique su respuesta.

10.Durante el mes de marzo, las temperaturas en grados Celsius registradas en


el cantón de Goicoechea fueron: 30, 25, 23, 31, 28, 32, 32, 26, 27, 24, 23, 22,
29, 30, 21, 19, 24, 29, 31, 33, 32, 30, 27, 26, 28, 31, 20, 24, 25, 21 y 24. Con
base en esta información, construya una tabla de distribución de frecuencias, un
histograma y un polígono de frecuencias.

11.En un laboratorio se midió la longitud de veinte hojas de una planta. Las


longitudes obtenidas fueron las siguientes:

13,9 12,3 10,8 13,1 10,6

12,9 12,3 12,5 14,2 10,9

12,7 14,6 12,7 12,6 14,2

11,5 12,5 13,1 14,1 13,8

a) Complete la siguiente tabla de frecuencias:

Frecuencia
Frecuencia Frecuencia
Clase relativa
absoluta relativa
porcentual

De 10 a 11

De 11 a 12

De 12 a 13

De 13 a 14

De 14 a 15

b) Determine la probabilidad de que la medida de la hoja sea entre 10 y 13 cm.


c) Determine la probabilidad de que la medida de la hoja sea entre 12 y 14 cm.

415
d) Determine la probabilidad de que la medida de la hoja sea menor a 11 cm.
e) Determine la probabilidad de que la medida de la hoja sea menor a 14 cm.

12. Determine para cada situación si es conveniente resolverla utilizando


probabilidad clásica o probabilidad frecuencial:
a) Se busca la probabilidad de elegir un estudiante con el cabello negro en el 9-
1.
b) En una empresa envasan refresco en cajas de 254 ml. En ocasiones el
empaque lleva un poco menos del contenido. Se busca determinar la
probabilidad de que un envase tenga menos de 253 ml de refresco.
c) En una encuesta nacional se busca determinar la probabilidad de que un
votante se abstenga de hacerlo.
d) Se busca determinar si la moneda con la que el profesor hace elecciones al
azar es justa o no.
e) En un banco se busca determinar la probabilidad de que un cliente que gana
menos de ₡200000 mantenga al día los pagos de sus deudas.
f) Se tiene 10 bolas de igual tamaño en una bolsa. De ellas, 4 son blancas y el
resto son negras. Se pretende calcular la probabilidad de sacar una bola de
la bolsa y que ésta sea blanca.
g) Se busca la probabilidad de obtener un múltiplo de 7 al lanzar un dado.
h) Se busca la probabilidad de obtener un divisor de 60 al lanzar un dado.

13.La siguiente tabla muestra las edades de 1362 conductores que portan su
licencia de conducir. Determine para cada afirmación si es verdadera o falsa:

Edad Frecuencia absoluta

De 18 a menos de 38 200

De 28 a menos de 28 305

De 38 a menos de 48 406

De 48 a menos de 58 200

De 58 a menos de 68 200

De 68 a menos de 78 46

De 78 a menos de 88 5

416
a) 251 conductores tienen más de 58 años.
b) No existe ningún conductor de 88 años.
c) 505 conductores son menores de 38 años.
d) 46 conductores tienen 68 años.
e) La amplitud de las clases es 10.

14. Complete los datos faltantes en la siguiente tabla:

Masa corporal de 40 estudiantes que participaron en un seminario


Masa corporal Frecuencia absoluta Porcentaje

[40,47[ 21

[47,54[ 5 12,5

[54,61[ 20

[61,68[ 4

6 15

[75,82[ 5 12,5

15.En el siguiente cuadro se resume la cantidad de nacimientos en un hospital de


acuerdo con la edad de las madres:

Edad Nacimientos

[15,24[ 8

[24,33[ 35

[33,42[ 9

[42,51[ 2

a) ¿Cuántas mujeres menores de 33 años dieron a luz en ese hospital?


b) ¿Cuántas madres tienen al menos 33 años?
c) ¿Cuántas clases tiene la tabla?
d) ¿Cuál es la amplitud de cada clase?

417
16. Para cada una de las siguientes situaciones determine la probabilidad como
frecuencia relativa:
a) Al lanzar 1 000veces un dado se obtienen los resultados de la tabla:
 ¿Cuál es la frecuencia absoluta del 4?
 Calcula las frecuencias relativas de cada suceso.
 Estima la probabilidad de obtener un 4 con ese dado.

Número Frecuencia absoluta Frecuencia relativa

1 2

2 166

3 171

4 160

5 157

6 171

18. Al extraer al azar 1000 veces una bola de una caja donde hay 10 bolas
numeradas del 0 al 9, se obtienen los resultados de la tabla:
 ¿Cuál es la frecuencia absoluta de 6?
 Calcula las frecuencias relativas de cada suceso.
 Estima la probabilidad de sacar un 4.

Número Frecuencia absoluta Frecuencia relativa

0 96

1 102

2 93

3 101

4 105

5 101

6 102

7 103

8 98

418
9 99

19.Al extraer al azar 1 000 veces una bola de una caja, en la que hay 4 bolas
verdes, 4 bolas azules y 4 bolas rojas, se obtienen los resultados de la tabla:
 ¿Cuál es la frecuencia absoluta de la bola roja?
 Calcula las frecuencias relativas de cada suceso.
 Estima la probabilidad de extraer una bola roja.

Bola Frecuencia absoluta Frecuencia relativa

Verde 341

Roja 332

Azul 327

20.Al lanzar 1 000 veces un dado, se obtienen los resultados de la tabla:


 ¿Cuál es la frecuencia absoluta de 2?
 Calcula las frecuencias relativas de cada suceso.
 Estima la probabilidad de obtener un 3 con ese dado.

Número Frecuencia absoluta Frecuencia relativa

1 169

2 165

3 166

4 172

5 160

6 168

419
Práctica de la Unidad 4

Considere el siguiente contexto para responder las preguntas 1 y 2:

El cuestionario

La profesora de Matemáticas de un colegio pide a sus estudiantes completar el


siguiente cuestionario para elaborar un expediente por cada estudiante:

Cuestionario individual

Complete el cuestionario en forma individual. La información es confidencial y será


parte del expediente de cada uno de ustedes.

Nombre completo:

Dirección del
hogar:

Números
Edad (años
telefónicos (casa y
cumplidos]:
celular):

Fecha de
Número de cédula:
nacimiento:

¿Con quién vive? Papá ( ) Mamá ( ) Abuelo(a) ( ) Otros ( )

Cantidad de Asignatura
hermanos: preferida:

Estatura: Masa (peso):

Enfermedades,
¿cuáles?

420
1) Un ejemplo de variable cuantitativa discreta corresponde a:

A) la estatura.
B) las enfermedades.
C) la asignatura preferida.
D) la cantidad de hermanos.
2) Un ejemplo de variable cuantitativa continua es la:
A) masa.
B) fecha de nacimiento.
C) edad en años cumplidos.
D) cantidad de personas con las que vive.

3) Considere las siguientes situaciones:


I. Un estudiante necesita comprar un cuaderno de 100 hojas (según se indica en la
portada del cuaderno) y desea conocer la probabilidad de que el cuaderno que
compre tenga más de 100 hojas.
II. Un grupo está conformado por 35 estudiantes y se desea conocer la
probabilidad de que al elegir uno de ellos, su edad sea igual o mayor a 15 años.
¿Cuáles de ellas corresponden a eventos para los cuales su probabilidad
únicamente puede ser estimada mediante el empleo de la frecuencia relativa
(probabilidad frecuencial)?
A) Ambas C)Ninguna
B) Solo la I D)Solo la II

421
Considere el siguiente contexto y responda las preguntas 4, 5 y 6:

Características de las viviendas

Por medio del Censo 2011 realizado por el Instituto Nacional de Estadística y Censos (INÉC), se
realizó un estudio sobre las características de las viviendas, en particular se obtuvo información
sobre si la persona posee o no vivienda y en caso afirmativo si esta es de uso colectivo o
individual. Los resultados por provincia se muestran en el siguiente cuadro:

Costa Rica: Población total por tipo de vivienda,


según provincia

Población Con vivienda


total de Sin
Provincia
la Vivienda
Individual Colectiva
provincia

1 404 1 393
San José 4800 360
242 082

Alajuela 848 146 843 023 5044 79

Cartago 490 903 488 671 2198 34

Heredia 433 677 432 427 1215 35

Guanacaste 326 953 325 772 1157 24

Puntarenas 410 929 409 525 1337 67

Limón 386 862 384 563 2267 32

4 301 4 233
Total 18 018 631
712 063

Adaptado de: http://www inec.go.cr

422
4) De la población total del país, ¿cuál es aproximadamente la frecuencia
porcentual que corresponde a la población total del país con vivienda
colectiva?
A) 0,34%
B) 0, 42%
C) 2,39%
D) 4,19%

5) De la población total con vivienda individual, ¿cuál es la provincia que posee la


menor frecuencia relativa?
A) Limón
B) Alajuela
C) Heredia
D) Guanacaste

6) Considere las siguientes proposiciones:


I. A menor población total de una provincia, menor es la población sin vivienda.
II. Más del 75% de la población sin vivienda se encuentra en las provincias de
San José, Alajuela, Cartago o Heredia.

¿Cuáles de ellas son verdaderas?


A) Ambas
B) Ninguna
C) Solo la I
D) Solo la II

423
Considere el siguiente contexto y responda las preguntas 7 y 8:

Los dados

Un estudiante de un colegio realiza un experimento que consiste en lanzar 10 veces un dado legal y registrar
el número que aparece en la cara superior. En el siguiente cuadro se muestra el resultado del experimento,
en donde la “ X ” indica el número que sale en cada lanzamiento.

Número de la cara superior del dado


N
1 2 3 4 5 6

1 X

2 X

3 X

4 X

5 X

6 X

7 X

8 X

9 X

10 X

N: El lanzamiento

424
7) ¿Cuál es la probabilidad frecuencial de que salga un 5 ?

A) 0,10%
B) 0,17%
C) 0,30%
D) 0,50%
8) ¿Cuántas veces más probable es que salga el número 6 respecto a que salga el
número 1 ?
A) Dos veces.
B) Tres veces.
C) Cinco veces.
D) Seis veces
9) Considere las siguientes variables cuantitativas:
I. La edad, en años cumplidos, de las mascotas del barrio.
II. La estatura, en centímetros, de las estudiantes de sétimo año del colegio.
De ellas, ¿cuál o cuáles son variables discretas?
A) Ambas
B) Ninguna
C) Solo la I
D) Solo la II

10)¿Cuál de las siguientes proposiciones representa una variable cuantitativa


continua?
a. Cantidad de estudiantes que les gusta jugar voleibol.
b. Cantidad de tiempo que se dura en una llamada telefónica.
c. Cantidad de vehículos parqueados en las afueras de un supermercado.
d. Cantidad de teléfonos inteligentes que se venden en una tienda durante una
semana.

425
11) ¿Cuál de las siguientes variables corresponde a una variable cuantitativa
discreta?
a. La cantidad de masa de una persona.
b. El área de construcción de una vivienda.
c. El ingreso mensual en colones de un empleado.
d. La suma de los puntos obtenidos al lanzar dos dados.

Considere la información que se representa en la siguiente tabla, para contestar


las preguntas 12, 13 y 14:

Distribución del total de personas


que asistieron a un centro médico
en Costa Rica, según la edad en
años cumplidos, durante el mes de
agosto del año 2016

Edad Frecuencia absoluta

0,9 18

9,18 36

18, 27 39

 27,36 12

36, 45 9

45,54 15

54,63 21

TOTAL 150

426
12) La frecuencia relativa de la clase  27,36 corresponde a:

a. 0, 06
b. 0, 08
c. 0,18

d. 0, 24

13) El porcentaje de personas que asistieron al centro médico con edades, igual o
superior a los 45 años, corresponde a:

a. 24
b. 30
c. 36
d. 42

14) De los diferentes grupos que asistieron al centro médico en el mes de


agosto de 2016 , la clase que presentó mayor frecuencia corresponde a:

a. 9,18

b. 18, 27

c. 36, 45

d. 
54,63

Considere el siguiente contexto, para responder las preguntas 15, 16 y 17:

427
Personal con o sin experiencia laboral en el
puesto, contratado en diferentes departamentos
en una empresa comercial

CON SIN
EXPERIENCIA EXPERIENCIA
DEPARTAMENTO TOTAL
EN EL EN EL
PUESTO PUESTO

Bodega 9 3 12

Informática 1 2 3

Contabilidad 12 4 16

Secretariado 8 6 14

TOTAL 30 15 45

15) Si del total de personas contratadas sin experiencia en el puesto, se


elige una al azar, entonces hay mayor probabilidad de que esta pertenezca
al departamento de:

a. bodega.
b. informática.
c. secretariado.
d. contabilidad
16) Considere las siguientes proposiciones:
I. Si del total de personas contratadas se elige una al azar, entonces, hay mayor
probabilidad de que esta tenga experiencia en el puesto.
II. Si de las personas contratadas en informática, se elige una al azar, entonces
hay menor probabilidad de que sea con experiencia en esa área.
¿Cuál o cuáles de ellas son verdaderas?
A) Ambas
B) Ninguna
C) Solo la I
D) Solo la II

428
17) Si de las personas contratadas en el departamento de contabilidad, se
elige una al azar, entonces, la probabilidad de que esta tuviera experiencia,
corresponde a:

4
A)
15

4
B)
16

12
C)
16

12
D)
30

429
Bibliografía
Arquímedes. (2011). Wikipedia. Recuperado en agosto 18, 2011, de
http://es.wikipedia.org/wiki/Arqu%C3%ADmedes

Berenger, J. (2011). Los sulbasutras de la India. Historia de las matemáticas.


Recuperado de http://astroseti.org/articulo/4585/los-sulbasutras-de-la-india
Hernández, V. (2002).

La geometría analítica de Descartes y Fermat: ¿Y Apolonio? Apuntes de historia de


las matemáticas.1 (1). Recuperado de
http://www.mat.uson.mx/depto/publicaciones/apuntes/pdf/1-1-4-analitica.pdf

Larson, R. &Hostetler, R. (1989). Cálculo y geometría analítica. México: McGraw-


Hill.

Mankiewicz, R. (2005). Historia de las matemáticas: Del cálculo al caos. España:


Ediciones Paidós Iberia, S.A.

Ruiz, A. (2003). Historia y filosofía de las matemáticas. San José, Costa Rica:
EUNED.

Ruiz, A. (1999). Geometrías no euclidianas: Breve historia de una gran revolución


intelectual. San José, Costa Rica: Editorial de la Universidad de Costa Rica.

Swokoski, P. (1995). Álgebra con geometría analítica. México D.F.: Grupo Editorial
Iberoamérica.

Vázquez, A. & De Santiago, J. (2007). Geometría Analítica. México: Pearson


Educación.

Barrantes, H., Díaz, P., Murillo, M. y Soto, A. (1998). Introducción a la teoría de


números. San José: EUNED
Enzensberger, H. (1998).

The National Council of Teachers of Mathematics (2006) Historical topics for the
Mathematics classroom. Reston: NCTM, Inc.
Golden Ratio. Recuperado en mayo, 10, 2011 de
http://en.wikipedia.org/wiki/Golden_ratio

Chaves,C. y León, A. (2003). La biblia de las matemáticas. México: Editorial


Letrarte, S.A.
Stewart, J. (2002). Cálculo. Trascendentes tempranas. México: International
Thomson learning editores, S.A.

430
García, M. Matemática: Función cuadrática, parábola y ecuación de segundo grado.
Aportes para la enseñanza. Nivel Medio. Buenos Aires: Gobierno de la ciudad.

Ministerio de Educación Pública. (2012). Programa de estudios Matemáticas. San


José, Costa Rica.

Batanero, C. (2001). Didáctica de la estadística. Granada, España: Grupo de


Educación Estadística de la Universidad de Granada.

Gómez, M. (1999). Elementos de Estadística Descriptiva. San José, Costa Rica:


EUNED.

Fernández, C. y Fuentes, F. (1995). Curso de estadística descriptiva: teoría y


práctica. España: Ariel.

Meter, J., y Wasserman, W. (1973). Fundamentos de Estadística. Traducción de la


tercera edición. Compañía Editorial Continental, S.A. (C.E.C.S.A).

Quintana, C. (1996). Elementos de Inferencia Estadística. San José, Costa Rica:


EUCR.

431

También podría gustarte